You are on page 1of 697

Series WX1YZ/2 SET~1

Q.P. Code 30/2/1


Roll No. narjmWu àíZ-nÌ H$moS> >H$mo CÎma-nwpñVH$m Ho$
_wI-n¥ð >na Adí` {bIo§ &
Candidates must write the Q.P. Code on
the title page of the answer-book.

J{UV (_mZH$)
MATHEMATICS (STANDARD)
*
:3 : 80
Time allowed : 3 hours Maximum Marks : 80

ZmoQ> / NOTE :
(i) - 23
Please check that this question paper contains 23 printed pages.
(ii) - - -
-
Q.P. Code given on the right hand side of the question paper should be written on the title
page of the answer-book by the candidate.
(iii) - 38
Please check that this question paper contains 38 questions.
(iv) -

Please write down the serial number of the question in the answer-book before
attempting it.
(v) - 15 -
10.15 10.15 10.30 -
-
15 minute time has been allotted to read this question paper. The question paper will be
distributed at 10.15 a.m. From 10.15 a.m. to 10.30 a.m., the students will read the
question paper only and will not write any answer on the answer-book during this period.

30/2/1 JJJJ Page 1 P.T.O.


:
:
(i) 38
(ii)
(iii) 1 18 (MCQ) 19 20

(iv) 21 25 (VSA)

(v) 26 31 (SA)
(vi) 32 35 (LA)
(vii) 36 38

(viii) 2 2
2 3

22
(ix) =
7

(x)

IÊS> H$

(MCQ) 1

1. {ZåZ{b{IV _| go {H$g {ÛKmV g_rH$aU Ho$ _ybm| H$m `moJ\$b 4 h¡ ?

(a) 2x2 4x + 8 = 0 (b) x2 + 4x + 4 = 0


4
(c) 2 x2 x+1=0 (d) 4x2 4x + 4 = 0
2

2. {ÌÁ`m 14 cm dmbo EH$ d¥Îm Ho$ {ÌÁ`I§S>, {OgH$m Ho$ÝÐr` H$moU 90 h¡, H$s g§JV Mmn H$s
?

(a) 22 cm (b) 44 cm
(c) 88 cm (d) 11 cm
30/2/1 JJJJ Page 2
General Instructions :
Read the following instructions very carefully and strictly follow them :
(i) This question paper contains 38 questions. All questions are compulsory.
(ii) This question paper is divided into five Sections A, B, C, D and E.
(iii) In Section A, Questions no. 1 to 18 are multiple choice questions (MCQs) and
questions number 19 and 20 are Assertion-Reason based questions of 1 mark
each.
(iv) In Section B, Questions no. 21 to 25 are very short answer (VSA) type
questions, carrying 2 marks each.
(v) In Section C, Questions no. 26 to 31 are short answer (SA) type questions,
carrying 3 marks each.
(vi) In Section D, Questions no. 32 to 35 are long answer (LA) type questions
carrying 5 marks each.
(vii) In Section E, Questions no. 36 to 38 are case study based questions carrying
4 marks each. Internal choice is provided in 2 marks questions in each
case-study.
(viii) There is no overall choice. However, an internal choice has been provided in
2 questions in Section B, 2 questions in Section C, 2 questions in Section D and
3 questions in Section E.
22
(ix) Draw neat diagrams wherever required. Take = wherever required, if not
7
stated.
(x) Use of calculators is not allowed.

SECTION A

This section comprises multiple choice questions (MCQs) of 1 mark each.

1. Which of the following quadratic equations has sum of its roots as 4 ?


(a) 2x2 4x + 8 = 0 (b) x2 + 4x + 4 = 0
4
(c) 2 x2 x+1=0 (d) 4x2 4x + 4 = 0
2

2. What is the length of the arc of the sector of a circle with radius 14 cm
and of central angle 90 ?
(a) 22 cm (b) 44 cm
(c) 88 cm (d) 11 cm
30/2/1 JJJJ Page 3 P.T.O.
3. `{X ABC PQR _|, A = 32 Am¡a R = 65 h¡, Vmo B H$s _mn h¡ :
(a) 32 (b) 65
(c) 83 (d) 97

4. `{X Am¡a àmH¥$V g§»`mE± h¢ Am¡a p g§»`m q H$m JwUO h¡, Vmo Am¡a q H$m HCF
?
(a) pq (b ) p
(c) q (d) p+q

5. EH$ Am`V ABCD {OgHo$ VrZ erf© B(0, 0), C(3, 0) Am¡a D(0, 4) h¢, CgHo$ erf© A Ho$
{ZX}em§H$ hm|Jo :
(a) (4, 0) (b) (0, 3)
(c) (3, 4) (d) (4, 3)

6. `{X g_rH$aU `w½_ 3x y+8=0 Am¡a 6x ry + 16 = 0 Ûmam {Zê${nV aoImE± g§nmVr


h¢, Vmo H$m _mZ hmoJm :
1 1
(a) (b)
2 2
(c) 2 (d) 2

7. EH$ W¡bo _| 100 nÎmo h¢ {OZ na 1 go 100 VH$ H$s g§»`mE± A§{H$V h¢ & Bg W¡bo _| go EH$
nÎmm `mÑÀN>`m {ZH$mbm OmVm h¡ & Bg nÎmo na EH$ nyU© KZ g§»`m A§{H$V hmoZo H$s àm{`H$Vm
?
1 3
(a) (b)
20 50
1 7
(c) (d)
25 100

8. g_rH$aU `w½_ x = a Am¡a y = b Ûmam {Zê${nV aoImE± J«m\$s` ê$n _| :


(a) nañna g_m§Va hmoVr h¢
(b) {~ÝXþ (b, a) na à{VÀN>oXr hmoVr h¢
(c) g§nmVr hmoVr h¢
(d) {~ÝXþ (a, b) na à{VÀN>oXr hmoVr h¢
30/2/1 JJJJ Page 4
3. If ABC A = 32 and R = 65 , then the measure of
B is :
(a) 32 (b) 65
(c) 83 (d) 97

4.

(a) pq (b ) p
(c) q (d) p+q

5. The coordinates of the vertex A of a rectangle ABCD whose three vertices


are given as B(0, 0), C(3, 0) and D(0, 4) are :
(a) (4, 0) (b) (0, 3)
(c) (3, 4) (d) (4, 3)

6. If the pair of equations 3x y + 8 = 0 and 6x ry + 16 = 0 represent


coincident lines, then t
1 1
(a) (b)
2 2
(c) 2 (d) 2

7. A bag contains 100 cards numbered 1 to 100. A card is drawn at random


from the bag. What is the probability that the number on the card is a
perfect cube ?
1 3
(a) (b)
20 50
1 7
(c) (d)
25 100

8. The pair of equations x = a and y = b graphically represents lines which


are :
(a) parallel
(b) intersecting at (b, a)
(c) coincident
(d) intersecting at (a, b)

30/2/1 JJJJ Page 5 P.T.O.


9. `{X ~hþnX 6x2 + 37x (k 2) H$m EH$ eyÝ`H$, Xÿgao eyÝ`H$ H$m ì`wËH«$_ hmo, Vmo k H$m
?

(a) 4 (b) 6

(c) 6 (d) 4

10. EH$ R>mog AY©-Jmobo, {OgH$m ì`mg h¡, H$m g§nyU© n¥ð>r` joÌ\ ?

2 2
(a) 3 d (b) 2 d

1 3
(c) d2 (d) d2
2 4

11. gmW CN>mbo OmVo h¢, Vmo A{YH$-go-A{YH$ EH$ nQ> àmßV hmoZo H$s
?

3 4
(a) (b)
8 8

5 7
(c) (d)
8 8

12. Xr JB© AmH¥${V _|, DE BC & `{X AD = 2 BH$mB©, DB = AE = 3 BH$mB© Am¡a


EC = x BH$mB© h¡, Vmo x H$m _mZ hmoJm :

(a) 2 (b) 3
9
(c) 5 (d)
2

30/2/1 JJJJ Page 6


9. If one zero of the polynomial 6x2 + 37x (k 2) is reciprocal of the other,
then what is the value of k ?

(a) 4 (b) 6

(c) 6 (d) 4

10.

2 2
(a) 3 d (b) 2 d

1 2 3 2
(c) d (d) d
2 4

11. If three coins are tossed simultaneously, what is the probability of getting
at most one tail ?

3 4
(a) (b)
8 8

5 7
(c) (d)
8 8

12. In the given figure, DE BC. If AD = 2 units, DB = AE = 3 units and


EC = x units, then the value of x is :

(a) 2 (b) 3
9
(c) 5 (d)
2

30/2/1 JJJJ Page 7 P.T.O.


13. 6 cm b§~r h¡ & Bg gwB© Ûmam 7:20 a.m. Am¡a 7:55 a.m. Ho$
~rM Omo H$moU a{MV hmoJm, dh h¡ :

35 35
(a) (b)
4 2

(c) 35 (d) 70

14. ~hþnX p(x) = x2 + 4x + 3 Ho$ eyÝ`H$ h¢ :


(a) 1, 3 (b) 1, 3
(c) 1, 3 (d) 1, 3

15. Xr JB© AmH¥${V _|, EH$ d¥Îm Ho$ n[aJV EH$ MVw^w©O PQRS ~Zm h¡ & `hm± PA + CS ~am~a
h¡ :

(a) QR Ho$ (b) PR Ho$


(c) PS Ho$ (d) PQ Ho$
16. `{X Am¡a , {ÛKmV ~hþnX p(x) = x2 ax b Ho$ eyÝ`H$ h¢, Vmo 2 + 2 H$m _mZ
hmoJm :
(a) a2 2b (b) a2 + 2b
2 2
(c) b 2a (d) b + 2a
x y
17. aoIm + = 1 VWm {ZX}em§H$ Ajm| go ~Zo {Ì^wO H$m joÌ\$b h¡ :
a b
1
(a) ab (b) ab
2
1
(c) ab (d) 2ab
4

30/2/1 JJJJ Page 8


13. The hour-hand of a clock is 6 cm long. The angle swept by it between
7:20 a.m. and 7:55 a.m. is :
35 35
(a) (b)
4 2

(c) 35 (d) 70
14. The zeroes of the polynomial p(x) = x2 + 4x + 3 are given by :
(a) 1, 3 (b) 1, 3
(c) 1, 3 (d) 1, 3
15. In the given figure, the quadrilateral PQRS circumscribes a circle. Here
PA + CS is equal to :

(a) QR (b) PR
(c) PS (d) PQ
16. If and are the zeroes of the quadratic polynomial p(x) = x2 ax b,
then the value of 2 + 2 is :
(a) a2 2b (b) a2 + 2b
2 2
(c) b 2a (d)
b + 2a
x y
17. The area of the triangle formed by the line = 1 with the coordinate
a b
axes is :
1
(a) ab (b) ab
2
1
(c) ab (d) 2ab
4
30/2/1 JJJJ Page 9 P.T.O.
18. Xr JB© AmH¥${V _|, AB PQ & `{X AB = 6 cm, PQ = 2 cm Am¡a OB = 3 cm h¡, Vmo
OP H$s bå~mB© hmoJr :

(a) 9 cm (b) 3 cm
(c) 4 cm (d) 1 cm

19 20 1
(A) (R)
(a), (b), (c) (d)
(a) A{^H$WZ (A) Am¡a VH©$ (R) XmoZm| ghr h¢ Am¡a VH©$ (R), A{^H$WZ (A) H$s ghr
ì¶m»¶m H$aVm h¡ &
(b) A{^H$WZ (A) Am¡a VH©$ (R) XmoZm| ghr h¢, naÝVw VH©$ (R), A{^H$WZ (A) H$s ghr
ì¶m»¶m H$aVm h¡ &
(c) A{^H$WZ (A) ghr h¡, naÝVw VH©$ (R) µJbV h¡ &
(d) A{^H$WZ (A) µJbV h¡, naÝVw VH©$ (R) ghr h¡ &
19. (A) : d¥Îm Ho$ {H$gr q~Xþ na ñne©-aoIm ñne© q~Xþ go OmZo dmbr {ÌÁ`m na bå~
hmoVr h¡ &
(R) : ~mø q~Xþ go d¥Îm na ItMr JB© ñne©-aoImAm| H$s bå~mB`m± ~am~a hmoVr h¢ &
20. (A) : ~hþnX p(x) = x2 + 3x + 3 Ho$ Xmo dmñV{dH$ eyÝ`H$ h¢ &
(R) : EH$ {ÛKmV ~hþnX Ho$ A{YH$-go-A{YH$ Xmo dmñV{dH$ eyÝ`H$ hmo gH$Vo h¢ &
30/2/1 JJJJ Page 10
18. In the given figure, AB PQ. If AB = 6 cm, PQ = 2 cm and OB = 3 cm,
then the length of OP is :

(a) 9 cm (b) 3 cm

(c) 4 cm (d) 1 cm
Questions number 19 and 20 are Assertion and Reason based questions carrying
1 mark each. Two statements are given, one labelled as Assertion (A) and the
other is labelled as Reason (R). Select the correct answer to these questions from
the codes (a), (b), (c) and (d) as given below.
(a) Both Assertion (A) and Reason (R) are true and Reason (R) is the
correct explanation of the Assertion (A).
(b) Both Assertion (A) and Reason (R) are true, but Reason (R) is not
the correct explanation of the Assertion (A).
(c) Assertion (A) is true, but Reason (R) is false.
(d) Assertion (A) is false, but Reason (R) is true.
19. Assertion (A) : A tangent to a circle is perpendicular to the radius
through the point of contact.
Reason (R) : The lengths of tangents drawn from an external point to a
circle are equal.

20. Assertion (A) : The polynomial p(x) = x2 + 3x + 3 has two real zeroes.
Reason (R): A quadratic polynomial can have at most two real zeroes.

30/2/1 JJJJ Page 11 P.T.O.


IÊS> I
(VSA) 2

21. {gÕ H$s{OE {H$ 2 + 3 EH$ An[a_o` g§»`m h¡, {X`m J`m h¡ {H$ 3 EH$ An[a_o`
g§»`m h¡ &

3
22. (H$) `{X 4 cot2 45 sec2 60 + sin2 60 + p = h¡, Vmo p H$m _mZ kmV H$s{OE &
4

AWdm
(I) `{X cos A + cos2 A = 1 h¡, Vmo sin2 A + sin4 A H$m _mZ kmV H$s{OE &

23. Xem©BE {H$ q~Xþ ( 2, 3), (8, 3) Am¡a (6, 7) EH$ g_H$moU {Ì^wO Ho$ erf© h¢ &

24. (H$) EH$ g_Vb O_rZ H$s 3 JwZr b§~r


h¡ & gy`© H$m CÞVm§e kmV H$s{OE &
AWdm
(I) ^y{_ Ho$ EH$ q~Xþ go, Omo _rZma Ho$ nmX-q~Xþ go 30 m H$s Xÿar na h¡, _rZma Ho$
{eIa H$m CÞ`Z H$moU 30 h¡ & _rZma H$s D±$MmB© kmV H$s{OE &

25. Xr JB© AmH¥${V _|, d¥Îm H$m H|$Ð O h¡ & q~Xþ A go Bg d¥Îm na AB Am¡a AC ñne©-aoImE±
ItMr JB© h¢ & `{X BAC = 65 h¡, Vmo BOC H$s _mn kmV H$s{OE &

30/2/1 JJJJ Page 12


SECTION B

This section comprises very short answer (VSA) type questions of 2 marks each.

21. Prove that 2 + 3 is an irrational number, given that 3 is an irrational


number.

3
22. (a) If 4 cot245 sec2 60 + sin2 60 + p = , then find the value of p.
4
OR
(b) If cos A + cos2 A = 1, then find the value of sin2 A + sin4 A.

23. Show that the points ( 2, 3), (8, 3) and (6, 7) are the vertices of a
right-angled triangle.

24. (a) The length of the shadow of a tower on the plane ground is 3
times the height of the tower. Find the angle of elevation of the
sun.
OR
(b) The angle of elevation of the top of a tower from a point on the
ground which is 30 m away from the foot of the tower, is 30 . Find
the height of the tower.

25. In the given figure, O is the centre of the circle. AB and AC are tangents
drawn to the circle from point A. If BAC = 65 , then find the measure of
BOC.

30/2/1 JJJJ Page 13 P.T.O.


IÊS> J

(SA) 3

26. (H$) g§»`mAm| 18180 Am¡a 7575 H$m A^mÁ` JwUZI§S>Z {d{Y Ûmam LCM kmV
H$s{OE & BZ Xmo g§»`mAm| H$m HCF ^r kmV H$s{OE &
AWdm
(I) VrZ K§{Q>`m± 6, 12 Am¡a 18 {_ZQ>m| Ho$ A§Vamb na ~OVt h¢ & `{X `o VrZm| K§{Q>`m±
EH$ gmW 6 a.m. na ~Ot hm|, Vmo CgHo$ níMmV² do VrZm| EH$ gmW H$~ ~O|Jr ?

27. {gÕ H$s{OE :


1 1 1
cos sin =
cos sin tan cot

28. `{X q~Xþ Q(0, 1), q~XþAm| P(5, 3) Am¡a R(x, 6) go EH$g_mZ Xÿar na hmo, Vmo x Ho$ _mZ
kmV H$s{OE &

29. EH$ H$ma Ho$ Xmo dmBna (wipers) h¢, Omo nañna H$^r AmÀN>m{XH$ Zht hmoVo h¢ & àË`oH$
dmBna H$s nÎmr H$s b§~mB© 21 cm h¡ Am¡a 120 Ho$ H$moU VH$ Ky_ H$a g\$mB© H$a gH$Vm
h¡ & XmoZm| n{Îm`m| H$s àË`oH$ ~whma Ho$ gmW {OVZm joÌ\$b gm\$ hmo OmVm h¡, dh kmV
H$s{OE &

30. (H$) `{X a¡{IH$ g_rH$aU {ZH$m`


2x + 3y = 7 VWm 2ax + (a + b)y = 28

Ho$ An[a{_V ê$n go AZoH$ hb hm|, Vmo Am¡a Ho$ _mZ kmV H$s{OE &
AWdm
(I) `{X 217x + 131y = 913 Am¡a 131x + 217y = 827 hm|, Vmo x Am¡a y Ho$ _mZ
kmV H$aZo Ho$ {bE g_rH$aU hb H$s{OE &
30/2/1 JJJJ Page 14
SECTION C

This section comprises of short answer (SA) type questions of 3 marks each.

26. (a) Find by prime factorisation the LCM of the numbers 18180 and
7575. Also, find the HCF of the two numbers.

OR

(b) Three bells ring at intervals of 6, 12 and 18 minutes. If all the


three bells rang at 6 a.m., when will they ring together again ?

27. Prove that :


1 1 1
cos sin = .
cos sin tan cot

28. If Q(0, 1) is equidistant from P(5, 3) and R(x, 6), find the values of x.

29. A car has two wipers which do not overlap. Each wiper has a blade of
length 21 cm sweeping through an angle of 120 . Find the total area
cleaned at each sweep of the two blades.

30. (a) If the system of linear equations


2x + 3y = 7 and 2ax + (a + b)y = 28

OR

(b) If 217x + 131y = 913 and

131x + 217y = 827,

then solve the equations for the values of x and y.

30/2/1 JJJJ Page 15 P.T.O.


31. Xr JB© AmH¥${V _|, d¥Îm H$m H|$Ð O VWm QPR d¥Îm Ho$ q~Xþ P na ñne©-aoIm h¡ & {gÕ
H$s{OE {H$ QAP + APR = 90 .

IÊS> K
(LA) 5

32. 45, 39, 33, ....... Ho$ {H$VZo nXm| H$m `moJ\$b 180 hmoJm ? Xmohao CÎma H$s
ì`m»`m H$s{OE &
33. (H$) g_wÐ-Vb go 75 m D±$Mr bmBQ>-hmD$g Ho$ {eIa go XoIZo na Xmo g_wÐr OhmOm| Ho$
AdZ_Z H$moU 30 Am¡a 60 h¢ & `{X bmBQ>-hmD$g Ho$ EH$ hr Amoa EH$
OhmO Xÿgao OhmO Ho$ R>rH$ nrN>o hmo, Vmo Xmo OhmOm| Ho$ ~rM H$s Xÿar kmV H$s{OE &
( 3 = 1·73 H$m à`moJ H$s{OE)
AWdm
(I) ^y{_ Ho$ EH$ q~Xþ go EH$ 30 m D±$Mo ^dZ Ho$ {eIa na bJr EH$ g§Mma _rZma Ho$
Vb Am¡a {eIa Ho$ CÞ`Z H$moU H«$_e: 30 Am¡a 60 h¢ & g§Mma _rZma H$s D±$MmB©
kmV H$s{OE & ( 3 = 1·73 H$m à`moJ H$s{OE)

34.
3 {_ZQ> H$s 100 Ad{Y`m| _| {H$VZr h¢ Am¡a Bgo ZrMo Xr JB© Vm{bH$m _| gmam§{eV {H$`m
J`m h¡
H$mam| H$s
0 10 10 20 20 30 30 40 40 50 50 60 60 70 70 80
g§»`m
~ma§~maVm
7 14 13 12 20 11 15 8
(Ad{Y`m±)

30/2/1 JJJJ Page 16


31. In the given figure, O is the centre of the circle and QPR is a tangent to it
at P. Prove that QAP + APR = 90 .

SECTION D

This section comprises long answer (LA) type questions of 5 marks each.

32. How many terms of the arithmetic progression 45, 39, 33, ........ must be
taken so that their sum is 180 ? Explain the double answer.

33. (a) As observed from the top of a 75 m high lighthouse from the
sea-level, the angles of depression of two ships are 30 and 60 . If
one ship is exactly behind the other on the same side of the
lighthouse, find the distance between the two ships.
(Use 3 = 1·73)

OR
(b) From a point on the ground, the angle of elevation of the bottom
and top of a transmission tower fixed at the top of 30 m high
building are 30 and 60 , respectively. Find the height of the
transmission tower. (Use 3 = 1·73)
34. A student noted the number of cars passing through a spot on a road for
100 periods each of 3 minutes and summarised it in the table given
below. Find the mean and median of the following data.
Number of
0 10 10 20 20 30 30 40 40 50 50 60 60 70 70 80
cars
Frequency
7 14 13 12 20 11 15 8
(periods)

30/2/1 JJJJ Page 17 P.T.O.


35. (H$) EH$ {Ì^wO H$s ^wOmE± AB Am¡a BC VWm _mpÜ`H$m AD EH$ AÝ` {Ì^wO
ABC
PQR ^wOmAm| PQ Am¡a QR VWm _mpÜ`H$m PM Ho$ g_mZwnmVr h¢ &
Xem©BE {H$ ABC PQR h¡ &
AWdm
(I) g_m§Va MVw^w©O ABCD H$s ^wOm CD Ho$ _Ü`-q~Xþ M go EH$ aoIm BM ItMr JB©
Omo {dH$U© AC H$mo q~Xþ L AD H$mo q~Xþ E na H$mQ>Vr h¡ &
{gÕ H$s{OE {H$ EL = 2BL.
IÊS> L>
3 4
àH$aU AÜ``Z 1
36. EH$ {dÚmb` Ho$ dm{f©H$ {Xdg na à~§YH$m| Zo AnZo g~go hmoZhma {dÚm{W©`m| H$mo ZH$X
nwañH$ma Ho$ gmW-gmW ñ_¥{V-
O¡gm ~Zdm`m J`m VWm BgH$m AmYma ABCD gm_Zo H$s Amoa go {XIVm Wm & {gëda
ßboqQ>J H$m IM© < 20 à{V dJ© go_r h¡ &

Cn`w©º$ Ho$ AmYma na, {ZåZ{b{IV àíZm| Ho$ CÎma Xr{OE :


(i) MVwWmªe ODCO H$m joÌ\ ? 1

(ii) AOB H$m joÌ\$b kmV H$s{OE & 1

(iii) (H$) ABCD N>m`m§{H$V ^mJ H$m {gëd ? 2


AWdm
(iii) (I) Mmn CD ? 2

30/2/1 JJJJ Page 18


35. (a) Sides AB and BC and median AD of a triangle ABC are
respectively proportional to sides PQ and QR and median PM of
~
OR
(b) Through the mid-point M of the side CD of a parallelogram ABCD,
the line BM is drawn intersecting AC in L and AD (produced) in E.
Prove that EL = 2BL.

SECTION E
This section comprises 3 case study based questions of 4 marks each.

Case Study 1
36. In an annual day function of a school, the organizers wanted to give a
cash prize along with a memento to their best students. Each memento is
made as shown in the figure and its base ABCD is shown from the front
side. The rate of silver plating is 20 per cm2.

Based on the above, answer the following questions :

(i) What is the area of the quadrant ODCO ? 1


(ii) 1

(iii) (a) What is the total cost of silver plating the shaded part
ABCD ? 2
OR

(iii) (b) What is the length of arc CD ? 2


30/2/1 JJJJ Page 19 P.T.O.
àH$aU AÜ``Z 2

37. EH$ H$m°\$s XþH$mZ _| H$m°\$s Xmo Vah Ho$ H$n _| namogr OmVr h¡ & EH$ H$n ~obZmH$ma h¡
{OgH$m ì`mg 7 cm VWm D±$MmB© 14 cm h¡ Am¡a Xÿgam H$n AY©Jmobr` AmH$ma H$m h¡
{OgH$m ì`mg 21 cm h¡ &

Cn`w©º$ Ho$ AmYma na, {ZåZ{b{IV àíZm| Ho$ CÎma Xr{OE :

(i) ~obZmH$ma H$n Ho$ AmYma H$m joÌ\$b kmV H$s{OE & 1

(ii) (H$) ? 2

AWdm

(ii) (I) ~obZmH$ma H$n H$s j_Vm kmV H$s{OE & 2

(iii) ~obZmH$ma H$n H$m dH«$ n¥ð>r` joÌ\ ? 1

30/2/1 JJJJ Page 20


Case Study 2

37. In a coffee shop, coffee is served in two types of cups. One is cylindrical in
shape with diameter 7 cm and height 14 cm and the other is
hemispherical with diameter 21 cm.

Based on the above, answer the following questions :

(i) Find the area of the base of the cylindrical cup. 1

(ii) (a) What is the capacity of the hemispherical cup ? 2

OR

(ii) (b) Find the capacity of the cylindrical cup. 2

(iii) What is the curved surface area of the cylindrical cup ? 1

30/2/1 JJJJ Page 21 P.T.O.


àH$aU AÜ``Z 3

38. H§$ß`yQ>a-AmYm[aV {ejU {H$gr ^r Eogr {ejU nÕ{V H$mo g§X{^©V H$aVm h¡ Omo gyMZm
àgmaU Ho$ {bE H§$ß`yQ>am| H$m Cn`moJ H$aVr h¡ & àmW{_H$ {dÚmb` ñVa na, _ëQ>r_r{S>`m
nmR> `moOZmAm| H$mo àX{e©V H$aZo Ho$ {bE H§$ß`yQ>a AZwà`moJm| H$m Cn`moJ {H$`m Om gH$Vm h¡ &
Ag_ Ho$ 1000 àmW{_H$ Am¡a _mÜ`{_H$ {dÚmb`m| na EH$ gd}jU {H$`m J`m Wm Am¡a
CZHo$ nmg {OVZo H§$ß`yQ>a Wo, CZHo$ AmYma na CÝh| dJuH¥$V {H$`m J`m Wm &

101 Am¡aBggo
H§$ß`yQ>am| H$s g§»`m 1 10 11 20 21 50 51 100
A{YH$
{dÚmb`m| H$s g§»`m 250 200 290 180 80
EH$ {dÚmb` H$m `mÑÀN>`m M`Z {H$`m J`m & Vmo :
(i) `mÑÀN>`m M`Z {H$E JE {dÚmb` _| 100 go A{YH$ H§$ß`yQ>a hmoZo H$s àm{`H$Vm
kmV H$s{OE & 1
(ii) (H$) `mÑÀN>`m M`Z {H$E JE {dÚmb` _| 50 `m 50 go H$_ H§$ß`yQ>a hmoZo H$s
àm{`H$Vm kmV H$s{OE & 2
AWdm
(ii) (I) `mÑÀN>`m M`Z {H$E JE {dÚmb` _| 20 go A{YH$ H§$ß`yQ>a Z hmoZo H$s
àm{`H$Vm kmV H$s{OE & 2
(iii) `mÑÀN>`m M`Z {H$E JE {dÚmb` _| 10 `m 10 go H$_ H§$ß`yQ>a hmoZo H$s àm{`H$Vm
kmV H$s{OE & 1

30/2/1 JJJJ Page 22


Case Study 3

38. Computer-based learning (CBL) refers to any teaching methodology that


makes use of computers for information transmission. At an elementary
school level, computer applications can be used to display multimedia
lesson plans. A survey was done on 1000 elementary and secondary
schools of Assam and they were classified by the number of computers
they had.

Number of 101 and


1 10 11 20 21 50 51 100
Computers more
Number of
250 200 290 180 80
Schools

One school is chosen at random. Then :


(i) Find the probability that the school chosen at random has more
than 100 computers. 1
(ii) (a) Find the probability that the school chosen at random has
50 or fewer computers. 2
OR
(ii) (b) Find the probability that the school chosen at random has
no more than 20 computers. 2
(iii) Find the probability that the school chosen at random has 10 or
less than 10 computers. 1

30/2/1 JJJJ Page 23 P.T.O.


Series WX1YZ/2 SET~2

Q.P. Code 30/2/2


Roll No. narjmWu àíZ-nÌ H$moS> >H$mo CÎma-nwpñVH$m Ho$
_wI-n¥ð >na Adí` {bIo§ &
Candidates must write the Q.P. Code on
the title page of the answer-book.

J{UV (_mZH$)
MATHEMATICS (STANDARD)
*
:3 : 80
Time allowed : 3 hours Maximum Marks : 80

ZmoQ> / NOTE :
(i) - 23
Please check that this question paper contains 23 printed pages.
(ii) - - -
-
Q.P. Code given on the right hand side of the question paper should be written on the title
page of the answer-book by the candidate.
(iii) - 38
Please check that this question paper contains 38 questions.
(iv) -

Please write down the serial number of the question in the answer-book before
attempting it.
(v) - 15 -
10.15 10.15 10.30 -
-
15 minute time has been allotted to read this question paper. The question paper will be
distributed at 10.15 a.m. From 10.15 a.m. to 10.30 a.m., the students will read the
question paper only and will not write any answer on the answer-book during this period.

30/2/2 JJJJ Page 1 P.T.O.


:
:
(i) 38
(ii)
(iii) 1 18 (MCQ) 19 20

(iv) 21 25 (VSA)

(v) 26 31 (SA)
(vi) 32 35 (LA)
(vii) 36 38

(viii) 2 2
2 3

22
(ix) =
7

(x)

IÊS> H$
(MCQ) 1

1. `{X ~hþnX x2 3kx + 4k H$m EH$ eyÝ`H$, Xÿgao eyÝ`H$ H$m XþJwZm h¡, Vmo k H$m _mZ
h¡ :
(a) 2 (b) 2
1 1
(c) (d)
2 2

2. x-Aj, {~ÝXþAm| ( 2, 3) Am¡a (6, 7)


{d^m{OV H$aVm h¡, dh h¡ :
(a) 1:3 (b) 3:7
(c) 7:3 (d) 1:2

30/2/2 JJJJ Page 2


General Instructions :
Read the following instructions very carefully and strictly follow them :
(i) This question paper contains 38 questions. All questions are compulsory.
(ii) This question paper is divided into five Sections A, B, C, D and E.
(iii) In Section A, Questions no. 1 to 18 are multiple choice questions (MCQs) and
questions number 19 and 20 are Assertion-Reason based questions of 1 mark
each.
(iv) In Section B, Questions no. 21 to 25 are very short answer (VSA) type
questions, carrying 2 marks each.
(v) In Section C, Questions no. 26 to 31 are short answer (SA) type questions,
carrying 3 marks each.
(vi) In Section D, Questions no. 32 to 35 are long answer (LA) type questions
carrying 5 marks each.
(vii) In Section E, Questions no. 36 to 38 are case study based questions carrying
4 marks each. Internal choice is provided in 2 marks questions in each
case-study.
(viii) There is no overall choice. However, an internal choice has been provided in
2 questions in Section B, 2 questions in Section C, 2 questions in Section D and
3 questions in Section E.
22
(ix) Draw neat diagrams wherever required. Take = wherever required, if not
7
stated.
(x) Use of calculators is not allowed.

SECTION A

This section comprises multiple choice questions (MCQs) of 1 mark each.

1. If one zero of the polynomial x2 3kx + 4k be twice the other, then the
value of k is :
(a) 2 (b) 2
1 1
(c) (d)
2 2

2. The ratio in which the x-axis divides the line segment joining the points
( 2, 3) and (6, 7) is :
(a) 1:3 (b) 3:7
(c) 7:3 (d) 1:2

30/2/2 JJJJ Page 3 P.T.O.


3. EH$ R>mog AY©-Jmobo, {OgH$m ì`mg h¡, H$m g§nyU© n¥ð>r` joÌ\ ?

(a) 3 d2 (b) 2 d2

1 3
(c) d2 (d) d2
2 4

4. {ÌÁ`m 14 cm dmbo EH$ d¥Îm Ho$ {ÌÁ`I§S>, {OgH$m Ho$ÝÐr` H$moU 90 h¡, H$s g§JV Mmn H$s
bå~m ?

(a) 22 cm (b) 44 cm
(c) 88 cm (d) 11 cm

5. `{X x = 0·3, g_rH$aU x2 0·9k = 0 H$m EH$ _yb h¡, Vmo k ~am~a h¡ :
(a) 1 (b) 10
(c) 0·1 (d) 100

6. Xr JB© AmH¥${V _|, AB PQ & `{X AB = 6 cm, PQ = 2 cm Am¡a OB = 3 cm h¡, Vmo


OP H$s bå~mB© hmoJr :

(a) 9 cm (b) 3 cm
(c) 4 cm (d) 1 cm

30/2/2 JJJJ Page 4


3.

(a) 3 d2 (b) 2 d2

1 3
(c) d2 (d) d2
2 4

4. What is the length of the arc of the sector of a circle with radius 14 cm
and of central angle 90 ?
(a) 22 cm (b) 44 cm
(c) 88 cm (d) 11 cm

5. If x = 0·3, is a root of the equation x2 0·9k = 0, then k is equal to :


(a) 1 (b) 10
(c) 0·1 (d) 100

6. In the given figure, AB PQ. If AB = 6 cm, PQ = 2 cm and OB = 3 cm,


then the length of OP is :

(a) 9 cm (b) 3 cm

(c) 4 cm (d) 1 cm

30/2/2 JJJJ Page 5 P.T.O.


7. `{X Am¡a àmH¥$V g§»`mE± h¢ Am¡a g§»`m H$m JwUO h¡, Vmo Am¡a H$m HCF
?
(a) pq (b ) p
(c) q (d) p+q

8. `{X ABC PQR _|, A = 32 Am¡a R = 65 h¡, Vmo B H$s _mn h¡ :


(a) 32 (b) 65
(c) 83 (d) 97

9. g_rH$aU `w½_ x = a Am¡a y = b Ûmam {Zê${nV aoImE± J«m\$s` ê$n _| :


(a) nañna g_m§Va hmoVr h¢
(b) {~ÝXþ (b, a) na à{VÀN>oXr hmoVr h¢
(c) g§nmVr hmoVr h¢
(d) {~ÝXþ (a, b) na à{VÀN>oXr hmoVr h¢
x y
10. aoIm + = 1 VWm {ZX}em§H$ Ajm| go ~Zo {Ì^wO H$m joÌ\$b h¡ :
a b
1
(a) ab (b) ab
2
1
(c) ab (d) 2ab
4

11. Xmo nmgm| H$mo EH$ ~ma CN>mbZo na, XmoZm| àmßV A§H$m| H$m JwUZ\$b 12 àmßV hmoZo H$s
àm{`H$Vm h¡ :
1 2
(a) (b)
9 9
4 5
(c) (d)
9 9

12. `{X Am¡a ~hþnX ax2 5x + c Ho$ eyÝ`H$ h¢ Am¡a + = = 10 h¡, Vmo :
1 5
(a) a = 5, c = (b) a = 1, c =
2 2
5 1
(c) a= , c=1 (d) a= , c=5
2 2

30/2/2 JJJJ Page 6


7.
q
(a) pq (b ) p
(c) q (d) p+q

8. If ABC A = 32 and R = 65 , then the measure of


B is :
(a) 32 (b) 65
(c) 83 (d) 97

9. The pair of equations x = a and y = b graphically represents lines which


are :
(a) parallel
(b) intersecting at (b, a)
(c) coincident
(d) intersecting at (a, b)
x y
10. The area of the triangle formed by the line = 1 with the coordinate
a b
axes is :
1
(a) ab (b) ab
2
1
(c) ab (d) 2ab
4
11. In a single throw of two dice, the probability of getting 12 as a product of
two numbers obtained is :
1 2
(a) (b)
9 9
4 5
(c) (d)
9 9

12. 2 5x + c and
+ = = 10, then :
1 5
(a) a = 5, c = (b) a = 1, c =
2 2
5 1
(c) a= , c=1 (d) a= , c=5
2 2
30/2/2 JJJJ Page 7 P.T.O.
13. EH$ W¡bo _| 100 nÎmo h¢ {OZ na 1 go 100 VH$ H$s g§»`mE± A§{H$V h¢ & Bg W¡bo _| go EH$
nÎmm `mÑÀN>`m {ZH$mbm OmVm h¡ & Bg nÎmo na EH$ nyU© KZ g§»`m A§{H$V hmoZo H$s àm{`H$Vm
?
1 3
(a) (b)
20 50
1 7
(c) (d)
25 100

14. Xr JB© AmH¥${V _|, DE BC & `{X AD = 2 BH$mB©, DB = AE = 3 BH$mB© Am¡a


EC = x BH$mB© h¡, Vmo x H$m _mZ hmoJm :

(a) 2 (b) 3
9
(c) 5 (d)
2

15. `{X g_rH$aU `w½_ 3x y+8=0 Am¡a 6x ry + 16 = 0 Ûmam {Zê${nV aoImE± g§nmVr
h¢, Vmo H$m _mZ hmoJm :
1 1
(a) (b)
2 2
(c) 2 (d) 2

16. 6 cm b§~r h¡ & Bg gwB© Ûmam 7:20 a.m. Am¡a 7:55 a.m. Ho$
~rM Omo H$moU a{MV hmoJm, dh h¡ :

35 35
(a) (b)
4 2

(c) 35 (d) 70
30/2/2 JJJJ Page 8
13. A bag contains 100 cards numbered 1 to 100. A card is drawn at random
from the bag. What is the probability that the number on the card is a
perfect cube ?
1 3
(a) (b)
20 50
1 7
(c) (d)
25 100

14. In the given figure, DE BC. If AD = 2 units, DB = AE = 3 units and


EC = x units, then the value of x is :

(a) 2 (b) 3
9
(c) 5 (d)
2

15. If the pair of equations 3x y + 8 = 0 and 6x ry + 16 = 0 represent

1 1
(a) (b)
2 2
(c) 2 (d) 2

16. The hour-hand of a clock is 6 cm long. The angle swept by it between


7:20 a.m. and 7:55 a.m. is :
35 35
(a) (b)
4 2

(c) 35 (d) 70

30/2/2 JJJJ Page 9 P.T.O.


17. Xr JB© AmH¥${V _|, EH$ d¥Îm Ho$ n[aJV EH$ MVw^w©O PQRS ~Zm h¡ & `hm± PA + CS ~am~a h¡ :

(a) QR Ho$ (b) PR Ho$


(c) PS Ho$ (d) PQ Ho$
18. `{X Am¡a , {ÛKmV ~hþnX p(x) = x2 ax b Ho$ eyÝ`H$ h¢, Vmo 2 + 2 H$m _mZ
hmoJm :
(a) a2 2b (b) a2 + 2b
(c) b2 2a (d) b2 + 2a

19 20 1
(A) (R)
(a), (b), (c) (d)
(a) A{^H$WZ (A) Am¡a VH©$ (R) XmoZm| ghr h¢ Am¡a VH©$ (R), A{^H$WZ (A) H$s ghr
ì¶m»¶m H$aVm h¡ &
(b) A{^H$WZ (A) Am¡a VH©$ (R) XmoZm| ghr h¢, naÝVw VH©$ (R), A{^H$WZ (A) H$s ghr
ì¶m»¶m H$aVm h¡ &
(c) A{^H$WZ (A) ghr h¡, naÝVw VH©$ (R) µJbV h¡ &
(d) A{^H$WZ (A) µJbV h¡, naÝVw VH©$ (R) ghr h¡ &
19. (A) : ~hþnX p(x) = x2 + 3x + 3 Ho$ Xmo dmñV{dH$ eyÝ`H$ h¢ &
(R) : EH$ {ÛKmV ~hþnX Ho$ A{YH$-go-A{YH$ Xmo dmñV{dH$ eyÝ`H$ hmo gH$Vo h¢ &
20. (A) : `{X H|$Ð O dmbo d¥Îm na ~mø q~Xþ P go Xmo ñne©-aoImE± PA Am¡a PB
ItMr JB© h¢, Vmo MVw^w©O AOBP MH«$s` hmoJm &
(R) : {H$gr ~mø q~Xþ go {H$gr d¥Îm na ItMr JB© Xmo ñne©-aoImAm| Ho$ ~rM H$m
H$moU ñne© q~XþAm| H$mo {_bmZo dmbo aoImI§S> Ûmam H|$Ð na A§V[aV H$moU
H$m g§nyaH$ hmoVm h¡ &
30/2/2 JJJJ Page 10
17. In the given figure, the quadrilateral PQRS circumscribes a circle. Here
PA + CS is equal to :

(a) QR (b) PR
(c) PS (d) PQ
18. If and are the zeroes of the quadratic polynomial p(x) = x2 ax b,
2
then the value of 2 + is :
(a) a2 2b (b) a2 + 2b
2 2
(c) b 2a (d) b + 2a
Questions number 19 and 20 are Assertion and Reason based questions carrying
1 mark each. Two statements are given, one labelled as Assertion (A) and the
other is labelled as Reason (R). Select the correct answer to these questions from
the codes (a), (b), (c) and (d) as given below.
(a) Both Assertion (A) and Reason (R) are true and Reason (R) is the
correct explanation of the Assertion (A).
(b) Both Assertion (A) and Reason (R) are true, but Reason (R) is not
the correct explanation of the Assertion (A).
(c) Assertion (A) is true, but Reason (R) is false.
(d) Assertion (A) is false, but Reason (R) is true.
19. Assertion (A) : The polynomial p(x) = x2 + 3x + 3 has two real zeroes.
Reason (R): A quadratic polynomial can have at most two real zeroes.
20. Assertion (A) : If PA and PB are tangents drawn from an external point
P to a circle with centre O, then the quadrilateral AOBP
is cyclic.
Reason (R): The angle between two tangents drawn from an external
point to a circle is supplementary to the angle subtended
by the line segment joining the points of contact at the
centre.
30/2/2 JJJJ Page 11 P.T.O.
IÊS> I
(VSA) 2

21. (H$) 3 JwZr b§~r


h¡ & gy`© H$m CÞVm§e kmV H$s{OE &
AWdm
(I) ^y{_ Ho$ EH$ q~Xþ go, Omo _rZma Ho$ nmX-q~Xþ go 30 m H$s Xÿar na h¡, _rZma Ho$
{eIa H$m CÞ`Z H$moU 30 h¡ & _rZma H$s D±$MmB© kmV H$s{OE &

22. Xr JB© AmH¥${V _|, d¥Îm H$m H|$Ð O h¡ & q~Xþ A go Bg d¥Îm na AB Am¡a AC ñne©-aoImE±
ItMr JB© h¢ & `{X BAC = 65 h¡, Vmo BOC H$s _mn kmV H$s{OE &

3
23. (H$) `{X 4 cot2 45 sec2 60 + sin2 60 + p = h¡, Vmo p H$m _mZ kmV H$s{OE &
4

AWdm
(I) `{X cos A + cos2 A = 1 h¡, Vmo sin2 A + sin4 A H$m _mZ kmV H$s{OE &

24. Xem©BE {H$ q~Xþ ( 2, 3), (8, 3) Am¡a (6, 7) EH$ g_H$moU {Ì^wO Ho$ erf© h¢ &
n
25. {gÕ H$s{OE {H$ 4 H$^r ^r eyÝ` (0) na g_mßV Zht hmoVm h¡, Ohm± n EH$ àmH¥$V g§»`m h¡ &

30/2/2 JJJJ Page 12


SECTION B

This section comprises very short answer (VSA) type questions of 2 marks each.

21. (a) The length of the shadow of a tower on the plane ground is 3
times the height of the tower. Find the angle of elevation of
the sun.
OR
(b) The angle of elevation of the top of a tower from a point on the
ground which is 30 m away from the foot of the tower, is 30 . Find
the height of the tower.

22. In the given figure, O is the centre of the circle. AB and AC are tangents
drawn to the circle from point A. If BAC = 65 , then find the measure of
BOC.

2 3
23. (a) If 4 cot 45 sec2 60 + sin2 60 + p = , then find the value of p.
4

OR
(b) If cos A + cos2 A = 1, then find the value of sin2 A + sin4 A.

24. Show that the points ( 2, 3), (8, 3) and (6, 7) are the vertices of a
right-angled triangle.

n
25. Prove that 4 can never end with digit 0, where n is a natural number.

30/2/2 JJJJ Page 13 P.T.O.


IÊS> J

(SA) 3

26. {gÕ H$s{OE :


sin 1 cos
+ = 2 cosec
1 cos sin

27. (H$) `{X a¡{IH$ g_rH$aU {ZH$m`


2x + 3y = 7 VWm 2ax + (a + b)y = 28
Ho$ An[a{_V ê$n go AZoH$ hb hm|, Vmo Am¡a Ho$ _mZ kmV H$s{OE &
AWdm
(I) `{X 217x + 131y = 913 Am¡a 131x + 217y = 827 hm|, Vmo x Am¡a y Ho$ _mZ
kmV H$aZo Ho$ {bE g_rH$aU hb H$s{OE &

28. (H$) g§»`mAm| 18180 Am¡a 7575 H$m A^mÁ` JwUZI§S>Z >{d{Y Ûmam LCM kmV
H$s{OE & BZ Xmo g§»`mAm| H$m HCF ^r kmV H$s{OE &
AWdm
(I) VrZ K§{Q>`m± 6, 12 Am¡a 18 {_ZQ>m| Ho$ A§Vamb na ~OVt h¢ & `{X `o VrZm| K§{Q>`m±
EH$ gmW 6 a.m. na ~Ot hm|, Vmo CgHo$ níMmV² do VrZm| EH$ gmW H$~ ~O|Jr ?

29. Xr JB© AmH¥${V _|, d¥Îm H$m H|$Ð O VWm QPR d¥Îm Ho$ q~Xþ P na ñne©-aoIm h¡ & {gÕ
H$s{OE {H$ QAP + APR = 90 .

30/2/2 JJJJ Page 14


SECTION C

This section comprises of short answer (SA) type questions of 3 marks each.

26. Prove that :


sin 1 cos
+ = 2 cosec
1 cos sin

27. (a) If the system of linear equations


2x + 3y = 7 and 2ax + (a + b)y = 28

OR

(b) If 217x + 131y = 913 and


131x + 217y = 827,
then solve the equations for the values of x and y.

28. (a) Find by prime factorisation the LCM of the numbers 18180 and
7575. Also, find the HCF of the two numbers.

OR

(b) Three bells ring at intervals of 6, 12 and 18 minutes. If all the


three bells rang at 6 a.m., when will they ring together again ?

29. In the given figure, O is the centre of the circle and QPR is a tangent to it
at P. Prove that QAP + APR = 90 .

30/2/2 JJJJ Page 15 P.T.O.


30. `{X q~Xþ Q(0, 1), q~XþAm| P(5, 3) Am¡a R(x, 6) go EH$g_mZ Xÿar na hmo, Vmo x Ho$ _mZ
kmV H$s{OE &
31. ar{V Zo AnZo ^mB© amo{ZV Ho$ {bE EH$ amIr V¡`ma H$s & `h amIr EH$ Am`V go ~Zr h¡ Omo
EH$ d¥Îm Ho$ A§VJ©V h¡ & Am`V H$s bå~mB© 8 cm 6 cm h¡ O¡gm {H$ AmH¥${V _|
{XIm`m J`m h¡ & N>m`m§{H$V ^mJ H$m joÌ\$b kmV H$s{OE &
( = 3·14 H$m à`moJ H$s{OE)

IÊS> K
(LA) 5

32.
3 {_ZQ> H$s 100 Ad{Y`m| _| {H$VZr h¢ Am¡a Bgo ZrMo Xr JB© Vm{bH$m _| gmam§{eV {H$`m
_mÜ`H$ kmV H$s{OE &
H$mam| H$s
0 10 10 20 20 30 30 40 40 50 50 60 60 70 70 80
g§»`m
~ma§~maVm
7 14 13 12 20 11 15 8
(Ad{Y`m±)
33. (H$) EH$ {Ì^wO ABC H$s ^wOmE± AB Am¡a BC VWm _mpÜ`H$m AD EH$ AÝ` {Ì^wO
PQR PQ Am¡a QR VWm _mpÜ`H$m PM Ho$ g_mZwnmVr h¢ &
Xem©BE {H$ ABC PQR h¡ &
AWdm
(I) g_m§Va MVw^w©O ABCD H$s ^wOm CD Ho$ _Ü`-q~Xþ M go EH$ aoIm BM ItMr JB©
Omo {dH$U© AC H$mo q~Xþ L AD H$mo q~Xþ E na H$mQ>Vr h¡ &
{gÕ H$s{OE {H$ EL = 2BL.
30/2/2 JJJJ Page 16
30. If Q(0, 1) is equidistant from P(5, 3) and R(x, 6), find the values of x.

31. Reeti prepares a Rakhi for her brother Ronit. The Rakhi consists of a
rectangle of length 8 cm and breadth 6 cm inscribed in a circle as shown
in the figure. Find the area of the shaded region. (Use = 3·14)

SECTION D

This section comprises long answer (LA) type questions of 5 marks each.
32. A student noted the number of cars passing through a spot on a road for
100 periods each of 3 minutes and summarised it in the table given
below. Find the mean and median of the following data.
Number of
0 10 10 20 20 30 30 40 40 50 50 60 60 70 70 80
cars
Frequency
7 14 13 12 20 11 15 8
(periods)

33. (a) Sides AB and BC and median AD of a triangle ABC are


respectively proportional to sides PQ and QR and median PM of
~
OR

(b) Through the mid-point M of the side CD of a parallelogram ABCD,


the line BM is drawn intersecting AC in L and AD (produced) in E.
Prove that EL = 2BL.

30/2/2 JJJJ Page 17 P.T.O.


34. x Ho$ {bE g_rH$aU hb H$s{OE :
1 + 4 + 7 + 10 + .... + x = 287

35. (H$) EH$ _rZma Ho$ nmX-q~Xþ go 30 m D±$Mr Xÿgar _rZma Ho$ {eIa H$m CÞ`Z H$moU 60
h¡ Am¡a Xÿgar _rZma Ho$ nmX-q~Xþ go nhbr _rZma Ho$ {eIa H$m CÞ`Z H$moU 30 h¡ &
XmoZm| _rZmam| Ho$ ~rM H$s Xÿar Am¡a nhbr _rZma H$s D±$MmB© ^r kmV H$s{OE &
AWdm
(I) 100 m
30 Am¡a 45 Ho$ AdZ_Z H$moU dmbr Xmo H$mam| H$mo XoIVm h¡ & XmoZm| H$mam| Ho$ ~rM
H$s Xÿar kmV H$s{OE & ( 3 = 1·73 H$m à`moJ H$s{OE)

IÊS> L>
3 4

àH$aU AÜ``Z 1

36. H§$ß`yQ>a-AmYm[aV {ejU {H$gr ^r Eogr {ejU nÕ{V H$mo g§X{^©V H$aVm h¡ Omo gyMZm
àgmaU Ho$ {bE H§$ß`yQ>am| H$m Cn`moJ H$aVr h¡ & àmW{_H$ {dÚmb` ñVa na, _ëQ>r_r{S>`m
nmR> `moOZmAm| H$mo àX{e©V H$aZo Ho$ {bE H§$ß`yQ>a AZwà`moJm| H$m Cn`moJ {H$`m Om gH$Vm h¡ &
Ag_ Ho$ 1000 àmW{_H$ Am¡a _mÜ`{_H$ {dÚmb`m| na EH$ gd}jU {H$`m J`m Wm Am¡a
CZHo$ nmg {OVZo H§$ß`yQ>a Wo, CZHo$ AmYma na CÝh| dJuH¥$V {H$`m J`m Wm &

30/2/2 JJJJ Page 18


34. Solve the equation for x :
1 + 4 + 7 + 10 + .... + x = 287
35. (a) The angle of elevation of the top of a tower 30 m high from the foot
of another tower in the same plane is 60 and the angle of
elevation of the top of the second tower from the foot of the first
tower is 30 . Find the distance between the two towers and also
the height of the other tower.
OR
(b) From the top of a tower 100 m high, a man observes two cars on
the opposite sides of the tower with angles of depression 30 and
45 respectively. Find the distance between the two cars.
(Use 3 = 1·73)

SECTION E
This section comprises 3 case study based questions of 4 marks each.

Case Study 1

36. Computer-based learning (CBL) refers to any teaching methodology that


makes use of computers for information transmission. At an elementary
school level, computer applications can be used to display multimedia
lesson plans. A survey was done on 1000 elementary and secondary
schools of Assam and they were classified by the number of computers
they had.

30/2/2 JJJJ Page 19 P.T.O.


101 Am¡a
Bggo
H§$ß`yQ>am| H$s g§»`m 1 10 11 20 21 50 51 100
A{YH$
{dÚmb`m| H$s g§»`m 250 200 290 180 80
EH$ {dÚmb` H$m `mÑÀN>`m M`Z {H$`m J`m & Vmo :
(i) `mÑÀN>`m M`Z {H$E JE {dÚmb` _| 100 go A{YH$ H§$ß`yQ>a hmoZo H$s àm{`H$Vm
kmV H$s{OE & 1
(ii) (H$) `mÑÀN>`m M`Z {H$E JE {dÚmb` _| 50 `m 50 go H$_ H§$ß`yQ>a hmoZo H$s
àm{`H$Vm kmV H$s{OE & 2
AWdm
(ii) (I) `mÑÀN>`m M`Z {H$E JE {dÚmb` _| 20 go A{YH$ H§$ß`yQ>a Z hmoZo H$s
àm{`H$Vm kmV H$s{OE & 2
(iii) `mÑÀN>`m M`Z {H$E JE {dÚmb` _| 10 `m 10 go H$_ H§$ß`yQ>a hmoZo H$s àm{`H$Vm
kmV H$s{OE & 1
àH$aU AÜ``Z 2
37. EH$ {dÚmb` Ho$ dm{f©H$ {Xdg na à~§YH$m| Zo AnZo g~go hmoZhma {dÚm{W©`m| H$mo ZH$X
nwañH$ma Ho$ gmW-gmW ñ_¥{V-
O¡gm ~Zdm`m J`m VWm BgH$m AmYma ABCD gm_Zo H$s Amoa go {XIVm Wm & {gëda
ßboqQ>J H$m IM© < 20 à{V dJ© go_r h¡ &

Cn`w©º$ Ho$ AmYma na, {ZåZ{b{IV àíZm| Ho$ CÎma Xr{OE :


(i) MVwWmªe ODCO H$m joÌ\ ? 1
(ii) AOB H$m joÌ\$b kmV H$s{OE & 1
(iii) (H$) ABCD N>m`m§{H$V ^mJ H$m {gëd ? 2
AWdm
(iii) (I) Mmn CD ? 2
30/2/2 JJJJ Page 20
Number of 101 and
1 10 11 20 21 50 51 100
Computers more
Number of
250 200 290 180 80
Schools
One school is chosen at random. Then :
(i) Find the probability that the school chosen at random has more
than 100 computers. 1
(ii) (a) Find the probability that the school chosen at random has
50 or fewer computers. 2
OR
(ii) (b) Find the probability that the school chosen at random has
no more than 20 computers. 2
(iii) Find the probability that the school chosen at random has 10 or
less than 10 computers. 1

Case Study 2
37. In an annual day function of a school, the organizers wanted to give a
cash prize along with a memento to their best students. Each memento is
made as shown in the figure and its base ABCD is shown from the front
side. The rate of silver plating is 20 per cm2.

Based on the above, answer the following questions :


(i) What is the area of the quadrant ODCO ? 1
(ii) 1
(iii) (a) What is the total cost of silver plating the shaded part
ABCD ? 2
OR
(iii) (b) What is the length of arc CD ? 2
30/2/2 JJJJ Page 21 P.T.O.
àH$aU AÜ``Z 3

38. EH$ H$m°\$s XþH$mZ _| H$m°\$s Xmo Vah Ho$ H$n _| namogr OmVr h¡ & EH$ H$n ~obZmH$ma h¡
{OgH$m ì`mg 7 cm VWm D±$MmB© 14 cm h¡ Am¡a Xÿgam H$n AY©Jmobr` AmH$ma H$m h¡
{OgH$m ì`mg 21 cm h¡ &

Cn`w©º$ Ho$ AmYma na, {ZåZ{b{IV àíZm| Ho$ CÎma Xr{OE :

(i) ~obZmH$ma H$n Ho$ AmYma H$m joÌ\$b kmV H$s{OE & 1

(ii) (H$) ? 2

AWdm
(ii) (I) ~obZmH$ma H$n H$s j_Vm kmV H$s{OE & 2

(iii) ~obZmH$ma H$n H$m dH«$ n¥ð>r` joÌ\ ? 1

30/2/2 JJJJ Page 22


Case Study 3

38. In a coffee shop, coffee is served in two types of cups. One is cylindrical in
shape with diameter 7 cm and height 14 cm and the other is
hemispherical with diameter 21 cm.

Based on the above, answer the following questions :

(i) Find the area of the base of the cylindrical cup. 1

(ii) (a) What is the capacity of the hemispherical cup ? 2

OR

(ii) (b) Find the capacity of the cylindrical cup. 2

(iii) What is the curved surface area of the cylindrical cup ? 1

30/2/2 JJJJ Page 23 P.T.O.


Series WX1YZ/2 SET~3

Q.P. Code 30/2/3


Roll No. narjmWu àíZ-nÌ H$moS> >H$mo CÎma-nwpñVH$m Ho$
_wI-n¥ð >na Adí` {bIo§ &
Candidates must write the Q.P. Code on
the title page of the answer-book.

J{UV (_mZH$)
MATHEMATICS (STANDARD)
*
:3 : 80
Time allowed : 3 hours Maximum Marks : 80

ZmoQ> / NOTE :
(i) - 23
Please check that this question paper contains 23 printed pages.
(ii) - - -
-
Q.P. Code given on the right hand side of the question paper should be written on the title
page of the answer-book by the candidate.
(iii) - 38
Please check that this question paper contains 38 questions.
(iv) -

Please write down the serial number of the question in the answer-book before
attempting it.
(v) - 15 -
10.15 10.15 10.30 -
-
15 minute time has been allotted to read this question paper. The question paper will be
distributed at 10.15 a.m. From 10.15 a.m. to 10.30 a.m., the students will read the
question paper only and will not write any answer on the answer-book during this period.

30/2/3 JJJJ Page 1 P.T.O.


:
:
(i) 38
(ii)
(iii) 1 18 (MCQ) 19 20

(iv) 21 25 (VSA)

(v) 26 31 (SA)
(vi) 32 35 (LA)
(vii) 36 38

(viii) 2 2
2 3

22
(ix) =
7

(x)

IÊS> H$
(MCQ) 1
1. `{X EH$ àmH¥$V g§»`m h¡, Vmo {ZåZ{b{IV _| go H$m¡Z-gr g§»`m eyÝ` (0) na g_mßV
hmoVr h¡ ?
n n
(a) (3 2) (b) (2 5)
n n
(c) (6 2) (d) (5 3)
2. EH$ bm°Q>ar _|, 5 nwañH$ma Am¡a 20 [aº$ ñWmZ h¢ & nwañH$ma {_bZo H$s àm{`H$Vm h¡ :
1 1
(a) (b)
4 20
1 1
(c) (d)
25 5
3. `{X 2x + 3y = 15 Am¡a 3x + 2y = 25 h¡, Vmo x y H$m _mZ h¡ :
(a) 10 (b) 8
(c) 10 (d) 8
30/2/3 JJJJ Page 2
General Instructions :
Read the following instructions very carefully and strictly follow them :
(i) This question paper contains 38 questions. All questions are compulsory.
(ii) This question paper is divided into five Sections A, B, C, D and E.
(iii) In Section A, Questions no. 1 to 18 are multiple choice questions (MCQs) and
questions number 19 and 20 are Assertion-Reason based questions of 1 mark
each.
(iv) In Section B, Questions no. 21 to 25 are very short answer (VSA) type
questions, carrying 2 marks each.
(v) In Section C, Questions no. 26 to 31 are short answer (SA) type questions,
carrying 3 marks each.
(vi) In Section D, Questions no. 32 to 35 are long answer (LA) type questions
carrying 5 marks each.
(vii) In Section E, Questions no. 36 to 38 are case study based questions carrying
4 marks each. Internal choice is provided in 2 marks questions in each
case-study.
(viii) There is no overall choice. However, an internal choice has been provided in
2 questions in Section B, 2 questions in Section C, 2 questions in Section D and
3 questions in Section E.
22
(ix) Draw neat diagrams wherever required. Take = wherever required, if not
7
stated.
(x) Use of calculators is not allowed.
SECTION A
This section comprises multiple choice questions (MCQs) of 1 mark each.
1.
zero ?
n n
(a) (3 2) (b) (2 5)
n n
(c) (6 2) (d) (5 3)
2. In a lottery, there are 5 prizes and 20 blanks. The probability of getting a
prize is :
1 1
(a) (b)
4 20
1 1
(c) (d)
25 5

3. If 2x + 3y = 15 and 3x + 2y = 25, then the value of x y is :


(a) 10 (b) 8
(c) 10 (d) 8
30/2/3 JJJJ Page 3 P.T.O.
4. Xr JB© AmH¥${V _|, A= C, AB = 6 cm, AP = 12 cm Am¡a CP = 4 cm h¡ & Vmo
CD H$s bå~mB© h¡ :

(a) 2 cm (b ) 6 cm
(c) 8 cm (d) 18 cm

5. ~hþnX 2 x2 17 Ho$ eyÝ`H$m| H$m `moJ\$b h¡ :


17 2 17 2
(a) (b)
2 2
(c) 0 (d) 1

6. `{X EH$ e§Hw$ Ho$ AmYma H$m joÌ\$b 51 cm2 Am¡a BgH$m Am`VZ 85 cm3 h¡, Vmo Bg e§Hw$
H$s D$Üdm©Ya D±$MmB© hmoJr :
5 5
(a) cm (b) cm
6 3
5
(c) cm (d) 5 cm
2

7. {ÌÁ`m 14 cm dmbo EH$ d¥Îm Ho$ {ÌÁ`I§S>, {OgH$m Ho$ÝÐr` H$moU 90 h¡, H$s g§JV Mmn H$s
?

(a) 22 cm (b) 44 cm
(c) 88 cm (d) 11 cm
30/2/3 JJJJ Page 4
4. In the given figure, A= C, AB = 6 cm, AP = 12 cm, CP = 4 cm. Then
length of CD is :

(a) 2 cm (b ) 6 cm
(c) 8 cm (d) 18 cm

5. The sum of zeroes of the polynomial 2 x2 17 are given as :


17 2 17 2
(a) (b)
2 2
(c) 0 (d) 1

6. If the area of the base of a cone is 51 cm2 and its volume is 85 cm3, then
the vertical height of the cone is given as :
5 5
(a) cm (b) cm
6 3
5
(c) cm (d) 5 cm
2

7. What is the length of the arc of the sector of a circle with radius 14 cm
and of central angle 90 ?
(a) 22 cm (b) 44 cm
(c) 88 cm (d) 11 cm

30/2/3 JJJJ Page 5 P.T.O.


8. EH$ Am`V ABCD {OgHo$ VrZ erf© B(0, 0), C(3, 0) Am¡a D(0, 4) h¢, CgHo$ erf© A Ho$
{ZX}em§H$ hm|Jo :
(a) (4, 0) (b) (0, 3)
(c) (3, 4) (d) (4, 3)
x y
9. aoIm + = 1 VWm {ZX}em§H$ Ajm| go ~Zo {Ì^wO H$m joÌ\$b h¡ :
a b
1
(a) ab (b) ab
2
1
(c) ab (d) 2ab
4
10. 6 cm b§~r h¡ & Bg gwB© Ûmam 7:20 a.m. Am¡a 7:55 a.m. Ho$
~rM Omo H$moU a{MV hmoJm, dh h¡ :
35 35
(a) (b)
4 2
(c) 35 (d) 70
11. ~hþnX p(x) = x2 + 4x + 3 Ho$ eyÝ`H$ h¢ :
(a) 1, 3 (b) 1, 3
(c) 1, 3 (d) 1, 3
12. Xr JB© AmH¥${V _|, AB PQ & `{X AB = 6 cm, PQ = 2 cm Am¡a OB = 3 cm h¡, Vmo
OP H$s bå~mB© hmoJr :

(a) 9 cm (b) 3 cm
(c) 4 cm (d) 1 cm
30/2/3 JJJJ Page 6
8. The coordinates of the vertex A of a rectangle ABCD whose three vertices
are given as B(0, 0), C(3, 0) and D(0, 4) are :
(a) (4, 0) (b) (0, 3)
(c) (3, 4) (d) (4, 3)
x y
9. The area of the triangle formed by the line = 1 with the coordinate
a b
axes is :
1
(a) ab (b) ab
2
1
(c) ab (d) 2ab
4
10. The hour-hand of a clock is 6 cm long. The angle swept by it between
7:20 a.m. and 7:55 a.m. is :
35 35
(a) (b)
4 2
(c) 35 (d) 70
11. The zeroes of the polynomial p(x) = x2 + 4x + 3 are given by :
(a) 1, 3 (b) 1, 3
(c) 1, 3 (d) 1, 3
12. In the given figure, AB PQ. If AB = 6 cm, PQ = 2 cm and OB = 3 cm,
then the length of OP is :

(a) 9 cm (b) 3 cm
(c) 4 cm (d) 1 cm
30/2/3 JJJJ Page 7 P.T.O.
13. Xr JB© AmH¥${V _|, EH$ d¥Îm Ho$ n[aJV EH$ MVw^w©O PQRS ~Zm h¡ & `hm± PA + CS ~am~a
h¡ :

(a) QR Ho$ (b) PR Ho$


(c) PS Ho$ (d) PQ Ho$

14. `{X ~hþnX 6x2 + 37x (k 2) H$m EH$ eyÝ`H$, Xÿgao eyÝ`H$ H$m ì`wËH«$_ hmo, Vmo k H$m
`m hmoJm ?
(a) 4 (b) 6

(c) 6 (d) 4

15. -go-A{YH$ EH$ nQ> àmßV hmoZo H$s


?

3 4
(a) (b)
8 8

5 7
(c) (d)
8 8

16. `{X g_rH$aU `w½_ 3x y+8=0 Am¡a 6x ry + 16 = 0 Ûmam {Zê${nV aoImE± g§nmVr
h¢, Vmo H$m _mZ hmoJm :
1 1
(a) (b)
2 2
(c) 2 (d) 2

30/2/3 JJJJ Page 8


13. In the given figure, the quadrilateral PQRS circumscribes a circle. Here
PA + CS is equal to :

(a) QR (b) PR
(c) PS (d) PQ

14. If one zero of the polynomial 6x2 + 37x (k 2) is reciprocal of the other,
then what is the value of k ?

(a) 4 (b) 6

(c) 6 (d) 4

15. If three coins are tossed simultaneously, what is the probability of getting
at most one tail ?

3 4
(a) (b)
8 8

5 7
(c) (d)
8 8

16. If the pair of equations 3x y + 8 = 0 and 6x ry + 16 = 0 represent


coincident lines, the
1 1
(a) (b)
2 2
(c) 2 (d) 2

30/2/3 JJJJ Page 9 P.T.O.


17. `{X ABC PQR _|, A = 32 Am¡a R = 65 h¡, Vmo B H$s _mn h¡ :
(a) 32 (b) 65
(c) 83 (d) 97
18. {ZåZ{b{IV _| go {H$g {ÛKmV g_rH$aU Ho$ _ybm| H$m `moJ\$b 4 h¡ ?
(a) 2x2 4x + 8 = 0 (b) x2 + 4x + 4 = 0
4
(c) 2 x2 x+1=0 (d) 4x2 4x + 4 = 0
2

19 20 1
(A) (R)
(a), (b), (c) (d)
(a) A{^H$WZ (A) Am¡a VH©$ (R) XmoZm| ghr h¢ Am¡a VH©$ (R), A{^H$WZ (A) H$s ghr
ì¶m»¶m H$aVm h¡ &
(b) A{^H$WZ (A) Am¡a VH©$ (R) XmoZm| ghr h¢, naÝVw VH©$ (R), A{^H$WZ (A) H$s ghr
ì¶m»¶m H$aVm h¡ &
(c) A{^H$WZ (A) ghr h¡, naÝVw VH©$ (R) µJbV h¡ &
(d) A{^H$WZ (A) µJbV h¡, naÝVw VH©$ (R) ghr h¡ &
19. (A) : d¥Îm Ho$ {H$gr q~Xþ na ñne©-aoIm ñne© q~Xþ go OmZo dmbr {ÌÁ`m na bå~
hmoVr h¡ &
(R) : ~mø q~Xþ go d¥Îm na ItMr JB© ñne©-aoImAm| H$s bå~mB`m± ~am~a hmoVr h¢ &
20. (A) : ~hþnX p(x) = x2 + 3x + 3 Ho$ Xmo dmñV{dH$ eyÝ`H$ h¢ &
(R) : EH$ {ÛKmV ~hþnX Ho$ A{YH$-go-A{YH$ Xmo dmñV{dH$ eyÝ`H$ hmo gH$Vo h¢ &
IÊS> I
(VSA) 2

21. (H$) 3 JwZr b§~r


h¡ & gy`© H$m CÞVm§e kmV H$s{OE &
AWdm
(I) ^y{_ Ho$ EH$ q~Xþ go, Omo _rZma Ho$ nmX-q~Xþ go 30 m H$s Xÿar na h¡, _rZma Ho$
{eIa H$m CÞ`Z H$moU 30 h¡ & _rZma H$s D±$MmB© kmV H$s{OE &

30/2/3 JJJJ Page 10


17. If ABC A = 32 and R = 65 , then the measure of
B is :
(a) 32 (b) 65
(c) 83 (d) 97
18. Which of the following quadratic equations has sum of its roots as 4 ?
(a) 2x2 4x + 8 = 0 (b) x2 + 4x + 4 = 0
4
(c) 2 x2 x+1=0 (d) 4x2 4x + 4 = 0
2
Questions number 19 and 20 are Assertion and Reason based questions carrying
1 mark each. Two statements are given, one labelled as Assertion (A) and the
other is labelled as Reason (R). Select the correct answer to these questions from
the codes (a), (b), (c) and (d) as given below.
(a) Both Assertion (A) and Reason (R) are true and Reason (R) is the
correct explanation of the Assertion (A).
(b) Both Assertion (A) and Reason (R) are true, but Reason (R) is not
the correct explanation of the Assertion (A).
(c) Assertion (A) is true, but Reason (R) is false.
(d) Assertion (A) is false, but Reason (R) is true.
19. Assertion (A) : A tangent to a circle is perpendicular to the radius
through the point of contact.
Reason (R) : The lengths of tangents drawn from an external point to a
circle are equal.

20. Assertion (A) : The polynomial p(x) = x2 + 3x + 3 has two real zeroes.
Reason (R): A quadratic polynomial can have at most two real zeroes.

SECTION B
This section comprises very short answer (VSA) type questions of 2 marks each.
21. (a) The length of the shadow of a tower on the plane ground is 3
times the height of the tower. Find the angle of elevation of the
sun.
OR
(b) The angle of elevation of the top of a tower from a point on the
ground which is 30 m away from the foot of the tower, is 30 . Find
the height of the tower.

30/2/3 JJJJ Page 11 P.T.O.


22. Xem©BE {H$ q~Xþ ( 2, 3), (8, 3) Am¡a (6, 7) EH$ g_H$moU {Ì^wO Ho$ erf© h¢ &

23. Xr JB© AmH¥${V _|, d¥Îm H$m H|$Ð O h¡ & q~Xþ A go Bg d¥Îm na AB Am¡a AC ñne©-aoImE±
ItMr JB© h¢ & `{X BAC = 65 h¡, Vmo BOC H$s _mn kmV H$s{OE &

3
24. (H$) `{X 4 cot2 45 sec2 60 + sin2 60 + p = h¡, Vmo p H$m _mZ kmV H$s{OE &
4
AWdm
(I) `{X cos A + cos2 A = 1 h¡, Vmo sin2 A + sin4 A H$m _mZ kmV H$s{OE &

25. {gÕ H$s{OE {H$ 6 7 EH$ An[a_o` g§»`m h¡, {X`m J`m h¡ {H$ 7 EH$ An[a_o`
g§»`m h¡ &

IÊS> J

(SA) 3

26. {gÕ H$s{OE {H$ :

cos 2 sin3
+ = 1 + sin cos
1 tan sin cos

27. 14 cm {ÌÁ`m dmbo EH$ d¥Îm H$s H$moB© Ordm Ho$ÝÐ na 60 H$m H$moU A§V[aV H$aVr h¡ & g§JV
bKw d¥ÎmI§S> H$m joÌ\$b kmV H$s{OE & ( = 3·14 Am¡a 3 = 1·73 H$m à`moJ H$s{OE)
30/2/3 JJJJ Page 12
22. Show that the points ( 2, 3), (8, 3) and (6, 7) are the vertices of a
right-angled triangle.

23. In the given figure, O is the centre of the circle. AB and AC are tangents
drawn to the circle from point A. If BAC = 65 , then find the measure of
BOC.

2 3
24. (a) If 4 cot 45 sec2 60 + sin2 60 + p = , then find the value of p.
4
OR
(b) If cos A + cos2 A = 1, then find the value of sin2 A + sin4 A.

25. Prove that 6 7 is irrational number, given that 7 is an irrational


number.

SECTION C

This section comprises of short answer (SA) type questions of 3 marks each.

26. Prove that :


cos 2 sin3
+ = 1 + sin cos
1 tan sin cos

27. A chord of a circle of radius 14 cm subtends an angle of 60 at the centre.


Find the area of the corresponding minor segment of the circle.
(Use = 3·14 and 3 = 1·73)

30/2/3 JJJJ Page 13 P.T.O.


28. (H$) g§»`mAm| 18180 Am¡a 7575 H$m A^mÁ` JwUZI§S>Z {d{Y Ûmam LCM kmV
H$s{OE & BZ Xmo g§»`mAm| H$m HCF ^r kmV H$s{OE &

AWdm

(I) VrZ K§{Q>`m± 6, 12 Am¡a 18 {_ZQ>m| Ho$ A§Vamb na ~OVt h¢ & `{X `o VrZm| K§{Q>`m±
EH$ gmW 6 a.m. na ~Ot hm|, Vmo CgHo$ níMmV² do VrZm| EH$ gmW H$~ ~O|Jr ?

29. Xr JB© AmH¥${V _|, d¥Îm H$m H|$Ð O VWm QPR d¥Îm Ho$ q~Xþ P na ñne©-aoIm h¡ & {gÕ
H$s{OE {H$ QAP + APR = 90 .

30. `{X q~Xþ Q(0, 1), q~XþAm| P(5, 3) Am¡a R(x, 6) go EH$g_mZ Xÿar na hmo, Vmo x Ho$ _mZ
kmV H$s{OE &

31. (H$) `{X a¡{IH$ g_rH$aU {ZH$m`


2x + 3y = 7 VWm 2ax + (a + b)y = 28

Ho$ An[a{_V ê$n go AZoH$ hb hm|, Vmo Am¡a Ho$ _mZ kmV H$s{OE &
AWdm

(I) `{X 217x + 131y = 913 Am¡a 131x + 217y = 827 hm|, Vmo x Am¡a y Ho$ _mZ
kmV H$aZo Ho$ {bE g_rH$aU hb H$s{OE &

30/2/3 JJJJ Page 14


28. (a) Find by prime factorisation the LCM of the numbers 18180 and
7575. Also, find the HCF of the two numbers.

OR

(b) Three bells ring at intervals of 6, 12 and 18 minutes. If all the


three bells rang at 6 a.m., when will they ring together again ?

29. In the given figure, O is the centre of the circle and QPR is a tangent to it
at P. Prove that QAP + APR = 90 .

30. If Q(0, 1) is equidistant from P(5, 3) and R(x, 6), find the values of x.

31. (a) If the system of linear equations


2x + 3y = 7 and 2ax + (a + b)y = 28

OR

(b) If 217x + 131y = 913 and

131x + 217y = 827,

then solve the equations for the values of x and y.

30/2/3 JJJJ Page 15 P.T.O.


IÊS> K
(LA) 5

32. {ZåZ ~ma§~maVm ~§Q>Z H$m ~hþbH$ 55 h¡ & bwßV ~ma§~maVmE± Am¡a kmV H$s{OE &
dJ© AÝVamb 0 15 15 30 30 45 45 60 60 75 75 90 `moJ
~ma§~maVm 6 7 a 15 10 b 51

33. àoaUm nhbo _hrZo _| < 32 ~MmVr h¡, Xÿgao _hrZo _| < 36 Am¡a Vrgao _hrZo _| < 40 &
`{X dh Bgr Vah go à{V _mh ~MV H$ao, Vmo dh {H$VZo _hrZm| _| < 2,000 H$s ~MV H$a
boJr ?

34. (H$) EH$ {Ì^wO ABC H$s ^wOmE± ABAm¡a BC VWm _mpÜ`H$m AD EH$ AÝ` {Ì^wO
PQR PQ Am¡a QR VWm _mpÜ`H$m PM Ho$ g_mZwnmVr h¢ &
Xem©BE {H$ ABC PQR h¡ &

AWdm

(I) g_m§Va MVw^w©O ABCD H$s ^wOm CD Ho$ _Ü`-q~Xþ go EH$ aoIm BM ItMr JB©
M
Omo {dH$U© AC H$mo q~Xþ L AD H$mo q~Xþ E na H$mQ>Vr h¡ &
{gÕ H$s{OE {H$ EL = 2BL.

35. (H$) g_wÐ-Vb go 75 m D±$Mr bmBQ>-hmD$g Ho$ {eIa go XoIZo na Xmo g_wÐr OhmOm| Ho$
AdZ_Z H$moU 30 Am¡a 60 h¢ & `{X bmBQ>-hmD$g Ho$ EH$ hr Amoa EH$
OhmO Xÿgao OhmO Ho$ R>rH$ nrN>o hmo, Vmo Xmo OhmOm| Ho$ ~rM H$s Xÿar kmV H$s{OE &
( 3 = 1·73 H$m à`moJ H$s{OE)
AWdm

(I) ^y{_ Ho$ EH$ q~Xþ go EH$ 30 m D±$Mo ^dZ Ho$ {eIa na bJr EH$ g§Mma _rZma Ho$
Vb Am¡a {eIa Ho$ CÞ`Z H$moU H«$_e: 30 Am¡a 60 h¢ & g§Mma _rZma H$s D±$MmB©
kmV H$s{OE & ( 3 = 1·73 H$m à`moJ H$s{OE)

30/2/3 JJJJ Page 16


SECTION D

This section comprises long answer (LA) type questions of 5 marks each.

32. The mode of the following frequency distribution is 55. Find the missing

Class Interval 0 15 15 30 30 45 45 60 60 75 75 90 Total


Frequency 6 7 a 15 10 b 51

33. Prerna saves < 32 during the first month, < 36 in the second month and
< 40 in the third month. If she continues to save in this manner, in how
many months will she save < 2,000 ?

34. (a) Sides AB and BC and median AD of a triangle ABC are


respectively proportional to sides PQ and QR and median PM of
~
OR
(b) Through the mid-point M of the side CD of a parallelogram ABCD,
the line BM is drawn intersecting AC in L and AD (produced) in E.
Prove that EL = 2BL.

35. (a) As observed from the top of a 75 m high lighthouse from the
sea-level, the angles of depression of two ships are 30 and 60 . If
one ship is exactly behind the other on the same side of the
lighthouse, find the distance between the two ships.
(Use 3 = 1·73)

OR
(b) From a point on the ground, the angle of elevation of the bottom
and top of a transmission tower fixed at the top of 30 m high
building are 30 and 60 , respectively. Find the height of the
transmission tower. (Use 3 = 1·73)

30/2/3 JJJJ Page 17 P.T.O.


IÊS> L>
3 4

àH$aU AÜ``Z 1

36. EH$ H$m°\$s XþH$mZ _| H$m°\$s Xmo Vah Ho$ H$n _| namogr OmVr h¡ & EH$ H$n ~obZmH$ma h¡
{OgH$m ì`mg 7 cm VWm D±$MmB© 14 cm h¡ Am¡a Xÿgam H$n AY©Jmobr` AmH$ma H$m h¡
{OgH$m ì`mg 21 cm h¡ &

Cn`w©º$ Ho$ AmYma na, {ZåZ{b{IV àíZm| Ho$ CÎma Xr{OE :

(i) ~obZmH$ma H$n Ho$ AmYma H$m joÌ\$b kmV H$s{OE & 1

(ii) (H$) ? 2

AWdm

(ii) (I) ~obZmH$ma H$n H$s j_Vm kmV H$s{OE & 2

(iii) ~obZmH$ma H$n H$m dH«$ n¥ð>r` joÌ\ ? 1

30/2/3 JJJJ Page 18


SECTION E
This section comprises 3 case study based questions of 4 marks each.

Case Study 1

36. In a coffee shop, coffee is served in two types of cups. One is cylindrical in
shape with diameter 7 cm and height 14 cm and the other is
hemispherical with diameter 21 cm.

Based on the above, answer the following questions :

(i) Find the area of the base of the cylindrical cup. 1

(ii) (a) What is the capacity of the hemispherical cup ? 2

OR

(ii) (b) Find the capacity of the cylindrical cup. 2

(iii) What is the curved surface area of the cylindrical cup ? 1

30/2/3 JJJJ Page 19 P.T.O.


àH$aU AÜ``Z 2

37. H§$ß`yQ>a-AmYm[aV {ejU {H$gr ^r Eogr {ejU nÕ{V H$mo g§X{^©V H$aVm h¡ Omo gyMZm
àgmaU Ho$ {bE H§$ß`yQ>am| H$m Cn`moJ H$aVr h¡ & àmW{_H$ {dÚmb` ñVa na, _ëQ>r_r{S>`m
nmR> `moOZmAm| H$mo àX{e©V H$aZo Ho$ {bE H§$ß`yQ>a AZwà`moJm| H$m Cn`moJ {H$`m Om gH$Vm h¡ &
Ag_ Ho$ 1000 àmW{_H$ Am¡a _mÜ`{_H$ {dÚmb`m| na EH$ gd}jU {H$`m J`m Wm Am¡a
CZHo$ nmg {OVZo H§$ß`yQ>a Wo, CZHo$ AmYma na CÝh| dJuH¥$V {H$`m J`m Wm &

101 Am¡aBggo
H§$ß`yQ>am| H$s g§»`m 1 10 11 20 21 50 51 100
A{YH$
{dÚmb`m| H$s g§»`m 250 200 290 180 80
EH$ {dÚmb` H$m `mÑÀN>`m M`Z {H$`m J`m & Vmo :
(i) `mÑÀN>`m M`Z {H$E JE {dÚmb` _| 100 go A{YH$ H§$ß`yQ>a hmoZo H$s àm{`H$Vm
kmV H$s{OE & 1
(ii) (H$) `mÑÀN>`m M`Z {H$E JE {dÚmb` _| 50 `m 50 go H$_ H§$ß`yQ>a hmoZo H$s
àm{`H$Vm kmV H$s{OE & 2
AWdm
(ii) (I) `mÑÀN>`m M`Z {H$E JE {dÚmb` _| 20 go A{YH$ H§$ß`yQ>a Z hmoZo H$s
àm{`H$Vm kmV H$s{OE & 2
(iii) `mÑÀN>`m M`Z {H$E JE {dÚmb` _| 10 `m 10 go H$_ H§$ß`yQ>a hmoZo H$s àm{`H$Vm
kmV H$s{OE & 1

30/2/3 JJJJ Page 20


Case Study 2

37. Computer-based learning (CBL) refers to any teaching methodology that


makes use of computers for information transmission. At an elementary
school level, computer applications can be used to display multimedia
lesson plans. A survey was done on 1000 elementary and secondary
schools of Assam and they were classified by the number of computers
they had.

Number of 101 and


1 10 11 20 21 50 51 100
Computers more
Number of
250 200 290 180 80
Schools

One school is chosen at random. Then :


(i) Find the probability that the school chosen at random has more
than 100 computers. 1
(ii) (a) Find the probability that the school chosen at random has
50 or fewer computers. 2
OR
(ii) (b) Find the probability that the school chosen at random has
no more than 20 computers. 2
(iii) Find the probability that the school chosen at random has 10 or
less than 10 computers. 1

30/2/3 JJJJ Page 21 P.T.O.


àH$aU AÜ``Z 3

38. EH$ {dÚmb` Ho$ dm{f©H$ {Xdg na à~§YH$m| Zo AnZo g~go hmoZhma {dÚm{W©`m| H$mo ZH$X
nwañH$ma Ho$ gmW-gmW ñ_¥{V-
O¡gm ~Zdm`m J`m VWm BgH$m AmYma ABCD gm_Zo H$s Amoa go {XIVm Wm & {gëda
ßboqQ>J H$m IM© < 20 à{V dJ© go_r h¡ &

Cn`w©º$ Ho$ AmYma na, {ZåZ{b{IV àíZm| Ho$ CÎma Xr{OE :


(i) MVwWmªe ODCO H$m joÌ\ ? 1
(ii) AOB H$m joÌ\$b kmV H$s{OE & 1

(iii) (H$) ABCD ? 2


AWdm
(iii) (I) Mmn CD ? 2

30/2/3 JJJJ Page 22


Case Study 3

38. In an annual day function of a school, the organizers wanted to give a


cash prize along with a memento to their best students. Each memento is
made as shown in the figure and its base ABCD is shown from the front
side. The rate of silver plating is 20 per cm2.

Based on the above, answer the following questions :


(i) What is the area of the quadrant ODCO ? 1

(ii) 1
(iii) (a) What is the total cost of silver plating the shaded part
ABCD ? 2
OR
(iii) (b) What is the length of arc CD ? 2

30/2/3 JJJJ Page 23 P.T.O.


SET ~ 1
Series WX1YZ/4
àíZ-nÌ H$moS>
Q.P. Code 30/4/1
amob Z§. narjmWu àíZ-nÌ H$moS> H$mo CÎma-nwpñVH$m Ho$
Roll No. _wI-n¥ð> na Adí` {bI|Ÿ&
Candidates must write the Q.P. Code
on the title page of the answer-book.

J{UV (_mZH$) - g¡ÕmpÝVH$


MATHEMATICS (STANDARD) - Theory
#

{ZYm©[aV g_` … 3 KÊQ>o A{YH$V_ A§H$ … 80


Time allowed : 3 hours Maximum Marks : 80

ZmoQ> / NOTE :
(i) H¥$n`m Om±M H$a b| {H$ Bg àíZ-nÌ _| _w{ÐV n¥ð> 15 h¢Ÿ&
Please check that this question paper contains 15 printed pages.
(ii) àíZ-nÌ _| Xm{hZo hmW H$s Amoa {XE JE àíZ-nÌ H$moS> H$mo N>mÌ CÎma-nwpñVH$m Ho$ _wI-n¥ð> na
{bI|&
Q.P. Code given on the right hand side of the question paper should be written on
the title page of the answer-book by the candidate.
(iii) H¥$n`m Om±M H$a b| {H$ Bg àíZ-nÌ _| 38 àíZ h¢&
Please check that this question paper contains 38 questions.
(iv) H¥$n`m àíZ H$m CÎma {bIZm ewê$ H$aZo go nhbo, CÎma-nwpñVH$m _| àíZ H$m H«$_m§H$ Adí`
{bI|&
Please write down the Serial Number of the question in the answer-book before
attempting it.
(v) Bg àíZ-nÌ H$mo n‹T>Zo Ho$ {bE 15 {_ZQ> H$m g_` {X`m J`m h¡& àíZ-nÌ H$m {dVaU nydm©• _|
10.15 ~Oo {H$`m OmEJm& 10.15 ~Oo go 10.30 ~Oo VH$ N>mÌ Ho$db àíZ-nÌ H$mo n‹T>|Jo Am¡a
Bg Ad{Y Ho$ Xm¡amZ do CÎma-nwpñVH$m na H$moB© CÎma Zht {bI|Jo&
15 minute time has been allotted to read this question paper. The question paper
will be distributed at 10.15 a.m. From 10.15 a.m. to 10.30 a.m., the students will
read the question paper only and will not write any answer on the answer-book
during this period.

30/4/1 ~~~~ 1 [P.T.O.


gm_mÝ` {ZX}e …
{ZåZ{b{IV {ZX}em| H$mo ~hþV gmdYmZr go n{‹T>E Am¡a CZH$m nmbZ H$s{OE …
(i) Bg àíZ-nÌ _| 38 àíZ h¢Ÿ& g^r àíZ A{Zdm`© h¢Ÿ&
(ii) àíZ-nÌ nm§M IÊS>m| _| {d^m{OV h¡ - IÊS> H$, I, J, K VWm ‹S>Ÿ&
(iii) IÊS> H$ _| àíZ g§»`m 1 go 18 VH$ ~hþ{dH$ënr` VWm àíZ g§»`m 19 Ed§ 20 A{^H$WZ Ed§
H$maU AmYm[aV EH$-EH$ A§H$ Ho$ àíZ h¢Ÿ&
(iv) IÊS> I _| àíZ g§»`m 21 go 25 VH$ A{V bKw-CÎmar` (VSA) àH$ma Ho$ Xmo-Xmo A§H$m| Ho$ àíZ h¢Ÿ&
(v) IÊS> J _| àíZ g§»`m 26 go 31 VH$ bKw-CÎmar` (SA) àH$ma Ho$ VrZ-VrZ A§H$m| Ho$
àíZ h¢Ÿ&
(vi) IÊS> K _| àíZ g§»`m 32 go 35 VH$ XrK©-CÎmar` (LA) àH$ma Ho$ nm§M-nm§M A§H$m| Ho$ àíZ h¢Ÿ&
(vii) IÊS> ‹S> _| àíZ g§»`m 36 go 38 VH$ òmoV / àH$aU BH$mB© AmYm[aV Mma-Mma A§H$m| Ho$ àíZ h¢Ÿ&
Am§V[aH$ {dH$ën Xmo-Xmo A§H$m| Ho$ àíZ _| {X`m J`m h¡Ÿ&
(viii) àíZ-nÌ _| g_J« {dH$ën Zht {X`m J`m h¡Ÿ& `Ú{n, IÊS> I Ho$ 2 àíZm| _|, IÊS> J Ho$ 2 àíZm| _|,
IÊS> K Ho$ 2 àíZm| _| VWm IÊS> ‹S> Ho$ 3 àíZm| _| Am§V[aH$ {dH$ën H$m àmdYmZ {X`m J`m h¡Ÿ&
(ix) Ohm§ Amdí`H$ hmo, ñdÀN> AmH¥${V`m§ ~ZmE§Ÿ& `{X Amdí`H$ hmo Vmo π=22/7 b|Ÿ&
(x) H¡$bHw$boQ>a H$m Cn`moJ d{O©V h¡Ÿ&

IÊS> - H$
IÊS> - H$ _| ~hþ{dH$ënr` àH$ma Ho$ àíZ h¢Ÿ& àË`oH$ àíZ 1 A§H$ H$m h¡Ÿ&
1. g~go N>moQ>r g§`wº$ g§»`m Am¡a g~go N>moQ>r A^mÁ` g§»`m Ho$ HCF H$m AZwnmV CZHo$
LCM go h¡ … 1
(a) 1:2 (b) 2:1 (c) 1:1 (d) 1:3
2. g_rH$aU x2 + 3x – 10 = 0 Ho$ _yb h¢ … 1
(a) 2, –5 (b) –2, 5 (c) 2, 5 (d) –2, –5
3. A.P. : 6 , 24 , 54 H$m AJbm nX h¡ … 1
(a) 60 (b) 96 (c) 72 (d) 216

4. {~§Xw (–1, 7) H$s x-Aj go Xyar h¡ … 1


(a) –1 (b) 7 (c) 6 (d) 50

5. ‘d’ ì`mg Ho$ AY©d¥Îm H$m joÌ\$b Š`m h¡ ? 1


1 2 1 2 1 2 1 2
(a) πd (b) πd (c) πd (d) πd
16 4 8 2

6. {H$gr ~§Q>Z Ho$ ~hþbH$, _mÜ` VWm _mÜ`H$ Ho$ {bE AmZw^m{dH$ g§~§Y h¡ … 1
(a) ~hþbH$ = 3 _mÜ`H$ - 2 _mÜ` (b) ~hþbH$ = 3 _mÜ` - 2 _mÜ`H$
(c) ~hþbH$ = 2 _mÜ`H$ - 3 _mÜ` (d) ~hþbH$ = 2 _mÜ` - 3 _mÜ`H$

30/4/1 2
GENERAL INSTRUCTIONS :
Read the following instructions carefully and follow them :
(i) This question paper contains 38 questions. All questions are compulsory.
(ii) Question paper is divided into FIVE sections – Section A, B, C, D and E.
(iii) In section A – question number 1 to 18 are multiple choice questions (MCQs) and
question number 19 and 20 are Assertion-Reason based questions of 1 mark each.
(iv) In section B – question number 21 to 25 are Very Short Answer (VSA) type questions
of 2 marks each.
(v) In section C – question number 26 to 31 are Short Answer (SA) type questions
carrying 3 marks each.
(vi) In section D – question number 32 to 35 are Long Answer (LA) type questions
carrying 5 marks each.
(vii) In section E – question number 36 to 38 are case based integrated units of
assessment questions carrying 4 marks each. Internal choice is provided in 2 marks
question in each case-study.
(viii) There is no overall choice. However, an internal choice has been provided in 2
questions in Section B, 2 questions in Section C, 2 questions in Section D and 3
questions in Section E.
(ix) Draw neat figures wherever required. Take π = 22/7 wherever required if not stated.
(x) Use of calculators is NOT allowed.
SECTION - A
Section - A consists of Multiple Choice type questions of 1 mark each.
1. The ratio of HCF to LCM of the least composite number and the least
prime number is : 1
(a) 1:2 (b) 2:1 (c) 1:1 (d) 1:3
2. The roots of the equation x2 + 3x – 10 = 0 are : 1
(a) 2, –5 (b) –2, 5 (c) 2, 5 (d) –2, –5

3. The next term of the A.P. : 6 , 24 , 54 is : 1


(a) 60 (b) 96 (c) 72 (d) 216
4. The distance of the point (–1, 7) from x-axis is : 1
(a) –1 (b) 7 (c) 6 (d) 50
5. What is the area of a semi-circle of diameter ‘d’ ? 1
1 2 1 2 1 1 2
(a) πd (b) πd (c) πd 2 (d) πd
16 4 8 2
6. The empirical relation between the mode, median and mean of a
distribution is : 1
(a) Mode = 3 Median – 2 Mean (b) Mode = 3 Mean – 2 Median
(c) Mode = 2 Median – 3 Mean (d) Mode = 2 Mean – 3 Median
30/4/1 3 [P.T.O.
7. g_rH$aU `w½_ … 2x = 5y + 6 Am¡a 15y = 6x – 18 {OZ Xmo aoImAm| H$mo {Zê${nV
H$aVm h¡, do h¢ … 1
(a) à{VÀN>oXr aoImE§ (b) g_m§Va aoImE§
(c) g§nmVr aoImE§ (d) à{VÀN>oXr `m g_m§Va aoImE§
8. `{X α, β {ÛKmV ~hþnX x –1 Ho$ eyÝ`m§H$ h¢, Vmo (α + β) H$m _mZ hmoJm …
2
1
(a) 2 (b) 1 (c) –1 (d) 0
9. `{X 6 _r. D±$Mo EH$ I§^o H$s N>m`m, ^y{_ na 2 3 _r. b§~r h¡, Vmo gy`© H$m CÞVm§e
hmoJm … 1
o o o o
(a) 60 (b) 45 (c) 30 (d) 90
10. O~ sec θ H$mo cot θ Ho$ ê$n _| ì`º$ {H$`m OmVm h¡, V~ `h ~am~a h¡ … 1
1+cot 2θ
(a) (b) 1+cot 2θ
cot θ
1+cot 2θ 1 − cot 2θ
(c) (d)
cot θ cot θ
11. Xmo nmgm| H$mo EH$ gmW \|$H$m J`mŸ& nmgm| Ho$ D$nar \$bH$m| na AmB© g§»`mAm| H$m AÝVa 3
hmoZo H$s àm{`H$Vm h¡ … 1
1 2 1 1
(a) (b) (c) (d)
9 9 6 12
12.

Xr JB© AmH¥${V _|, ∆ABC ~ ∆QPRŸ& `{X AC = 6 go_r., BC = 5 go_r.,


QR = 3 go_r. Am¡a PR = x h¡, Vmo x H$m _mZ hmoJm … 1
(a) 3.6 go_r. (b) 2.5 go_r. (c) 10 go_r. (d) 3.2 go_r.
13. _yb {~ÝXw go {~ÝXw (– 6, 8) H$s Xyar h¡ … 1
(a) 6 (b) – 6 (c) 8 (d) 10
14. Xr JB© AmH¥${V _|, Ho$ÝÐ O dmbo EH$ d¥Îm na PQ EH$ ñne© aoIm h¡Ÿ& `{X
∠OPQ = x Am¡a ∠POQ = y h¡, Vmo x + y H$m _mZ hmoJm … 1
(a) 45o
(b) 90 o
(c) 60 o
(d) 180 o
30/4/1 4
7. The pair of linear equations 2x = 5y + 6 and 15y = 6x – 18 represents
two lines which are : 1
(a) intersecting (b) parallel
(c) coincident (d) either intersecting or parallel
8. If α, β are zeroes of the polynomial x2–1, then value of (α + β) is : 1
(a) 2 (b) 1 (c) –1 (d) 0
9. If a pole 6 m high casts a shadow 2 3 m long on the ground, then
sun’s elevation is : 1
(a) 60o (b) 45 o (c) 30 o (d) 90 o
10. sec θ when expressed in terms of cot θ, is equal to : 1
1+cot 2θ
(a) (b) 1+cot 2θ
cot θ
1+cot 2θ 1 − cot 2θ
(c) (d)
cot θ cot θ
11. Two dice are thrown together. The probability of getting the
difference of numbers on their upper faces equals to 3 is : 1
1 2 1 1
(a) (b) (c) (d)
9 9 6 12
12.

In the given figure, ∆ABC ~ ∆QPR. If AC = 6 cm, BC = 5 cm,


QR = 3 cm and PR = x; then the value of x is : 1
(a) 3.6 cm (b) 2.5 cm (c) 10 cm (d) 3.2 cm
13. The distance of the point (– 6, 8) from origin is : 1
(a) 6 (b) – 6 (c) 8 (d) 10
14. In the given figure, PQ is a tangent to the circle with centre O. If
∠OPQ = x, ∠POQ = y, then x + y is : 1
(a) 45o
(b) 90 o
(c) 60 o
(d) 180 o

30/4/1 5 [P.T.O.
15. Xr JB© AmH¥${V _|, Ho$ÝÐ O dmbo EH$ d¥Îm na TA EH$ ñne© aoIm h¡ Ohm± OT = 4 go_r.
Am¡a ∠OTA = 30o & TA H$s bå~mB© h¡ … 1
(a) 2 3 go_r.
(b) 2 go_r.
(c) 2 2 go_r.
(d) 3 go_r.
16. {XE JE ∆ ABC _| PQ || BC h¡Ÿ& `{X PB = 6 go_r., AP = 4 go_r. Am¡a AQ = 8
go_r. h¡, Vmo AC H$s bå~mB© hmoJr … 1
(a) 12 go_r.
(b) 20 go_r.
(c) 6 go_r.
(d) 14 go_r.

17. `{X α, β ~hþnX p(x) = 4x2 – 3x – 7 Ho$ eyÝ`m§H$ h¢ Vmo  1 + 1  H$m _mZ h¡ … 1
α β 
7 −7 3 −3
(a) (b) (c) (d)
3 3 7 7
18. 52 nÎmm| H$s AÀN>r àH$ma go \|$Q>r JB© EH$ JÈ>r go EH$ nÎmm `mÑÀN>`m {ZH$mbm OmVm h¡Ÿ&
{ZH$mbm J`m nÎmm EH$ BŠH$m Zht hmoZo H$s àm{`H$Vm h¡ … 1
1 9 4 12
(a) (b) (c) (d)
13 13 13 13
{ZX}e … àíZ g§»`m 19 VWm 20 _| EH$ A{^H$WZ (A) Ho$ níMmV² EH$ VH©$-
H$WZ (R) {X`m h¡Ÿ& {ZåZ _| go ghr {dH$ën Mw{ZE …
2
19. A{^H$WZ (A) … EH$ brn-df© _| 53 a{ddma hmoZo H$s àm{`H$Vm h¡Ÿ&
7
5
VH©$ (R) … EH$ J¡a brn-df© _| 53 a{ddma hmoZo H$s àm{`H$Vm
h¡Ÿ& 1
7
(a) A{^H$WZ (A) VWm H$maU (R) XmoZm| gË` h¢Ÿ& H$maU (R) A{^H$WZ (A) H$s
ì`m»`m H$aVm h¡Ÿ&
(b) A{^H$WZ (A) VWm H$maU (R) XmoZm| gË` h¢Ÿ& H$maU (R) A{^H$WZ (A) H$s
ì`m»`m Zht H$aVm h¡Ÿ&
(c) A{^H$WZ (A) gË` h¡ naÝVw H$maU (R) AgË` h¡Ÿ&
(d) A{^H$WZ (A) AgË` h¡ O~{H$ H$maU (R) gË` h¡Ÿ&

30/4/1 6
15. In the given figure, TA is a tangent to the circle with centre O such
that OT = 4 cm, ∠OTA = 30o, then length of TA is : 1
(a) 2 3 cm
(b) 2 cm
(c) 2 2 cm
(d) 3 cm

16. In ∆ ABC, PQ || BC. If PB = 6 cm, AP = 4 cm, AQ = 8 cm, find the


length of AC. 1
(a) 12 cm
(b) 20 cm
(c) 6 cm
(d) 14 cm

17. If α, β are the zeroes of the polynomial p(x) = 4x2 – 3x – 7, then


1 1
 α + β  is equal to : 1
 
7 −7 3 −3
(a) (b) (c) (d)
3 3 7 7
18. A card is drawn at random from a well-shuffled pack of 52 cards. The
probability that the card drawn is not an ace is : 1
1 9 4 12
(a) (b) (c) (d)
13 13 13 13
DIRECTIONS : In the question number 19 and 20, a statement of
Assertion (A) is followed by a statement of Reason (R). Choose the
correct option out of the following :
2
19. Assertion (A) : The probability that a leap year has 53 Sundays is .
7
5
Reason (R) : The probability that a non-leap year has 53 Sundays is . 1
7
(a) Both Assertion (A) and Reason (R) are true and Reason (R) is
the correct explanation of Assertion (A).
(b) Both Assertion (A) and Reason (R) are true and Reason (R) is
not the correct explanation of Assertion (A).
(c) Assertion (A) is true but Reason (R) is false.
(d) Assertion (A) is false but Reason (R) is true.

30/4/1 7 [P.T.O.
20. A{^H$WZ (A) … a, b, c EH$ A.P. Ho$ nX hm|Jo AJa Am¡a Ho$db AJa 2b = a + c &
VH©$ (R) … nhbr "n' {df_ àmH¥$V g§»`mAm| H$m `moJ n2 h¡Ÿ& 1
(a) A{^H$WZ (A) VWm H$maU (R) XmoZm| gË` h¢Ÿ& H$maU (R) A{^H$WZ (A) H$s
ì`m»`m H$aVm h¡Ÿ&
(b) A{^H$WZ (A) VWm H$maU (R) XmoZm| gË` h¢Ÿ& H$maU (R) A{^H$WZ (A) H$s
ì`m»`m Zht H$aVm h¡Ÿ&
(c) A{^H$WZ (A) gË` h¡ naÝVw H$maU (R) AgË` h¡Ÿ&
(d) A{^H$WZ (A) AgË` h¡ O~{H$ H$maU (R) gË` h¡Ÿ&
IÊS> - I
IÊS> - I _| A{V bKw-CÎma (VSA) àH$ma Ho$ àíZ h¢Ÿ& àË`oH$ àíZ Ho$ 2 A§H$ h¢Ÿ&
21. Xmo g§»`mE§ 2…3 Ho$ AZwnmV _| h¢ Am¡a CZH$m LCM 180 h¡Ÿ& BZ g§»`mAm| H$m HCF
Š`m hmoJm ? 2
22. `{X ~hþnX p(x) = 6x2 + 37x – (k – 2) H$m EH$ eyÝ`m§H$ Xygao eyÝ`m§H$ H$m ì`wËH«$_
hmo, Vmo k H$m _mZ kmV H$s{OEŸ& 2
23. (A) {ÛKmV g_rH$aU 2x2 – 9x + 4 = 0 Ho$ _ybm| H$m `moJ Am¡a JwUZ\$b kmV H$s{OEŸ& 2
AWdm
(B) {ÛKmV g_rH$aU 4x2 – 5 = 0 H$m {d{dº$H$a (discriminant) kmV H$s{OE
Am¡a g_rH$aU Ho$ _ybm| H$s àH¥${V {b{IEŸ& 2
24. EH$ {Zînj {gŠH$m Xmo ~ma CN>mbm OmVm h¡Ÿ& A{YH$ go A{YH$ EH$ {MV AmZo H$s
àm{`H$Vm kmV H$s{OEŸ& 2
5cos 2 60o + 4sec2 30o − tan 2 45o
25. (A) _mZ kmV H$s{OE … 2
sin 2 30o + cos 2 30o
AWdm
(B) `{X A Am¡a B Xmo Eogo Ý`yZ H$moU h¢ {OZHo$ {bE sin (A – B) = 0 Am¡a
2 cos (A + B) – 1 = 0 h¡, Vmo H$moU A Am¡a B kmV H$s{OEŸ& 2
IÊS> - J
IÊS> - J _| bKw-CÎma (SA) àH$ma Ho$ àíZ h¢ Am¡a àË`oH$ àíZ 3 A§H$ H$m h¡Ÿ&
26. (A) EH$ A.P. {OgH$m nhbm Am¡a nm±Mdm§ nX H«$_e… -14 Am¡a 2 h¡ VWm ApÝV_ nX
62 h¡, Vmo A.P. _| {H$VZo nX h¢ ? 3
AWdm
(B) A.P. : 65, 61, 57, 53, .......... H$m H$m¡Zgm nX g~go nhbm G$UmË_H$ nX hmoJm? 3
27. {gÕ H$s{OE {H$ 5 EH$ An[a_o` g§»`m h¡Ÿ& 3
28. {gÕ H$s{OE {H$ {H$gr ~mø {~§Xw go {H$gr d¥Îm na ItMr JB© Xmo ñne© aoImAm| Ho$ ~rM H$m
H$moU ñne© q~XwAm| H$mo {_bmZo dmbo aoImIÊS> Ûmam Ho$ÝÐ na A§V[aV H$moU H$m g§nyaH$ hmoVm
h¡Ÿ& 3
30/4/1 8
20. Assertion (A) : a, b, c are in A.P. if and only if 2b = a + c.
Reason (R) : The sum of first n odd natural numbers is n2. 1
(a) Both Assertion (A) and Reason (R) are true and Reason (R) is
the correct explanation of Assertion (A).
(b) Both Assertion (A) and Reason (R) are true and Reason (R) is
not the correct explanation of Assertion (A).
(c) Assertion (A) is true but Reason (R) is false.
(d) Assertion (A) is false but Reason (R) is true.
SECTION – B
Section - B consists of Very Short Answer (VSA) type questions of 2
marks each.
21. Two numbers are in the ratio 2 : 3 and their LCM is 180. What is the
HCF of these numbers ? 2
22. If one zero of the polynomial p(x) = 6x2 + 37x – (k – 2) is reciprocal
of the other, then find the value of k. 2
23. (A) Find the sum and product of the roots of the quadratic equation
2x2 – 9x + 4 = 0. 2
OR
(B) Find the discriminant of the quadratic equation 4x2 – 5 = 0 and
hence comment on the nature of roots of the equation. 2
24. If a fair coin is tossed twice, find the probability of getting ‘atmost
one head’. 2
5cos 2 60o + 4sec2 30o − tan 2 45o
25. (A) Evaluate 2
sin 2 30o + cos 2 30o
OR
(B) If A and B are acute angles such that sin (A – B) = 0 and
2 cos (A + B) – 1 = 0, then find angles A and B. 2
SECTION – C
Section - C consists of Short Answer (SA) type questions of 3 marks each.
26. (A) How many terms are there in an A.P. whose first and fifth terms
are – 14 and 2, respectively and the last term is 62. 3
OR
(B) Which term of the A.P. : 65, 61, 57, 53, .................. is the first
negative term ? 3
27. Prove that 5 is an irrational number. 3
28. Prove that the angle between the two tangents drawn from an external
point to a circle is supplementary to the angle subtended by the line-
segment joining the points of contact at the centre. 3

30/4/1 9 [P.T.O.
sin A − 2 sin 3A
29. (A) {gÕ H$s{OE {H$ = tan A 3
2cos3A − cos A
AWdm
(B) {gÕ H$s{OE {H$ sec A (1 – sin A) (sec A + tan A) = 1 3

30. Xmo g§H|$Ðr` d¥Îmm| H$s {ÌÁ`mE§ 5 go_r. Am¡a 3 go_r. h¢Ÿ& ~‹S>o d¥Îm H$s Ordm, Omo N>moQ>o d¥Îm
H$mo ñne© H$aVr h¡, H$s b§~mB© kmV H$s{OEŸ& 3

31. ‘p’ H$m _mZ kmV H$s{OE {OgHo$ {bE {ÛKmV g_rH$aU px(x – 2) + 6 = 0 Ho$ Xmo
~am~a dmñV{dH$ _yb hm|Ÿ& 3
IÊS> - K
IÊS> - K _| XrK©-CÎma (LA) àH$ma Ho$ àíZ h¢Ÿ& àË`oH$ àíZ Ho$ 5 A§H$ h¢Ÿ&
32. (A) EH$ grYm amO_mJ© EH$ _rZma Ho$ nmX VH$ OmVm h¡Ÿ& EH$ 75 _r. D±$Mo Q>m°da Ho$
erf© na I‹S>m EH$ ì`{º$ Xmo H$mam| H$mo 30o Am¡a 60o Ho$ AdZ_Z H$moUm| na XoIVm
h¡, Omo Q>m°da Ho$ nmX H$s Amoa Om ahr h¢Ÿ& `{X EH$ H$ma Q>m°da Ho$ Cgr Va\$ Xygar
H$ma Ho$ R>rH$ nrN>o hmo, Vmo XmoZm| H$mam| Ho$ ~rM H$s Xyar kmV H$s{OEŸ& ( 3 = 1.73
br{OE) 5
AWdm
(B) 7 _r. D±$Mo ^dZ Ho$ {eIa go EH$ Ho$~b Q>m°da Ho$ {eIa H$m CÞ`Z H$moU 60o h¡
Am¡a BgHo$ nmX H$m AdZ_Z H$moU 30o h¡Ÿ& Q>m°da H$s D±$MmB© kmV H$s{OEŸ& 5
33. (A) EH$ {Ì^wO ABC H$s ^wOm BC na EH$ {~§Xw D Bg àH$ma pñWV h¡ {H$
∠ADC = ∠BAC h¡Ÿ& Xem©BE {H$ CA2 = CB.CD h¡Ÿ& 5
AWdm
(B) AD Am¡a PM {Ì^wOm| ABC Am¡a PQR H$s H«$_e… _mpÜ`H$mE§ h¢, O~{H$
AB AD
∆ABC ~ ∆PQR h¡Ÿ& {gÕ H$s{OE {H$ = h¡Ÿ& 5
PQ PM
34. EH$ N>mÌ H$mo EH$ nVbr Eë`w{_{Z`_ erQ> H$m Cn`moJ H$aHo$ BgHo$ {gam| go Ow‹S>o e§Hw$Am|
Ho$ gmW EH$ ~obZ Ho$ AmH$ma H$m EH$ _m°S>b ~ZmZo Ho$ {b`o H$hm J`m WmŸ& _m°S>b H$m
ì`mg 3 go_r. Am¡a b§~mB© 12 go_r. h¡Ÿ& `{X àË`oH$ e§Hw$ H$s D±$MmB© 2 go_r. h¡, Vmo
_m°S>b Ho$ AÝXa H$s hdm H$m Am`VZ kmV H$s{OEŸ& 5
35. EH$ hmCqgJ gmogm`Q>r Ho$ 200 n[admam| _| XyY na _m{gH$ IM© XO© {H$E Omo ZrMo {XE JE
h¢ …
_m{gH$ IM© 1000- 1500- 2000- 2500- 3000- 3500- 4000- 4500-
(`. _|) 1500 2000 2500 3000 3500 4000 4500 5000
n[admam| H$s 24 40 33 x 30 22 16 7
g§»`m
x H$m _mZ kmV H$s{OE Am¡a XyY na _mÜ`H$ Am¡a _mÜ` IM© ^r kmV H$s{OEŸ& 5
30/4/1 10
sin A − 2 sin 3A
29. (A) Prove that = tan A 3
2cos3A − cos A
OR
(B) Prove that sec A (1 – sin A) (sec A + tan A) = 1. 3
30. Two concentric circles are of radii 5 cm and 3 cm. Find the length of
the chord of the larger circle which touches the smaller circle. 3
31. Find the value of ‘p’ for which the quadratic equation
px(x – 2) + 6 = 0 has two equal real roots. 3
SECTION – D
Section - D consists of Long Answer (LA) type questions of 5 marks each.
32. (A) A straight highway leads to the foot of a tower. A man standing
on the top of the 75 m high tower observes two cars at angles of
depression of 30o and 60o, which are approaching the foot of the
tower. If one car is exactly behind the other on the same side of
the tower, find the distance between the two cars. (use 3 = 1.73) 5
OR
(B) From the top of a 7 m high building, the angle of elevation of the
top of a cable tower is 60o and the angle of depression of its foot
is 30o. Determine the height of the tower. 5
33. (A) D is a point on the side BC of a triangle ABC such that
∠ADC = ∠BAC, prove that CA2 = CB.CD 5
OR
(B) If AD and PM are medians of triangles ABC and PQR,
AB AD
respectively where ∆ABC ~ ∆PQR, prove that = . 5
PQ PM
34. A student was asked to make a model shaped like a cylinder with two
cones attached to its ends by using a thin aluminium sheet. The
diameter of the model is 3 cm and its total length is 12 cm. If each
cone has a height of 2 cm, find the volume of air contained in the
model. 5
35. The monthly expenditure on milk in 200 families of a Housing
Society is given below :
Monthly 1000- 1500- 2000- 2500- 3000- 3500- 4000- 4500-
Expenditure 1500 2000 2500 3000 3500 4000 4500 5000
(in `)
Number of 24 40 33 x 30 22 16 7
families

Find the value of x and also, find the median and mean expenditure
on milk. 5
30/4/1 11 [P.T.O.
IÊS> - ‹S>
IÊS> - ‹S> _| àH$aU AÜ``Z/n[aÀN>oX AmYm[aV 3 àíZ h¢Ÿ& àË`oH$ àíZ Ho$ 4 A§H$ h¢Ÿ&
36. Xmo ñHy$bm| ‘P’ Am¡a ‘Q’ Zo AnZo N>mÌm| H$mo hm°H$s ` x à{V N>mÌ
Am¡a {H«$Ho$Q> ` y à{V N>mÌ Xmo Iobm| Ho$ {bE nwañH$ma XoZo H$m
\¡$gbm {H$`mŸ& ñHy$b ‘P’ Zo Xmo Iobm| Ho$ {bE H«$_e… 5 Am¡a 4
N>mÌm| H$mo Hw$b 9,500 é. H$m nwañH$ma XoZo H$m \¡$gbm {H$`m,
O~{H$ ñHy$b ‘Q’ Zo Xmo Iobm| Ho$ {bE H«$_e… 4 Am¡a 3 N>mÌm|
H$mo Hw$b 7,370 é. H$m nwañH$ma XoZo H$m \¡$gbm {H$`mŸ&
Cnamoº$ gyMZm Ho$ AmYma na, {ZåZ{b{IV àíZm| Ho$ CÎma Xr{OE …
(i) Cnamoº$ gyMZm H$mo, Mam| x Am¡a y H$m à`moJ H$aHo$,
~rOJ{UVr` ê$n _| ì`º$ H$s{OEŸ& 1
(ii) (a) hm°H$s Ho$ {bE nwañH$ma am{e Š`m h¡ ? 2
AWdm
(b) nwañH$ma am{e {H$g Iob H$s A{YH$ h¡ Am¡a {H$VZr A{YH$ ? 2
(iii) `{X àË`oH$ Iob go 2 N>mÌ hm|, Vmo Hw$b nwañH$ma am{e Š`m hmoJr ? 1
37. OJXre Ho$ nmg EH$ IoV h¡ Omo EH$ g_H$moU {Ì^wO AQC Ho$ AmH$ma H$m h¡Ÿ& dh IoV
Ho$ A§Xa EH$ dJm©H$ma PQRS Ho$ ê$n _| Johÿ± CJmZo Ho$ {bE Am¡a eof gpãO`m± CJmZo Ho$
{bE ({MÌ _| Xem©`o AZwgma) OJh N>mo‹S>Zm MmhVm h¡Ÿ& IoV _|, O Ho$ ê$n _| {M{•V EH$
I§^m h¡Ÿ&

Cnamoº$ gyMZm Ho$ AmYma na, {ZåZ{b{IV àíZm| Ho$ CÎma Xr{OE …
(i) O H$mo _yb q~Xw _mZ H$a, {~§XwAm| P Am¡a Q Ho$ {ZX}em§H$ H«$_e… (-200, 0) Am¡a
(200, 0) h¢Ÿ& PQRS EH$ dJ© hmoZo Ho$ H$maU, R Am¡a S Ho$ {ZX}em§H$ Š`m hm|JoŸ? 1

30/4/1 12
SECTION – E
Section – E consists of three Case Study Based questions of 4 marks each.
36. Two schools ‘P’ and ‘Q’ decided to award
prizes to their students for two games of
Hockey ` x per student and Cricket ` y per
student. School ‘P’ decided to award a total
of ` 9,500 for the two games to 5 and 4
students respectively; while school ‘Q’
decided to award ` 7,370 for the two games
to 4 and 3 students respectively.
Based on the above information, answer the following questions :
(i) Represent the following information algebraically (in terms of x
and y). 1
(ii) (a) What is the prize amount for hockey ? 2
OR
(b) Prize amount on which game is more and by how much ? 2
(iii) What will be the total prize amount if there are 2 students each
from two games ? 1
37. Jagdish has a field which is in the shape of a right angled triangle
AQC. He wants to leave a space in the form of a square PQRS inside
the field for growing wheat and the remaining for growing vegetables
(as shown in the figure). In the field, there is a pole marked as O.

Based on the above information, answer the following questions :


(i) Taking O as origin, coordinates of P are (–200, 0) and of Q are
(200, 0). PQRS being a square, what are the coordinates of R
and S ? 1
30/4/1 13 [P.T.O.
(ii) (a) dJ© PQRS H$m joÌ\$b Š`m h¡ ? 2
AWdm
(b) dJ© PQRS _| {dH$U© PR H$s bå~mB© Š`m h¡Ÿ? 2
(iii) `{X q~Xw S, aoImIÊS> CA H$mo AZwnmV K:1 _| {d^m{OV H$ao, Vmo K H$m _mZ
Š`m hmoJm, `{X q~Xw A Ho$ {ZX}em§H$ (200, 800) hm| ? 1

38. XohamXyZ Ho$ EH$ ñWmZr` gmd©O{ZH$ {dH$mg àm{YH$aU H$s Jd{ZªJ H$mC§{gb Zo EH$ nhm‹S>r
H$s MmoQ>r na EH$ gmh{gH$ Iob H$m _¡XmZ ~ZmZo H$m \¡$gbm {H$`m, {Og_| nm{Hª$J Ho$ {bE
n`m©á OJh hmoJrŸ&

gd}jU Ho$ níMmV², Am`VmH$ma Iob H$m _¡XmZ ~ZmZo H$m {ZU©` {b`m J`m, {Og_| EH$
Va\$ nm{Hª$J Ho$ {bE EH$ AY©-d¥ÎmmH$ma joÌ A§{H$V hmoŸ& Am`VmH$ma Iob Ho$ _¡XmZ H$s
bå~mB© Am¡a Mm¡‹S>mB© H«$_e… 14 BH$mB© Am¡a 7 BH$mB© h¡Ÿ& Iob Ho$ _¡XmZ Ho$ EH$ Amoa
2 BH$mB© {ÌÁ`m Ho$ Xmo MVwWmªe {deof grQ>m| Ho$ {bE h¢Ÿ&
Cnamoº$ gyMZm Ho$ AmYma na, {ZåZ{b{IV àíZm| Ho$ CÎma Xr{OE …
(i) nm{Hª$J joÌ H$m Hw$b n[a_mn {H$VZm h¡ ? 1
(ii) (a) nm{Hª$J joÌ Am¡a Xmo MVwWmªem| H$mo {_bmH$a Hw$b joÌ\$b {H$VZm h¡ ? 2
AWdm
(b) Iob Ho$ _¡XmZ Ho$ joÌ\$b Am¡a nm{Hª$J joÌ Ho$ joÌ\$b _| Š`m AZwnmV h¡Ÿ? 2
(iii) Iob Ho$ _¡XmZ Am¡a nm{Hª$J joÌ Ho$ Mmam| Amoa Vma bJdmZo H$m IM©, 2 é. à{V
BH$mB© H$s Xa go kmV H$s{OEŸ& 1

30/4/1 14
(ii) (a) What is the area of square PQRS ? 2
OR
(b) What is the length of diagonal PR in square PQRS ? 2
(iii) If S divides CA in the ratio K:1, what is the value of K, where
point A is (200, 800) ? 1
38. Governing council of a local public development authority of
Dehradun decided to build an adventurous playground on the top of a
hill, which will have adequate space for parking.

After survey, it was decided to build rectangular playground, with a


semi-circular area allotted for parking at one end of the playground.
The length and breadth of the rectangular playground are 14 units and
7 units, respectively. There are two quadrants of radius 2 units on one
side for special seats.
Based on the above information, answer the following questions :
(i) What is the total perimeter of the parking area ? 1
(ii) (a) What is the total area of parking and the two quadrants ? 2
OR
(b) What is the ratio of area of playground to the area of
parking area ? 2
(iii) Find the cost of fencing the playground and parking area at the
rate of ` 2 per unit. 1

30/4/1 15 [P.T.O.
30/4/1 16
SET ~ 2
Series WX1YZ/4
àíZ-nÌ H$moS>
Q.P. Code 30/4/2
amob Z§. narjmWu àíZ-nÌ H$moS> H$mo CÎma-nwpñVH$m Ho$
Roll No. _wI-n¥ð> na Adí` {bI|Ÿ&
Candidates must write the Q.P. Code
on the title page of the answer-book.

J{UV (_mZH$) - g¡ÕmpÝVH$


MATHEMATICS (STANDARD) - Theory
#

{ZYm©[aV g_` … 3 KÊQ>o A{YH$V_ A§H$ … 80


Time allowed : 3 hours Maximum Marks : 80

ZmoQ> / NOTE :
(i) H¥$n`m Om±M H$a b| {H$ Bg àíZ-nÌ _| _w{ÐV n¥ð> 15 h¢Ÿ&
Please check that this question paper contains 15 printed pages.
(ii) àíZ-nÌ _| Xm{hZo hmW H$s Amoa {XE JE àíZ-nÌ H$moS> H$mo N>mÌ CÎma-nwpñVH$m Ho$ _wI-n¥ð> na
{bI|&
Q.P. Code given on the right hand side of the question paper should be written on
the title page of the answer-book by the candidate.
(iii) H¥$n`m Om±M H$a b| {H$ Bg àíZ-nÌ _| 38 àíZ h¢&
Please check that this question paper contains 38 questions.
(iv) H¥$n`m àíZ H$m CÎma {bIZm ewê$ H$aZo go nhbo, CÎma-nwpñVH$m _| àíZ H$m H«$_m§H$ Adí`
{bI|&
Please write down the Serial Number of the question in the answer-book before
attempting it.
(v) Bg àíZ-nÌ H$mo n‹T>Zo Ho$ {bE 15 {_ZQ> H$m g_` {X`m J`m h¡& àíZ-nÌ H$m {dVaU nydm©• _|
10.15 ~Oo {H$`m OmEJm& 10.15 ~Oo go 10.30 ~Oo VH$ N>mÌ Ho$db àíZ-nÌ H$mo n‹T>|Jo Am¡a
Bg Ad{Y Ho$ Xm¡amZ do CÎma-nwpñVH$m na H$moB© CÎma Zht {bI|Jo&
15 minute time has been allotted to read this question paper. The question paper
will be distributed at 10.15 a.m. From 10.15 a.m. to 10.30 a.m., the students will
read the question paper only and will not write any answer on the answer-book
during this period.

30/4/2 ~~~~ 1 [P.T.O.


gm_mÝ` {ZX}e …
{ZåZ{b{IV {ZX}em| H$mo ~hþV gmdYmZr go n{‹T>E Am¡a CZH$m nmbZ H$s{OE …
(i) Bg àíZ-nÌ _| 38 àíZ h¢Ÿ& g^r àíZ A{Zdm`© h¢Ÿ&
(ii) àíZ-nÌ nm§M IÊS>m| _| {d^m{OV h¡ - IÊS> H$, I, J, K VWm ‹S>Ÿ&
(iii) IÊS> H$ _| àíZ g§»`m 1 go 18 VH$ ~hþ{dH$ënr` VWm àíZ g§»`m 19 Ed§ 20 A{^H$WZ Ed§
H$maU AmYm[aV EH$-EH$ A§H$ Ho$ àíZ h¢Ÿ&
(iv) IÊS> I _| àíZ g§»`m 21 go 25 VH$ A{V bKw-CÎmar` (VSA) àH$ma Ho$ Xmo-Xmo A§H$m| Ho$ àíZ h¢Ÿ&
(v) IÊS> J _| àíZ g§»`m 26 go 31 VH$ bKw-CÎmar` (SA) àH$ma Ho$ VrZ-VrZ A§H$m| Ho$
àíZ h¢Ÿ&
(vi) IÊS> K _| àíZ g§»`m 32 go 35 VH$ XrK©-CÎmar` (LA) àH$ma Ho$ nm§M-nm§M A§H$m| Ho$ àíZ h¢Ÿ&
(vii) IÊS> ‹S> _| àíZ g§»`m 36 go 38 VH$ òmoV / àH$aU BH$mB© AmYm[aV Mma-Mma A§H$m| Ho$ àíZ h¢Ÿ&
Am§V[aH$ {dH$ën Xmo-Xmo A§H$m| Ho$ àíZ _| {X`m J`m h¡Ÿ&
(viii) àíZ-nÌ _| g_J« {dH$ën Zht {X`m J`m h¡Ÿ& `Ú{n, IÊS> I Ho$ 2 àíZm| _|, IÊS> J Ho$ 2 àíZm| _|,
IÊS> K Ho$ 2 àíZm| _| VWm IÊS> ‹S> Ho$ 3 àíZm| _| Am§V[aH$ {dH$ën H$m àmdYmZ {X`m J`m h¡Ÿ&
(ix) Ohm§ Amdí`H$ hmo, ñdÀN> AmH¥${V`m§ ~ZmE§Ÿ& `{X Amdí`H$ hmo Vmo π=22/7 b|Ÿ&
(x) H¡$bHw$boQ>a H$m Cn`moJ d{O©V h¡Ÿ&
IÊS> - H$
IÊS> - H$ _| ~hþ{dH$ënr` àH$ma Ho$ àíZ h¢Ÿ& àË`oH$ àíZ 1 A§H$ H$m h¡Ÿ&
1. θ (0o ≤ θ ≤ 90o) Ho$ g^r _mZm| Ho$ {bE {ZåZ _| go H$m¡Z ghr h¡ ? 1
(a) cos2 θ – sin2 θ = 1 (b) cosec2 θ – sec2 θ = 1
(c) sec2 θ – tan2 θ = 1 (d) cot2 θ – tan2 θ = 1
2. `{X k + 2, 4k – 6 Am¡a 3k – 2 {H$gr EH$ A.P. Ho$ VrZ H«$_mJV nX h¢, Vmo k H$m
_mZ hmoJm … 1
(a) 3 (b) – 3 (c) 4 (d) – 4
3. {XE JE ∆ ABC _| PQ || BC h¡Ÿ& `{X PB = 6 go_r., AP = 4 go_r. Am¡a AQ = 8
go_r. h¡, Vmo AC H$s bå~mB© hmoJr … 1
(a) 12 go_r.
(b) 20 go_r.
(c) 6 go_r.
(d) 14 go_r.
4. g~go N>moQ>r g§`wº$ g§»`m Am¡a g~go N>moQ>r A^mÁ` g§»`m Ho$ HCF H$m AZwnmV CZHo$
LCM go h¡ … 1
(a) 1:2 (b) 2:1 (c) 1:1 (d) 1:3
5. 52 nÎmm| H$s AÀN>r àH$ma go \|$Q>r JB© EH$ JÈ>r go EH$ nÎmm `mÑÀN>`m {ZH$mbm OmVm h¡Ÿ&
{ZH$mbm J`m nÎmm EH$ BŠH$m Zht hmoZo H$s àm{`H$Vm h¡ … 1
1 9 4 12
(a) (b) (c) (d)
13 13 13 13

30/4/2 2
GENERAL INSTRUCTIONS :
Read the following instructions carefully and follow them :
(i) This question paper contains 38 questions. All questions are compulsory.
(ii) Question paper is divided into FIVE sections – Section A, B, C, D and E.
(iii) In section A – question number 1 to 18 are multiple choice questions (MCQs) and
question number 19 and 20 are Assertion-Reason based questions of 1 mark each.
(iv) In section B – question number 21 to 25 are Very Short Answer (VSA) type questions
of 2 marks each.
(v) In section C – question number 26 to 31 are Short Answer (SA) type questions
carrying 3 marks each.
(vi) In section D – question number 32 to 35 are Long Answer (LA) type questions
carrying 5 marks each.
(vii) In section E – question number 36 to 38 are case based integrated units of
assessment questions carrying 4 marks each. Internal choice is provided in 2 marks
question in each case-study.
(viii) There is no overall choice. However, an internal choice has been provided in 2
questions in Section B, 2 questions in Section C, 2 questions in Section D and 3
questions in Section E.
(ix) Draw neat figures wherever required. Take π = 22/7 wherever required if not stated.
(x) Use of calculators is NOT allowed.
SECTION - A
Section - A consists of Multiple Choice type questions of 1 mark each.
1. Which of the following is true for all values of θ (0o ≤ θ ≤ 90o) ? 1
(a) cos2 θ – sin2 θ = 1 (b) cosec2 θ – sec2 θ = 1
(c) sec2 θ – tan2 θ = 1 (d) cot2 θ – tan2 θ = 1
2. If k + 2, 4k – 6 and 3k – 2 are three consecutive terms of an A.P.,
then the value of k is : 1
(a) 3 (b) – 3 (c) 4 (d) – 4
3. In ∆ ABC, PQ || BC. If PB = 6 cm, AP = 4 cm, AQ = 8 cm, find the
length of AC. 1
(a) 12 cm
(b) 20 cm
(c) 6 cm
(d) 14 cm
4. The ratio of HCF to LCM of the least composite number and the least
prime number is : 1
(a) 1:2 (b) 2:1 (c) 1:1 (d) 1:3
5. A card is drawn at random from a well-shuffled pack of 52 cards. The
probability that the card drawn is not an ace is : 1
1 9 4 12
(a) (b) (c) (d)
13 13 13 13
30/4/2 3 [P.T.O.
6.

Xr JB© AmH¥${V _|, ∆ABC~∆QPRŸ& `{X AC = 6 go_r., BC = 5 go_r.,


QR = 3 go_r. Am¡a PR = x h¡, Vmo x H$m _mZ hmoJm … 1
(a) 3.6 go_r. (b) 2.5 go_r. (c) 10 go_r. (d) 3.2 go_r.
7. g_rH$aU x2 + 3x – 10 = 0 Ho$ _yb h¢ … 1
(a) 2, –5 (b) –2, 5 (c) 2, 5 (d) –2, –5
8. `{X 6 _r. D±$Mo EH$ I§^o H$s N>m`m, ^y{_ na 2 3 _r. b§~r h¡, Vmo gy`© H$m CÞVm§e
hmoJm … 1
o o o o
(a) 60 (b) 45 (c) 30 (d) 90
9. _yb {~ÝXw go {~ÝXw (– 6, 8) H$s Xyar h¡ … 1
(a) 6 (b) – 6 (c) 8 (d) 10
10. ‘d’ ì`mg Ho$ AY©d¥Îm H$m joÌ\$b Š`m h¡ ? 1
1 2 1 2 1 2 1 2
(a) πd (b) πd (c) πd (d) πd
16 4 8 2
11. {ZåZ ~§Q>Z Ho$ {bE _mÜ`H$ Am¡a ~hþbH$ dJm] H$s {ZMbr gr_mAm| H$m `moJ h¡ … 1
dJ© A§Vamb 0-5 5-10 10-15 15-20 20-25
~ma§~maVm 10 15 12 20 9
(a) 15 (b) 25 (c) 30 (d) 35
12. 9 go_r. {ÌÁ`m Ho$ EH$ d¥Îm Ho$ Ho$ÝÐ go 41 go_r. Xya pñWV EH$ q~Xw go d¥Îm na ItMr JB©
ñne© aoIm H$s b§~mB© h¡ … 1
(a) 40 go_r. (b) 9 go_r. (c) 41 go_r. (d) 50 go_r.
13. Xr JB© AmH¥${V _|, O d¥Îm H$m Ho$ÝÐ Am¡a PQ d¥Îm H$s Ordm h¡Ÿ& `{X P na ñne© aoIm
PR, Ordm PQ go 50o H$m H$moU ~ZmVr h¡, Vmo ∠POQ H$s _mn h¡ … 1

(a) 50 o
(b) 40o
(c) 100 o
(d) 130 o
30/4/2 4
6.

In the given figure, ∆ABC~∆QPR. If AC = 6 cm, BC = 5 cm,


QR = 3 cm and PR = x; then the value of x is : 1
(a) 3.6 cm (b) 2.5 cm (c) 10 cm (d) 3.2 cm
7. The roots of the equation x2 + 3x – 10 = 0 are : 1
(a) 2, –5 (b) –2, 5 (c) 2, 5 (d) –2, –5
8. If a pole 6 m high casts a shadow 2 3 m long on the ground, then
sun’s elevation is : 1
(a) 60o (b) 45 o (c) 30 o (d) 90 o
9. The distance of the point (– 6, 8) from origin is : 1
(a) 6 (b) – 6 (c) 8 (d) 10
10. What is the area of a semi-circle of diameter ‘d’ ? 1
1 2 1 2 1 1
(a) πd (b) πd (c) πd 2 (d) πd 2
16 4 8 2
11. For the following distribution : 1
Class 0-5 5-10 10-15 15-20 20-25
Frequency 10 15 12 20 9
The sum of lower limits of median class and modal class is :
(a) 15 (b) 25 (c) 30 (d) 35
12. The length of tangent drawn to a circle of radius 9 cm from a point
41 cm from the centre is : 1
(a) 40 cm (b) 9 cm (c) 41 cm (d) 50 cm
13. In the given figure, O is the centre of the circle and PQ is the chord.
If the tangent PR at P makes an angle of 50o with PQ, then the
measure of ∠POQ is : 1
(a) 50 o
(b) 40o
(c) 100 o
(d) 130 o

30/4/2 5 [P.T.O.
14. EH$ W¡bo _| 5 bmb J|X Am¡a "n' har J|X h¢Ÿ& `{X har J|X {ZH$mbZo H$s àm{`H$Vm, bmb
J|X {ZH$mbZo H$s àm{`H$Vm H$s VrZ JwZm hmo, Vmo "n' H$m _mZ h¡ … 1
(a) 18 (b) 15 (c) 10 (d) 20
15. `{X α, β {ÛKmV ~hþnX x –1 Ho$ eyÝ`m§H$ h¢, Vmo (α + β) H$m _mZ hmoJm …
2
1
(a) 2 (b) 1 (c) –1 (d) 0

16. `{X α, β ~hþnX p(x) = 4x2 – 3x – 7 Ho$ eyÝ`m§H$ h¢ Vmo  1 + 1  H$m _mZ h¡ … 1
 α β 
7 −7 3 −3
(a) (b) (c) (d)
3 3 7 7
17. g_rH$aU `w½_ … 2x = 5y + 6 Am¡a 15y = 6x – 18 {OZ Xmo aoImAm| H$mo {Zê${nV
H$aVm h¡, do h¢ … 1
(a) à{VÀN>oXr aoImE§ (b) g_m§Va aoImE§
(c) g§nmVr aoImE§ (d) à{VÀN>oXr `m g_m§Va aoImE§
18. {~§Xw (–1, 7) H$s x-Aj go Xyar h¡ … 1
(a) –1 (b) 7 (c) 6 (d) 50
{ZX}e … àíZ g§»`m 19 VWm 20 _| EH$ A{^H$WZ (A) Ho$ níMmV² EH$ VH©$-
H$WZ (R) {X`m h¡Ÿ& {ZåZ _| go ghr {dH$ën Mw{ZE …
19. A{^H$WZ (A) … a, b, c EH$ A.P. Ho$ nX hm|Jo AJa Am¡a Ho$db AJa 2b = a + c &
VH©$ (R) … nhbr "n' {df_ àmH¥$V g§»`mAm| H$m `moJ n2 h¡Ÿ& 1
(a) A{^H$WZ (A) VWm H$maU (R) XmoZm| gË` h¢Ÿ& H$maU (R) A{^H$WZ (A) H$s
ì`m»`m H$aVm h¡Ÿ&
(b) A{^H$WZ (A) VWm H$maU (R) XmoZm| gË` h¢Ÿ& H$maU (R) A{^H$WZ (A) H$s
ì`m»`m Zht H$aVm h¡Ÿ&
(c) A{^H$WZ (A) gË` h¡ naÝVw H$maU (R) AgË` h¡Ÿ&
(d) A{^H$WZ (A) AgË` h¡ O~{H$ H$maU (R) gË` h¡Ÿ&
2
20. A{^H$WZ (A) … EH$ brn-df© _| 53 a{ddma hmoZo H$s àm{`H$Vm h¡Ÿ&
7
5
VH©$ (R) … EH$ J¡a brn-df© _| 53 a{ddma hmoZo H$s àm{`H$Vm
h¡Ÿ& 1
7
(a) A{^H$WZ (A) VWm H$maU (R) XmoZm| gË` h¢Ÿ& H$maU (R) A{^H$WZ (A) H$s
ì`m»`m H$aVm h¡Ÿ&
(b) A{^H$WZ (A) VWm H$maU (R) XmoZm| gË` h¢Ÿ& H$maU (R) A{^H$WZ (A) H$s
ì`m»`m Zht H$aVm h¡Ÿ&
(c) A{^H$WZ (A) gË` h¡ naÝVw H$maU (R) AgË` h¡Ÿ&
(d) A{^H$WZ (A) AgË` h¡ O~{H$ H$maU (R) gË` h¡Ÿ&

30/4/2 6
14. A bag contains 5 red balls and n green balls. If the probability of
drawing a green ball is three times that of a red ball, then the value of
n is : 1
(a) 18 (b) 15 (c) 10 (d) 20
15. If α, β are zeroes of the polynomial x2–1, then value of (α + β) is : 1
(a) 2 (b) 1 (c) –1 (d) 0
16. If α, β are the zeroes of the polynomial p(x) = 4x2 – 3x – 7, then
1 1
 α + β  is equal to : 1
 
7 −7 3 −3
(a) (b) (c) (d)
3 3 7 7
17. The pair of linear equations 2x = 5y + 6 and 15y = 6x – 18 represents
two lines which are : 1
(a) intersecting (b) parallel
(c) coincident (d) either intersecting or parallel
18. The distance of the point (–1, 7) from x-axis is : 1
(a) –1 (b) 7 (c) 6 (d) 50
DIRECTIONS : In the question number 19 and 20, a statement of
Assertion (A) is followed by a statement of Reason (R). Choose the
correct option out of the following :
19. Assertion (A) : a, b, c are in A.P. if and only if 2b = a + c.
Reason (R) : The sum of first n odd natural numbers is n2. 1
(a) Both Assertion (A) and Reason (R) are true and Reason (R) is
the correct explanation of Assertion (A).
(b) Both Assertion (A) and Reason (R) are true and Reason (R) is
not the correct explanation of Assertion (A).
(c) Assertion (A) is true but Reason (R) is false.
(d) Assertion (A) is false but Reason (R) is true.
2
20. Assertion (A) : The probability that a leap year has 53 Sundays is .
7
5
Reason (R) : The probability that a non-leap year has 53 Sundays is . 1
7
(a) Both Assertion (A) and Reason (R) are true and Reason (R) is
the correct explanation of Assertion (A).
(b) Both Assertion (A) and Reason (R) are true and Reason (R) is
not the correct explanation of Assertion (A).
(c) Assertion (A) is true but Reason (R) is false.
(d) Assertion (A) is false but Reason (R) is true.
30/4/2 7 [P.T.O.
IÊS> - I
IÊS> - I _| A{V bKw-CÎma (VSA) àH$ma Ho$ àíZ h¢Ÿ& àË`oH$ àíZ Ho$ 2 A§H$ h¢Ÿ&
5 1
21. (A) _mZ kmV H$s{OE … 2 o
+ 2 o
– cot2 45o + 2 sin290o 2
cot 30 sin 60
AWdm
(B) `{X Ý`yZ H$moU θ Ho$ {bE sin θ = cos θ h¡, Vmo tan2 θ + cot2 θ – 2 H$m _mZ
kmV H$s{OEŸ& 2
22. EH$ {Zînj {gŠH$m Xmo ~ma CN>mbm OmVm h¡Ÿ& A{YH$ go A{YH$ EH$ {MV AmZo H$s
àm{`H$Vm kmV H$s{OEŸ& 2
23. Xmo g§»`mE§ 2…3 Ho$ AZwnmV _| h¢ Am¡a CZH$m LCM 180 h¡Ÿ& BZ g§»`mAm| H$m HCF
Š`m hmoJm ? 2
24. (A) {ÛKmV g_rH$aU 2x2 – 9x + 4 = 0 Ho$ _ybm| H$m `moJ Am¡a JwUZ\$b kmV H$s{OEŸ& 2
AWdm
2
(B) {ÛKmV g_rH$aU 4x – 5 = 0 H$m {d{dº$H$a (discriminant) kmV H$s{OE Am¡a
g_rH$aU Ho$ _ybm| H$s àH¥${V {b{IEŸ& 2
25. `{X ~hþnX p(x) = 6x2 + 37x – (k – 2) H$m EH$ eyÝ`m§H$ Xygao eyÝ`m§H$ H$m ì`wËH«$_
hmo, Vmo k H$m _mZ kmV H$s{OEŸ& 2

IÊS> - J
IÊS> - J _| bKw-CÎma (SA) àH$ma Ho$ àíZ h¢ Am¡a àË`oH$ àíZ 3 A§H$ H$m h¡Ÿ&
26. Xmo g§H|$Ðr` d¥Îmm| H$s {ÌÁ`mE§ 5 go_r. Am¡a 3 go_r. h¢Ÿ& ~‹S>o d¥Îm H$s Ordm, Omo N>moQ>o d¥Îm
H$mo ñne© H$aVr h¡, H$s b§~mB© kmV H$s{OEŸ& 3
27. {gÕ H$s{OE {H$ {H$gr ~mø {~§Xw go {H$gr d¥Îm na ItMr JB© Xmo ñne© aoImAm| Ho$ ~rM H$m
H$moU ñne© q~XwAm| H$mo {_bmZo dmbo aoImIÊS> Ûmam Ho$ÝÐ na A§V[aV H$moU H$m g§nyaH$ hmoVm
h¡Ÿ& 3
28. ‘p’ H$m _mZ kmV H$s{OE {OgHo$ {bE {ÛKmV g_rH$aU px(x – 2) + 6 = 0 Ho$ Xmo
~am~a dmñV{dH$ _yb hm|Ÿ& 3

29. (A) EH$ A.P. Ho$ nhbo 15 nXm| H$m `moJ 750 Am¡a nhbm nX 15 h¡Ÿ& BgH$m 20dm§ nX
kmV H$s{OEŸ& 3

AWdm
(B) amohZ 1,000 é. H$s nhbr {H$ñV go ha _hrZo ^wJVmZ H$aHo$ AnZm Hw$b
1,18,000 é. H$m G$U MwH$mVm h¡Ÿ& `{X dh ha _hrZo {H$ñV H$s am{e _| 100 é.
H$s d¥{Õ H$aVm h¡, Vmo 30dt {H$ñV H$s am{e Š`m hmoJr ? 30dt {H$ñV XoZo na G$U
H$s {H$VZr am{e MwH$Vm hmo OmEJrŸ? 3
30/4/2 8
SECTION – B
Section - B consists of Very Short Answer (VSA) type questions of 2
marks each.
5 1
21. (A) Evaluate : 2 o
+ 2 o
– cot245o + 2 sin290o 2
cot 30 sin 60
OR
(B) If θ is an acute angle and sin θ = cos θ, find the value of
tan2 θ + cot2 θ – 2. 2
22. If a fair coin is tossed twice, find the probability of getting ‘atmost
one head’. 2
23. Two numbers are in the ratio 2 : 3 and their LCM is 180. What is the
HCF of these numbers ? 2
24. (A) Find the sum and product of the roots of the quadratic equation
2x2 – 9x + 4 = 0. 2
OR
(B) Find the discriminant of the quadratic equation 4x2 – 5 = 0 and
hence comment on the nature of roots of the equation. 2
25. If one zero of the polynomial p(x) = 6x2 + 37x – (k – 2) is reciprocal
of the other, then find the value of k. 2
SECTION – C
Section - C consists of Short Answer (SA) type questions of 3 marks
each.
26. Two concentric circles are of radii 5 cm and 3 cm. Find the length of
the chord of the larger circle which touches the smaller circle. 3
27. Prove that the angle between the two tangents drawn from an external
point to a circle is supplementary to the angle subtended by the line-
segment joining the points of contact at the centre. 3
28. Find the value of ‘p’ for which the quadratic equation
px(x – 2) + 6 = 0 has two equal real roots. 3
29. (A) The sum of first 15 terms of an A.P. is 750 and its first term is
15. Find its 20th term. 3
OR
(B) Rohan repays his total loan of ` 1,18,000 by paying every month
starting with the first instalment of ` 1,000. If he increases the
instalment by ` 100 every month, what amount will be paid by
him in the 30th instalment ? What amount of loan has he paid
after 30th instalment ? 3

30/4/2 9 [P.T.O.
30. {gÕ H$s{OE {H$ 3 EH$ An[a_o` g§»`m h¡Ÿ& 3
sin A − 2 sin 3A
31. (A) {gÕ H$s{OE {H$ = tan A 3
2cos3A − cos A
AWdm
(B) {gÕ H$s{OE {H$ sec A (1 – sin A) (sec A + tan A) = 1 3

IÊS> - K
IÊS> - K _| XrK©-CÎma (LA) àH$ma Ho$ àíZ h¢Ÿ& àË`oH$ àíZ Ho$ 5 A§H$ h¢Ÿ&
32. 20 go_r. D±$MmB© Am¡a 12 go_r. ì`mg dmbo EH$ R>mog ~obZ go 8 go_r. D±$MmB© Am¡a
6 go_r. {ÌÁ`m dmbr EH$ e§ŠdmH$ma Jwhm H$mo ImoIbm {H$`m OmVm h¡Ÿ& eof R>mog H$m Hw$b
n¥ð>r` joÌ\$b kmV H$s{OEŸ& 5
33. EH$ hmCqgJ gmogm`Q>r Ho$ 200 n[admam| _| XyY na _m{gH$ IM© XO© {H$E Omo ZrMo {XE JE
h¢ …
_m{gH$ IM© 1000- 1500- 2000- 2500- 3000- 3500- 4000- 4500-
(`. _|) 1500 2000 2500 3000 3500 4000 4500 5000
n[admam| H$s 24 40 33 x 30 22 16 7
g§»`m
x H$m _mZ kmV H$s{OE Am¡a XyY na _mÜ`H$ Am¡a _mÜ` IM© ^r kmV H$s{OEŸ& 5
34. (A) EH$ grYm amO_mJ© EH$ _rZma Ho$ nmX VH$ OmVm h¡Ÿ& EH$ 75 _r. D±$Mo Q>m°da Ho$
erf© na I‹S>m EH$ ì`{º$ Xmo H$mam| H$mo 30o Am¡a 60o Ho$ AdZ_Z H$moUm| na XoIVm
h¡, Omo Q>m°da Ho$ nmX H$s Amoa Om ahr h¢Ÿ& `{X EH$ H$ma Q>m°da Ho$ Cgr Va\$ Xygar
H$ma Ho$ R>rH$ nrN>o hmo, Vmo XmoZm| H$mam| Ho$ ~rM H$s Xyar kmV H$s{OEŸ& ( 3 = 1.73
br{OE) 5
AWdm
(B) 7 _r. D±$Mo ^dZ Ho$ {eIa go EH$ Ho$~b Q>m°da Ho$ {eIa H$m CÞ`Z H$moU 60o h¡
Am¡a BgHo$ nmX H$m AdZ_Z H$moU 30o h¡Ÿ& Q>m°da H$s D±$MmB© kmV H$s{OEŸ& 5

35. (A) Xr JB© AmH¥${V _|, ∠ADC = ∠BCA h¡Ÿ& {gÕ H$s{OE {H$ ∆ACB ~ ∆ADC Ÿ&
AV… BD H$s bå~mB© kmV H$s{OE, `{X AC = 8 go_r. VWm AD = 3 go_r. Ÿ& 5

AWdm
30/4/2 10
30. Prove that 3 is an irrational number. 3
sin A − 2 sin 3A
31. (A) Prove that = tan A 3
2cos3A − cos A
OR
(B) Prove that sec A (1 – sin A) (sec A + tan A) = 1. 3
SECTION – D
Section – D consists of Long Answer (LA) type questions of 5 marks each.
32. From a solid cylinder of height 20 cm and diameter 12 cm, a conical
cavity of height 8 cm and radius 6 cm is hallowed out. Find the total
surface area of the remaining solid. 5
33. The monthly expenditure on milk in 200 families of a Housing
Society is given below :
Monthly 1000- 1500- 2000- 2500- 3000- 3500- 4000- 4500-
Expenditure 1500 2000 2500 3000 3500 4000 4500 5000
(in `)
Number of 24 40 33 x 30 22 16 7
families
Find the value of x and also, find the median and mean expenditure
on milk. 5
34. (A) A straight highway leads to the foot of a tower. A man standing
on the top of the 75 m high tower observes two cars at angles of
depression of 30o and 60o, which are approaching the foot of the
tower. If one car is exactly behind the other on the same side of
the tower, find the distance between the two cars. (use 3 = 1.73) 5
OR
(B) From the top of a 7 m high building, the angle of elevation of the
top of a cable tower is 60o and the angle of depression of its foot
is 30o. Determine the height of the tower. 5
35. (A) In the given figure, ∠ADC = ∠BCA ; prove that
∆ACB ~ ∆ADC. Hence find BD if AC = 8 cm and AD = 3 cm. 5

OR
30/4/2 11 [P.T.O.
(B) `{X {H$gr {Ì^wO H$s EH$ ^wOm Ho$ g_m§Va AÝ` Xmo ^wOmAm| H$mo {^Þ-{^Þ q~XwAm|
na à{VÀN>oX H$aZo Ho$ {bE EH$ aoIm ItMr OmE, Vmo {gÕ H$s{OE {H$ `o AÝ` Xmo
^wOmE§ EH$ hr AZwnmV _| {d^m{OV hmo OmVr h¢Ÿ& 5

IÊS> - ‹S>
IÊS> - ‹S> _| àH$aU AÜ``Z/n[aÀN>oX AmYm[aV 3 àíZ h¢Ÿ& àË`oH$ àíZ Ho$ 4 A§H$ h¢Ÿ&
36. OJXre Ho$ nmg EH$ IoV h¡ Omo EH$ g_H$moU {Ì^wO AQC Ho$ AmH$ma H$m h¡Ÿ& dh IoV
Ho$ A§Xa EH$ dJm©H$ma PQRS Ho$ ê$n _| Johÿ± CJmZo Ho$ {bE Am¡a eof gpãO`m± CJmZo Ho$
{bE ({MÌ _| Xem©`o AZwgma) OJh N>mo‹S>Zm MmhVm h¡Ÿ& IoV _|, O Ho$ ê$n _| {M{•V EH$
I§^m h¡Ÿ&

Cnamoº$ gyMZm Ho$ AmYma na, {ZåZ{b{IV àíZm| Ho$ CÎma Xr{OE …
(i) O H$mo _yb q~Xw _mZ H$a, {~§XwAm| P Am¡a Q Ho$ {ZX}em§H$ H«$_e… (-200, 0) Am¡a
(200, 0) h¢Ÿ& PQRS EH$ dJ© hmoZo Ho$ H$maU, R Am¡a S Ho$ {ZX}em§H$ Š`m hm|JoŸ? 1

(ii) (a) dJ© PQRS H$m joÌ\$b Š`m h¡ ? 2

AWdm
(b) dJ© PQRS _| {dH$U© PR H$s bå~mB© Š`m h¡Ÿ? 2

(iii) `{X q~Xw S, aoImIÊS> CA H$mo AZwnmV K:1 _| {d^m{OV H$ao, Vmo K H$m _mZ
Š`m hmoJm, `{X q~Xw A Ho$ {ZX}em§H$ (200, 800) hm| ? 1

30/4/2 12
(B) If a line is drawn parallel to one side of a triangle to intersect the
other two sides in distinct points, then prove that the other two
sides are divided in the same ratio. 5
SECTION – E
Section – E consists of three Case Study Based questions of 4 marks each.
36. Jagdish has a field which is in the shape of a right angled triangle
AQC. He wants to leave a space in the form of a square PQRS inside
the field for growing wheat and the remaining for growing vegetables
(as shown in the figure). In the field, there is a pole marked as O.

Based on the above information, answer the following questions :


(i) Taking O as origin, coordinates of P are (–200, 0) and of Q are
(200, 0). PQRS being a square, what are the coordinates of R
and S ? 1
(ii) (a) What is the area of square PQRS ? 2
OR
(b) What is the length of diagonal PR in square PQRS ? 2
(iii) If S divides CA in the ratio K:1, what is the value of K, where
point A is (200, 800) ? 1

30/4/2 13 [P.T.O.
37. XohamXyZ Ho$ EH$ ñWmZr` gmd©O{ZH$ {dH$mg àm{YH$aU H$s Jd{ZªJ H$mC§{gb Zo EH$ nhm‹S>r
H$s MmoQ>r na EH$ gmh{gH$ Iob H$m _¡XmZ ~ZmZo H$m \¡$gbm {H$`m, {Og_| nm{Hª$J Ho$ {bE
n`m©á OJh hmoJrŸ&

gd}jU Ho$ níMmV², Am`VmH$ma Iob H$m _¡XmZ ~ZmZo H$m {ZU©` {b`m J`m, {Og_| EH$
Va\$ nm{Hª$J Ho$ {bE EH$ AY©-d¥ÎmmH$ma joÌ A§{H$V hmoŸ& Am`VmH$ma Iob Ho$ _¡XmZ H$s
bå~mB© Am¡a Mm¡‹S>mB© H«$_e… 14 BH$mB© Am¡a 7 BH$mB© h¡Ÿ& Iob Ho$ _¡XmZ Ho$ EH$ Amoa
2 BH$mB© {ÌÁ`m Ho$ Xmo MVwWmªe {deof grQ>m| Ho$ {bE h¢Ÿ&
Cnamoº$ gyMZm Ho$ AmYma na, {ZåZ{b{IV àíZm| Ho$ CÎma Xr{OE …
(i) nm{Hª$J joÌ H$m Hw$b n[a_mn {H$VZm h¡ ? 1
(ii) (a) nm{Hª$J joÌ Am¡a Xmo MVwWmªem| H$mo {_bmH$a Hw$b joÌ\$b {H$VZm h¡ ? 2
AWdm
(b) Iob Ho$ _¡XmZ Ho$ joÌ\$b Am¡a nm{Hª$J joÌ Ho$ joÌ\$b _| Š`m AZwnmV h¡Ÿ? 2
(iii) Iob Ho$ _¡XmZ Am¡a nm{Hª$J joÌ Ho$ Mmam| Amoa Vma bJdmZo H$m IM©, 2 é. à{V
BH$mB© H$s Xa go kmV H$s{OEŸ& 1
38. Xmo ñHy$bm| ‘P’ Am¡a ‘Q’ Zo AnZo N>mÌm| H$mo hm°H$s ` x à{V N>mÌ
Am¡a {H«$Ho$Q> ` y à{V N>mÌ Xmo Iobm| Ho$ {bE nwañH$ma XoZo H$m
\¡$gbm {H$`mŸ& ñHy$b ‘P’ Zo Xmo Iobm| Ho$ {bE H«$_e… 5 Am¡a 4
N>mÌm| H$mo Hw$b 9,500 é. H$m nwañH$ma XoZo H$m \¡$gbm {H$`m,
O~{H$ ñHy$b ‘Q’ Zo Xmo Iobm| Ho$ {bE H«$_e… 4 Am¡a 3 N>mÌm|
H$mo Hw$b 7,370 é. H$m nwañH$ma XoZo H$m \¡$gbm {H$`mŸ&
Cnamoº$ gyMZm Ho$ AmYma na, {ZåZ{b{IV àíZm| Ho$ CÎma Xr{OE …
(i) Cnamoº$ gyMZm H$mo, Mam| x Am¡a y H$m à`moJ H$aHo$,
~rOJ{UVr` ê$n _| ì`º$ H$s{OEŸ& 1
(ii) (a) hm°H$s Ho$ {bE nwañH$ma am{e Š`m h¡ ? 2
AWdm
(b) nwañH$ma am{e {H$g Iob H$s A{YH$ h¡ Am¡a {H$VZr A{YH$ ? 2
(iii) `{X àË`oH$ Iob go 2 N>mÌ hm|, Vmo Hw$b nwañH$ma am{e Š`m hmoJr ? 1
30/4/2 14
37. Governing council of a local public development authority of
Dehradun decided to build an adventurous playground on the top of a
hill, which will have adequate space for parking.

After survey, it was decided to build rectangular playground, with a


semi-circular area allotted for parking at one end of the playground.
The length and breadth of the rectangular playground are 14 units and
7 units, respectively. There are two quadrants of radius 2 units on one
side for special seats.
Based on the above information, answer the following questions :
(i) What is the total perimeter of the parking area ? 1
(ii) (a) What is the total area of parking and the two quadrants ? 2
OR
(b) What is the ratio of area of playground to the area of
parking area ? 2
(iii) Find the cost of fencing the playground and parking area at the
rate of ` 2 per unit. 1
38. Two schools ‘P’ and ‘Q’ decided to award
prizes to their students for two games of
Hockey ` x per student and Cricket ` y per
student. School ‘P’ decided to award a total
of ` 9,500 for the two games to 5 and 4
students respectively; while school ‘Q’
decided to award ` 7,370 for the two games
to 4 and 3 students respectively.
Based on the above information, answer the following questions :
(i) Represent the following information algebraically (in terms of x
and y). 1
(ii) (a) What is the prize amount for hockey ? 2
OR
(b) Prize amount on which game is more and by how much ? 2
(iii) What will be the total prize amount if there are 2 students each
from two games ? 1
30/4/2 15 [P.T.O.
30/4/2 16
SET ~ 3
Series WX1YZ/4
àíZ-nÌ H$moS>
Q.P. Code 30/4/3
amob Z§. narjmWu àíZ-nÌ H$moS> H$mo CÎma-nwpñVH$m Ho$
Roll No. _wI-n¥ð> na Adí` {bI|Ÿ&
Candidates must write the Q.P. Code
on the title page of the answer-book.

J{UV (_mZH$) - g¡ÕmpÝVH$


MATHEMATICS (STANDARD) - Theory
#

{ZYm©[aV g_` … 3 KÊQ>o A{YH$V_ A§H$ … 80


Time allowed : 3 hours Maximum Marks : 80

ZmoQ> / NOTE :
(i) H¥$n`m Om±M H$a b| {H$ Bg àíZ-nÌ _| _w{ÐV n¥ð> 15 h¢Ÿ&
Please check that this question paper contains 15 printed pages.
(ii) àíZ-nÌ _| Xm{hZo hmW H$s Amoa {XE JE àíZ-nÌ H$moS> H$mo N>mÌ CÎma-nwpñVH$m Ho$ _wI-n¥ð> na
{bI|&
Q.P. Code given on the right hand side of the question paper should be written on
the title page of the answer-book by the candidate.
(iii) H¥$n`m Om±M H$a b| {H$ Bg àíZ-nÌ _| 38 àíZ h¢&
Please check that this question paper contains 38 questions.
(iv) H¥$n`m àíZ H$m CÎma {bIZm ewê$ H$aZo go nhbo, CÎma-nwpñVH$m _| àíZ H$m H«$_m§H$ Adí`
{bI|&
Please write down the Serial Number of the question in the answer-book before
attempting it.
(v) Bg àíZ-nÌ H$mo n‹T>Zo Ho$ {bE 15 {_ZQ> H$m g_` {X`m J`m h¡& àíZ-nÌ H$m {dVaU nydm©• _|
10.15 ~Oo {H$`m OmEJm& 10.15 ~Oo go 10.30 ~Oo VH$ N>mÌ Ho$db àíZ-nÌ H$mo n‹T>|Jo Am¡a
Bg Ad{Y Ho$ Xm¡amZ do CÎma-nwpñVH$m na H$moB© CÎma Zht {bI|Jo&
15 minute time has been allotted to read this question paper. The question paper
will be distributed at 10.15 a.m. From 10.15 a.m. to 10.30 a.m., the students will
read the question paper only and will not write any answer on the answer-book
during this period.

30/4/3 ~~~~ 1 [P.T.O.


gm_mÝ` {ZX}e …
{ZåZ{b{IV {ZX}em| H$mo ~hþV gmdYmZr go n{‹T>E Am¡a CZH$m nmbZ H$s{OE …
(i) Bg àíZ-nÌ _| 38 àíZ h¢Ÿ& g^r àíZ A{Zdm`© h¢Ÿ&
(ii) àíZ-nÌ nm§M IÊS>m| _| {d^m{OV h¡ - IÊS> H$, I, J, K VWm ‹S>Ÿ&
(iii) IÊS> H$ _| àíZ g§»`m 1 go 18 VH$ ~hþ{dH$ënr` VWm àíZ g§»`m 19 Ed§ 20 A{^H$WZ Ed§
H$maU AmYm[aV EH$-EH$ A§H$ Ho$ àíZ h¢Ÿ&
(iv) IÊS> I _| àíZ g§»`m 21 go 25 VH$ A{V bKw-CÎmar` (VSA) àH$ma Ho$ Xmo-Xmo A§H$m| Ho$ àíZ h¢Ÿ&
(v) IÊS> J _| àíZ g§»`m 26 go 31 VH$ bKw-CÎmar` (SA) àH$ma Ho$ VrZ-VrZ A§H$m| Ho$
àíZ h¢Ÿ&
(vi) IÊS> K _| àíZ g§»`m 32 go 35 VH$ XrK©-CÎmar` (LA) àH$ma Ho$ nm§M-nm§M A§H$m| Ho$ àíZ h¢Ÿ&
(vii) IÊS> ‹S> _| àíZ g§»`m 36 go 38 VH$ òmoV / àH$aU BH$mB© AmYm[aV Mma-Mma A§H$m| Ho$ àíZ h¢Ÿ&
Am§V[aH$ {dH$ën Xmo-Xmo A§H$m| Ho$ àíZ _| {X`m J`m h¡Ÿ&
(viii) àíZ-nÌ _| g_J« {dH$ën Zht {X`m J`m h¡Ÿ& `Ú{n, IÊS> I Ho$ 2 àíZm| _|, IÊS> J Ho$ 2 àíZm| _|,
IÊS> K Ho$ 2 àíZm| _| VWm IÊS> ‹S> Ho$ 3 àíZm| _| Am§V[aH$ {dH$ën H$m àmdYmZ {X`m J`m h¡Ÿ&
(ix) Ohm§ Amdí`H$ hmo, ñdÀN> AmH¥${V`m§ ~ZmE§Ÿ& `{X Amdí`H$ hmo Vmo π=22/7 b|Ÿ&
(x) H¡$bHw$boQ>a H$m Cn`moJ d{O©V h¡Ÿ&
IÊS> - H$
IÊS> - H$ _| ~hþ{dH$ënr` àH$ma Ho$ àíZ h¢Ÿ& àË`oH$ àíZ 1 A§H$ H$m h¡Ÿ&
1. g_rH$aU `w½_ … 2x = 5y + 6 Am¡a 15y = 6x – 18 {OZ Xmo aoImAm| H$mo {Zê${nV
H$aVm h¡, do h¢ … 1
(a) à{VÀN>oXr aoImE§ (b) g_m§Va aoImE§
(c) g§nmVr aoImE§ (d) à{VÀN>oXr `m g_m§Va aoImE§
2. Xr JB© AmH¥${V _|, Ho$ÝÐ O dmbo EH$ d¥Îm na TA EH$ ñne© aoIm h¡ Ohm± OT = 4 go_r.
Am¡a ∠OTA = 30o & TA H$s bå~mB© h¡ … 1
(a) 2 3 go_r.
(b) 2 go_r.
(c) 2 2 go_r.
(d) 3 go_r.
3. g~go N>moQ>r g§`wº$ g§»`m Am¡a g~go N>moQ>r A^mÁ` g§»`m Ho$ HCF H$m AZwnmV CZHo$
LCM go h¡ … 1
(a) 1:2 (b) 2:1 (c) 1:1 (d) 1:3
4. `{X 6 _r. D±$Mo EH$ I§^o H$s N>m`m, ^y{_ na 2 3 _r. b§~r h¡, Vmo gy`© H$m CÞVm§e
hmoJm … 1
o o o o
(a) 60 (b) 45 (c) 30 (d) 90
30/4/3 2
GENERAL INSTRUCTIONS :
Read the following instructions carefully and follow them :
(i) This question paper contains 38 questions. All questions are compulsory.
(ii) Question paper is divided into FIVE sections – Section A, B, C, D and E.
(iii) In section A – question number 1 to 18 are multiple choice questions (MCQs) and
question number 19 and 20 are Assertion-Reason based questions of 1 mark each.
(iv) In section B – question number 21 to 25 are Very Short Answer (VSA) type questions
of 2 marks each.
(v) In section C – question number 26 to 31 are Short Answer (SA) type questions
carrying 3 marks each.
(vi) In section D – question number 32 to 35 are Long Answer (LA) type questions
carrying 5 marks each.
(vii) In section E – question number 36 to 38 are case based integrated units of
assessment questions carrying 4 marks each. Internal choice is provided in 2 marks
question in each case-study.
(viii) There is no overall choice. However, an internal choice has been provided in 2
questions in Section B, 2 questions in Section C, 2 questions in Section D and 3
questions in Section E.
(ix) Draw neat figures wherever required. Take π = 22/7 wherever required if not stated.
(x) Use of calculators is NOT allowed.

SECTION - A
Section - A consists of Multiple Choice type questions of 1 mark each.
1. The pair of linear equations 2x = 5y + 6 and 15y = 6x – 18 represents
two lines which are : 1
(a) intersecting (b) parallel
(c) coincident (d) either intersecting or parallel
2. In the given figure, TA is a tangent to the circle with centre O such
that OT = 4 cm, ∠OTA = 30o, then length of TA is : 1
(a) 2 3 cm
(b) 2 cm
(c) 2 2 cm
(d) 3 cm

3. The ratio of HCF to LCM of the least composite number and the least
prime number is : 1
(a) 1:2 (b) 2:1 (c) 1:1 (d) 1:3

4. If a pole 6 m high casts a shadow 2 3 m long on the ground, then


sun’s elevation is : 1
(a) 60o (b) 45 o (c) 30 o (d) 90 o
30/4/3 3 [P.T.O.
5.

Xr JB© AmH¥${V _|, ∆ABC ~ ∆QPRŸ& `{X AC = 6 go_r., BC = 5 go_r.,


QR = 3 go_r. Am¡a PR = x h¡, Vmo x H$m _mZ hmoJm … 1
(a) 3.6 go_r. (b) 2.5 go_r. (c) 10 go_r. (d) 3.2 go_r.
6. _yb {~ÝXw go {~ÝXw (– 6, 8) H$s Xyar h¡ … 1
(a) 6 (b) – 6 (c) 8 (d) 10
7. A.P. : 7, 28, 63 H$m AJbm nX h¡ … 1
(a) 70 (b) 80 (c) 97 (d) 112

8. (sec2 θ – 1) (cosec2 θ – 1) ~am~a h¡ … 1


(a) – 1 (b) 1 (c) 0 (d) 2
9. Xmo nmgm| H$mo EH$ gmW \|$H$m J`mŸ& nmgm| Ho$ D$nar \$bH$m| na AmB© g§»`mAm| H$m AÝVa 3
hmoZo H$s àm{`H$Vm h¡ … 1
1 2 1 1
(a) (b) (c) (d)
9 9 6 12
10. 52 nÎmm| H$s AÀN>r àH$ma go \|$Q>r JB© EH$ JÈ>r go EH$ nÎmm `mÑÀN>`m {ZH$mbm OmVm h¡Ÿ&
{ZH$mbm J`m nÎmm EH$ BŠH$m Zht hmoZo H$s àm{`H$Vm h¡ … 1
1 9 4 12
(a) (b) (c) (d)
13 13 13 13
11. g_rH$aU x2 + 3x – 10 = 0 Ho$ _yb h¢ … 1
(a) 2, –5 (b) –2, 5 (c) 2, 5 (d) –2, –5
12. `{X α, β {ÛKmV ~hþnX x –1 Ho$ eyÝ`m§H$ h¢, Vmo (α + β) H$m _mZ hmoJm …
2
1
(a) 2 (b) 1 (c) –1 (d) 0

13. `{X α, β ~hþnX p(x) = 4x2 – 3x – 7 Ho$ eyÝ`m§H$ h¢ Vmo  1 + 1  H$m _mZ h¡ … 1
α β 
7 −7 3 −3
(a) (b) (c) (d)
3 3 7 7
14. ‘d’ ì`mg Ho$ AY©d¥Îm H$m joÌ\$b Š`m h¡ ? 1
1 2 1 2 1 2 1 2
(a) πd (b) πd (c) πd (d) πd
16 4 8 2
30/4/3 4
5.

In the given figure, ∆ABC ~ ∆QPR. If AC = 6 cm, BC = 5 cm,


QR = 3 cm and PR = x; then the value of x is : 1
(a) 3.6 cm (b) 2.5 cm (c) 10 cm (d) 3.2 cm
6. The distance of the point (– 6, 8) from origin is : 1
(a) 6 (b) – 6 (c) 8 (d) 10
7. The next term of the A.P. : 7, 28, 63 is : 1
(a) 70 (b) 80 (c) 97 (d) 112
8. (sec2 θ – 1) (cosec2 θ – 1) is equal to : 1
(a) – 1 (b) 1 (c) 0 (d) 2
9. Two dice are thrown together. The probability of getting the
difference of numbers on their upper faces equals to 3 is : 1
1 2 1 1
(a) (b) (c) (d)
9 9 6 12
10. A card is drawn at random from a well-shuffled pack of 52 cards. The
probability that the card drawn is not an ace is : 1
1 9 4 12
(a) (b) (c) (d)
13 13 13 13
11. The roots of the equation x2 + 3x – 10 = 0 are : 1
(a) 2, –5 (b) –2, 5 (c) 2, 5 (d) –2, –5
12. If α, β are zeroes of the polynomial x2–1, then value of (α + β) is : 1
(a) 2 (b) 1 (c) –1 (d) 0
13. If α, β are the zeroes of the polynomial p(x) = 4x2 – 3x – 7, then
1 1
 α + β  is equal to : 1
 
7 −7 3 −3
(a) (b) (c) (d)
3 3 7 7
14. What is the area of a semi-circle of diameter ‘d’ ? 1
1 2 1 2 1 1
(a) πd (b) πd (c) πd 2 (d) πd 2
16 4 8 2
30/4/3 5 [P.T.O.
15. {ZåZ ~§Q>Z Ho$ {bE …
àmám§H$ 10 go 20 go 30 go 40 go 50 go 60 go
H$_ H$_ H$_ H$_ H$_ H$_
N>mÌm| H$s g§»`m 3 12 27 57 75 80
~hþbH$ dJ© h¡ … 1
(a) 10-20 (b) 20-30 (c) 30-40 (d) 50-60
16. Xr JB© AmH¥${V _|, Ho$ÝÐ O dmbo d¥Îm Ho$ q~Xw T na PT EH$ ñne© aoIm h¡Ÿ& `{X
∠TPO = 25o h¡, Vmo x H$m _mZ h¡ … 1
o
(a) 25
(b) 65 o
(c) 90 o
(d) 115 o
17. Xr JB© AmH¥${V _|, PQ || AC Ÿ& `{X BP = 4 go_r., AP = 2.4 go_r. Am¡a
BQ = 5 go_r., Vmo BC H$s bå~mB© hmoJr … 1
(a) 8 go_r.

(b) 3 go_r.

(c) 0.3 go_r.

25
(d) go_r.
3
18. q~Xw (– 4, 0), (4, 0) Am¡a (0, 3) erf© h¢ EH$ … 1
(a) g_H$moU {Ì^wO Ho$ (b) g_{Û~mhþ {Ì^wO Ho$
(c) g_~mhþ {Ì^wO Ho$ (d) {df_ {Ì^wO Ho$
{ZX}e … àíZ g§»`m 19 VWm 20 _| EH$ A{^H$WZ (A) Ho$ níMmV² EH$ VH©$-
H$WZ (R) {X`m h¡Ÿ& {ZåZ _| go ghr {dH$ën Mw{ZE …
2
19. A{^H$WZ (A) … EH$ brn-df© _| 53 a{ddma hmoZo H$s àm{`H$Vm h¡Ÿ&
7
5
VH©$ (R) … EH$ J¡a brn-df© _| 53 a{ddma hmoZo H$s àm{`H$Vm
h¡Ÿ& 1
7
(a) A{^H$WZ (A) VWm H$maU (R) XmoZm| gË` h¢Ÿ& H$maU (R) A{^H$WZ (A) H$s
ì`m»`m H$aVm h¡Ÿ&
(b) A{^H$WZ (A) VWm H$maU (R) XmoZm| gË` h¢Ÿ& H$maU (R) A{^H$WZ (A) H$s
ì`m»`m Zht H$aVm h¡Ÿ&
(c) A{^H$WZ (A) gË` h¡ naÝVw H$maU (R) AgË` h¡Ÿ&
(d) A{^H$WZ (A) AgË` h¡ O~{H$ H$maU (R) gË` h¡Ÿ&
30/4/3 6
15. For the following distribution :
Marks Below 10 20 30 40 50 60
Number of Students 3 12 27 57 75 80
The modal class is : 1
(a) 10-20 (b) 20-30 (c) 30-40 (d) 50-60
16. In the given figure, PT is a tangent at T to the circle with centre O. If
∠TPO = 25o, then x is equal to : 1
o
(a) 25
(b) 65 o
(c) 90 o
(d) 115 o
17. In the given figure, PQ || AC. If BP = 4 cm, AP = 2.4 cm and
BQ = 5 cm, then length of BC is : 1
(a) 8 cm
(b) 3 cm
(c) 0.3 cm
25
(d) cm
3
18. The points (– 4, 0), (4, 0) and (0, 3) are the vertices of a : 1
(a) right triangle (b) isosceles triangle
(c) equilateral triangle (d) scalene triangle
DIRECTIONS : In the question number 19 and 20, a statement of
Assertion (A) is followed by a statement of Reason (R). Choose the
correct option out of the following :
2
19. Assertion (A) : The probability that a leap year has 53 Sundays is .
7
5
Reason (R) : The probability that a non-leap year has 53 Sundays is . 1
7
(a) Both Assertion (A) and Reason (R) are true and Reason (R) is
the correct explanation of Assertion (A).
(b) Both Assertion (A) and Reason (R) are true and Reason (R) is
not the correct explanation of Assertion (A).
(c) Assertion (A) is true but Reason (R) is false.
(d) Assertion (A) is false but Reason (R) is true.

30/4/3 7 [P.T.O.
20. A{^H$WZ (A) … a, b, c EH$ A.P. Ho$ nX hm|Jo AJa Am¡a Ho$db AJa 2b = a + c &
VH©$ (R) … nhbr "n' {df_ àmH¥$V g§»`mAm| H$m `moJ n2 h¡Ÿ& 1
(a) A{^H$WZ (A) VWm H$maU (R) XmoZm| gË` h¢Ÿ& H$maU (R) A{^H$WZ (A) H$s
ì`m»`m H$aVm h¡Ÿ&
(b) A{^H$WZ (A) VWm H$maU (R) XmoZm| gË` h¢Ÿ& H$maU (R) A{^H$WZ (A) H$s
ì`m»`m Zht H$aVm h¡Ÿ&
(c) A{^H$WZ (A) gË` h¡ naÝVw H$maU (R) AgË` h¡Ÿ&
(d) A{^H$WZ (A) AgË` h¡ O~{H$ H$maU (R) gË` h¡Ÿ&

IÊS> - I
IÊS> - I _| A{V bKw-CÎma (VSA) àH$ma Ho$ àíZ h¢Ÿ& àË`oH$ àíZ Ho$ 2 A§H$ h¢Ÿ&
21. (A) {ÛKmV g_rH$aU 2x2 – 9x + 4 = 0 Ho$ _ybm| H$m `moJ Am¡a JwUZ\$b kmV H$s{OEŸ& 2
AWdm
2
(B) {ÛKmV g_rH$aU 4x – 5 = 0 H$m {d{dº$H$a (discriminant) kmV H$s{OE Am¡a
g_rH$aU Ho$ _ybm| H$s àH¥${V {b{IEŸ& 2

22. (A) _mZ kmV H$s{OE … 2sec2 θ + 3cosec2 θ – 2sin θcos θ, if θ = 45o. 2
AWdm
(B) `{X sin θ – cos θ = 0 h¡, Vmo sin4 θ + cos4 θ H$m _mZ kmV H$s{OEŸ& 2
23. EH$ {Zînj {gŠH$m Xmo ~ma CN>mbm OmVm h¡Ÿ& A{YH$ go A{YH$ EH$ {MV AmZo H$s
àm{`H$Vm kmV H$s{OEŸ& 2
24. Xmo g§»`mE§ 2…3 Ho$ AZwnmV _| h¢ Am¡a CZH$m LCM 180 h¡Ÿ& BZ g§»`mAm| H$m HCF
Š`m hmoJm ? 2
25. `{X ~hþnX p(x) = 6x2 + 37x – (k – 2) H$m EH$ eyÝ`m§H$ Xygao eyÝ`m§H$ H$m ì`wËH«$_
hmo, Vmo k H$m _mZ kmV H$s{OEŸ& 2

IÊS> - J
IÊS> - J _| bKw-CÎma (SA) àH$ma Ho$ àíZ h¢ Am¡a àË`oH$ àíZ 3 A§H$ H$m h¡Ÿ&
26. ‘p’ H$m _mZ kmV H$s{OE {OgHo$ {bE {ÛKmV g_rH$aU px2 – 14x + 8 = 0 H$m EH$
_yb Xygao _yb H$m N>… JwZm h¡Ÿ& 3
27. EH$ ~mø q~Xw go, {H$gr d¥Îm na Xmo ñne© aoImE§ ItMr JBªŸ& {gÕ H$s{OE {H$ ~mø q~Xw
Am¡a d¥Îm Ho$ H|$Ð H$mo {_bmZo dmbr aoIm ñne© aoImAm| Ho$ ~rM Ho$ H$moU g_{Û^m{OV H$aVr
h¡Ÿ& 3
28. Xmo g§H|$Ðr` d¥Îmm| H$s {ÌÁ`mE§ 5 go_r. Am¡a 3 go_r. h¢Ÿ& ~‹S>o d¥Îm H$s Ordm, Omo N>moQ>o d¥Îm
H$mo ñne© H$aVr h¡, H$s b§~mB© kmV H$s{OEŸ& 3
29. (A) EH$ A.P. {OgH$m nhbm Am¡a nm±Mdm§ nX H«$_e… -14 Am¡a 2 h¡ VWm ApÝV_ nX
62 h¡, Vmo A.P. _| {H$VZo nX h¢ ? 3
30/4/3 8
20. Assertion (A) : a, b, c are in A.P. if and only if 2b = a + c.
Reason (R) : The sum of first n odd natural numbers is n2. 1
(a) Both Assertion (A) and Reason (R) are true and Reason (R) is
the correct explanation of Assertion (A).
(b) Both Assertion (A) and Reason (R) are true and Reason (R) is
not the correct explanation of Assertion (A).
(c) Assertion (A) is true but Reason (R) is false.
(d) Assertion (A) is false but Reason (R) is true.
SECTION – B
Section - B consists of Very Short Answer (VSA) type questions of 2
marks each.
21. (A) Find the sum and product of the roots of the quadratic equation
2x2 – 9x + 4 = 0. 2
OR
(B) Find the discriminant of the quadratic equation 4x2 – 5 = 0 and
hence comment on the nature of roots of the equation. 2
22. (A) Evaluate 2sec2 θ + 3cosec2 θ – 2sin θcos θ if θ = 45o. 2
OR
(B) If sin θ – cos θ = 0, then find the value of sin4 θ + cos4 θ. 2
23. If a fair coin is tossed twice, find the probability of getting ‘atmost
one head’. 2
24. Two numbers are in the ratio 2 : 3 and their LCM is 180. What is the
HCF of these numbers ? 2
25. If one zero of the polynomial p(x) = 6x2 + 37x – (k – 2) is reciprocal
of the other, then find the value of k. 2
SECTION – C
Section - C consists of Short Answer (SA) type questions of 3 marks each.
26. Find the value of ‘p’ for which one root of the quadratic equation
px2 – 14x + 8 = 0 is 6 times the other. 3
27. From an external point, two tangents are drawn to a circle. Prove that
the line joining the external point to the centre of the circle bisects the
angle between the two tangents. 3
28. Two concentric circles are of radii 5 cm and 3 cm. Find the length of
the chord of the larger circle which touches the smaller circle. 3
29. (A) How many terms are there in A.P. whose first and fifth term are
– 14 and 2, respectively and the last term is 62. 3

30/4/3 9 [P.T.O.
AWdm
(B) A.P. : 65, 61, 57, 53, .......... H$m H$m¡Zgm nX g~go nhbm G$UmË_H$ nX hmoJm? 3
sin A − 2 sin 3A
30. (A) {gÕ H$s{OE {H$ = tan A 3
2cos3A − cos A
AWdm
(B) {gÕ H$s{OE {H$ sec A (1 – sin A) (sec A + tan A) = 1 3
31. {gÕ H$s{OE {H$ 5 EH$ An[a_o` g§»`m h¡Ÿ& 3
IÊS> - K
IÊS> - K _| XrK©-CÎma (LA) àH$ma Ho$ àíZ h¢Ÿ& àË`oH$ àíZ Ho$ 5 A§H$ h¢Ÿ&
32. (A) EH$ {Ì^wO PQR _| N, ^wOm PR na EH$ Eogm q~Xw h¡ {OgHo$ {bE QN ⊥ PR
h¡Ÿ& `{X PN × NR = QN2 h¡, Vmo {gÕ H$s{OE {H$ ∠PQR = 90o . 5
AWdm
(B) Xr JB© AmH¥${V _|, {Ì^wO ABC Am¡a {Ì^wO
DBC EH$ hr AmYma BC na h¢Ÿ&
`{X AD, BC H$mo q~Xw O na H$mQ>o, Vmo
{gÕ H$s{OE {H$
∆ABC H$m joÌ\$b AO
= 5
∆DBC H$m joÌ\$b DO

33. bH$‹S>r Ho$ EH$ R>mog ~obZ Ho$ àË`oH$ {gao na EH$ AY©Jmobm ImoXH$a {ZH$mbVo
hþE EH$ dñVw ~ZmB© JB© h¡, O¡gm AmH¥${V _| {XIm`m J`m h¡Ÿ& `{X ~obZ H$s
D±$MmB© 10 go_r. h¡ Am¡a AmYma H$s {ÌÁ`m 3.5 go_r. h¡, Vmo Bg dñVw H$m
g§nyU© n¥ð>r` joÌ\$b kmV H$s{OEŸ& 5

34. EH$ hmCqgJ gmogm`Q>r Ho$ 200 n[admam| _| XyY na _m{gH$ IM© XO© {H$E Omo
ZrMo {XE JE h¢ …
_m{gH$ IM© 1000- 1500- 2000- 2500- 3000- 3500- 4000- 4500-
(`. _|) 1500 2000 2500 3000 3500 4000 4500 5000
n[admam| H$s 24 40 33 x 30 22 16 7
g§»`m
x H$m _mZ kmV H$s{OE Am¡a XyY na _mÜ`H$ Am¡a _mÜ` IM© ^r kmV H$s{OEŸ& 5

30/4/3 10
OR
(B) Which term of the A.P. : 65, 61, 57, 53, .................. is the first
negative term ? 3
sin A − 2 sin 3A
30. (A) Prove that = tan A 3
2cos3A − cos A
OR
(B) Prove that sec A (1 – sin A) (sec A + tan A) = 1. 3
31. Prove that 5 is an irrational number. 3
SECTION – D
Section - D consists of Long Answer (LA) type questions of 5 marks each.
32. (A) In a ∆PQR, N is a point on PR, such that
QN ⊥ PR. If PN × NR = QN2, prove that ∠PQR = 90o. 5
OR
(B) In the given figure, ∆ABC and
∆DBC are on the same base BC.
If AD intersects BC at O, prove
ar (∆ABC) AO
that = 5
ar (∆DBC) DO

33. A wooden article was made by scooping out a hemisphere from


each end of a solid cylinder, as shown in the figure. If the height
of the cylinder is 10 cm and its base is of radius 3.5 cm, find the
total surface area of the article. 5

34. The monthly expenditure on milk in 200 families of a Housing


Society is given below :
Monthly 1000- 1500- 2000- 2500- 3000- 3500- 4000- 4500-
Expenditure 1500 2000 2500 3000 3500 4000 4500 5000
(in `)
Number of 24 40 33 x 30 22 16 7
families
Find the value of x and also, find the median and mean expenditure
on milk. 5

30/4/3 11 [P.T.O.
35. (A) EH$ grYm amO_mJ© EH$ _rZma Ho$ nmX VH$ OmVm h¡Ÿ& EH$ 75 _r. D±$Mo Q>m°da Ho$ erf©
na I‹S>m EH$ ì`{º$ Xmo H$mam| H$mo 30o Am¡a 60o Ho$ AdZ_Z H$moUm| na XoIVm h¡, Omo
Q>m°da Ho$ nmX H$s Amoa Om ahr h¢Ÿ& `{X EH$ H$ma Q>m°da Ho$ Cgr Va\$ Xygar H$ma Ho$
R>rH$ nrN>o hmo, Vmo XmoZm| H$mam| Ho$ ~rM H$s Xyar kmV H$s{OEŸ& ( 3 = 1.73 br{OE) 5
AWdm
(B) 7 _r. D±$Mo ^dZ Ho$ {eIa go EH$ Ho$~b Q>m°da Ho$ {eIa H$m CÞ`Z H$moU 60o h¡
Am¡a BgHo$ nmX H$m AdZ_Z H$moU 30o h¡Ÿ& Q>m°da H$s D±$MmB© kmV H$s{OEŸ& 5
IÊS> - ‹S>
IÊS> - ‹S> _| àH$aU AÜ``Z/n[aÀN>oX AmYm[aV 3 àíZ h¢Ÿ& àË`oH$ àíZ Ho$ 4 A§H$ h¢Ÿ&
36. XohamXyZ Ho$ EH$ ñWmZr` gmd©O{ZH$ {dH$mg àm{YH$aU H$s Jd{ZªJ H$mC§{gb Zo EH$ nhm‹S>r
H$s MmoQ>r na EH$ gmh{gH$ Iob H$m _¡XmZ ~ZmZo H$m \¡$gbm {H$`m, {Og_| nm{Hª$J Ho$ {bE
n`m©á OJh hmoJrŸ&

gd}jU Ho$ níMmV², Am`VmH$ma Iob H$m _¡XmZ ~ZmZo H$m {ZU©` {b`m J`m, {Og_| EH$
Va\$ nm{Hª$J Ho$ {bE EH$ AY©-d¥ÎmmH$ma joÌ A§{H$V hmoŸ& Am`VmH$ma Iob Ho$ _¡XmZ H$s
bå~mB© Am¡a Mm¡‹S>mB© H«$_e… 14 BH$mB© Am¡a 7 BH$mB© h¡Ÿ& Iob Ho$ _¡XmZ Ho$ EH$ Amoa
2 BH$mB© {ÌÁ`m Ho$ Xmo MVwWmªe {deof grQ>m| Ho$ {bE h¢Ÿ&
Cnamoº$ gyMZm Ho$ AmYma na, {ZåZ{b{IV àíZm| Ho$ CÎma Xr{OE …
(i) nm{Hª$J joÌ H$m Hw$b n[a_mn {H$VZm h¡ ? 1
(ii) (a) nm{Hª$J joÌ Am¡a Xmo MVwWmªem| H$mo {_bmH$a Hw$b joÌ\$b {H$VZm h¡ ? 2
AWdm
(b) Iob Ho$ _¡XmZ Ho$ joÌ\$b Am¡a nm{Hª$J joÌ Ho$ joÌ\$b _| Š`m AZwnmV h¡Ÿ? 2
(iii) Iob Ho$ _¡XmZ Am¡a nm{Hª$J joÌ Ho$ Mmam| Amoa Vma bJdmZo H$m IM©, 2 é. à{V
BH$mB© H$s Xa go kmV H$s{OEŸ& 1
37. Xmo ñHy$bm| ‘P’ Am¡a ‘Q’ Zo AnZo N>mÌm| H$mo hm°H$s ` x à{V N>mÌ
Am¡a {H«$Ho$Q> ` y à{V N>mÌ Xmo Iobm| Ho$ {bE nwañH$ma XoZo H$m
\¡$gbm {H$`mŸ& ñHy$b ‘P’ Zo Xmo Iobm| Ho$ {bE H«$_e… 5 Am¡a 4
N>mÌm| H$mo Hw$b 9,500 é. H$m nwañH$ma XoZo H$m \¡$gbm {H$`m,
O~{H$ ñHy$b ‘Q’ Zo Xmo Iobm| Ho$ {bE H«$_e… 4 Am¡a 3 N>mÌm| H$mo
Hw$b 7,370 é. H$m nwañH$ma XoZo H$m \¡$gbm {H$`mŸ&

30/4/3 12
35. (A) A straight highway leads to the foot of a tower. A man standing
on the top of the 75 m high tower observes two cars at angles of
depression of 30o and 60o, which are approaching the foot of the
tower. If one car is exactly behind the other on the same side of
the tower, find the distance between the two cars. (use 3 = 1.73) 5
OR
(B) From the top of a 7 m high building, the angle of elevation of the
top of a cable tower is 60o and the angle of depression of its foot
is 30o. Determine the height of the tower. 5
SECTION – E
Section – E consists of three Case Study Based questions of 4 marks each.
36. Governing council of a local public development authority of
Dehradun decided to build an adventurous playground on the top of a
hill, which will have adequate space for parking.

After survey, it was decided to build rectangular playground, with a


semi-circular area allotted for parking at one end of the playground.
The length and breadth of the rectangular playground are 14 units and
7 units, respectively. There are two quadrants of radius 2 units on one
side for special seats.
Based on the above information, answer the following questions :
(i) What is the total perimeter of the parking area ? 1
(ii) (a) What is the total area of parking and the two quadrants ? 2
OR
(b) What is the ratio of area of playground to the area of
parking area ? 2
(iii) Find the cost of fencing the playground and parking area at the
rate of ` 2 per unit. 1
37. Two schools ‘P’ and ‘Q’ decided to award prizes
to their students for two games of Hockey ` x per
student and Cricket ` y per student. School ‘P’
decided to award a total of ` 9,500 for the two
games to 5 and 4 students respectively; while
school ‘Q’ decided to award ` 7,370 for the two
games to 4 and 3 students respectively.
30/4/3 13 [P.T.O.
àmá gyMZm Ho$ AmYma na, {ZåZ{b{IV àíZm| Ho$ CÎma Xr{OE …
(i) Cnamoº$ gyMZm H$mo, Mam| x Am¡a y H$m à`moJ H$aHo$, ~rOJ{UVr` ê$n _| ì`º$
H$s{OEŸ& 1
(ii) (a) hm°H$s Ho$ {bE nwañH$ma am{e Š`m h¡ ? 2
AWdm
(b) nwañH$ma am{e {H$g Iob H$s A{YH$ h¡ Am¡a {H$VZr A{YH$ ? 2
(iii) `{X àË`oH$ Iob go 2 N>mÌ hm|, Vmo Hw$b nwañH$ma am{e Š`m hmoJr ? 1

38. OJXre Ho$ nmg EH$ IoV h¡ Omo EH$ g_H$moU {Ì^wO AQC Ho$ AmH$ma H$m h¡Ÿ& dh IoV
Ho$ A§Xa EH$ dJm©H$ma PQRS Ho$ ê$n _| Johÿ± CJmZo Ho$ {bE Am¡a eof gpãO`m± CJmZo Ho$
{bE ({MÌ _| Xem©`o AZwgma) OJh N>mo‹S>Zm MmhVm h¡Ÿ& IoV _|, O Ho$ ê$n _| {M{•V EH$
I§^m h¡Ÿ&

Cnamoº$ gyMZm Ho$ AmYma na, {ZåZ{b{IV àíZm| Ho$ CÎma Xr{OE …
(i) O H$mo _yb q~Xw _mZ H$a, {~§XwAm| P Am¡a Q Ho$ {ZX}em§H$ H«$_e… (-200, 0) Am¡a
(200, 0) h¢Ÿ& PQRS EH$ dJ© hmoZo Ho$ H$maU, R Am¡a S Ho$ {ZX}em§H$ Š`m hm|JoŸ? 1
(ii) (a) dJ© PQRS H$m joÌ\$b Š`m h¡ ? 2
AWdm
(b) dJ© PQRS _| {dH$U© PR H$s bå~mB© Š`m h¡Ÿ? 2
(iii) `{X q~Xw S, aoImIÊS> CA H$mo AZwnmV K:1 _| {d^m{OV H$ao, Vmo K H$m _mZ
Š`m hmoJm, `{X q~Xw A Ho$ {ZX}em§H$ (200, 800) hm| ? 1

30/4/3 14
Based on the given information, answer the following questions :
(i) Represent the following information algebraically (in terms of x
and y). 1
(ii) (a) What is the prize amount for hockey ? 2
OR
(b) Prize amount on which game is more and by how much ? 2
(iii) What will be the total prize amount if there are 2 students each
from two games ? 1
38. Jagdish has a field which is in the shape of a right angled triangle
AQC. He wants to leave a space in the form of a square PQRS inside
the field for growing wheat and the remaining for growing vegetables
(as shown in the figure). In the field, there is a pole marked as O.

Based on the above information, answer the following questions :


(i) Taking O as origin, coordinates of P are (–200, 0) and of Q are
(200, 0). PQRS being a square, what are the coordinates of R
and S ? 1
(ii) (a) What is the area of square PQRS ? 2
OR
(b) What is the length of diagonal PR in square PQRS ? 2
(iii) If S divides CA in the ratio K:1, what is the value of K, where
point A is (200, 800) ? 1

30/4/3 15 [P.T.O.
30/4/3 16
Series WX1YZ/5 SET~1

Q.P. Code 30/5/1


Roll No. narjmWu àíZ-nÌ H$moS> >H$mo CÎma-nwpñVH$m Ho$
_wI-n¥ð >na Adí` {bIo§ &
Candidates must write the Q.P. Code on
the title page of the answer-book.

J{UV (_mZH$)$
MATHEMATICS (STANDARD)
*
:3 : 80
Time allowed : 3 hours Maximum Marks : 80

NOTE :
(i) - 27
Please check that this question paper contains 27 printed pages.
(ii) - - -
-
Q.P. Code given on the right hand side of the question paper should be written on the title
page of the answer-book by the candidate.
(iii) - 38
Please check that this question paper contains 38 questions.
(iv) -

Please write down the serial number of the question in the answer-book before
attempting it.
(v) - 15 -
10.15 10.15 10.30 -
-
15 minute time has been allotted to read this question paper. The question paper will be
distributed at 10.15 a.m. From 10.15 a.m. to 10.30 a.m., the students will read the
question paper only and will not write any answer on the answer-book during this period.

30/5/1 JJJJ Page 1 P.T.O.


:
:
(i) 38
(ii)
(iii) 1 18 (MCQ) 19 20

(iv) 21 25 (VSA)

(v) 26 31 (SA)
(vi) 32 35 (LA)
(vii) 36 38

(viii) 2 2
2 3

22
(ix) =
7

(x)

IÊS> H$
(MCQ) 1

1. eyÝ`H$ 3 Am¡a 5 dmbo ~hþnXm| H$s g§»`m h¡ :


(a) Ho$db EH$ AZÝV$
(b)
(c) R>rH$ Xmo (d) A{YH$-go-A{YH$ Xmo
2. g_rH$aU `w½_ ax + 2y = 9 Am¡a 3x + by = 18 g_m§Va aoImE± {Zê${nV H$aoJm, Ohm±
a, b nyUmªH$ h¡§, `{X :
(a) a=b (b) 3a = 2b
(c) 2a = 3b (d) ab = 6
3. EH$ A.P., {OgH$m ndm± nX an = 3n + 7 Ûmam {X`m J`m h¡, H$m gmd© A§Va hmoJm :
(a) 7 (b) 3
(c) 3n (d) 1
30/5/1 JJJJ Page 2
General Instructions :
Read the following instructions carefully and follow them :
(i) This question paper contains 38 questions. All questions are compulsory.
(ii) This question paper is divided into five Sections A, B, C, D and E.
(iii) In Section A, Questions no. 1 to 18 are multiple choice questions (MCQs) and
questions number 19 and 20 are Assertion-Reason based questions of 1 mark
each.
(iv) In Section B, Questions no. 21 to 25 are very short answer (VSA) type
questions, carrying 2 marks each.
(v) In Section C, Questions no. 26 to 31 are short answer (SA) type questions,
carrying 3 marks each.
(vi) In Section D, Questions no. 32 to 35 are long answer (LA) type questions
carrying 5 marks each.
(vii) In Section E, Questions no. 36 to 38 are case study based questions carrying
4 marks each. Internal choice is provided in 2 marks questions in each
case-study.
(viii) There is no overall choice. However, an internal choice has been provided in
2 questions in Section B, 2 questions in Section C, 2 questions in Section D and
3 questions in Section E.
22
(ix) Draw neat diagrams wherever required. Take = wherever required, if not
7
stated.
(x) Use of calculators is not allowed.

SECTION A
This section comprises multiple choice questions (MCQs) of 1 mark each.
1. The number of polynomials having zeroes 3 and 5 is :
(a) only one (b) infinite
(c) exactly two (d) at most two
2. The pair of equations ax + 2y = 9 and 3x + by = 18 represent parallel
lines, where a, b are integers, if :
(a) a=b (b) 3a = 2b
(c) 2a = 3b (d) ab = 6
3. The common difference of the A.P. whose nth term is given by an = 3n + 7,
is :
(a) 7 (b) 3
(c) 3n (d) 1
30/5/1 JJJJ Page 3 P.T.O.
4. Xr JB© AmH¥${V _|, DE BC. x H$m _mZ h¡ :

(a) 6 (b ) 12·5
(c) 8 (d) 10
5. _yb (2 + 3) Am¡a (2 3) dmbm EH$ {ÛKmV g_rH$aU h¡ :
(a) x2 4x + 1 = 0 (b) x2 + 4x + 1 = 0
(c) 4x2 3=0 (d) x2 1=0

5 sin cos
6. `{X tan = h¡, Vmo H$m _mZ h¡ :
12 sin cos
17 17
(a) (b)
7 7
17 7
(c) (d)
13 13

11 2
7. {~ÝXþAm| P ,5 Am¡a Q ,5 Ho$ ~rM H$s Xÿar h¡ :
3 3
(a) 6 BH$mB© (b) 4 BH$mB©
(c) 2 BH$mB© (d) 3 BH$mB©
8. Xr JB© AmH¥${V _|, AB = BC = 10 cm & `{X AC = 7 cm h¡, Vmo BP H$s bå~mB© hmoJr :

(a) 3·5 cm (b) 7 cm


(c) 6·5 cm (d) 5 cm
30/5/1 JJJJ Page 4
4. In the given figure, DE BC. The value of x is :

(a) 6 (b ) 12·5
(c) 8 (d) 10
5. A quadratic equation whose roots are (2 + 3 ) and (2 3 ) is :
(a) x2 4x + 1 = 0 (b) x2 + 4x + 1 = 0
(c) 4x2 3=0 (d) x2 1=0
5 sin cos
6. If tan = , then the value of is :
12 sin cos
17 17
(a) (b)
7 7
17 7
(c) (d)
13 13
11 2
7. The distance between the points P , 5 and Q , 5 is :
3 3
(a) 6 units (b) 4 units
(c) 2 units (d) 3 units
8. In the given figure, AB = BC = 10 cm. If AC = 7 cm, then the length of BP
is :

(a) 3·5 cm (b) 7 cm


(c) 6·5 cm (d) 5 cm
30/5/1 JJJJ Page 5 P.T.O.
9. 3 m Jhar Am¡a 40 m ZXr _| nmZr 2 km/h H$s Xa go ~h ahm h¡ & 2 {_ZQ> _| g_wÐ
_| {H$VZm nmZr {Ja OmEJm ?
(a) 800 m3 (b) 4000 m3
(c) 8000 m3 (d) 2000 m3

10. mÜ` Am¡a _mÜ`H$ H«$_e: 12 Am¡a 15 h¢, Vmo ~hþbH$ hmoJm :
(a) 13·5 (b) 21
(c) 6 (d) 14

11. Xr JB© AmH¥${V _|, Ho$ÝÐ O Ho$ EH$ d¥Îm na AB EH$ ñne©-aoIm h¡ & `{X OA = 6 cm Am¡a
OAB = 30 h¡, Vmo d¥Îm H$s {ÌÁ`m hmoJr :

(a) 3 cm (b) 3 3 cm

(c) 2 cm (d) 3 cm

2 tan 30
12. ~am~a h¡ :
1 tan 2 30

(a) sin 60 (b) cos 60

(c) tan 60 (d) sin 30

AB BC
13. {Ì^wOm| ABC Am¡a DEF _|, & {ZåZ{b{IV _| go H$m¡Z-gm BZ Xmo {Ì^wOm| H$mo
DE FD
g_ê$n ~ZmEJm ?

(a) A= D (b) B= D

(c) B= E (d) A= F

30/5/1 JJJJ Page 6


9. Water in a river which is 3 m deep and 40 m wide is flowing at the rate of
2 km/h. How much water will fall into the sea in 2 minutes ?

(a) 800 m3 (b) 4000 m3

(c) 8000 m3 (d) 2000 m3

10. If the mean and the median of a data are 12 and 15 respectively, then its
mode is :

(a) 13·5 (b) 21

(c) 6 (d) 14

11. In the given figure, AB is a tangent to the circle centered at O. If


OA = 6 cm and OAB = 30 , then the radius of the circle is :

(a) 3 cm (b) 3 3 cm

(c) 2 cm (d) 3 cm

2 tan 30
12. is equal to :
1 tan 2 30

(a) sin 60 (b) cos 60

(c) tan 60 (d) sin 30

AB BC
13. . Which of the following makes the two
DE FD
triangles similar ?

(a) A= D (b) B= D

(c) B= E (d) A= F

30/5/1 JJJJ Page 7 P.T.O.


14. A.P. : 10, 7, 4, ......., 62 _| A§{V_ nX go 11dm± nX h¡ :
(a) 25 (b) 16
(c) 32 (d) 0

15. -go-H$_ EH$ nQ> AmZo H$s àm{`H$Vm hmoJr :


1 1
(a) (b)
4 2
3
(c) (d) 1
4

16. Xr JB© AmH¥${V _|, Ho$ÝÐ O dmbo EH$ d¥Îm na AC Am¡a AB ñne©-aoImE± h¢ & `{X
COD = 120 h¡, Vmo BAO ~am~a h¡ :

(a) 30 (b) 60
(c) 45 (d) 90

17. {ZåZ{b{IV _| go H$m¡Z-gr g§»`m, {H$gr KQ>Zm Ho$ KQ>Zo H$s àm{`H$Vm hmo gH$Vr h¡ ?
7
(a) 0 (b)
0·01
0·07
(c) 0·07 (d)
3

18. `{X {H$gr gm§p»`H$s , {Og_| n nX h¢, Ho$ àË`oH$ nX H$mo 2 go H$_ H$a {X`m OmE,
:
(a) 2 H$_ hmo OmEJm
(b) An[ad{V©V ahoJm
(c) 2n H$_ hmo OmEJm
(d) 1 H$_ hmo OmEJm

30/5/1 JJJJ Page 8


14. The 11th term from the end of the A.P. : 10, 7, 4, ......., 62 is :
(a) 25 (b) 16
(c) 32 (d) 0
15. Two coins are tossed together. The probability of getting at least one tail
is :
1 1
(a) (b)
4 2
3
(c) (d) 1
4

16. In the given figure, AC and AB are tangents to a circle centered at O. If


COD = 120 , then BAO is equal to :

(a) 30 (b) 60
(c) 45 (d) 90
17. Which of the following numbers cannot be the probability of happening
of an event ?
7
(a) 0 (b)
0·01

0·07
(c) 0·07 (d)
3

18. If every term of the statistical data consisting of n terms is decreased by


2, then the mean of the data :
(a) decreases by 2
(b) remains unchanged
(c) decreases by 2n
(d) decreases by 1
30/5/1 JJJJ Page 9 P.T.O.
19 20 1
(A) (R)
(a), (b), (c) (d)
(a) A{^H$WZ (A) Am¡a VH©$ (R) XmoZm| ghr h¢ Am¡a VH©$ (R), A{^H$WZ (A) H$s ghr
ì¶m»¶m H$aVm h¡ &
(b) A{^H$WZ (A) Am¡a VH©$ (R) XmoZm| ghr h¢, naÝVw VH©$ (R), A{^H$WZ (A) H$s ghr
ì¶m»¶m H$aVm h¡ &
(c) A{^H$WZ (A) ghr h¡, naÝVw VH©$ (R) µJbV h¡ &
(d) A{^H$WZ (A) µJbV h¡, naÝVw VH©$ (R) ghr h¡ &

19. (A) : `{X Ho$ÝÐ O(2, 3) dmbo EH$ d¥Îm na q~Xþ A(4, 3) Am¡a B(x, 5) pñWV h¢,
Vmo x H$m _mZ 2 hmoJm &
(R) : d¥Îm H$s àË`oH$ Ordm H$m _Ü`-q~Xþ, d¥Îm H$m Ho$ÝÐ hmoVm h¡ &
n
20. (A) : g§»`m 5 H$^r ^r A§H$ eyÝ` (0) na g_má Zht hmoVr h¡, Ohm± n H$moB© ^r
àmH¥$V g§»`m h¡ &
(R) : 5 Ho$ A^mÁ` JwUZI§S>Z _| Ho$db Xmo hr JwUZI§S> h¢, 1 Am¡a 5 &

IÊS> I
(VSA) 2

21. (H$) {~ÝXþ A(4, 5) Am¡a B(4, 5) H$mo {_bmZo dmbo aoImI§S> H$mo {~ÝXþ P go Bg àH$ma
{d^m{OV {H$`m OmVm h¡ {H$ AP : AB = 2 : 5 h¡ & {~ÝXþ P Ho$ {ZX}em§H$ kmV
H$s{OE &
AWdm
(I) {~ÝXþ P(x, y), {~ÝXþAm| A(5, 1) VWm B(1, 5) go g_XÿañW h¡ & {gÕ H$s{OE {H$
x = y.
30/5/1 JJJJ Page 10
Questions number 19 and 20 are Assertion and Reason based questions carrying
1 mark each. Two statements are given, one labelled as Assertion (A) and the
other is labelled as Reason (R). Select the correct answer to these questions from
the codes (a), (b), (c) and (d) as given below.

(a) Both Assertion (A) and Reason (R) are true and Reason (R) is the
correct explanation of the Assertion (A).

(b) Both Assertion (A) and Reason (R) are true, but Reason (R) is not
the correct explanation of the Assertion (A).

(c) Assertion (A) is true, but Reason (R) is false.

(d) Assertion (A) is false, but Reason (R) is true.

19. Assertion (A) : If the points A(4, 3) and B(x, 5) lie on a circle with centre
O(2, 3), then the value of x is 2.
Reason (R) : Centre of a circle is the mid-point of each chord of the
circle.

n
20. Assertion (A) : The number 5 cannot end with the digit 0, where n is a
natural number.
Reason (R): Prime factorisation of 5 has only two factors, 1 and 5.

SECTION B

This section comprises very short answer (VSA) type questions of 2 marks each.

21. (a) The line segment joining the points A(4, 5) and B(4, 5) is divided
by the point P such that AP : AB = 2 : 5. Find the coordinates of P.

OR

(b) Point P(x, y) is equidistant from points A(5, 1) and B(1, 5). Prove
that x = y.

30/5/1 JJJJ Page 11 P.T.O.


22. Xr JB© AmH¥${V _|, Ho$ÝÐ O dmbo d¥Îm na PT EH$ ñne©-aoIm h¡ & Ordm AB na OC b§~dV
h¡ & {gÕ H$s{OE {H$ PA . PB = PC2 AC2.

23. A^mÁ` JwUZI§S>Z go 96 Am¡a 120 H$m HCF Am¡a LCM kmV H$s{OE &

24. {~ÝXþAm| (5, 6) Am¡a ( 1, 4) H$mo {_bmZo dmbo aoImI§S> H$mo y-Aj {Og AZwnmV _|
{d^m{OV H$aVm h¡, Cg AZwnmV H$mo kmV H$s{OE &

25. (H$) `{X a cos + b sin = m VWm a sin b cos =n h¡, Vmo {gÕ H$s{OE
{H$ a2 + b2 = m2 + n2.

AWdm
(I) {gÕ H$s{OE :
sec A 1 sec A 1
+ = 2 cosec A
sec A 1 sec A 1

IÊS> J

(SA) 3

26. (H$) {gÕ H$s{OE {H$ 3 EH$ An[a_o` g§»`m h¡ &


AWdm
(I) VrZ AbJ-AbJ amoS> H«$m°qgJ na Q´>¡{\$H$ bmBQ> H«$_e: àË`oH$ 48 goH$ÊS>,
72 goH$ÊS> Am¡a 108 goH$ÊS> Ho$ ~mX ~Xb OmVr h¢ & `{X do EH$ gmW gw~h 7 ~Oo
~XbVr h¢, Vmo do AmJo {H$g g_` EH$ gmW ~Xbo§Jr ?

30/5/1 JJJJ Page 12


22. In the given figure, PT is a tangent to the circle centered at O. OC is
2 2
perpendicular to chord AB. Prove that PA . PB = PC AC .

23. Using prime factorisation, find HCF and LCM of 96 and 120.

24. Find the ratio in which y-axis divides the line segment joining the points
(5, 6) and ( 1, 4).

25. (a) If a cos + b sin = m and a sin b cos = n, then prove that
a2 + b2 = m2 + n2.

OR
(b) Prove that :
sec A 1 sec A 1
+ = 2 cosec A
sec A 1 sec A 1

SECTION C

This section comprises short answer (SA) type questions of 3 marks each.

26. (a) Prove that 3 is an irrational number.

OR

(b) The traffic lights at three different road crossings change after
every 48 seconds, 72 seconds and 108 seconds respectively. If they
change simultaneously at 7 a.m., at what time will they change
together next ?

30/5/1 JJJJ Page 13 P.T.O.


27. `{X EH$ A.P. H$m pdm± nX q Am¡a qdm± nX p h¡, Vmo {gÕ H$s{OE {H$ BgH$m ndm± nX
(p + q n) hmoJm &

28. (H$) Xr JB© AmH¥${V _|, CD, AB H$m b§~ g_{Û^mOH$ h¡ & EF, CD Ho$ b§~dV h¡ &
CF FG
AE, CD H$mo G na H$mQ>Vr h¡ & {gÕ H$s{OE {H$ .
CD DG

AWdm
(I) Xr JB© AmH¥${V _|, ABCD EH$ g_m§Va MVw^w©O h¡ & BE, CD H$mo M na
g_{Û^m{OV H$aVr h¡ Am¡a AC H$mo L na H$mQ>Vr h¡ & {gÕ H$s{OE {H$
EL = 2BL.

29. 16 km H$s Xÿar na h¢ & do EH$ hr g_` na MbZm Amaå^


H$aVo h¢ & `{X do EH$-Xÿgao H$s Amoa {^Þ Mmb go MbVo h¢, Vmo 2 K§Q>o níMmV² {_bVo h¢ &
`{X do EH$ hr {Xem _| nhbo O¡gr Mmb go MbVo h¢, Vmo 8 K§Q>o níMmV² {_bVo h¢ & XmoZm|
ì`{º$`m| Ho$ MbZo H$s Mmb kmV H$s{OE &

30. {gÕ H$s{OE :


tan cot
+ = 1 + sec cosec
1 cot 1 tan

30/5/1 JJJJ Page 14


27. If pth term of an A.P. is q and qth term is p, then prove that its nth term is
(p + q n).

28. (a) In the given figure, CD is the perpendicular bisector of AB. EF is


CF FG
perpendicular to CD. AE intersects CD at G. Prove that .
CD DG

OR

(b) In the given figure, ABCD is a parallelogram. BE bisects CD at M


and intersects AC at L. Prove that EL = 2BL.

29. Two people are 16 km apart on a straight road. They start walking at the
same time. If they walk towards each other with different speeds, they
will meet in 2 hours. Had they walked in the same direction with same
speeds as before, they would have met in 8 hours. Find their walking
speeds.

30. Prove that :


tan cot
+ = 1 + sec cosec
1 cot 1 tan

30/5/1 JJJJ Page 15 P.T.O.


31. {ZåZ{b{IV ~ma§~maVm ~§Q>Z H$m _mÜ` kmV H$s{OE :

dJ© A§Vamb 25 30 30 35 35 40 40 45 45 50 50 55 55 60

~ma§~maVm 14 22 16 6 5 3 4

IÊS> K
(LA) 5

32. EH$ àojH$ EH$ J_© hdm Ho$ Jwã~mao H$s Q>moH$ar Ho$ CÞ`Z H$moU H$m AZw_mZ 60 bJmVm h¡,
O~{H$ EH$ AÝ` n`©dojH$ Omo 100 m Xÿa h¡, CÞ`Z H$moU H$m AZw_mZ 30 bJmVm h¡ &
kmV H$s{OE :
(H$) O_rZ go Q>moH$ar H$s D±$MmB© &
(I) nhbo àojH$ H$s Am±I go Q>moH$ar H$s$ Xÿar &
(J) Q>moH$ar go Xÿgao àojH$ H$s j¡{VO Xÿar &

33. (H$) 4 cm {ÌÁ`m dmbo EH$ d¥Îm Ho$ n[aJV EH$ {Ì^wO ABC Bg àH$ma ItMm J`m h¡ {H$
aoImI§S BD Am¡a DC H$s b§~mB`m± H«$_e: 10 cm Am¡a 8 cm h¢ & ^wOmE± AB Am¡a
AC H$s b§~mB`m± kmV H$r{OE, `{X {X`m J`m h¡ {H$ ABC H$m joÌ\$b
90 cm2 h¡ &

AWdm
30/5/1 JJJJ Page 16
31. Find the mean of the following frequency distribution :

Classes 25 30 30 35 35 40 40 45 45 50 50 55 55 60

Frequency 14 22 16 6 5 3 4

SECTION D

This section comprises long answer (LA) type questions of 5 marks each.

32. One observer estimates the angle of elevation to the basket of a hot air
balloon to be 60 , while another observer 100 m away estimates the angle
of elevation to be 30 . Find :

(a) The height of the basket from the ground.

(b) The distance of the basket

(c) The horizontal distance of the second observer from the basket.

33. (a) A triangle ABC is drawn to circumscribe a circle of radius 4 cm


such that the segments BD and DC are of lengths 10 cm and 8 cm
respectively. Find the lengths of the sides AB and AC, if it is given
2.

OR

30/5/1 JJJJ Page 17 P.T.O.


(I) Xmo d¥Îm h¢ {OZHo$ H|$Ð O Am¡a O h¢, Am¡a {ÌÁ`mE± H«$_e: 6 cm Am¡a 8 cm h¢ & Xmo
{~ÝXþAm| P Am¡a Q na do Bg àH$ma à{VÀN>oX H$aVo h¢ {H$ OP Am¡a O P Xmo d¥Îmm| H$s
ñne©-aoImE± h¢ & C^`{ZîR> Ordm PQ H$s bå~mB© kmV H$s{OE &

34. (H$) 54 km H$s Xÿar V` H$aVr h¡ Am¡a ~mX _|


63 km H$s Xÿar nhbr J{V go 6 km/h A{YH$ Am¡gVZ J{V go V` H$aVr h¡ & `{X
nyar `mÌm 3 K§Q>m| _| V` H$aVr h¡, Vmo BgH$s nhbr Am¡gVZ J{V ?

AWdm

15
(I) Xmo nmBn {_bH$a EH$ Q>¢H$ H$mo K§Q>m| _| ^a gH$Vo h¢ & A{YH$ ì`mg dmbm
8
nmBn, N>moQ>o ì`mg dmbo nmBn go, 2 K§Q>o H$_ _| Q>¢H$ H$mo ^a gH$Vm h¡ & XmoZm| nmBn
AbJ-AbJ Q>¢H$ H$mo {H$VZo g_` _| ^a gH$Vo h¢, kmV H$s{OE &

35. 15 m ^wOm dmbo EH$ dJm©H$ma Kmg Ho$ _¡XmZ Ho$ EH$ H$moZo na b H$mo
5m b§~r añgr go ~m±Y {X`m J`m h¡ & Bg _¡XmZ Ho$ Cg ^mJ H$m joÌ\$b,
Ma gH$Vm h¡, kmV H 10 m H$a Xr OmE Vmo Kmg Ma
gH$Zo dmbo joÌ\ ^r kmV H$s{OE & ( = 3.14 H$m à`moJ H$s{OE)

30/5/1 JJJJ Page 18


(b) Two circles with centres O and O of radii 6 cm and 8 cm,
respectively intersect at two points P and Q such that OP and O P
are tangents to the two circles. Find the length of the common
chord PQ.

34. (a) A train travels at a certain average speed for a distance of 54 km


and then travels a distance of 63 km at an average speed of 6 km/h
more than the first speed. If it takes 3 hours to complete the
journey, what was its first average speed ?

OR
15
(b) Two pipes together can fill a tank in hours. The pipe with
8
larger diameter takes 2 hours less than the pipe with smaller
diameter to fill the tank separately. Find the time in which each
pipe can fill the tank separately.

35. A horse is tied to a peg at one corner of a square shaped grass field of side
15 m by means of a 5 m long rope. Find the area of that part of the field
in which the horse can graze. Also, find the increase in grazing area if
length of rope is increased to 10 m. (Use = 3·14)

30/5/1 JJJJ Page 19 P.T.O.


IÊS> L>
3 4

àH$aU AÜ``Z 1

36. EH$ Jmoë\$ H$s J|X bJ^J 300 500 {S>ånb Ho$ gmW JmobmH$ma hmoVr h¡, Omo IobZo Ho$
Xm¡amZ CgHo$ doJ H$mo & Jmoë\$ H$s J|X nma§n[aH$ ê$n go g\o$X hmoVr h¡,
bo{H$Z a§Jm| _| ^r CnbãY hmoVr h¡ & Xr JB© AmH¥${V _|, EH$ Jmoë\$ H$s J|X H$m ì`mg
4·2 cm Am¡a BgH$s gVh na {ÌÁ`m 2 mm Ho$ 315 {S>ånb (AY©-JmobmH$ma) h¢ &

Cn`w©º$ Ho$ AmYma na, {ZåZ àíZm| Ho$ CÎma Xr{OE :


(i) EH$ Eogo {S>ånb H$m n¥îR>r` joÌ\$b kmV H$s{OE & 1

(ii) EH$ {S>ånb ~ZmZo Ho$ {bE ImoXr JB© gm_J«r H$m Am`VZ kmV H$s{OE & 1

(iii) (H$) n[adoe Ho$ g§nH©$ _| AmZo dmbm Hw$b n¥îR>r` joÌ\$b kmV H$s{OE & 2
AWdm
(iii) (I) Jmoë\$ H$s J§oX H$m Am`VZ kmV H$s{OE & 2

30/5/1 JJJJ Page 20


SECTION E

This section comprises 3 case study based questions of 4 marks each.

Case Study 1

36. A golf ball is spherical with about 300 500 dimples that help increase
its velocity while in play. Golf balls are traditionally white but available
in colours also. In the given figure, a golf ball has diameter
4·2 cm and the surface has 315 dimples (hemi-spherical) of radius 2 mm.

Based on the above, answer the following questions :


(i) Find the surface area of one such dimple. 1

(ii) Find the volume of the material dug out to make one dimple. 1

(iii) (a) Find the total surface area exposed to the surroundings. 2

OR

(iii) (b) Find the volume of the golf ball. 2

30/5/1 JJJJ Page 21 P.T.O.


àH$aU AÜ``Z 2

37. EH$ {_{S>b ñHy$b Zo {H«$g_g H$m{Z©db na {ZåZ{b{IV pñnZa Jo_ H$mo µ\$§S>-aoµOa Ho$ ê$n _|
MbmZo H$m {ZU©` {b`m &

: àË`oH$ pñnZa H$mo EH$ ~ma pñnZ H$s{OE & Zrbm Am¡a bmb, ~¢JZr ~ZmVo
h¢ & Bg{bE, `{X EH$ pñnZa bmb (R) Am¡a Xÿgam Zrbm (B) {XImVm h¡, Vmo Amn OrVVo
hmo & Eogo n[aUm_ na {bIm OmVm h¡ &
Cn`w©º$ Ho$ AmYma na, {ZåZ àíZm| Ho$ CÎma Xr{OE :
(i) Jo_ Ho$ g^r g§^d n[aUm_m| H$s gyMr ~ZmBE & 1

(ii) ~¢JZr a§J ~ZmZo H$s àm{`H$Vm kmV H$s{OE & 1

(iii) (H$) àË`oH$ OrV Ho$ {bE, à{V^mJr H$mo < 10 {_bVo h¢, bo{H$Z AJa dh hma
OmVm h¡, Vmo Cgo ñHy$b H$mo < 5 H$m ^wJVmZ H$aZm hmoJm &
`{X 99 à{V^m{J`m| Zo Jo_ Iobm hmo, Vmo ñHy$b Zo {H$VZm \§$S> EH$Ì {H$`m
hmoJm ? 2
AWdm
(iii) (I) `{X Iob _| OrV `m hma Ho$ {bE < 5 H$s g_mZ am{e V` H$s JB© h¡, Vmo
ñHy$b Zo {H$VZm \§$S> EH$Ì {H$`m hmoJm ? (à{V^m{J`m| H$s g§»`m = 99) 2

30/5/1 JJJJ Page 22


Case Study 2

37. A middle school decided to run the following spinner game as a


fund-raiser on Christmas Carnival.

Making Purple : Spin each spinner once. Blue and red make purple. So, if

Based on the above, answer the following questions :

(i) List all possible outcomes of the game. 1

(ii) 1

(iii) (a) For each win, a participant gets < 10, but if he/she loses,
he/she has to pay < 5 to the school.
If 99 participants played, calculate how much fund could the
school have collected. 2
OR
(iii) (b) If the same amount of < 5 has been decided for winning or
losing the game, then how much fund had been collected by
school ? (Number of participants = 99) 2

30/5/1 JJJJ Page 23 P.T.O.


àH$aU AÜ``Z 3
38. yb _|, EH$ S>m°pë\$Z 20 go_r à{V goH$ÊS> H$s J{V go `mÌm H$aVo hþE,
nmZr go ~mha Hy$XVr h¡ & t goH$ÊS> Ho$ ~mX Ob ñVa go CgH$s D±$MmB© h = 20t 16t2
Ûmam Xr OmVr h¡ &

Cn`w©º$ Ho$ AmYma na, {ZåZ àíZm| Ho$ CÎma Xr{OE :


(i) ~hþnX p(t) = 20t 16t2 Ho$ eyÝ`H$ kmV H$s{OE & 1
(ii) {ZåZ{b{IV _| go H$m¡Z-gm J«m\$ ~hþnX p(t) H$mo {Zê${nV H$aVm h¡ ? 1

30/5/1 JJJJ Page 24


Case Study 3

38. In a pool at an aquarium, a dolphin jumps out of the water travelling at


20 cm per second. Its height above water level after t seconds is given by
2
h = 20t 16t .

Based on the above, answer the following questions :


2
(i) Find zeroes of polynomial p(t) = 20t 16t . 1
(ii) Which of the following types of graph represents p(t) ? 1

30/5/1 JJJJ Page 25 P.T.O.


3
(iii) (H$) t= na h hmoJm ? n[aUm_ H$s ì`m»`m H$s{OE & 2
2
AWdm
(iii) (I) Xmo~mam Ob ñVa go Q>H$amZo go nhbo S>m°pë\$Z Zo {H$VZr Xÿar V` H$s h¡ ? 2

30/5/1 JJJJ Page 26


3
(iii) (a) What would be the value of h at t = ? Interpret the result. 2
2
OR

(iii) (b) How much distance has the dolphin covered before hitting
the water level again ? 2

30/5/1 JJJJ Page 27 P.T.O.


Series WX1YZ/5 SET~2

Q.P. Code 30/5/2


Roll No. narjmWu àíZ-nÌ H$moS> >H$mo CÎma-nwpñVH$m Ho$
_wI-n¥ð >na Adí` {bIo§ &
Candidates must write the Q.P. Code on
the title page of the answer-book.

J{UV (_mZH$)$
MATHEMATICS (STANDARD)
*
:3 : 80
Time allowed : 3 hours Maximum Marks : 80

NOTE :
(i) - 27
Please check that this question paper contains 27 printed pages.
(ii) - - -
-
Q.P. Code given on the right hand side of the question paper should be written on the title
page of the answer-book by the candidate.
(iii) - 38
Please check that this question paper contains 38 questions.
(iv) -

Please write down the serial number of the question in the answer-book before
attempting it.
(v) - 15 -
10.15 10.15 10.30 -
-
15 minute time has been allotted to read this question paper. The question paper will be
distributed at 10.15 a.m. From 10.15 a.m. to 10.30 a.m., the students will read the
question paper only and will not write any answer on the answer-book during this period.

30/5/2 JJJJ Page 1 P.T.O.


:
:
(i) 38
(ii)
(iii) 1 18 (MCQ) 19 20

(iv) 21 25 (VSA)

(v) 26 31 (SA)
(vi) 32 35 (LA)
(vii) 36 38

(viii) 2 2
2 3

22
(ix) =
7

(x)

IÊS> H$
(MCQ) 1

1 tan 2 30
1. ~am~a h¡ :
1 tan 2 30

(a) sin 60 (b) cos 60


(c) tan 60 (d) cos 30

AB BC
2. {Ì^wOm| ABC Am¡a DEF _|, & {ZåZ{b{IV _| go H$m¡Z-gm BZ Xmo {Ì^wOm| H$mo
DE FD
g_ê$n ~ZmEJm ?
(a) A= D (b) B= D
(c) B= E (d) A= F
30/5/2 JJJJ Page 2
General Instructions :
Read the following instructions carefully and follow them :
(i) This question paper contains 38 questions. All questions are compulsory.
(ii) This question paper is divided into five Sections A, B, C, D and E.
(iii) In Section A, Questions no. 1 to 18 are multiple choice questions (MCQs) and
questions number 19 and 20 are Assertion-Reason based questions of 1 mark
each.
(iv) In Section B, Questions no. 21 to 25 are very short answer (VSA) type
questions, carrying 2 marks each.
(v) In Section C, Questions no. 26 to 31 are short answer (SA) type questions,
carrying 3 marks each.
(vi) In Section D, Questions no. 32 to 35 are long answer (LA) type questions
carrying 5 marks each.
(vii) In Section E, Questions no. 36 to 38 are case study based questions carrying
4 marks each. Internal choice is provided in 2 marks questions in each
case-study.
(viii) There is no overall choice. However, an internal choice has been provided in
2 questions in Section B, 2 questions in Section C, 2 questions in Section D and
3 questions in Section E.
22
(ix) Draw neat diagrams wherever required. Take = wherever required, if not
7
stated.
(x) Use of calculators is not allowed.

SECTION A
This section comprises multiple choice questions (MCQs) of 1 mark each.

1 tan 2 30
1. is equal to :
1 tan 2 30

(a) sin 60 (b) cos 60


(c) tan 60 (d) cos 30

AB BC
2. . Which of the following makes the two
DE FD
triangles similar ?

(a) A= D (b) B= D

(c) B= E (d) A= F

30/5/2 JJJJ Page 3 P.T.O.


3. A.P. : 20, 13, 6, 1, ....., 148 _| A§{V_ nX go 13dm± nX h¡ :
(a) 57 (b) 57

(c) 64 (d) 64

4. Xmo nmgo EH$ gmW \|$Ho$ JE & `moJ\$b 10 ?


1 1
(a) (b )
9 6
1 5
(c) (d)
12 18

5. Xr JB© AmH¥${V _|, AB = BC = 10 cm & `{X AC = 7 cm h¡, Vmo BP H$s bå~mB© hmoJr :

(a) 3·5 cm (b) 7 cm


(c) 6·5 cm (d) 5 cm

6. 3 m Jhar Am¡a 40 m 2 km/h H$s Xa go ~h ahm h¡ & 2 {_ZQ> _| g_wÐ


_| {H$VZm nmZr {Ja OmEJm ?

(a) 800 m3 (b) 4000 m3


(c) 8000 m3 (d) 2000 m3

7. _e: 12 Am¡a 15 h¢, Vmo ~hþbH$ hmoJm :


(a) 13·5 (b) 21
(c) 6 (d) 14

30/5/2 JJJJ Page 4


3. The 13th term from the end of the A.P. : 20, 13, 6, 1, ....., 148 is :
(a) 57 (b) 57
(c) 64 (d) 64

4. Two dice are rolled together. What is the probability of getting a sum
greater than 10 ?
1 1
(a) (b )
9 6
1 5
(c) (d)
12 18

5. In the given figure, AB = BC = 10 cm. If AC = 7 cm, then the length of BP


is :

(a) 3·5 cm (b) 7 cm


(c) 6·5 cm (d) 5 cm

6. Water in a river which is 3 m deep and 40 m wide is flowing at the rate of


2 km/h. How much water will fall into the sea in 2 minutes ?

(a) 800 m3 (b) 4000 m3

(c) 8000 m3 (d) 2000 m3

7. If the mean and the median of a data are 12 and 15 respectively, then its
mode is :

(a) 13·5 (b) 21

(c) 6 (d) 14
30/5/2 JJJJ Page 5 P.T.O.
8. Xr JB© AmH¥${V _|, Ho$ÝÐ O Ho$ EH$ d¥Îm na AB EH$ ñne©-aoIm h¡ & `{X OA = 6 cm Am¡a
OAB = 30 h¡, Vmo d¥Îm H$s {ÌÁ`m hmoJr :

(a) 3 cm (b) 3 3 cm

(c) 2 cm (d) 3 cm

9. Xr JB© AmH¥${V _|, Ho$ÝÐ O dmbo EH$ d¥Îm na AC Am¡a AB ñne©-aoImE± h¢ & `{X
COD = 120 h¡, Vmo BAO ~am~a h¡ :

(a) 30 (b) 60
(c) 45 (d) 90

10. {ZåZ{b{IV _| go H$m¡Z-gr g§»`m, {H$gr KQ>Zm Ho$ KQ>Zo H$s àm{`H$Vm hmo gH$Vr h¡ ?
7
(a) 0 (b)
0·01

0·07
(c) 0·07 (d)
3

11. n nX h¢, Ho$ àË`oH$ nX H$mo 2 go H$_ H$a {X`m OmE,


:
(a) 2 H$_ hmo OmEJm
(b) An[ad{V©V ahoJm
(c) 2n H$_ hmo OmEJm
(d) 1 H$_ hmo OmEJm

30/5/2 JJJJ Page 6


8. In the given figure, AB is a tangent to the circle centered at O. If
OA = 6 cm and OAB = 30 , then the radius of the circle is :

(a) 3 cm (b) 3 3 cm

(c) 2 cm (d) 3 cm

9. In the given figure, AC and AB are tangents to a circle centered at O. If


COD = 120 , then BAO is equal to :

(a) 30 (b) 60
(c) 45 (d) 90

10. Which of the following numbers cannot be the probability of happening


of an event ?
7
(a) 0 (b)
0·01
0·07
(c) 0·07 (d)
3

11. If every term of the statistical data consisting of n terms is decreased by


2, then the mean of the data :
(a) decreases by 2
(b) remains unchanged
(c) decreases by 2n
(d) decreases by 1
30/5/2 JJJJ Page 7 P.T.O.
12. Xr JB© AmH¥${V _|, DE BC. x H$m _mZ h¡ :

(a) 6 (b ) 12·5
(c) 8 (d) 10
13. _yb (2 + 3) Am¡a (2 3) dmbm EH$ {ÛKmV g_rH$aU h¡ :
(a) x2 4x + 1 = 0 (b) x2 + 4x + 1 = 0
(c) 4x2 3=0 (d) x2 1=0

5 sin cos
14. `{X tan = h¡, Vmo H$m _mZ h¡ :
12 sin cos
17 17
(a) (b)
7 7
17 7
(c) (d)
13 13

15. `{X EH$ d¥Îm Ho$ EH$ ì`mg Ho$ A§{V_ {~ÝXþ ( 5, 4) Am¡a (1, 0) h¢, Vmo d¥Îm H$s {ÌÁ`m h¡ :

(a) 2 13 BH$mB© (b) 13 BH$mB©


(c) 4 2 BH$mB© (d) 2 2 BH$mB©

16. eyÝ`H$ 1 Am¡a 2 dmbo ~hþnXm| H$s g§»`m h¡ :


(a) R>rH$ 2 (b) Ho$db 1
(c) A{YH$-go-A{YH$ 2 (d) AZÝV

17. g_rH$aU `w½_ ax + 2y = 9 Am¡a 3x + by = 18 g_m§Va aoImE± {Zê${nV H$aoJm, Ohm±


a, b nyUmªH$ h¡§, `{X :
(a) a=b (b) 3a = 2b
(c) 2a = 3b (d) ab = 6

30/5/2 JJJJ Page 8


12. In the given figure, DE BC. The value of x is :

(a) 6 (b ) 12·5
(c) 8 (d) 10
13. A quadratic equation whose roots are (2 + 3 ) and (2 3 ) is :
(a) x2 4x + 1 = 0 (b) x2 + 4x + 1 = 0
(c) 4x2 3=0 (d) x2 1=0

5 sin cos
14. If tan = , then the value of is :
12 sin cos
17 17
(a) (b)
7 7
17 7
(c) (d)
13 13

15. If end points of a diameter of a circle are ( 5, 4) and (1, 0), then the
radius of the circle is :
(a) 2 13 units (b) 13 units
(c) 4 2 units (d) 2 2 units

16. The number of polynomials having zeroes 1 and 2 is :


(a) exactly 2 (b) only 1
(c) at most 2 (d) infinite

17. The pair of equations ax + 2y = 9 and 3x + by = 18 represent parallel


lines, where a, b are integers, if :

(a) a=b (b) 3a = 2b


(c) 2a = 3b (d) ab = 6

30/5/2 JJJJ Page 9 P.T.O.


18. EH$ A.P. {OgH$m ndm± nX an = 5n 7 Ûmam {X`m J`m h¡, H$m gmd© A§Va hmoJm :
(a) 7 (b) 7
(c) 5 (d) 2
19 20 1
(A) (R)
(a), (b), (c) (d)
(a) A{^H$WZ (A) Am¡a VH©$ (R) XmoZm| ghr h¢ Am¡a VH©$ (R), A{^H$WZ (A) H$s ghr
ì¶m»¶m H$aVm h¡ &
(b) A{^H$WZ (A) Am¡a VH©$ (R) XmoZm| ghr h¢, naÝVw VH©$ (R), A{^H$WZ (A) H$s ghr
ì¶m»¶m H$aVm h¡ &
(c) A{^H$WZ (A) ghr h¡, naÝVw VH©$ (R) µJbV h¡ &
(d) A{^H$WZ (A) µJbV h¡, naÝVw VH©$ (R) ghr h¡ &

19. (A) : g§»`m 5n H$^r ^r A§H$ eyÝ` (0) na g_má Zht hmoVr h¡, Ohm± n H$moB© ^r
àmH¥$V g§»`m h¡ &
(R) : 5 Ho$ A^mÁ` JwUZI§S>Z _| Ho$db Xmo hr JwUZI§S> h¢, 1 Am¡a 5 &

20. (A) : `{X Ho$ÝÐ O(2, 3) dmbo EH$ d¥Îm na q~Xþ A(4, 3) Am¡a B(x, 5) pñWV h¢,
Vmo x H$m _mZ 2 hmoJm &
(R) : d¥Îm H$s àË`oH$ Ordm H$m _Ü`-q~Xþ, d¥Îm H$m Ho$ÝÐ hmoVm h¡ &

IÊS> I
(VSA) 2

21. A^mÁ` JwUZI§S>Z go 96 Am¡a 120 H$m HCF Am¡a LCM kmV H$s{OE &
22. {~ÝXþAm| (6, 3) VWm (1, 6) y = x {Og AZwnmV _|
{d^m{OV H$aVr h¡, dh AZwnmV kmV H$s{OE &
23. (H$) `{X a cos + b sin = m VWm a sin b cos =n h¡, Vmo {gÕ H$s{OE
{H$ a2 + b2 = m2 + n2.
AWdm

30/5/2 JJJJ Page 10


18. The common difference of the A.P. whose nth term is given by an = 5n 7
is :
(a) 7 (b) 7
(c) 5 (d) 2

Questions number 19 and 20 are Assertion and Reason based questions carrying
1 mark each. Two statements are given, one labelled as Assertion (A) and the
other is labelled as Reason (R). Select the correct answer to these questions from
the codes (a), (b), (c) and (d) as given below.
(a) Both Assertion (A) and Reason (R) are true and Reason (R) is the
correct explanation of the Assertion (A).
(b) Both Assertion (A) and Reason (R) are true, but Reason (R) is not
the correct explanation of the Assertion (A).
(c) Assertion (A) is true, but Reason (R) is false.
(d) Assertion (A) is false, but Reason (R) is true.
n
19. Assertion (A) : The number 5 cannot end with the digit 0, where n is a
natural number.
Reason (R): Prime factorisation of 5 has only two factors, 1 and 5.

20. Assertion (A) : If the points A(4, 3) and B(x, 5) lie on a circle with centre
O(2, 3), then the value of x is 2.
Reason (R) : Centre of a circle is the mid-point of each chord of the
circle.

SECTION B

This section comprises very short answer (VSA) type questions of 2 marks each.

21. Using prime factorisation, find HCF and LCM of 96 and 120.

22. Find the ratio in which line y = x divides the line segment joining the
points (6, 3) and (1, 6).

23. (a) If a cos + b sin = m and a sin b cos = n, then prove that
a2 + b2 = m2 + n2.
OR
30/5/2 JJJJ Page 11 P.T.O.
(I) {gÕ H$s{OE :
sec A 1 sec A 1
+ = 2 cosec A
sec A 1 sec A 1

24. (H$) {~ÝXþ A(4, 5) Am¡a B(4, 5) H$mo {_bmZo dmbo aoImI§S> H$mo {~ÝXþ P go Bg àH$ma
{d^m{OV {H$`m OmVm h¡ {H$ AP : AB = 2 : 5 h¡ & {~ÝXþ P Ho$ {ZX}em§H$ kmV
H$s{OE &
AWdm
(I) {~ÝXþ P(x, y), {~ÝXþAm| A(5, 1) VWm B(1, 5) go g_XÿañW h¡ & {gÕ H$s{OE {H$
x = y.

25. Xr JB© AmH¥${V _|, Ho$ÝÐ O dmbo EH$ d¥Îm H$s Ordm PQ h¡ & d¥Îm Ho$ {~ÝXþ P na, PT EH$
ñne©-aoIm h¡ & `{X QPT = 55 h¡, Vmo PRQ kmV H$s{OE &

IÊS> J

(SA) 3

26. {ZåZ ~§Q>Z H$m _mÜ` kmV H$s{OE :


dJ© 0 15 15 30 30 45 45 60 60 75 75 90
~ma§~maVm 17 20 18 21 15 9

27. EH$ 2-A§H$s` g§»`m, AnZo A§H$m| Ho$ `moJ\$b Ho$ gmV JwZm h¡ & A§H$m| Ho$ ñWmZ ~XbZo na
àmá g§»`m, nhbr g§»`m go 18 H$_ h¡ & nhbr g§»`m kmV H$s{OE &

30/5/2 JJJJ Page 12


(b) Prove that :
sec A 1 sec A 1
+ = 2 cosec A
sec A 1 sec A 1

24. (a) The line segment joining the points A(4, 5) and B(4, 5) is divided
by the point P such that AP : AB = 2 : 5. Find the coordinates of P.

OR

(b) Point P(x, y) is equidistant from points A(5, 1) and B(1, 5). Prove

that x = y.

25. In the given figure, PQ is a chord of the circle centered at O. PT is a


tangent to the circle at P. If QPT = 55 , then find PRQ.

SECTION C

This section comprises short answer (SA) type questions of 3 marks each.

26. Find the mean of the following distribution :


Classes 0 15 15 30 30 45 45 60 60 75 75 90
Frequency 17 20 18 21 15 9

27. A 2-digit number is seven times the sum of its digits. The number formed
by reversing the digits is 18 less than the given number. Find the given
number.
30/5/2 JJJJ Page 13 P.T.O.
28. {gÕ H$s{OE :
tan cot
+ = 1 + sec cosec
1 cot 1 tan

29. (H$) {gÕ H$s{OE {H$ 3 EH$ An[a_o` g§»`m h¡ &


AWdm
(I) VrZ AbJ-AbJ amoS> H«$m°qgJ na Q´>{¡ \$H$ bmBQ> H«$_e: àË`oH$ 48 goH$ÊS>,
72 goH$ÊS> Am¡a 108 goH$ÊS> Ho$ ~mX ~Xb OmVr h¢ & `{X do EH$ gmW gw~h 7 ~Oo
~XbVr h¢, Vmo do AmJo {H$g g_` EH$ gmW ~Xbo§Jr ?
30. EH$ A.P. _| nhbo n nXm| H$m `moJ\$b Sn = 6n n2 Ûmam {X`m J`m h¡ & Bg A.P. H$m
30dm± nX kmV H$s{OE &
31. (H$) Xr JB© AmH¥${V _|, CD, AB H$m b§~ g_{Û^mOH$ h¡ & EF, CD Ho$ b§~dV h¡ &
CF FG
AE, CD H$mo G na H$mQ>Vr h¡ & {gÕ H$s{OE {H$ .
CD DG

AWdm
(I) Xr JB© AmH¥${V _|, ABCD EH$ g_m§Va MVw^w©O h¡ & BE, CD H$mo M na
g_{Û^m{OV H$aVr h¡ Am¡a AC H$mo L na H$mQ>Vr h¡ & {gÕ H$s{OE {H$
EL = 2BL.

30/5/2 JJJJ Page 14


28. Prove that :
tan cot
+ = 1 + sec cosec
1 cot 1 tan

29. (a) Prove that 3 is an irrational number.

OR
(b) The traffic lights at three different road crossings change after
every 48 seconds, 72 seconds and 108 seconds respectively. If they
change simultaneously at 7 a.m., at what time will they change
together next ?

30. In an A.P., the sum of the first n terms is given by Sn = 6n n2. Find its
th
30 term.

31. (a) In the given figure, CD is the perpendicular bisector of AB. EF is


CF FG
perpendicular to CD. AE intersects CD at G. Prove that .
CD DG

OR

(b) In the given figure, ABCD is a parallelogram. BE bisects CD at M


and intersects AC at L. Prove that EL = 2BL.

30/5/2 JJJJ Page 15 P.T.O.


IÊS> K

(LA) 5

32. (H$) 4 cm {ÌÁ`m dmbo EH$ d¥Îm Ho$ n[aJV EH$ {Ì^wO ABC Bg àH$ma ItMm J`m h¡ {H$
aoImI§S BD Am¡a DC H$s b§~mB`m± H«$_e: 10 cm Am¡a 8 cm h¢ & ^wOmE± AB Am¡a
AC H$s b§~mB`m± kmV H$r{OE, `{X {X`m J`m h¡ {H$ ABC H$m joÌ\$b
90 cm2 h¡ &

AWdm
(I) Xmo d¥Îm h¢ {OZHo$ H|$Ð O Am¡a O h¢, Am¡a {ÌÁ`mE± H«$_e: 6 cm Am¡a 8 cm h¢ & Xmo
{~ÝXþAm| P Am¡a Q na do Bg àH$ma à{VÀN>oX H$aVo h¢ {H$ OP Am¡a O P Xmo d¥Îmm| H$s
ñne©-aoImE± h¢ & C^`{ZîR> Ordm PQ H$s bå~mB© kmV H$s{OE &

30/5/2 JJJJ Page 16


SECTION D

This section comprises long answer (LA) type questions of 5 marks each.

32. (a) A triangle ABC is drawn to circumscribe a circle of radius 4 cm


such that the segments BD and DC are of lengths 10 cm and 8 cm
respectively. Find the lengths of the sides AB and AC, if it is given
that 2.

OR

(b) Two circles with centres O and O of radii 6 cm and 8 cm,


respectively intersect at two points P and Q such that OP and O P
are tangents to the two circles. Find the length of the common
chord PQ.

30/5/2 JJJJ Page 17 P.T.O.


33. 15 m ^wOm dmbo E
5m b§~r añgr go ~m±Y {X`m J`m h¡ & Bg _¡XmZ Ho$ Cg ^mJ H$m joÌ\
10 m H$a Xr OmE Vmo Kmg Ma
gH$Zo dmbo joÌ\ ( = 3.14 H$m à`moJ H$s{OE)

34. ^y{_ Ho$ EH$ q~Xþ P h¡ & {~ÝXþ Q, Omo


60
b§~dV ê$n go nhbo {~ÝXþ P Ho$ 10 m D$na h¡, go {eIa H$m CÞ`Z H$moU 30 h¡ & kmV
H$s{OE :

(a) _rZma H$s D±$MmB© &

(b) _rZma Ho$ nmX go {~ÝXþ P H$s Xÿar &

(c) _rZma Ho$ {eIa go {~ÝXþ P H$s Xÿar &

35. (H$) H$s Xÿar V` H$aVr h¡ Am¡a ~mX _|


54 km
63 km H$s Xÿar nhbr J{V go 6 km/h A{YH$ Am¡gVZ J{V go V` H$aVr h¡ & `{X
>r nyar `mÌm 3 K§Q>m| _| V` H$aVr h¡, Vmo BgH$s nhbr Am¡gVZ J{V ?

AWdm

15
(I) Xmo nmBn {_bH$a EH$ Q>¢H$ H$mo K§Q>m| _| ^a gH$Vo h¢ & A{YH$ ì`mg dmbm
8
nmBn, N>moQ>o ì`mg dmbo nmBn go, 2 K§Q>o H$_ _| Q>¢H$ H$mo ^a gH$Vm h¡ & XmoZm| nmBn
AbJ-AbJ Q>¢H$ H$mo {H$VZo g_` _| ^a gH$Vo h¢, kmV H$s{OE &

30/5/2 JJJJ Page 18


33. A horse is tied to a peg at one corner of a square shaped grass field of side
15 m by means of a 5 m long rope. Find the area of that part of the field
in which the horse can graze. Also, find the increase in grazing area if
length of rope is increased to 10 m. (Use = 3·14)

34. The angle of elevation of the top of a vertical tower from a point P on the
ground is 60 . From another point Q, 10 m vertically above the first point
P, its angle of elevation is 30 . Find :

(a) The height of the tower.

(b) The distance of the point P from the foot of the tower.

(c) The distance of the point P from the top of the tower.

35. (a) A train travels at a certain average speed for a distance of 54 km


and then travels a distance of 63 km at an average speed of 6 km/h
more than the first speed. If it takes 3 hours to complete the
journey, what was its first average speed ?

OR

15
(b) Two pipes together can fill a tank in hours. The pipe with
8
larger diameter takes 2 hours less than the pipe with smaller
diameter to fill the tank separately. Find the time in which each
pipe can fill the tank separately.

30/5/2 JJJJ Page 19 P.T.O.


IÊS> L>
3 4

àH$aU AÜ``Z 1

36. EH$ {_{S>b ñHy$b Zo {H«$g_g H$m{Z©db na {ZåZ{b{IV pñnZa Jo_ H$mo µ\$§S>-aoµOa Ho$ ê$n _|
MbmZo H$m {ZU©` {b`m &

: àË`oH$ pñnZa H$mo EH$ ~ma pñnZ H$s{OE & Zrbm Am¡a bmb, ~¢JZr ~ZmVo
h¢ & Bg{bE, `{X EH$ pñnZa bmb (R) Am¡a Xÿgam Zrbm (B) {XImVm h¡, Vmo Amn OrVVo
hmo & Eogo n[aUm_ na {bIm OmVm h¡ &
Cn`w©º$ Ho$ AmYma na, {ZåZ àíZm| Ho$ CÎma Xr{OE :
(i) Jo_ Ho$ g^r g§^d n[aUm_m| H$s gyMr ~ZmBE & 1

(ii) ~¢JZr a§J ~ZmZo H$s àm{`H$Vm kmV H$s{OE & 1


(iii) (H$) àË`oH$ OrV Ho$ {bE, à{V^mJr H$mo < 10 {_bVo h¢, bo{H$Z AJa dh hma
OmVm h¡, Vmo Cgo ñHy$b H$mo < 5 H$m ^wJVmZ H$aZm hmoJm &
`{X 99 à{V^m{J`m| Zo Jo_ Iobm hmo, Vmo ñHy$b Zo {H$VZm \§$S> EH$Ì {H$`m
hmoJm ? 2
AWdm
(iii) (I) `{X Iob _| OrV `m hma Ho$ {bE < 5 H$s g_mZ am{e V` H$s JB© h¡, Vmo
ñHy$b Zo {H$VZm \§$S> EH$Ì {H$`m hmoJm ? (à{V^m{J`m| H$s g§»`m = 99) 2

30/5/2 JJJJ Page 20


SECTION E

This section comprises 3 case study based questions of 4 marks each.

Case Study 1

36. A middle school decided to run the following spinner game as a


fund-raiser on Christmas Carnival.

Making Purple : Spin each spinner once. Blue and red make purple. So, if

Based on the above, answer the following questions :


(i) List all possible outcomes of the game. 1

(ii) 1
(iii) (a) For each win, a participant gets < 10, but if he/she loses,
he/she has to pay < 5 to the school.
If 99 participants played, calculate how much fund could the
school have collected. 2
OR
(iii) (b) If the same amount of < 5 has been decided for winning or
losing the game, then how much fund had been collected by
school ? (Number of participants = 99) 2

30/5/2 JJJJ Page 21 P.T.O.


àH$aU AÜ``Z 2

37. EH$ Jmoë\$ H$s J|X bJ^J 300 500 {S>ånb Ho$ gmW JmobmH$ma hmoVr h¡, Omo IobZo Ho$
Xm¡amZ CgHo$ doJ H$mo & Jmoë\$ H$s J|X nma§n[aH$ ê$n go g\o$X hmoVr h¡,
bo{H$Z a§Jm| _| ^r CnbãY hmoVr h¡ & Xr JB© AmH¥${V _|, EH$ Jmoë\$ H$s J|X H$m ì`mg
4·2 cm Am¡a BgH$s gVh na {ÌÁ`m 2 mm Ho$ 315 {S>ånb (AY©-JmobmH$ma) h¢ &

Cn`w©º$ Ho$ AmYma na, {ZåZ àíZm| Ho$ CÎma Xr{OE :


(i) EH$ Eogo {S>ånb H$m n¥îR>r` joÌ\$b kmV H$s{OE & 1

(ii) EH$ {S>ånb ~ZmZo Ho$ {bE ImoXr JB© gm_J«r H$m Am`VZ kmV H$s{OE & 1

(iii) (H$) n[adoe Ho$ g§nH©$ _| AmZo dmbm Hw$b n¥îR>r` joÌ\$b kmV H$s{OE & 2

AWdm

(iii) (I) Jmoë\$ H$s J§oX H$m Am`VZ kmV H$s{OE & 2

30/5/2 JJJJ Page 22


Case Study 2

37. A golf ball is spherical with about 300 500 dimples that help increase
its velocity while in play. Golf balls are traditionally white but available
in colours also. In the given figure, a golf ball has diameter
4·2 cm and the surface has 315 dimples (hemi-spherical) of radius 2 mm.

Based on the above, answer the following questions :


(i) Find the surface area of one such dimple. 1

(ii) Find the volume of the material dug out to make one dimple. 1

(iii) (a) Find the total surface area exposed to the surroundings. 2

OR

(iii) (b) Find the volume of the golf ball. 2

30/5/2 JJJJ Page 23 P.T.O.


àH$aU AÜ``Z 3

38. yb _|, EH$ S>m°pë\$Z 20 go_r> à{V goH$ÊS> H$s J{V go `mÌm H$aVo hþE,
nmZr go ~mha Hy$XVr h¡ & t goH$ÊS> Ho$ ~mX Ob ñVa go CgH$s D±$MmB© h = 20t 16t2
Ûmam Xr OmVr h¡ &

Cn`w©º$ Ho$ AmYma na, {ZåZ àíZm| Ho$ CÎma Xr{OE :


(i) ~hþnX p(t) = 20t 16t2 Ho$ eyÝ`H$ kmV H$s{OE & 1
(ii) {ZåZ{b{IV _| go H$m¡Z-gm J«m\$ ~hþnX p(t) H$mo {Zê${nV H$aVm h¡ ? 1

30/5/2 JJJJ Page 24


Case Study 3

38. In a pool at an aquarium, a dolphin jumps out of the water travelling at


20 cm per second. Its height above water level after t seconds is given by
2
h = 20t 16t .

Based on the above, answer the following questions :


(i) Find zeroes of polynomial p(t) = 20t 16t2. 1
(ii) Which of the following types of graph represents p(t) ? 1

30/5/2 JJJJ Page 25 P.T.O.


3
(iii) (H$) t= na h hmoJm ? n[aUm_ H$s ì`m»`m H$s{OE & 2
2
AWdm
(iii) (I) Xmo~mam Ob ñVa go Q>H$amZo go nhbo S>m°pë\$Z Zo {H$VZr Xÿar V` H$s h¡ ? 2

30/5/2 JJJJ Page 26


3
(iii) (a) What would be the value of h at t = ? Interpret the result. 2
2
OR

(iii) (b) How much distance has the dolphin covered before hitting
the water level again ? 2

30/5/2 JJJJ Page 27 P.T.O.


Series WX1YZ/5 SET~3

Q.P. Code 30/5/3


Roll No. narjmWu àíZ-nÌ H$moS> >H$mo CÎma-nwpñVH$m Ho$
_wI-n¥ð >na Adí` {bIo§ &
Candidates must write the Q.P. Code on
the title page of the answer-book.

J{UV (_mZH$)$
MATHEMATICS (STANDARD)
*
:3 : 80
Time allowed : 3 hours Maximum Marks : 80

NOTE :
(i) - 27
Please check that this question paper contains 27 printed pages.
(ii) - - -
-
Q.P. Code given on the right hand side of the question paper should be written on the title
page of the answer-book by the candidate.
(iii) - 38
Please check that this question paper contains 38 questions.
(iv) -

Please write down the serial number of the question in the answer-book before
attempting it.
(v) - 15 -
10.15 10.15 10.30 -
-
15 minute time has been allotted to read this question paper. The question paper will be
distributed at 10.15 a.m. From 10.15 a.m. to 10.30 a.m., the students will read the
question paper only and will not write any answer on the answer-book during this period.

30/5/3 JJJJ Page 1 P.T.O.


:
:
(i) 38
(ii)
(iii) 1 18 (MCQ) 19 20

(iv) 21 25 (VSA)

(v) 26 31 (SA)
(vi) 32 35 (LA)
(vii) 36 38

(viii) 2 2
2 3

22
(ix) =
7

(x)
IÊS> H$
(MCQ) 1
11 2
1. {~ÝXþAm| P ,5 Am¡a Q ,5 Ho$ ~rM H$s Xÿar h¡ :
3 3
(a) BH$mB©
6 BH$mB©
(b) 4
(c) BH$mB©
2 BH$mB©
(d) 3
2. Xr JB© AmH¥${V _|, AB = BC = 10 cm & `{X AC = 7 cm h¡, Vmo BP H$s bå~mB© hmoJr :

(a) 3·5 cm (b) 7 cm


(c) 6·5 cm (d) 5 cm
30/5/3 JJJJ Page 2
General Instructions :
Read the following instructions carefully and follow them :
(i) This question paper contains 38 questions. All questions are compulsory.
(ii) This question paper is divided into five Sections A, B, C, D and E.
(iii) In Section A, Questions no. 1 to 18 are multiple choice questions (MCQs) and
questions number 19 and 20 are Assertion-Reason based questions of 1 mark
each.
(iv) In Section B, Questions no. 21 to 25 are very short answer (VSA) type
questions, carrying 2 marks each.
(v) In Section C, Questions no. 26 to 31 are short answer (SA) type questions,
carrying 3 marks each.
(vi) In Section D, Questions no. 32 to 35 are long answer (LA) type questions
carrying 5 marks each.
(vii) In Section E, Questions no. 36 to 38 are case study based questions carrying
4 marks each. Internal choice is provided in 2 marks questions in each
case-study.
(viii) There is no overall choice. However, an internal choice has been provided in
2 questions in Section B, 2 questions in Section C, 2 questions in Section D and
3 questions in Section E.
22
(ix) Draw neat diagrams wherever required. Take = wherever required, if not
stated. 7
(x) Use of calculators is not allowed.
SECTION A
This section comprises multiple choice questions (MCQs) of 1 mark each.
11 2
1. The distance between the points P , 5 and Q , 5 is :
3 3
(a) 6 units (b) 4 units
(c) 2 units (d) 3 units
2. In the given figure, AB = BC = 10 cm. If AC = 7 cm, then the length of BP
is :

(a) 3·5 cm (b) 7 cm


(c) 6·5 cm (d) 5 cm
30/5/3 JJJJ Page 3 P.T.O.
3. 3 m Jhar Am¡a 40 m 2 km/h H$s Xa go ~h ahm h¡ & 2 {_ZQ> _| g_wÐ
_| {H$VZm nmZr {Ja OmEJm ?

(a) 800 m3 (b) 4000 m3


(c) 8000 m3 (d) 2000 m3

4. `{X {H$gr ~§Q>Z H$m _mÜ` Am¡a ~hþbH$ H«$_e: 15 Am¡a 18 hm|, Vmo Bg ~§Q>Z H$m _mÜ`H$
hmoJm :
(a) 17 (b ) 15
(c) 16 (d) 18

5. A.P. : 10, 7, 4, ......., 62 _| A§{V_ nX go 11dm± nX h¡ :


(a) 25 (b) 16
(c) 32 (d) 0

6. 52 Vme Ho$ nÎmm| H$s AÀN>r Vah go \|$Q>r JB© EH$ JÈ>r go EH$ nÎmm `mÑÀN>`m {ZH$mbm OmVm
h¡ & Bg {ZH$mbo JE nÎmo Ho$ ~oJ_ hmoZo H$s àm{`H$Vm h¡ :
4 4
(a) (b)
13 52
2 1
(c) (d)
13 26

7. Ho$ÝÐ O dmbo EH$ d¥Îm na PQ EH$ ñne©-aoIm h¡ & `{X d¥Îm H$s {ÌÁ`m 5 cm h¡, Vmo
ñne©-aoIm PQ H$s b§~mB© hmoJr :

10
(a) 5 3 cm (b) cm
3
5
(c) 10 cm (d) cm
3
30/5/3 JJJJ Page 4
3. Water in a river which is 3 m deep and 40 m wide is flowing at the rate of
2 km/h. How much water will fall into the sea in 2 minutes ?

(a) 800 m3 (b) 4000 m3

(c) 8000 m3 (d) 2000 m3

4. If the mean and the mode of a distribution are 15 and 18 respectively,


then the median of the distribution is :
(a) 17 (b) 15
(c) 16 (d) 18

5. The 11th term from the end of the A.P. : 10, 7, 4, ......., 62 is :
(a) 25 (b) 16
(c) 32 (d) 0

6. One card is drawn at random from a well shuffled pack of 52 playing


cards. The probability that the drawn card is a queen, is :
4 4
(a) (b)
13 52
2 1
(c) (d)
13 26

7. PQ is tangent to a circle centered at O. If the radius of the circle is 5 cm,


then the length of the tangent PQ is :

10
(a) 5 3 cm (b) cm
3
5
(c) 10 cm (d) cm
3
30/5/3 JJJJ Page 5 P.T.O.
8. {ZåZ{b{IV _| go H$m¡Z-gr g§»`m, {H$gr KQ>Zm Ho$ KQ>Zo H$s àm{`H$Vm hmo gH$Vr h¡ ?
7
(a) 0 (b)
0·01

0·07
(c) 0·07 (d)
3

1
9. `{X sec tan = , Vmo (sec + tan ) H$m _mZ hmoJm :
3
4 2
(a) (b)
3 3
1
(c) (d) 3
3

10. EH$ {ÛKmV g_rH$aU, {OgHo$ _yb (3 2 ) VWm (3 + 2) h¢, h¡ :


(a) x2 6x + 7 = 0 (b) x2 + 6x + 7 = 0
(c) 9x2 2=0 (d) x2 7=0

11. n nX h¢, Ho$ àË`oH$ nX H$mo 2 go H$_ H$a {X`m OmE,


m _mÜ` :
(a) 2 H$_ hmo OmEJm
(b) An[ad{V©V ahoJm
(c) 2n H$_ hmo OmEJm
(d) 1 H$_ hmo OmEJm

12. eyÝ`H$ 3 Am¡a 5 dmbo ~hþnXm| H$s g§»`m h¡ :


(a) Ho$db EH$ (b) AZÝV$
(c) R>rH$ Xmo (d) A{YH$-go-A{YH$ Xmo
2 2
13. g_rH$aU `w½_ x+y=a+b Am¡a ax by = a b H$m hb h¡ :
(a) x = b, y = a (b) x = a, y = b
(c) x = a, y = b (d) x = a, y = b

14. EH$ A.P., {OgH$m ndm± nX an = 3n + 7 Ûmam {X`m J`m h¡, H$m gmd© A§Va hmoJm :
(a) 7 (b) 3
(c) 3n (d) 1

30/5/3 JJJJ Page 6


8. Which of the following numbers cannot be the probability of happening
of an event ?
7
(a) 0 (b)
0·01

0·07
(c) 0·07 (d)
3
1
9. If sec tan = , then the value of (sec + tan ) is :
3
4 2
(a) (b)
3 3
1
(c) (d) 3
3

10. A quadratic equation whose roots are (3 2 ) and (3 + 2 ) is :


(a) x2 6x + 7 = 0 (b) x2 + 6x + 7 = 0
(c) 9x2 2=0 (d) x2 7=0

11. If every term of the statistical data consisting of n terms is decreased by


2, then the mean of the data :
(a) decreases by 2
(b) remains unchanged
(c) decreases by 2n
(d) decreases by 1
12. The number of polynomials having zeroes 3 and 5 is :
(a) only one (b) infinite
(c) exactly two (d) at most two
2
13. The solution of the pair of equations x + y = a + b and ax by = a2 b
is :
(a) x = b, y = a (b) x = a, y = b
(c) x = a, y = b (d) x = a, y = b

14. The common difference of the A.P. whose nth term is given by an = 3n + 7,
is :
(a) 7 (b) 3
(c) 3n (d) 1
30/5/3 JJJJ Page 7 P.T.O.
15. Xr JB© AmH¥${V _|, DE BC. x H$m _mZ h¡ :

(a) 6 (b ) 12·5
(c) 8 (d) 10

AB BC
16. {Ì^wOm| ABC Am¡a DEF _|, & {ZåZ{b{IV _| go H$m¡Z-gm BZ Xmo {Ì^wOm| H$mo
DE FD
g_ê$n ~ZmEJm ?

(a) A= D (b) B= D

(c) B= E (d) A= F

2 tan 30
17. ~am~a h¡ :
1 tan 2 30

(a) sin 60 (b) cos 60

(c) tan 60 (d) sin 30

18. Xr JB© AmH¥${V _|, Ho$ÝÐ O Ho$ EH$ d¥Îm na AB EH$ ñne©-aoIm h¡ & `{X OA = 6 cm Am¡a
OAB = 30 h¡, Vmo d¥Îm H$s {ÌÁ`m hmoJr :

(a) 3 cm (b) 3 3 cm

(c) 2 cm (d) 3 cm

30/5/3 JJJJ Page 8


15. In the given figure, DE BC. The value of x is :

(a) 6 (b ) 12·5

(c) 8 (d) 10

AB BC
16. . Which of the following makes the two
DE FD
triangles similar ?

(a) A= D (b) B= D

(c) B= E (d) A= F

2 tan 30
17. is equal to :
1 tan 2 30

(a) sin 60 (b) cos 60

(c) tan 60 (d) sin 30

18. In the given figure, AB is a tangent to the circle centered at O. If


OA = 6 cm and OAB = 30 , then the radius of the circle is :

(a) 3 cm (b) 3 3 cm

(c) 2 cm (d) 3 cm
30/5/3 JJJJ Page 9 P.T.O.
19 20 1
(A) (R)
(a), (b), (c) (d)
(a) A{^H$WZ (A) Am¡a VH©$ (R) XmoZm| ghr h¢ Am¡a VH©$ (R), A{^H$WZ (A) H$s ghr
ì¶m»¶m H$aVm h¡ &
(b) A{^H$WZ (A) Am¡a VH©$ (R) XmoZm| ghr h¢, naÝVw VH©$ (R), A{^H$WZ (A) H$s ghr
ì¶m»¶m H$aVm h¡ &
(c) A{^H$WZ (A) ghr h¡, naÝVw VH©$ (R) µJbV h¡ &
(d) A{^H$WZ (A) µJbV h¡, naÝVw VH©$ (R) ghr h¡ &
n
19. (A) : g§»`m 5 H$^r ^r A§H$ eyÝ` (0) na g_má Zht hmoVr h¡, Ohm± n H$moB© ^r
àmH¥$V g§»`m h¡ &
(R) : 5 Ho$ A^mÁ` JwUZI§S>Z _| Ho$db Xmo hr JwUZI§S> h¢, 1 Am¡a 5 &

20. (A) : `{X Ho$ÝÐ O(2, 3) dmbo EH$ d¥Îm na q~Xþ A(4, 3) Am¡a B(x, 5) pñWV h¢,
Vmo x H$m _mZ 2 hmoJm &
(R) : d¥Îm H$s àË`oH$ Ordm H$m _Ü`-q~Xþ, d¥Îm H$m Ho$ÝÐ hmoVm h¡ &
IÊS> I
(VSA) 2

21. 3- -go- $s{OE, Omo 18, 24 Am¡a 36 go {d^m{OV hmoVr h¡ &

22. (H$) `{X a cos + b sin = m VWm a sin b cos =n h¡, Vmo {gÕ H$s{OE
{H$ a2 + b2 = m2 + n2.

AWdm
(I) {gÕ H$s{OE :
sec A 1 sec A 1
+ = 2 cosec A
sec A 1 sec A 1

30/5/3 JJJJ Page 10


Questions number 19 and 20 are Assertion and Reason based questions carrying
1 mark each. Two statements are given, one labelled as Assertion (A) and the
other is labelled as Reason (R). Select the correct answer to these questions from
the codes (a), (b), (c) and (d) as given below.

(a) Both Assertion (A) and Reason (R) are true and Reason (R) is the
correct explanation of the Assertion (A).
(b) Both Assertion (A) and Reason (R) are true, but Reason (R) is not
the correct explanation of the Assertion (A).
(c) Assertion (A) is true, but Reason (R) is false.
(d) Assertion (A) is false, but Reason (R) is true.

19. Assertion (A) : The number 5n cannot end with the digit 0, where n is a
natural number.
Reason (R): Prime factorisation of 5 has only two factors, 1 and 5.

20. Assertion (A) : If the points A(4, 3) and B(x, 5) lie on a circle with centre
O(2, 3), then the value of x is 2.
Reason (R) : Centre of a circle is the mid-point of each chord of the
circle.

SECTION B

This section comprises very short answer (VSA) type questions of 2 marks each.

21. Find the greatest 3-digit number which is divisible by 18, 24 and 36.

22. (a) If a cos + b sin = m and a sin b cos = n, then prove that
a2 + b2 = m2 + n2.

OR
(b) Prove that :
sec A 1 sec A 1
+ = 2 cosec A
sec A 1 sec A 1

30/5/3 JJJJ Page 11 P.T.O.


23. {~ÝXþAm| (5, 6) Am¡a ( 1, 4) H$mo {_bmZo dmbo aoImI§S> H$mo y-Aj {Og AZwnmV _|
{d^m{OV H$aVm h¡, Cg AZwnmV H$mo kmV H$s{OE &

24. {gÕ H$s{OE {H$ {H$gr d¥Îm Ho$ {H$gr ì`mg Ho$ {gam| na ItMr JB© ñne© -aoImE± g_mÝVa hmoVr
h¢ &
25. (H$) {~ÝXþ A(4, 5) Am¡a B(4, 5) H$mo {_bmZo dmbo aoImI§S> H$mo {~ÝXþ P go Bg àH$ma
{d^m{OV {H$`m OmVm h¡ {H$ AP : AB = 2 : 5 h¡ & {~ÝXþ P Ho$ {ZX}em§H$ kmV
H$s{OE &

AWdm

(I) {~ÝXþ P(x, y), {~ÝXþAm| A(5, 1) VWm B(1, 5) go g_XÿañW h¡ & {gÕ H$s{OE {H$
x = y.

IÊS> J

(SA) 3

26. (H$) Xr JB© AmH¥${V _|, CD, AB H$m b§~ g_{Û^mOH$ h¡ & EF, CD Ho$ b§~dV h¡ &
CF FG
AE, CD H$mo G na H$mQ>Vr h¡ & {gÕ H$s{OE {H$ .
CD DG

AWdm

30/5/3 JJJJ Page 12


23. Find the ratio in which y-axis divides the line segment joining the points
(5, 6) and ( 1, 4).

24. Prove that the tangents drawn at the ends of a diameter of a circle are
parallel.

25. (a) The line segment joining the points A(4, 5) and B(4, 5) is divided
by the point P such that AP : AB = 2 : 5. Find the coordinates of P.

OR

(b) Point P(x, y) is equidistant from points A(5, 1) and B(1, 5). Prove

that x = y.

SECTION C

This section comprises short answer (SA) type questions of 3 marks each.

26. (a) In the given figure, CD is the perpendicular bisector of AB. EF is


CF FG
perpendicular to CD. AE intersects CD at G. Prove that .
CD DG

OR

30/5/3 JJJJ Page 13 P.T.O.


(I) Xr JB© AmH¥${V _|, ABCD EH$ g_m§Va MVw^w©O h¡ & BE, CD H$mo M na
g_{Û^m{OV H$aVr h¡ Am¡a AC H$mo L na H$mQ>Vr h¡ & {gÕ H$s{OE {H$
EL = 2BL.

1 1
27. EH$ {^Þ Ho$ A§e _| go 1 KQ>m XoZo na {^Þ ~Z OmVr h¡ & `h {^Þ ~Z OmVr h¡ O~
3 4
{^Þ Ho$ ha _| 8

28. {gÕ H$s{OE :


tan A tan A
= 2 cosec A
1 sec A 1 sec A

29. {ZåZ{b{IV ~ma§~maVm ~§Q>Z H$m _mÜ` kmV H$s{OE :

dJ© A§Vamb 25 30 30 35 35 40 40 45 45 50 50 55 55 60

~ma§~maVm 14 22 16 6 5 3 4

30. (H$) {gÕ H$s{OE {H$ 3 EH$ An[a_o` g§»`m h¡ &

AWdm

30/5/3 JJJJ Page 14


(b) In the given figure, ABCD is a parallelogram. BE bisects CD at M
and intersects AC at L. Prove that EL = 2BL.

1
27. A fraction becomes when 1 is subtracted from the numerator. It
3
1
becomes when 8 is added to the denominator. Find the fraction.
4

28. Prove that :

tan A tan A
= 2 cosec A
1 sec A 1 sec A

29. Find the mean of the following frequency distribution :

Classes 25 30 30 35 35 40 40 45 45 50 50 55 55 60

Frequency 14 22 16 6 5 3 4

30. (a) Prove that 3 is an irrational number.

OR

30/5/3 JJJJ Page 15 P.T.O.


(I) VrZ AbJ-AbJ amoS> H«$m°qgJ na Q´>¡{\$H$ bmBQ> H«$_e: àË`oH$ 48 goH$ÊS>,
72 goH$ÊS> Am¡a 108 goH$ÊS> Ho$ ~mX ~Xb OmVr h¢ & `{X do EH$ gmW gw~h 7 ~Oo
~XbVr h¢, Vmo do AmJo {H$g g_` EH$ gmW ~Xbo§Jr ?

31. Cg A.P. H$m gmd© AÝVa kmV H$s{OE, {OgH$m nhbm nX 8, A§{V_ nX 65 VWm BgHo$ g^r
nXm| H$m `moJ\$b 730 h¡ &

IÊS> K

(LA) 5

32. (H$) 54 km H$s Xÿar V` H$aVr h¡ Am¡a ~mX _|


63 km H$s Xÿar nhbr J{V go 6 km/h A{YH$ Am¡gVZ J{V go V` H$aVr h¡ & `{X
nyar `mÌm 3 K§Q>m| _| V` H$aVr h¡, Vmo BgH$s nhbr Am¡g ?

AWdm

15
(I) Xmo nmBn {_bH$a EH$ Q>¢H$ H$mo K§Q>m| _| ^a gH$Vo h¢ & A{YH$ ì`mg dmbm
8
nmBn, N>moQ>o ì`mg dmbo nmBn go, 2 K§Q>o H$_ _| Q>¢H$ H$mo ^a gH$Vm h¡ & XmoZm| nmBn
AbJ-AbJ Q>¢H$ H$mo {H$VZo g_` _| ^a gH$Vo h¢, kmV H$s{OE &

33. 15 m

5m b§~r añgr go ~m±Y {X`m J`m h¡ & Bg _¡XmZ Ho$ Cg ^mJ H$m joÌ\
10 m H$a Xr OmE Vmo Kmg Ma
gH$Zo dmbo joÌ\ ( = 3.14 H$m à`moJ H$s{OE)

30/5/3 JJJJ Page 16


(b) The traffic lights at three different road crossings change after
every 48 seconds, 72 seconds and 108 seconds respectively. If they
change simultaneously at 7 a.m., at what time will they change
together next ?

31. Find the common difference of an A.P. whose first term is 8, the last term
is 65 and the sum of all its terms is 730.

SECTION D

This section comprises long answer (LA) type questions of 5 marks each.

32. (a) A train travels at a certain average speed for a distance of 54 km


and then travels a distance of 63 km at an average speed of 6 km/h
more than the first speed. If it takes 3 hours to complete the
journey, what was its first average speed ?

OR
15
(b) Two pipes together can fill a tank in hours. The pipe with
8
larger diameter takes 2 hours less than the pipe with smaller
diameter to fill the tank separately. Find the time in which each
pipe can fill the tank separately.

33. A horse is tied to a peg at one corner of a square shaped grass field of side
15 m by means of a 5 m long rope. Find the area of that part of the field
in which the horse can graze. Also, find the increase in grazing area if
length of rope is increased to 10 m. (Use = 3·14)

30/5/3 JJJJ Page 17 P.T.O.


34. (H$) 4 cm {ÌÁ`m dmbo EH$ d¥Îm Ho$ n[aJV EH$ {Ì^wO ABC Bg àH$ma ItMm J`m h¡ {H$
aoImI§S BD Am¡a DC H$s b§~mB`m± H«$_e: 10 cm Am¡a 8 cm h¢ & ^wOmE± AB Am¡a
AC H$s b§~mB`m± kmV H$r{OE, `{X {X`m J`m h¡ {H$ ABC H$m joÌ\$b
90 cm2 h¡ &

AWdm
(I) Xmo d¥Îm h¢ {OZHo$ H|$Ð O Am¡a O h¢, Am¡a {ÌÁ`mE± H«$_e: 6 cm Am¡a 8 cm h¢ & Xmo
{~ÝXþAm| P Am¡a Q na do Bg àH$ma à{VÀN>oX H$aVo h¢ {H$ OP Am¡a O P Xmo d¥Îmm| H$s
ñne©-aoImE± h¢ & C^`{ZîR> Ordm PQ H$s bå~mB© kmV H$s{OE &

35. EH$ 80 m -gm_Zo Xmo I§^o bJoo hþE h¢ & EH$ I§^o H$s D±$MmB©
Xÿgao I§^o H$s D±$MmB© go 20 m

Ho$ {eIa Ho$ CÞ`Z H$moU H«$_e: 60 Am¡a 30 h¢ & I§^m| go {~ÝXþ H$s Xÿar Am¡a àË`oH$
I§^o H$s D±$MmB© kmV H$s{OE & ( 3 = 1·73 H$m à`moJ H$s{OE)

30/5/3 JJJJ Page 18


34. (a) A triangle ABC is drawn to circumscribe a circle of radius 4 cm
such that the segments BD and DC are of lengths 10 cm and 8 cm
respectively. Find the lengths of the sides AB and AC, if it is given
2.

OR

(b) Two circles with centres O and O of radii 6 cm and 8 cm,


respectively intersect at two points P and Q such that OP and O P
are tangents to the two circles. Find the length of the common
chord PQ.

35. Two pillars are standing on either side of a 80 m wide road. Height of one
pillar is 20 m more than the height of the other pillar. From a point on
the road between the pillars, the angle of elevation of the higher pillar is
60 , whereas that of the other pillar is 30 . Find the position of the point
between the pillars and the height of each pillar. (Use 3 = 1·73)

30/5/3 JJJJ Page 19 P.T.O.


IÊS> L>
3 4
àH$aU AÜ``Z 1
36. \$Z 20 go_r à{V goH$ÊS> H$s J{V go `mÌm H$aVo hþE,
nmZr go ~mha Hy$XVr h¡ & t goH$ÊS> Ho$ ~mX Ob ñVa go CgH$s D±$MmB© h = 20t 16t2
Ûmam Xr OmVr h¡ &

Cn`w©º$ Ho$ AmYma na, {ZåZ àíZm| Ho$ CÎma Xr{OE :


(i) ~hþnX p(t) = 20t 16t2 Ho$ eyÝ`H$ kmV H$s{OE & 1
(ii) {ZåZ{b{IV _| go H$m¡Z-gm J«m\$ ~hþnX p(t) H$mo {Zê${nV H$aVm h¡ ? 1

30/5/3 JJJJ Page 20


SECTION E
This section comprises 3 case study based questions of 4 marks each.
Case Study 1
36. In a pool at an aquarium, a dolphin jumps out of the water travelling at
20 cm per second. Its height above water level after t seconds is given by
2
h = 20t 16t .

Based on the above, answer the following questions :


2
(i) Find zeroes of polynomial p(t) = 20t 16t . 1
(ii) Which of the following types of graph represents p(t) ? 1

30/5/3 JJJJ Page 21 P.T.O.


3
(iii) (H$) t= na h hmoJm ? n[aUm_ H$s ì`m»`m H$s{OE & 2
2
AWdm
(iii) (I) Xmo~mam Ob ñVa go Q>H$amZo go nhbo S>m°pë\$Z Zo {H$VZr Xÿar V` H$s h¡ ? 2

àH$aU AÜ``Z 2

37. EH$ Jmoë\$ H$s J|X bJ^J 300 500 {S>ånb Ho$ gmW JmobmH$ma hmoVr h¡, Omo IobZo Ho$
Xm¡amZ CgHo$ doJ H$mo & Jmoë\$ H$s J|X nma§n[aH$ ê$n go g\o$X hmoVr h¡,
bo{H$Z a§Jm| _| ^r CnbãY hmoVr h¡ & Xr JB© AmH¥${V _|, EH$ Jmoë\$ H$s J|X H$m ì`mg
4·2 cm Am¡a BgH$s gVh na {ÌÁ`m 2 mm Ho$ 315 {S>ånb (AY©-JmobmH$ma) h¢ &

Cn`w©º$ Ho$ AmYma na, {ZåZ àíZm| Ho$ CÎma Xr{OE :


(i) EH$ Eogo {S>ånb H$m n¥îR>r` joÌ\$b kmV H$s{OE & 1

(ii) EH$ {S>ånb ~ZmZo Ho$ {bE ImoXr JB© gm_J«r H$m Am`VZ kmV H$s{OE & 1

(iii) (H$) n[adoe Ho$ g§nH©$ _| AmZo dmbm Hw$b n¥îR>r` joÌ\$b kmV H$s{OE & 2
AWdm
(iii) (I) Jmoë\$ H$s J§oX H$m Am`VZ kmV H$s{OE & 2

30/5/3 JJJJ Page 22


3
(iii) (a) What would be the value of h at t = ? Interpret the result. 2
2
OR

(iii) (b) How much distance has the dolphin covered before hitting
the water level again ? 2

Case Study 2

37. A golf ball is spherical with about 300 500 dimples that help increase
its velocity while in play. Golf balls are traditionally white but available
in colours also. In the given figure, a golf ball has diameter
4·2 cm and the surface has 315 dimples (hemi-spherical) of radius 2 mm.

Based on the above, answer the following questions :

(i) Find the surface area of one such dimple. 1

(ii) Find the volume of the material dug out to make one dimple. 1

(iii) (a) Find the total surface area exposed to the surroundings. 2

OR

(iii) (b) Find the volume of the golf ball. 2

30/5/3 JJJJ Page 23 P.T.O.


àH$aU AÜ``Z 3

38. EH$ {_{S>b ñHy$b Zo {H«$g_g H$m{Z©db na {ZåZ{b{IV pñnZa Jo_ H$mo µ\$§S>-aoµOa Ho$ ê$n _|
MbmZo H$m {ZU©` {b`m &

: àË`oH$ pñnZa H$mo EH$ ~ma pñnZ H$s{OE & Zrbm Am¡a bmb, ~¢JZr ~ZmVo
h¢ & Bg{bE, `{X EH$ pñnZa bmb (R) Am¡a Xÿgam Zrbm (B) {XImVm h¡, Vmo Amn OrVVo
hmo & Eogo n[aUm_ na {bIm OmVm h¡ &

Cn`w©º$ Ho$ AmYma na, {ZåZ àíZm| Ho$ CÎma Xr{OE :

(i) Jo_ Ho$ g^r g§^d n[aUm_m| H$s gyMr ~ZmBE & 1

(ii) ~¢JZr a§J ~ZmZo H$s àm{`H$Vm kmV H$s{OE & 1

30/5/3 JJJJ Page 24


Case Study 3

38. A middle school decided to run the following spinner game as a


fund-raiser on Christmas Carnival.

Making Purple : Spin each spinner once. Blue and red make purple. So, if

outcome

Based on the above, answer the following questions :

(i) List all possible outcomes of the game. 1

(ii) 1

30/5/3 JJJJ Page 25 P.T.O.


(iii) (H$) àË`oH$ OrV Ho$ {bE, à{V^mJr H$mo < 10 {_bVo h¢, bo{H$Z AJa dh hma
OmVm h¡, Vmo Cgo ñHy$b H$mo < 5 H$m ^wJVmZ H$aZm hmoJm &
`{X 99 à{V^m{J`m| Zo Jo_ Iobm hmo, Vmo ñHy$b Zo {H$VZm \§$S> EH$Ì {H$`m
hmoJm ? 2
AWdm
(iii) (I) `{X Iob _| OrV `m hma Ho$ {bE < 5 H$s g_mZ am{e V` H$s JB© h¡, Vmo
ñHy$b Zo {H$VZm \§$S> EH$Ì {H$`m hmoJm ? (à{V^m{J`m| H$s g§»`m = 99) 2

30/5/3 JJJJ Page 26


(iii) (a) For each win, a participant gets < 10, but if he/she loses,
he/she has to pay < 5 to the school.
If 99 participants played, calculate how much fund could the
school have collected. 2

OR
(iii) (b) If the same amount of < 5 has been decided for winning or
losing the game, then how much fund had been collected by
school ? (Number of participants = 99) 2

30/5/3 JJJJ Page 27 P.T.O.


ANSWERS
Marking Scheme
Strictly Confidential
(For Internal and Restricted use only)
Secondary School Examination, 2023
MATHEMATICS PAPER CODE 30/1/1

General Instructions: -

1 You are aware that evaluation is the most important process in the actual and correct assessment of
the candidates. A small mistake in evaluation may lead to serious problems which may affect the
future of the candidates, education system and teaching profession. To avoid mistakes, it is requested
that before starting evaluation, you must read and understand the spot evaluation guidelines carefully.
2 “Evaluation policy is a confidential policy as it is related to the confidentiality of the
examinations conducted, Evaluation done and several other aspects. Its’ leakage to public in
any manner could lead to derailment of the examination system and affect the life and future
of millions of candidates. Sharing this policy/document to anyone, publishing in any magazine
and printing in News Paper/Website etc may invite action under various rules of the Board and
IPC.”
3 Evaluation is to be done as per instructions provided in the Marking Scheme. It should not be done
according to one’s own interpretation or any other consideration. Marking Scheme should be strictly
adhered to and religiously followed. However, while evaluating, answers which are based on
latest information or knowledge and/or are innovative, they may be assessed for their
correctness otherwise and due marks be awarded to them.
4 The Marking scheme carries only suggested value points for the answers.
These are in the nature of Guidelines only and do not constitute the complete answer. The students
can have their own expression and if the expression is correct, the due marks should be awarded
accordingly.
5 The Head-Examiner must go through the first five answer books evaluated by each evaluator on the
first day, to ensure that evaluation has been carried out as per the instructions given in the Marking
Scheme. If there is any variation, the same should be zero after deliberation and discussion. The
remaining answer books meant for evaluation shall be given only after ensuring that there is no
significant variation in the marking of individual evaluators.
6 Evaluators will mark ( √ ) wherever answer is correct. For wrong answer CROSS ‘X” be marked.
Evaluators will not put right (✓) while evaluating which gives an impression that answer is correct
and no marks are awarded. This is most common mistake which evaluators are committing.
7 If a question has parts, please award marks on the right-hand side for each part. Marks awarded for
different parts of the question should then be totaled up and written in the left-hand margin and
encircled. This may be followed strictly.
8 If a question does not have any parts, marks must be awarded in the left-hand margin and encircled.
This may also be followed strictly.
9 In Q1-Q20, if a candidate attempts the question more than once (without canceling the previous
attempt), marks shall be awarded for the first attempt only and the other answer scored out
with a note “Extra Question”.
10 In Q21-Q38, if a student has attempted an extra question, answer of the question deserving
more marks should be retained and the other answer scored out with a note “Extra Question”.
11 No marks to be deducted for the cumulative effect of an error. It should be penalized only once.
12 A full scale of marks __________(example 0 to 80/70/60/50/40/30 marks as given in Question Paper)
has to be used. Please do not hesitate to award full marks if the answer deserves it.

1
MS_X_Mathematics_041_30/1/1_2022-23
13 Every examiner has to necessarily do evaluation work for full working hours i.e., 8 hours every day
and evaluate 20 answer books per day in main subjects and 25 answer books per day in other subjects
(Details are given in Spot Guidelines). This is in view of the reduced syllabus and number of
questions in question paper.
14 Ensure that you do not make the following common types of errors committed by the Examiner in
the past:-
● Leaving answer or part thereof unassessed in an answer book.
● Giving more marks for an answer than assigned to it.
● Wrong totaling of marks awarded on an answer.
● Wrong transfer of marks from the inside pages of the answer book to the title page.
● Wrong question wise totaling on the title page.
● Wrong totaling of marks of the two columns on the title page.
● Wrong grand total.
● Marks in words and figures not tallying/not same.
● Wrong transfer of marks from the answer book to online award list.
● Answers marked as correct, but marks not awarded. (Ensure that the right tick mark is correctly
and clearly indicated. It should merely be a line. Same is with the X for incorrect answer.)
● Half or a part of answer marked correct and the rest as wrong, but no marks awarded.
15 While evaluating the answer books if the answer is found to be totally incorrect, it should be marked
as cross (X) and awarded zero (0)Marks.
16 Any un assessed portion, non-carrying over of marks to the title page, or totaling error detected by
the candidate shall damage the prestige of all the personnel engaged in the evaluation work as also
of the Board. Hence, in order to uphold the prestige of all concerned, it is again reiterated that the
instructions be followed meticulously and judiciously.
17 The Examiners should acquaint themselves with the guidelines given in the “Guidelines for spot
Evaluation” before starting the actual evaluation.
18 Every Examiner shall also ensure that all the answers are evaluated, marks carried over to the title
page, correctly totaled and written in figures and words.
19 The candidates are entitled to obtain photocopy of the Answer Book on request on payment of the
prescribed processing fee. All Examiners/Additional Head Examiners/Head Examiners are once
again reminded that they must ensure that evaluation is carried out strictly as per value points for
each answer as given in the Marking Scheme.

MS_X_Mathematics_041_30/1/1_2022-23
2
MARKING SCHEME
MATHEMATICS (Subject Code–041)
(PAPER CODE: 30/1/1)

Q. No. EXPECTED OUTCOMES/VALUE POINTS Marks


SECTION A
Questions no. 1 to 18 are multiple choice questions (MCQs) and questions
number 19 and 20 are Assertion-Reason based questions of 1 mark each

The graph of y = p ( x ) is given, for a polynomial p ( x) . The number of


1.
zeroes of p(x) from the graph is

(A) 3 (B) 1 (C) 2 (D) 0

Sol. (B) 1 1
The value of k for which the pair of equations kx = y + 2 and 6 x = 2 y + 3
2.
has infinitely many solutions,

(A) is k = 3 (B) does not exist (C) is k = −3 (D) is k = 4

Sol. (B) does not exist 1


3. If p − 1, p + 1 and 2p + 3 are in A.P., then the value of p is
(A) − 2 (B) 4 (C) 0 (D) 2

Sol. (C) 0 1
In what ratio, does x-axis divide the line segment joining the points
4.
A(3, 6) and b(−12, −3) ?
(A) 1: 2 (B) 1: 4 (C) 4 :1 (D) 2 :1

3
MS_X_Mathematics_041_30/1/1_2022-23
Sol. (D) 2 : 1 1
In the given figure, PQ is tangent to the circle centred at O.
5.
If AOB = 95 , then the measure of ABQ will be

(A) 47.5 (B) 42.5 (C) 85 (D) 95

Sol. (A) 47.5 1


4sin A + 3cos A
6. If 2 tan A = 3 , then the value of is
4sin A − 3cos A
7 1
(A) (B) (C) 3 (D) does not exist
13 13

Sol. (C) 3 1
1 1
7. If  ,  are the zeroes of a polynomial p( x) = x 2 + x − 1 , then 𝛼 +𝛽 equals to

−1
(A) 1 (B) 2 (C) − 1 (D)
2

Sol. (A) 1 1
The least positive value of k , for which the quadratic equation
8.
2 x 2 + kx − 4 = 0 has rational roots, is
(A) 2 2 (B) 2 (C) 2 (D) 2
Sol. (B) 2 1
3 2 0 0
 4 tan 30 − sec 45 + sin 60  is equal to
9. 2 0 2

5 −3 1
(A) −1 (B) (C) (D)
6 2 6
Sol. (A) – 1 1

MS_X_Mathematics_041_30/1/1_2022-23
4
Curved surface area of a cylinder of height 5 cm is 94.2 cm 2 . Radius of the
10.
cylinder is (Take  = 3.14)
(A) 2 cm (B) 3 cm (C) 2.9 cm (D) 6 cm

Sol. (B) 3 1
The distribution below gives the marks obtained by 80 students on a test :
11.
Marks Less Less Less Less Less Less
than 10 than than than 40 than than
20 30 50 60
Number of 3 12 27 57 75 80
Students
The modal class of this distribution is :
(A) 10-20 (B) 20-30
(C) 30-40 (D) 50-60
Sol. (C) 30 – 40 1
The curved surface area of a cone having height 24 cm and radius 7 cm, is
12.
(A) 528 cm 2 (B) 1056 cm 2 (C) 550 cm 2 (D) 500 cm 2

Sol. (C)550cm2 1

13. The distance between the points (0,2√5) and (−2√5, 0) is


(A) 2√10 units (B) 4√10units (C) 2√20 units (D) 0
Sol. (A) 2√10 units 1
−2 2
14. Which of the following is a quadratic polynomial having zeroes and ?
3 3
4 9
(A) 4 x 2 − 9 (D) 5(9 x − 4)
2
(B) (9 x 2 + 4) (C) x 2 +
9 4
Sol. (D) 5 (9x2 – 4) 1
If the value of each observation of a statistical data is increased by 3, then the
15.
mean of the data
(A) remains unchanged (B) increases by 3
(C) increases by 6 (D) increases by 3n

Sol. (B) increases by 3 1


5
MS_X_Mathematics_041_30/1/1_2022-23
Probability of happening of an event is denoted by p and probability of non-
16.
happening of the event is denoted by q . Relation between p and q is
(A) p + q = 1 (B) p = 1, q = 1
(C) p = q − 1 (D) p + q + 1 = 0

Sol. (A) p + q = 1 1
A girl calculates that the probability of her winning the first prize in a lottery
17.
is 0.08. If 6000 tickets are sold, how many tickets has she bought?
(A) 40 (B) 240 (C) 480 (D) 750
Sol. (C) 480 1

In a group of 20 people, 5 can't swim. If one person is selected at random,


18.
then the probability that he/she can swim, is
3 1 1
(A) (B) (C) 1 (D)
4 3 4
Sol. (A) 3/4 1

19. Assertion (A): Point P ( 0, 2 ) is the point of intersection of y-axis with the
line 3 x + 2 y = 4 .
Reason (R): The distance of point P ( 0, 2 ) from x-axis is 2 units.

(B) Both Assertion (A) and Reason ( R) are correct but Reason ( R) is
Sol. 1
not the correct explanation of Assertion (A)

MS_X_Mathematics_041_30/1/1_2022-23
6
Assertion (A): The perimeter of ABC is a rational number.
20.
Reason (R): The sum of the squares of two rational numbers is always
rational.

Sol. (D) Assertion (A) is false but Reason (R) is true 1

SECTION B
This section comprises of Very Short Answer (VSA) type questions of 2
marks each.
Solve the pair of equations x = 3 and y = −4 graphically.
21(a).
Sol. Correct graph of both the equations. 1
Solution of equation is x = 3, y = – 4 1

OR
Using graphical method, find whether following system of linear equations
21(b).
is consistent or not:
x = 0 and y = −7

Sol. Correct graph of y = – 7 and x = 0 1


As y = – 7 is intersecting x = 0 at (0, – 7)
So, system of equations is consistent 1
In the given figure, XZ is parallel to BC. AZ = 3 cm, ZC = 2 cm,
22.
BM =3 cm and MC = 5 cm. Find the length of XY.

7
MS_X_Mathematics_041_30/1/1_2022-23
AX 3 AZ ½
Sol. As XZ || BC Therefore = = – (i)
XB 2 ZC
 AXY   ABM ½

𝐴𝑋 𝑋𝑌
⇒ 𝐴𝐵 = 𝐵𝑀 or 5 =
3 𝑋𝑌 ½
3

9
½
 𝑋𝑌 = 5 or 1·8 cm

23(a). If sin + cos = 3 , then find the value of sin  cos .

Sol. sin  + cos  = 3


squaring both sides
sin2  + cos2  + 2 sin cos  = 3 1
 1 + 2 sin  cos  = 3 ½
½
 sin  cos  = 1

OR
1
23(b). If sin  = and cot  = 3 , then find the value of cosecα+ cosecβ
2

MS_X_Mathematics_041_30/1/1_2022-23
8
Sol. 1 ½
cosec  = = 2
sin 
1
cosec  = 1 + cot 2  = 1+3 =2

½
 cosec  + cosec  = 2 + 2 or 2 ( 2 + 1)

Find the greatest number which divides 85 and 72 leaving remainders 1 and
24.
2 respectively.

We have to find HCF of 85 – 1 = 84 and 72 – 2 = 70.


Sol. 1
HCF of 84 and 70 = 14 1

A bag contains 4 red, 3 blue and 2 yellow balls. One ball is drawn at random
25.
from the bag. Find the probability that drawn ball is
(i) red (ii) yellow.
Total No of Balls=9
Sol.
4 1
(i) P(drawn ball is red) =
9
2 1
(ii) P(drawn ball is yellow) =
9

SECTION C
This section comprises of Short Answer (SA) type questions of 3 marks
each.
Half of the difference between two numbers is 2. The sum of the greater
26.
number and twice the smaller number is 13. Find the numbers.
Let the numbers be x and y, x > y
Sol.
1
Therefore (x – y) = 2 — (i) 1
2
and 2y + x = 13 — (ii) 1
Solving equations (i) and (ii)
x = 7, y = 3 1

9
MS_X_Mathematics_041_30/1/1_2022-23
27. Prove that 5 is an irrational number.
Sol. Let √𝟓 be a rational number.
𝐩 ½
∴ √𝟓 = 𝐪 , where q≠0 and let p & q be co-primes.

5q2 = p2 ⟹ p2 is divisible by 5 ⟹ p is divisible by 5


1
⟹ p = 5a, where ‘a’ is some integer ----- (i)
25a2 = 5q2 ⟹ q2 = 5a2 ⟹q2 is divisible by 5 ⟹ q is divisible by 5
½
⟹ q = 5b, where ‘b’ is some integer ----- (ii)
(i) and (ii) leads to contradiction as ‘p’ and ‘q’ are co-primes.
1
∴ √𝟓 is an irrational number.
If ( −5,3) and (5,3) are two vertices of an equilateral triangle, then find
28.
coordinates of the third vertex, given that origin lies inside the triangle. (Take
3 = 1.7)

Let the third vertex be (x,y)


Sol.
A (-5,3) B(5,3) C(x,y)
AB=10=AC 1
AC2=100
(-5-x)2+(3-y)2 = (5-x)2+(3-y)2
1
20x =0

x=0 ½

(3-y)2=75
3-y = ±5√3
y=3-5√3
y= -5.5
½
The coordinates of the third vertex are (0,-5.5)

Two tangents TP and TQ are drawn to a circle with centre O from an external
29(a).
point T. Prove that PTQ = 2OPQ .

MS_X_Mathematics_041_30/1/1_2022-23
10
TP = TQ
Sol.
⇒  TPQ =  TQP 1

Let  PTQ be 
180° – 𝜃 𝜃
⇒  TPQ =  TQP = = 90 – 2 1
2

Now  OPT = 90


𝜃 𝜃
⇒ OPQ = 90 – (90 – 2 ) = 2
1
 PTQ = 2  OPQ

OR
In the given figure, a circle is inscribed in a quadrilateral ABCD in which
29(b).
B = 900 . If AD 17 cm, AB = 20 cm and DS = 3 cm, then find the radius of
the circle.

11
MS_X_Mathematics_041_30/1/1_2022-23
Sol.

DR = DS = 3 cm ½

 AR = AD – DR = 17 – 3 = 14 cm 1
 AQ = AR = 14 cm ½

 QB = AB – AQ = 20 – 14 = 6 cm ½

Since QB = OP = r  radius = 6 cm ½

tan  + sec  − 1 1 + sin 


30. Prove that: =
tan  − sec  +  cos 

(𝑡𝑎𝑛 𝜃 + 𝑠𝑒𝑐 𝜃) – (𝑠𝑒𝑐 2 𝜃 – 𝑡𝑎𝑛 2 𝜃)


Sol. LHS= 1
𝑡𝑎𝑛 𝜃 – 𝑠𝑒𝑐 𝜃 + 1
(𝑡𝑎𝑛 𝜃 + 𝑠𝑒𝑐 𝜃) (1 – 𝑠𝑒𝑐 𝜃 + 𝑡𝑎𝑛 𝜃) 1
= 𝑡𝑎𝑛 𝜃 – 𝑠𝑒𝑐 𝜃 + 1

= 𝑡𝑎𝑛𝜃 + 𝑠𝑒𝑐𝜃 ½
1 + 𝑠𝑖𝑛 𝜃
= = RHS ½
𝑐𝑜𝑠 𝜃

A room is in the form of cylinder surmounted by a hemi-spherical dome.


31(a).
The base radius of hemisphere is one-half the height of cylindrical part.
 1408  3
Find total height of the room if it contains   m of air. Take
 21 
 22 
 = 
 7 

MS_X_Mathematics_041_30/1/1_2022-23
12
Sol. Let h be height of cylindrical part and r be radius of hemisphere ½
2 1408 1
Volume of room = 2 r 3 +  r 3 =
3 21

r = 2 ½
Therefore, h=4 ½
Height of the room is = 6m ½

OR
An empty cone is of radius 3 cm and height 12 cm. Ice-cream is filled
31(b).
th
1
in it so that lower part of the cone which is   of the volume of the
6
cone is unfilled but hemisphere is formed on the top. Find volume of the
ice-cream. (Take  = 3.14)

Sol. 1 1
Volume of the cone = =    9 12 = 36 cm3
3
5
Volume of ice-cream in the cone = 6 × 36 × 𝜋 = 30𝜋𝑐𝑚3 ½
2 1
Volume of ice-cream on top = 3 × 27 × 𝜋 = 18𝜋𝑐𝑚3

Total volume of the ice-cream = (30 + 18 ) = 48 cm3

= 48  3.14 = 150.72cm3 ½

SECTION D
This section comprises of Long Answer (LA) type questions of 5 marks
each.

13
MS_X_Mathematics_041_30/1/1_2022-23
If a line is drawn parallel to one side of a triangle to intersect the other
32.
two sides at distinct points, prove that the other two sides are divided in
the same ratio.

Sol. Correct Given, to prove, figure, construction 2


Correct proof 3
33(a). The angle of elevation of the top of a tower 24 m high from the foot of
another tower in the same plane is 600. The angle of elevation of the top
of second tower from the foot of the first tower is 300. Find the distance
between two towers and the height of the other tower. Also, find the
length of the wire attached to the tops of both the towers.

Sol.

1 mark
for
correct
figure

Let AB and CD be the given towers.


1 h
tan 30 = =  x = h 3 __ (i) 1
3 x
24 24
tan 60 = 3 = x= or 8√3 __ (ii) 1
x 3
using (i) and (ii)
1 1
x = 8 3 and h = 8 +
2 2

length of wire = BE 2 + x 2 = 256 + 192 = 448 m = 8 7 m 1

MS_X_Mathematics_041_30/1/1_2022-23
14
OR
A spherical balloon of radius r subtends an angle of 600 at the eye of an
33(b).
observer. If the angle of elevation of its centre is 450 from the same point,
then prove that height of the centre of the balloon is 2 times its radius.

Sol.

1 mark
for
correct
figure

Let Point B represents observer.


  QBP = 60;  ABO = 45
1
Using geometry  PBO =  60 = 30 1
2
r 1
Now, = sin 30 =  OB = 2r — (i)
OB 2 1

OA 1
Also = sin 45 =  OB = OA 2 (ii) 1
OB 2

Using (i) and (ii) OA = 2 r 1


or height of center of balloon = 2 r units

A chord of a circle of radius 14 cm subtends an angle of 600 at the centre.


34.
Find the area of the corresponding minor segment of the circle. Also find
the area of the major segment of the circle.

Sol. 22 60 1 3 1+1
Area of minor segment = 14 14  − 14 14 
7 360 2 2
 308 
= − 49 3  cm2 or 17.9cm2 1
 3 

15
MS_X_Mathematics_041_30/1/1_2022-23
22 308
Area of major segment = × 14 × 14 − ( − 49√3)
7 3
308 1
= 616 − + 49 3
3
 1540 
= + 49 3  cm2 or 598.1cm2 1
 3 
th th
The ratio of the 11 term to 17 term of an A.P. is 3:4. Find the ratio of
35(a).
5th term to 21st term of the same A.P. Also, find the ratio of the sum of
first 5 terms to that of first 21 terms.

Sol. a + 10 d 3 1
Given =
a + 16 d 4
 4a + 40d = 3a + 48d
1
 a = 8d (i)
a a + 4d 3
therefore 5 = =7 using( i)
a 21 a + 20 d 1

a5 : a21 = 3 : 7
5
s5 (2a + 4 d ) 5  20 d 25
= 2 = = 2
s 21 21
(2a + 20 d ) 21  36 d 189
2
Therefore, S5:S21=25:189
OR
250 logs are stacked in the following manner:
35(b).
22 logs in the bottom row, 21 in the next row, 20 in the row next to it and so
on (as shown by an example). In how many rows, are the 250 logs placed and
how many logs are there in the top row?

Sol.
Let the number of rows be n.
A.P. formed is 22, 21, 20, 19, .........
MS_X_Mathematics_041_30/1/1_2022-23
16
Here a = 22, d = – 1 Sn = 250 1
n
 250 = [44 + (n – 1) (– 1)] 1
2
1
 n2 – 45n + 500 = 0
 (n – 25) (n – 20) = 0
n  25  n = 20 1
logs in top row = a20 = 22 + 19 (– 1) = 3 1

SECTION E
This section comprises of 3 case-study based questions of 4 marks each.
While designing the school year book, a teacher asked the student that the
36.
length and width of a particular photo is increased by x units each to double
the area of the photo. The original photo is 18 cm long and 12 cm wide.
Based on the above information, answer the following questions:
(I) Write an algebraic equation depicting the above information.
(II) Write the corresponding quadratic equation in standard form.
(III) What should be the new dimensions of the enlarged photo?

OR
Can any rational value of x make the new area equal to 220cm 2

Sol. (i) (18 + x) (12 + x) = 2(18 ×12) 1


(ii) x2 + 30x – 216 = 0
1
(iii) Solving : x2 + 30x – 216 = 0
 (x + 36) (x – 6) = 0
x  -36   x = 6. 1
1
new dimensions are 24 cm  18 cm

17
MS_X_Mathematics_041_30/1/1_2022-23
OR
(iii) If (18 + x) (12 + x) = 220
1
then x2 + 30x – 4 = 0
1
Here D = 900 + 16 = 916 which is not a perfect square.

Thus we can’t have any such rational value of x.

India meteorological department observes seasonal and annual rainfall


37.
every year in different sub-divisions of our country.

It helps them to compare and analyse the results. The table given below shows
sub-division wise seasonal (monsoon) rainfall (mm) in 2018:
Rainfall (mm) Number of Sub-divisions
200-400 2
400-600 4
600-800 7
800-1000 4
1000-1200 2
1200-1400 3
1400 -1600 1
1600-1800 1
Based on the above information, answer the following questions:
(I) Write the modal class.
(II) Find the median of the given data.
MS_X_Mathematics_041_30/1/1_2022-23
18
OR
(II) Find the mean rainfall in this season.
(III) If sub-division having at least 1000 mm rainfall during monsoon season,
is considered good rainfall sub-division, then how many sub-divisions
had good rainfall?

Sol. (i) Modal Class is 600-800 1

𝑁
(ii) = 12, median class is 600 – 800 ½
2

Rainfall xi fi cf.
200 – 400 300 2 2
400 – 600 500 4 6
600 – 800 700 7 13
800 – 1000 900 4 17
1000 – 1200 1100 2 19
1200 – 1400 1300 3 22
½ for
1400 – 1600 1500 1 23 correct
table
1600 – 1800 1700 1 24
24

200
Median = 600 + (12 – 6)
7
5400
= or 771·4 1
7

OR
(ii)
Rainfall xi fi f ixi
200 – 400 300 2 600
400 – 600 500 4 2000

19
MS_X_Mathematics_041_30/1/1_2022-23
600 – 800 700 7 4900
800 – 1000 900 4 3600
1000 – 1200 1100 2 2200 1 for
correct
1200 – 1400 1300 3 3900 table
1400 – 1600 1500 1 1500
1600 – 1800 1700 1 1700
24 20400

20400 1
Mean = = 850
24

1
(iii) Sub-divisions having good rainfall = 2 + 3 + 1 + 1 = 7.

The discus throw is an event in which an athlete attempts to


38.
throw a discus. The athlete spins anti-clockwise around one and
a half times through a circle, then releases the throw. When released,
the discus travels along tangent to the circular spin orbit.

In the given figure, AB is one such tangent to a circle of radius 75 cm. Point
O is centre of the circle and ABO = 300 . PQ is parallel to OA.

MS_X_Mathematics_041_30/1/1_2022-23
20
Based on above information:
(a) find the length of AB.
(b) find the length of OB.
(c) find the length of AP.
OR
Find the length of PQ

Sol. 1 75 1
(i)tan 30 = =
3 AB 2
 AB = 75 3 cm 1
2
1 75 1
(ii)sin 30 = 2 = 𝑂𝐵
2
1
 OB = 150 cm
2

(iii) QB = 150 – 75 = 75 cm 1
 Q is mid point. of OB

Since PQ ll AO therefore P is mid point of AB


75 3 1
Hence AP = cm.
2
OR
1
(iii) QB = 150 – 75 = 75 cm
2
Now,  BQP   BOA

𝑄𝐵 𝑃𝑄
 =
𝑂𝐵 𝑂𝐴
1 𝑃𝑄
2= 1
75
75 1
 PQ = cm
2 2

21
MS_X_Mathematics_041_30/1/1_2022-23
MS_X_Mathematics_041_30/1/1_2022-23
22
Marking Scheme
Strictly Confidential
(For Internal and Restricted use only)
Secondary School Examination, 2023
MATHEMATICS PAPER CODE 30/1/2
General Instructions: -

1 You are aware that evaluation is the most important process in the actual and correct
assessment of the candidates. A small mistake in evaluation may lead to serious problems
which may affect the future of the candidates, education system and teaching profession. To
avoid mistakes, it is requested that before starting evaluation, you must read and understand
the spot evaluation guidelines carefully.
2 “Evaluation policy is a confidential policy as it is related to the confidentiality of the
examinations conducted, Evaluation done and several other aspects. Its’ leakage to
public in any manner could lead to derailment of the examination system and affect the
life and future of millions of candidates. Sharing this policy/document to anyone,
publishing in any magazine and printing in News Paper/Website etc may invite action
under various rules of the Board and IPC.”
3 Evaluation is to be done as per instructions provided in the Marking Scheme. It should not
be done according to one’s own interpretation or any other consideration. Marking Scheme
should be strictly adhered to and religiously followed. However, while evaluating, answers
which are based on latest information or knowledge and/or are innovative, they may be
assessed for their correctness otherwise and due marks be awarded to them.
4 The Marking scheme carries only suggested value points for the answers.
These are in the nature of Guidelines only and do not constitute the complete answer. The
students can have their own expression and if the expression is correct, the due marks should
be awarded accordingly.
5 The Head-Examiner must go through the first five answer books evaluated by each evaluator
on the first day, to ensure that evaluation has been carried out as per the instructions given
in the Marking Scheme. If there is any variation, the same should be zero after deliberation
and discussion. The remaining answer books meant for evaluation shall be given only after
ensuring that there is no significant variation in the marking of individual evaluators.
6 Evaluators will mark ( √ ) wherever answer is correct. For wrong answer CROSS ‘X” be
marked. Evaluators will not put right (✓) while evaluating which gives an impression that
answer is correct and no marks are awarded. This is most common mistake which
evaluators are committing.
7 If a question has parts, please award marks on the right-hand side for each part. Marks
awarded for different parts of the question should then be totaled up and written in the left-
hand margin and encircled. This may be followed strictly.
8 If a question does not have any parts, marks must be awarded in the left-hand margin and
encircled. This may also be followed strictly.
9 In Q1-Q20, if a candidate attempts the question more than once (without canceling the
previous attempt), marks shall be awarded for the first attempt only and the other
answer scored out with a note “Extra Question”.

1
MS_X_Mathematics_041_30/1/2_2022-23
10 In Q21-Q38, if a student has attempted an extra question, answer of the question
deserving more marks should be retained and the other answer scored out with a note
“Extra Question”.
11 No marks to be deducted for the cumulative effect of an error. It should be penalized only
once.
12 A full scale of marks __________ (example 0 to 80/70/60/50/40/30 marks as given in
Question Paper) has to be used. Please do not hesitate to award full marks if the answer
deserves it.
13 Every examiner has to necessarily do evaluation work for full working hours i.e., 8 hours
every day and evaluate 20 answer books per day in main subjects and 25 answer books per
day in other subjects (Details are given in Spot Guidelines). This is in view of the reduced
syllabus and number of questions in question paper.
14 Ensure that you do not make the following common types of errors committed by the
Examiner in the past:-
● Leaving answer or part thereof unassessed in an answer book.
● Giving more marks for an answer than assigned to it.
● Wrong totaling of marks awarded on an answer.
● Wrong transfer of marks from the inside pages of the answer book to the title page.
● Wrong question wise totaling on the title page.
● Wrong totaling of marks of the two columns on the title page.
● Wrong grand total.
● Marks in words and figures not tallying/not same.
● Wrong transfer of marks from the answer book to online award list.
● Answers marked as correct, but marks not awarded. (Ensure that the right tick mark is
correctly and clearly indicated. It should merely be a line. Same is with the X for
incorrect answer.)
● Half or a part of answer marked correct and the rest as wrong, but no marks awarded.
15 While evaluating the answer books if the answer is found to be totally incorrect, it should be
marked as cross (X) and awarded zero (0) Marks.
16 Any un assessed portion, non-carrying over of marks to the title page, or totaling error
detected by the candidate shall damage the prestige of all the personnel engaged in the
evaluation work as also of the Board. Hence, in order to uphold the prestige of all concerned,
it is again reiterated that the instructions be followed meticulously and judiciously.
17 The Examiners should acquaint themselves with the guidelines given in the “Guidelines for
spot Evaluation” before starting the actual evaluation.
18 Every Examiner shall also ensure that all the answers are evaluated, marks carried over to
the title page, correctly totaled and written in figures and words.
19 The candidates are entitled to obtain photocopy of the Answer Book on request on payment
of the prescribed processing fee. All Examiners/Additional Head Examiners/Head
Examiners are once again reminded that they must ensure that evaluation is carried out
strictly as per value points for each answer as given in the Marking Scheme.

2
MS_X_Mathematics_041_30/1/2_2022-23
MARKING SCHEME
MATHEMATICS (Subject Code–041)
(PAPER CODE: 30/1/2)

Q. No. EXPECTED OUTCOMES/VALUE POINTS Marks


SECTION A
Questions no. 1 to 18 are multiple choice questions (MCQs) and questions
number 19 and 20 are Assertion-Reason based questions of 1 mark each

1. In what ratio, does x-axis divide the line segment joining the points
A(3, 6) and 𝐵(−12, −3) ?
(A) 1: 2 (B) 1: 4 (C) 4 :1 (D) 2 :1

Sol. (D) 2 : 1 1
2. In the given figure, PQ is tangent to the circle centered at O.
If AOB = 95 , then the measure of ABQ will be
(A) 47.5 (B) 42.5 (C) 85 (D) 95

Sol. (A) 47·5 1


4sin A + 3cos A
3. If 2 tan A = 3 , then the value of
4sin A − 3cos A
7 1
(A) (B) (C) 3 (D) does not exist
13 13

Sol. (C) 3 1

3
MS_X_Mathematics_041_30/1/2_2022-23
4. In a group of 20 people, 5 can't swim. If one person is selected at random,
then the probability that he/she can swim, is
3 1 1
(A) (B) (C) 1 (D)
4 3 4

Sol. (A) 3/4 1


5. The distribution below gives the marks obtained by 80 students on a test:
Marks Less Less Less Less Less Less
than 10 than than than 40 than than
20 30 50 60
Number of 3 12 27 57 75 80
Students
The modal class of this distribution is :
(A) 10-20 (B) 20 - 30
(C) 30-40 (D) 50 - 60
Sol. (C) 30 – 40 1

6. The curved surface area of a cone having height 24 cm and radius


7 cm, is
(A) 528 cm2 (B) 1056 cm2 (C) 550 cm2 (D) 500 cm2

Sol. (C) 550 cm2 1

7. The end-points of a diameter of a circle are (2, 4) and (-3, -1). The radius of
the circle is
5 5
(A) 2 5 (B) 2 √5 (C) 2 √2 (D) 5 2
Sol. 5 2 1
(C) 2

8. 5
Which of the following is a quadratic polynomial with zeroes and 0?
3
5 5 2
(A) 3x (3x – 5) (B) 3x (x – 5) (C) x 2 − (D) x
3 3
Sol. (A) 3x (3x – 5) 1

4
MS_X_Mathematics_041_30/1/2_2022-23
9. The graph of y = p ( x ) is given, for a polynomial p ( x) . The number of
zeroes of p(x) from the graph is:

(A) 3 (B) 1 (C) 2 (D) 0

Sol. (B) 1 1
10. The value of k for which the pair of equations kx = y + 2 and 6 x = 2 y + 3
has infinitely many solutions,
(A) is k = 3 (B) does not exist (C) is k = −3 (D) is k = 4

Sol. (B) does not exist 1


11. If a, b , c form a A.P. with common difference d, then the value of a - 2b – c
is equal to
(A) 2a + 4d (B) 0 (C) − 2a − 4d (D) − 2a − 3d
Sol. (C) − 2a − 4d 1

12. If the value of each observation of a statistical data is increased by 3, then


the mean of the data
(A) remains unchanged (B) increases by 3
(C) increases by 6 (D) increases by 3n
Sol. (B) increases by 3 1
13. Probability of happening of an event is denoted by p and probability of non-
happening of the event is denoted by q . Relation between p and q is
(A) p + q = 1 (B) p = 1, q = 1 (C) p = q − 1 (D) p + q + 1 = 0

Sol. (A) p + q = 1 1

14. A girl calculates that the probability of her winning the first prize in a lottery
is 0.08. If 6000 tickets are sold, how many tickets has she bought?
(A) 40 (B) 240 (C) 480 (D) 750
5
MS_X_Mathematics_041_30/1/2_2022-23
Sol. (C) 480 1
If  ,  are the zeroes of a polynomial p( x) = x + x − 1 , then
2
15.
1 1
+𝛽 equals to
𝛼

(C) − 1
−1
(A) 1 (B) 2 (D) 2

Sol. (A) 1 1
16. The least positive value of k , for which the quadratic equation
2 x 2 + kx − 4 = 0 has rational roots, is

(A) 2 2 (B) 2 (C) 2 (D) 2

Sol. (B) 2 1

17. 5 2 
 8 sec 60 − tan 60 + cos 45 
2 2

is equal to
−5 −1 −1
(A) (B) (C) 0 (D)
3 2 4

Sol. (C) 0 1

Curved surface area of a cylinder of height 5 cm is 94.2 cm 2 . Radius of the


18.
cylinder is (Take  = 3.14)
(A) 2 cm (B) 3 cm (C) 2.9 cm (D) 6 cm

Sol. (B) 3cm 1

6
MS_X_Mathematics_041_30/1/2_2022-23
19. Assertion (A) : The perimeter of ABC is a rational number.
Reason (R) : The sum of the squares of two rational numbers is always
rational.

Sol. (D) Assertion (A) is false but Reason (R) is true 1

20. Assertion (A): Point P ( 0, 2 ) is the point of intersection of y-axis with the
line 3 x + 2 y = 4 .
Reason (R): The distance of point P ( 0, 2 ) from x-axis is 2 units.

Sol. (B) Both Assertion (A) and Reason (R) are correct but Reason (R) is 1
not the correct explanation of Assertion (A)

SECTION B
This section comprises of Very Short Answer (VSA) type questions of 2
marks each.
21. Find the least number which when divided by 12, 16 and 24 leaves remainder
7 in each case
Sol. LCM of 12, 16, 24 = 48 1

Required number is 48 + 7 = 55. 1

22. A bag contains 4 red, 3 blue and 2 yellow balls. One ball is drawn at random
from the bag. Find the probability that drawn ball is
(i) red (ii) yellow.

Sol. Total No of Balls=9


4 1
(i) P(drawn ball is red) =
9

7
MS_X_Mathematics_041_30/1/2_2022-23
2 1
(ii) P(drawn ball is yellow) =
9

23(a). Solve the pair of equations x=5 and y=7 graphically.


Sol. Drawing correct graph 1
Solution is x = 5, y = 7 1

OR

23(b). Using graphical method, find whether pair of equations x=0 and y = −3 is
consistent or not
Sol. Drawing correct graph 1
As 𝑥 = 0 and 𝑦 = −3 are intersecting
∴ Pair of equations is consistent 1

24(a). If sin + cos = 3 , then find the value of sin  cos .

Sol. 3
sin  + cos  =

squaring both sides

sin2  + cos2  + 2 sin cos  = 3 1


1
 1 + 2 sin  cos  = 3 2
 sin  cos  = 1 1
2

OR
1
24(b). If sin  = and cot  = 3 , then find the value of cosecα+ cosecβ
2

8
MS_X_Mathematics_041_30/1/2_2022-23
Sol. 1 1
cosec  = = 2
sin  2
1
cosec  = 1 + cot 2  = 1 + 3 = 2
1
 cosec  + cosec  = 2 + 2 or 2 ( 2 + 1) 2

25. In the given figure, XZ is parallel to BC. AZ = 3 cm, ZC = 2 cm, BM = 3 cm


and MC = 5 cm. Find the length of XY.

Sol. 𝐴𝑋 3 𝐴𝑍 ½
As 𝑋𝑍|| 𝐵𝐶 Therefore, 𝑋𝐵 = 2 = 𝑍𝐶 – (i)

 AXY   ABM ½
𝐴𝑋 𝑋𝑌 3 𝑋𝑌 ½
⇒ 𝐴𝐵 = 𝐵𝑀 or 5 = 3
9 ½
 𝑋𝑌 = 5 or 1·8 cm

SECTION C
This section comprises of Short Answer (SA) type questions of 3 marks
each.
26. The centre of a circle is (2a, a-7). Find the values of ‘a’ if the circle passes
through the point (11,-9). Radius of the circle is 5 2 cm.
Sol. (2a – 11)2 + (a – 7 + 9)2 = (5 2 )2 1

 5a2 – 40a + 75 = 0 1
 (a – 5) (5a – 15) = 0
a = 5, a = 3 1
9
MS_X_Mathematics_041_30/1/2_2022-23
27(a). (a) Two tangents TP and TQ are drawn to a circle with centre O from an
external point T. Prove that PTQ = 2OPQ .

Sol. TP = TQ
⇒  TPQ =  TQP 1
Let  PTQ be 
180° – 𝜃 𝜃 1
⇒  TPQ =  TQP = = 90 – 2
2

Now  OPT = 90


𝜃 𝜃
⇒ OPQ = 90 – (90 – ) =
2 2

 PTQ = 2  OPQ 1

OR

27(b). In the given figure, a circle is inscribed in a quadrilateral ABCD in which


B = 900 .If AD 17 cm, AB = 20 cm and DS = 3 cm, then find the radius of
the circle.

10
MS_X_Mathematics_041_30/1/2_2022-23
Sol.

DR = DS = 3 cm ½
 AR = AD – DR = 17 – 3 = 14 cm
 AQ = AR = 14 cm 1
 QB = AB – AQ = 20 – 14 = 6 cm ½
Since QB = OP = r ½
 radius = 6 cm ½

28. Half of the difference between two numbers is 2. The sum of the greater
number and twice the smaller number is 13. Find the numbers.
Sol. Let the numbers be x and y, x > y
1
Therefore (x – y) = 2 — (i) 1
2
and 2y + x = 13 — (ii) 1
Solving equations (i) and (ii)
x = 7, y = 3 1

29(a). A room is in the form of cylinder surmounted by a hemi-spherical


dome. The base radius of hemisphere is one-half the height of
 1408  3
cylindrical part. Find total height of the room if it contains  m
 21 
 22 
of air. Take   = 
 7 

Let h be height of cylindrical part and r be radius of hemisphere


Sol. ½
2 1408
Volume of room = 2 r 3 +  r 3 = 1
3 21
r = 2 ½
Therefore, h=4 ½
Height of the room is = 6m ½
11
MS_X_Mathematics_041_30/1/2_2022-23
OR

29(b). An empty cone is of radius 3 cm and height 12 cm. Ice-cream is filled


th
1
in it so that lower part of the cone which is   of the volume of the
6
cone is unfilled but hemisphere is formed on the top. Find volume of
the ice-cream. (Take  = 3.14)

Sol. 1 1
Volume of the cone = =    9 12 = 36 cm3
3
5
Volume of ice-cream in the cone = 6 × 36 × 𝜋 = 30𝜋𝑐𝑚3 ½
2 1
Volume of ice-cream on top = 3 × 27 × 𝜋 = 18𝜋𝑐𝑚3

Total volume of the ice-cream = (30 + 18 ) = 48 cm3

= 48  3.14 = 150.72cm3 ½

30. Prove that 5 is an irrational number


Sol. Let √𝟓 be a rational number.
𝐩 ½
∴ √𝟓 = , where q≠0 and let p & q be co-primes.
𝐪

5q2 = p2 ⟹ p2 is divisible by 5 ⟹ p is divisible by 5


1
⟹ p = 5a, where ‘a’ is some integer ----- (i)
25a2 = 5q2 ⟹ q2 = 5a2 ⟹q2 is divisible by 5 ⟹ q is divisible by 5
½
⟹ q = 5b, where ‘b’ is some integer ----- (ii)
(i) and (ii) leads to contradiction as ‘p’ and ‘q’ are co-primes.
1
∴ √𝟓 is an irrational number.
31. 1
Prove that (𝑐𝑜𝑠 𝑒 𝑐𝐴 − 𝑠𝑖𝑛 𝐴)(𝑠𝑒𝑐 𝐴 − 𝑐𝑜𝑠 𝐴) = 𝑐𝑜𝑡 𝐴−𝑡𝑎𝑛 𝐴

12
MS_X_Mathematics_041_30/1/2_2022-23
Sol. 1 1 ½
LHS =(sin 𝐴 – 𝑠𝑖𝑛 𝐴) (𝑐𝑜𝑠 𝐴 – 𝑐𝑜𝑠 𝐴)
1 – 𝑠𝑖𝑛 2 𝐴 1 – 𝑐𝑜𝑠 2 𝐴
½
= ×
sin A cos A
½
= sin A cos A
RHS = cos A
1 ½
sin A
+
sin A cos A

𝑠𝑖𝑛 𝐴 𝑐𝑜𝑠 𝐴
= sin2 A + 𝑐𝑜𝑠 2 A ½
= sin A cos A = LHS ½

SECTION D
This section comprises of Long Answer (LA) type questions of 5 marks
each.
32. A ladder set against a wall at an angle 450 to the ground. If the foot of the
ladder is pulled away from the wall through a distance of 4 m, its top slides
a distance of 3 m down the wall making an angle 300 with the ground. Find
the final height of the top of the ladder from the ground and length of the
ladder.

Sol.

1 for
correct
figure

𝐴𝐵 ℎ+ 3
sin 45 = 𝐵𝐷 = 𝐵𝐷
1
 BD = (h + 3) 2 ------ (i)
1 ℎ
sin 30 = 2 = 𝐶𝐸

 CE = 2h ------ (ii) 1

13
MS_X_Mathematics_041_30/1/2_2022-23
length of ladder remains same
Therefore BD = CE  (h + 3) 2 = 2h
3 2 1
h= = 3( 2 + 1)
2– 2

Final height of the top of the ladder = 3( 2 + 1) m

and length of ladder = 2h = 6( 2 + 1) m 1

33(a). The ratio of the 11th term to 17th term of an A.P. is 3:4. Find the ratio of
5th term to 21st term of the same A.P. Also, find the ratio of the sum of
first 5 terms to that of first 21 terms.

Sol. a + 10 d 3 1
Given =
a + 16 d 4
 4a + 40d = 3a + 48d
1
 a = 8d (i)
a a + 4d 3
therefore 5 = =7 using( i) 1
a 21 a + 20 d

a5 : a21 = 3 : 7
5
s5 (2a + 4 d ) 5  20 d 25
= 2 = = 2
s 21 21
(2a + 20 d ) 21  36 d 189
2
Therefore, S5:S21=25:189

OR

14
MS_X_Mathematics_041_30/1/2_2022-23
33(b). 250 logs are stacked in the following manner:
22 logs in the bottom row, 21 in the next row, 20 in the row next to it
and so on (as shown by an example). In how many rows, are the 250
logs placed and how many logs are there in the top row?

Sol. Let the number of rows be n.


A.P. formed is 22, 21, 20, 19, .........
Here a = 22, d = – 1 Sn = 250 1
n 1
 250 = [44 + (n – 1) (– 1)]
2
 n2 – 45n + 500 = 0 1
 (n – 25) (n – 20) = 0
n  25  n = 20 1
1
logs in top row = a20 = 22 + 19 (– 1) = 3

34(a). PA, QB and RC are each perpendicular to AC. If AP = x, QB = z, RC = Y,


1 1 1
AB = a and BC = b, then prove that + =
x y z

Sol. (a) CQB   CPA

15
MS_X_Mathematics_041_30/1/2_2022-23
b z 1
 = __ (i) 12
a+b x
Also  AQB   ARC
a 𝑧
 = 𝑦 __ (ii) 1
a+b 12

z z a +b
from (i) and (ii) + = =1
x y a +b 1
1 1 1
 + =
x y z 1

OR

34(b). In the given figure, CD and RS are respectively the medians of ABC and
PQR . If ABC ~ PQR then prove that:
(i) ∆ ADC~∆PSR
(ii) 𝐴𝐷 × 𝑃𝑅 = 𝐴𝐶 × 𝑃𝑆

Sol. (i)  ABC   PQR


A=P 1
AB AC 1
and =
PQ PR 2
2 AD AC 1
= 2
 2 PS PR

16
MS_X_Mathematics_041_30/1/2_2022-23
AD AC
 = and  A =  P 1
PS PR
Therefore  ADC   PSR

𝐴𝐷 𝐴𝐶
(ii)Hence = 𝑃𝑅 1
𝑃𝑆

 AD × PR = AC × PS 1

35. A chord of a circle of radius 14 cm subtends an angle of 600 at the


center. Find the area of the corresponding minor segment of the circle.
Also find the area of the major segment of the circle.

Sol. 22 60 1 3 1+1
Area of minor segment = 14 14  − 14 14 
7 360 2 2
 308  1
= − 49 3  cm2 or 17.9cm2
 3 
22 308
Area of major segment = × 14 × 14 − ( − 49√3)
7 3

308
= 616 − + 49 3 1
3
 1540 
= + 49 3  cm2 or 598.1cm2 1
 3 

SECTION E
This section comprises of 3 case-study based questions of 4 marks each.
36. The discus throw is an event in which an athlete attempts to
throw a discus. The athlete spins anti-clockwise around one and
a half times through a circle, then releases the throw. When released,
the discus travels along tangent to the circular spin orbit.

17
MS_X_Mathematics_041_30/1/2_2022-23
In the given figure, AB is one such tangent to a circle of radius 75 cm.
Point O is centre of the circle and ABO = 300 . PQ is parallel to OA.

Based on above information:


(a) find the length of AB.
(b) find the length of OB.
(c) find the length of AP.
OR
Find the length of PQ

18
MS_X_Mathematics_041_30/1/2_2022-23
Sol. 1 75 1
(i) tan 30 = =
3 AB 2
1
 AB = 75 3 cm
2
1
1 75 2
(ii) sin 30 = 2 = 𝑂𝐵
1
 OB = 150 cm
2

(iii) QB = 150 – 75 = 75 cm
 Q is mid-point. of OB 1

Since PQ ll AO therefore P is mid-point of AB


75 3 1
Hence AP = cm.
2
OR
(iii) QB = 150 – 75 = 75 cm 1
Now,  BQP   BOA 2
𝑄𝐵 𝑃𝑄
 = 𝑂𝐴
𝑂𝐵

1 𝑃𝑄
2= 1
75
1
75
 PQ = cm 2
2

While designing the school year book, a teacher asked the student that the
37.
length and width of a particular photo is increased by x units each to double
the area of the photo. The original photo is 18 cm long and 12 cm wide.
Based on the above information, answer the following questions:
(I) Write an algebraic equation depicting the above information.
(II) Write the corresponding quadratic equation in standard form.
(III) What should be the new dimensions of the enlarged photo?

19
MS_X_Mathematics_041_30/1/2_2022-23
OR
Can any rational value of x make the new area equal to 220cm 2

Sol. (i) (18 + x) (12 + x) = 2(18 ×12) 1

(ii) x2 + 30x – 216 = 0 1

(iii) Solving: x2 + 30x – 216 = 0


 (x + 36) (x – 6) = 0
1
x ≠-36   x = 6.

new dimensions are 24 cm  18 cm 1

OR
(iii) If (18 + x) (12 + x) = 220
1
then x2 + 30x – 4 = 0
1
Here D = 900 + 16 = 916 which is not a perfect square.

Thus, we can’t have any such rational value of x.

38. India meteorological department observes seasonal and annual rainfall


every year in different sub-divisions of our country.

20
MS_X_Mathematics_041_30/1/2_2022-23
It helps them to compare and analyse the results. The table given below
shows sub-division wise seasonal (monsoon) rainfall (mm) in 2018:
Rainfall (mm) Number of Sub-divisions
200-400 2
400-600 4
600-800 7
800-1000 4
1000-1200 2
1200-1400 3
1400 -1600 1
1600-1800 1
Based on the above information, answer the following questions:
(I) Write the modal class.
(II) Find the median of the given data.
OR
Find the mean rainfall in this season.
(III) If sub-division having at least 1000 mm rainfall during monsoon
season, is considered good rainfall sub-division, then how many sub-
divisions had good rainfall?

Sol. (i) Modal Class is 600-800 1

21
MS_X_Mathematics_041_30/1/2_2022-23
𝑁
(ii) = 12, median class is 600 – 800 ½
2

Rainfall xi fi cf.
200 – 400 300 2 2
400 – 600 500 4 6
600 – 800 700 7 13
800 – 1000 900 4 17
1000 – 1200 1100 2 19
1200 – 1400 1300 3 22
1400 – 1600 1500 1 23 ½ for
correct
1600 – 1800 1700 1 24 table
24

200
Median = 600 + (12 – 6)
7
5400 1
= or 771·4
7

OR
(ii)
Rainfall xi fi f ixi
200 – 400 300 2 600
400 – 600 500 4 2000
600 – 800 700 7 4900
800 – 1000 900 4 3600
1000 – 1200 1100 2 2200
1200 – 1400 1300 3 3900
1400 – 1600 1500 1 1500

22
MS_X_Mathematics_041_30/1/2_2022-23
1600 – 1800 1700 1 1700 1 for
24 20400 correct
table
20400 1
Mean = = 850
24
(iii) Sub-divisions having good rainfall = 2 + 3 + 1 + 1 = 7. 1

23
MS_X_Mathematics_041_30/1/2_2022-23
Marking Scheme
Strictly Confidential
(For Internal and Restricted use only)
Secondary School Examination, 2023
MATHEMATICS PAPER CODE 30/1/3

General Instructions: -

1 You are aware that evaluation is the most important process in the actual and correct assessment of
the candidates. A small mistake in evaluation may lead to serious problems which may affect the
future of the candidates, education system and teaching profession. To avoid mistakes, it is requested
that before starting evaluation, you must read and understand the spot evaluation guidelines carefully.
2 “Evaluation policy is a confidential policy as it is related to the confidentiality of the
examinations conducted, Evaluation done and several other aspects. Its’ leakage to public in
any manner could lead to derailment of the examination system and affect the life and future
of millions of candidates. Sharing this policy/document to anyone, publishing in any magazine
and printing in News Paper/Website etc may invite action under various rules of the Board and
IPC.”
3 Evaluation is to be done as per instructions provided in the Marking Scheme. It should not be done
according to one’s own interpretation or any other consideration. Marking Scheme should be strictly
adhered to and religiously followed. However, while evaluating, answers which are based on
latest information or knowledge and/or are innovative, they may be assessed for their
correctness otherwise and due marks be awarded to them.
4 The Marking scheme carries only suggested value points for the answers.
These are in the nature of Guidelines only and do not constitute the complete answer. The students
can have their own expression and if the expression is correct, the due marks should be awarded
accordingly.
5 The Head-Examiner must go through the first five answer books evaluated by each evaluator on the
first day, to ensure that evaluation has been carried out as per the instructions given in the Marking
Scheme. If there is any variation, the same should be zero after deliberation and discussion. The
remaining answer books meant for evaluation shall be given only after ensuring that there is no
significant variation in the marking of individual evaluators.
6 Evaluators will mark ( √ ) wherever answer is correct. For wrong answer CROSS ‘X” be marked.
Evaluators will not put right (✓) while evaluating which gives an impression that answer is correct
and no marks are awarded. This is most common mistake which evaluators are committing.
7 If a question has parts, please award marks on the right-hand side for each part. Marks awarded for
different parts of the question should then be totaled up and written in the left-hand margin and
encircled. This may be followed strictly.
8 If a question does not have any parts, marks must be awarded in the left-hand margin and encircled.
This may also be followed strictly.
9 In Q1-Q20, if a candidate attempts the question more than once (without canceling the previous
attempt), marks shall be awarded for the first attempt only and the other answer scored out
with a note “Extra Question”.
10 In Q21-Q38, if a student has attempted an extra question, answer of the question deserving
more marks should be retained and the other answer scored out with a note “Extra Question”.
11 No marks to be deducted for the cumulative effect of an error. It should be penalized only once.
12 A full scale of marks __________(example 0 to 80/70/60/50/40/30 marks as given in Question Paper)
has to be used. Please do not hesitate to award full marks if the answer deserves it.

1
MS_X_Mathematics_041_30/1/3_2022-23
13 Every examiner has to necessarily do evaluation work for full working hours i.e., 8 hours every day
and evaluate 20 answer books per day in main subjects and 25 answer books per day in other subjects
(Details are given in Spot Guidelines). This is in view of the reduced syllabus and number of
questions in question paper.
14 Ensure that you do not make the following common types of errors committed by the Examiner in
the past:-
● Leaving answer or part thereof unassessed in an answer book.
● Giving more marks for an answer than assigned to it.
● Wrong totaling of marks awarded on an answer.
● Wrong transfer of marks from the inside pages of the answer book to the title page.
● Wrong question wise totaling on the title page.
● Wrong totaling of marks of the two columns on the title page.
● Wrong grand total.
● Marks in words and figures not tallying/not same.
● Wrong transfer of marks from the answer book to online award list.
● Answers marked as correct, but marks not awarded. (Ensure that the right tick mark is correctly
and clearly indicated. It should merely be a line. Same is with the X for incorrect answer.)
● Half or a part of answer marked correct and the rest as wrong, but no marks awarded.
15 While evaluating the answer books if the answer is found to be totally incorrect, it should be marked
as cross (X) and awarded zero (0) Marks.
16 Any un assessed portion, non-carrying over of marks to the title page, or totaling error detected by
the candidate shall damage the prestige of all the personnel engaged in the evaluation work as also
of the Board. Hence, in order to uphold the prestige of all concerned, it is again reiterated that the
instructions be followed meticulously and judiciously.
17 The Examiners should acquaint themselves with the guidelines given in the “Guidelines for spot
Evaluation” before starting the actual evaluation.
18 Every Examiner shall also ensure that all the answers are evaluated, marks carried over to the title
page, correctly totaled and written in figures and words.
19 The candidates are entitled to obtain photocopy of the Answer Book on request on payment of the
prescribed processing fee. All Examiners/Additional Head Examiners/Head Examiners are once
again reminded that they must ensure that evaluation is carried out strictly as per value points for
each answer as given in the Marking Scheme.

2
MS_X_Mathematics_041_30/1/3_2022-23
MARKING SCHEME
MATHEMATICS (Subject Code–041)
(PAPER CODE: 30/1/3)

Q. No. EXPECTED OUTCOMES/VALUE POINTS Marks


SECTION A
Questions no. 1 to 18 are multiple choice questions (MCQs) and questions
number 19 and 20 are Assertion-Reason based questions of 1 mark each

1. Curved surface area of a cylinder of height 5 cm is 94.2 cm 2. Radius of the


cylinder is (Take  = 3.14)
(A) 2 cm (B) 3 cm (C) 2.9 cm (D) 6 cm
Sol. (B) 3 cm 1
2. The distribution below gives the marks obtained by 80 students on a test:
Marks Less Less Less Less Less Less
than 10 than than than 40 than than
20 30 50 60
Number of 3 12 27 57 75 80
Students
The modal class of this distribution is:
(A) 10-20 (B) 20-30
(C) 30-40 (D) 50-60
Sol. (C) 30 – 40 1
3. The area of metal sheet required to make a closed hollow cylinder of height
2.4m and base radius 0.7m, is
(A)10.56 𝑚2 (B)13.52 𝑚2 (C)13.64 𝑚2 (D)14.08 𝑚2
Sol. (C) 13.64 𝑚2 1
4. The distance between the points (0, 2 5) and (−2 5, 0) is

(A) 2 10 units (B) 4 10 units (C) 2 20 units (D) 0


Sol. (A) 2 10 units 1

3
MS_X_Mathematics_041_30/1/3_2022-23
If    are zeros of a polynomial p ( x) = 2 x − x − 1 then  +  is equal to
5. 2 2 2

−3 5 1 3
(A) 4 (B)4 (C) 4 (D) 4
Sol. 5 1
(B) 4
6. The least positive value of k , for which the quadratic equation
2 x 2 + kx − 4 = 0 has rational roots, is
(A) 2 2 (B) 2 (C) 2 (D) 2
Sol. (B) 2 1

7. 3 2 0 0
 4 tan 30 − sec 45 + sin 60  is equal to
2 0 2

5 −3 1
(A) −1 (B) (C) (D)
6 2 6
Sol. (A) – 1 1

8. The graph of y = p ( x ) is given in the adjoining figure. Zeroes of the


polynomial p ( x) are

−5 −7
(A)-5, 7 (B) ,
2 2
−5 7
(C)-5, 0, 7 (D) -5, , ,7
2 2
Sol. (C) -5, 0, 7 1
9. The value of k for which the pair of equations kx = y + 2 and 6 x = 2 y + 3
has infinitely many solutions,

(A) is k = 3 (B) does not exist (C) is k = −3 (D) is k = 4


Sol. (B) does not exist 1

10. If p − 1, p + 1 and 2p + 3 are in A.P., then the value of p is


(A) − 2 (B) 4 (C) 0 (D) 2
Sol. (C) 0 1
11. In what ratio, does x-axis divide the line segment joining the points

4
MS_X_Mathematics_041_30/1/3_2022-23
A(3, 6) and 𝐵(−12, −3) ?
(A) 1: 2 (B) 1: 4 (C) 4 :1 (D) 2 :1
Sol. (D) 2 : 1 1

12. Probability of happening of an event is denoted by p and probability of non-


happening of the event is denoted by q . Relation between p and q is
(A) 𝑝 + 𝑞 = 1 (B) 𝑝 = 1, 𝑞 = 1 (C) 𝑝 = 𝑞 − 1 (D) 𝑝 + 𝑞 + 1 = 0
Sol. (A) p + q = 1 1
13. A girl calculates that the probability of her winning the first prize in a lottery
is 0.08. If 6000 tickets are sold, how many tickets has she bought ?
(A) 40 (B) 240 (C) 480 (D) 750

Sol. (C) 480 1

14. In the given figure, PQ is tangent to the circle centred at O.


If AOB = 95 , then the measure of ABQ will be
(A) 47.5 (B) 42.5 (C) 85 (D) 95

Sol. (A) 47.5 1


15. 4sin A + 3cos A
If 2 tan A = 3 , then the value of
4sin A − 3cos A
7 1
(A) (B) (C) 3 (D) does not exist
13 13

Sol. (C) 3 1
16. In a survey, it is found that every fifth person has a vehicle. The probability
of a person NOT having a vehicle is,
1 4
(A) 5 (B) 5% (C) 5 (D) 95%

5
MS_X_Mathematics_041_30/1/3_2022-23
Sol. 4 1
(C) 5

17. If the value of each observation of a statistical data is increased by 3, then the
mean of the data
(A) remains unchanged (B) increases by 3
(C) increases by 6 (D) increases by 3n
Sol. (B) increases by 3 1

18. −2 2
Which of the following is a quadratic polynomial having zeroes and ?
3 3
4 9
(A) 4 x 2 − 9 (B) (9 x 2 + 4) (C) x 2 + (D) 5(9 x 2 − 4)
9 4
Sol. (D) 5 (9x2 – 4) 1

19. Assertion (A) : The perimeter of ABC is a rational number.


Reason (R) : The sum of the squares of two rational numbers is always
rational.

Sol. (D) Assertion (A) is false but Reason ( R) is true 1

20. Assertion (A): Point P ( 0, 2 ) is the point of intersection of y-axis with the
line 3 x + 2 y = 4 .
Reason (R) : The distance of point P ( 0, 2 ) from x-axis is 2 units.

6
MS_X_Mathematics_041_30/1/3_2022-23
Sol. (B) Both Assertion (A) and Reason ( R) are correct but Reason ( R) is not the 1
correct explanation of Assertion (A)
SECTION B
This section comprises of Very Short Answer (VSA) type questions of 2
marks each.
21(a). If 𝑠𝑖𝑛 𝜃 + 𝑠𝑖𝑛2 𝜃 = 1, then prove that 𝑐𝑜𝑠 2 𝜃 + 𝑐𝑜𝑠 4 𝜃 = 1.
Sol.
sin  + sin2  = 1
1
 sin  = 1 – sin2  = cos2 
2

 cos2  + cos4  = cos2  (1 + cos2 )


1
= sin  (1 + sin )
1
= sin  + sin2 = 1 2

OR
21(b). 1 cos ec 2 − sec2  3
If tan  = , then show that =
7 cos ec 2 + sec2  4
Sol.
1 8
sec2  = 1 + = 1
7 7
2
1
cot  = 7  cosec2  = 1 + 7 = 8
2
8
8–
 LHS = 7 = 48
8 64
8+ 1
7
3
= =RHS
4

22. Find the greatest number which divides 85 and 72 leaving remainders 1 and
2 respectively.

7
MS_X_Mathematics_041_30/1/3_2022-23
Sol.
We have to find HCF of 85 – 1 = 84 and 72 – 2 = 70. 1
1
HCF of 84 and 70 = 14

23. A bag contains 4 red, 3 blue and 2 yellow balls. One ball is drawn at random
from the bag. Find the probability that drawn ball is
(i) red (ii) yellow.
Sol. Total No of Balls=9
4 1
(i) P(drawn ball is red) =
9
2 1
(ii) P(drawn ball is yellow) =
9

24. In the given figure, ABCD is a parallelogram. AE divides the line segment
BD in the ratio 1 : 2. If BE = 1.5cm, then find the length of BC.

Sol.
 OBE   ODA 1
2
𝑂𝐵 𝐵𝐸
 𝑂𝐷 = 𝐴𝐷 1
2
1 𝐵𝐸 1
 2 = 𝐵𝐶 (AD = BC)
2
1
BE = 1·5 cm  BC = 3 cm
2
25(a). Solve the pair of equations x = 3 and y = −4 graphically.
Sol.
Correct graph of both the equations. 1
Solution of equation is x = 3, y = – 4 1

8
MS_X_Mathematics_041_30/1/3_2022-23
OR
25(b). Using graphical method, find whether following system of linear equations
is consistent or not:
𝑥 = 0 and 𝑦 = −7
Sol.
Correct graph of y = – 7 and x = 0 1
As y = – 7 is intersecting x = 0 at (0, – 7)

So system of equations is consistent 1

SECTION C
This section comprises of Short Answer (SA) type questions of 3 marks
each.
26(a). Two tangents TP and TQ are drawn to a circle with centre O from an
external point T. Prove that PTQ = 2OPQ .

Sol.
TP = TQ

⇒  TPQ =  TQP 1

Let  PTQ be 

180° – 𝜃 𝜃
⇒  TPQ =  TQP = = 90 – 2 1
2

Now  OPT = 90

𝜃 𝜃
⇒ OPQ = 90 – (90 – 2 ) = 2

 PTQ = 2  OPQ 1

OR
9
MS_X_Mathematics_041_30/1/3_2022-23
26(b). In the given figure, a circle is inscribed in a quadrilateral ABCD in which
B = 900 . If AD 17 cm, AB = 20 cm and DS = 3 cm, then find the radius of
the circle.

Sol.

DR = DS = 3 cm ½

 AR = AD – DR = 17 – 3 = 14 cm 1
 AQ = AR = 14 cm ½

 QB = AB – AQ = 20 – 14 = 6 cm ½

Since QB = OP = r  radius = 6 cm ½

Prove that: 2(sin  + cos  ) − 3(sin  + cos  ) + 1 = 0 .


6 6 4 4
27.
Sol.
LHS = 2(𝑠𝑖𝑛6 𝜃 + 𝑐𝑜𝑠 6 𝜃) − 3(𝑠𝑖𝑛4 𝜃 + 𝑐𝑜𝑠 4 𝜃) + 1

= 2[(𝑠𝑖𝑛2 𝜃)3 + (𝑐𝑜𝑠 2 𝜃)3 ] − 3(𝑠𝑖𝑛4 𝜃 + 𝑐𝑜𝑠 4 𝜃) + 1

= 2[(𝑠𝑖𝑛2 𝜃 + 𝑐𝑜𝑠 2 𝜃)(𝑠𝑖𝑛4 𝜃 − 𝑠𝑖𝑛2 𝜃 𝑐𝑜𝑠 2 𝜃 + 𝑐𝑜𝑠 4 𝜃)] −


1
3(𝑠𝑖𝑛4 𝜃 + 𝑐𝑜𝑠 4 𝜃) + 1

10
MS_X_Mathematics_041_30/1/3_2022-23
= 2[𝑠𝑖𝑛4 𝜃 + 𝑐𝑜𝑠 4 𝜃 − 𝑠𝑖𝑛2 𝜃 𝑐𝑜𝑠 2 𝜃] − 3(𝑠𝑖𝑛4 𝜃 + 𝑐𝑜𝑠 4 𝜃) + 1

= −[𝑠𝑖𝑛4 𝜃 + 𝑐𝑜𝑠 4 𝜃 + 2 𝑠𝑖𝑛2 𝜃 𝑐𝑜𝑠 2 𝜃] + 1 1

= −(𝑠𝑖𝑛2 𝜃 + 𝑐𝑜𝑠 2 𝜃)2 + 1

= −1 + 1 = 0 1
28(a). A room is in the form of cylinder surmounted by a hemi-spherical dome.
The base radius of hemisphere is one-half the height of cylindrical part.
 1408  3
Find total height of the room if it contains   m of air. Take
 21 
 22 
 = 
 7 

Sol. Let h be height of cylindrical part and r be radius of hemisphere


½
2 1408
Volume of room = 2 r 3 +  r 3 = 1
3 21
½
⇒r = 2
½
Therefore, h=4
Height of the room is = 6m ½

OR
28(b). An empty cone is of radius 3 cm and height 12 cm. Ice-cream is filled
th
1
in it so that lower part of the cone which is   of the volume of the
6
cone is unfilled but hemisphere is formed on the top. Find volume of the
ice-cream. (Take  = 3.14)

11
MS_X_Mathematics_041_30/1/3_2022-23
Sol. 1 1
Volume of the cone = =    9 12 = 36 cm3
3
5
Volume of ice-cream in the cone = 6 × 36 × 𝜋 = 30𝜋𝑐𝑚3 ½
2
Volume of ice-cream on top = 3 × 27 × 𝜋 = 18𝜋𝑐𝑚3 1
Total volume of the ice-cream = (30 + 18 ) = 48 cm 3

= 48  3.14 = 150.72cm3 ½

29. If ( −5, 3) and (5,3) are two vertices of an equilateral triangle, then find
coordinates of the third vertex, given that origin lies inside the triangle. (Take
3 = 1.7)

12
MS_X_Mathematics_041_30/1/3_2022-23
Sol.
Let the third vertex be (x,y)
A (-5,3) B(5,3) C(x,y)

AB=10=AC
1

AC2=100

(-5-x)2+(3-y)2 = (5-x)2+(3-y)2 1

20x =0

x=0 ½

(3-y)2=75

3-y = ±5√3

y=3-5√3

y= -5.5

The coordinates of the third vertex are (0,-5.5) ½

30. Prove that 5 is an irrational number.


Sol. Let √𝟓 be a rational number.
𝐩 ½
∴ √𝟓 = 𝐪 , where q≠0 and let p & q be co-primes.
5q2 = p2 ⟹ p2 is divisible by 5 ⟹ p is divisible by 5 1
⟹ p = 5a, where ‘a’ is some integer ----- (i)
25a2 = 5q2 ⟹ q2 = 5a2 ⟹q2 is divisible by 5 ⟹ q is divisible by 5 ½
⟹ q = 5b, where ‘b’ is some integer ----- (ii)
(i) and (ii) leads to contradiction as ‘p’ and ‘q’ are co-primes. 1
∴ √𝟓 is an irrational number.

13
MS_X_Mathematics_041_30/1/3_2022-23
31. Jaya scored 40 marks in a test getting 3 marks for each correct answer and
losing 1 mark for each incorrect answer. Had 4 marks being awarded for each
correct answer and 2 marks were deducted for each incorrect answer then
Jaya again would have scored 40 marks. How many questions were there in
the Test?

Sol.
Let number of questions answered correctly be x

and number of questions answered wrong be y


1
Therefore 3x – y = 40 __ (i)
1
and 4x – 2y = 40 __ (ii)
2
solving, x = 20, y = 20 1
1
Total number of questions = x + y = 40 2

SECTION D
This section comprises of Long Answer (LA) type questions of 5 marks
each.
32(a). A chord of a circle of radius 14 cm subtends an angle of 600 at the centre. Find
the area of the corresponding minor segment of the circle. Also find the area
of the major segment of the circle.
Sol.
22 60 1 3
Area of minor segment = 14 14  − 14 14  1+1
7 360 2 2

 308 
= − 49 3  cm2 or 17.9cm2 1
 3 

22 308
Area of major segment = × 14 × 14 − ( − 49√3)
7 3

1
308
= 616 − + 49 3
3
 1540 
= + 49 3  cm2 or 598.1cm2 1
 3 
14
MS_X_Mathematics_041_30/1/3_2022-23
33(a). The ratio of the 11th term to 17th term of an A.P. is 3:4. Find the ratio of
5th term to 21st term of the same A.P. Also, find the ratio of the sum of
first 5 terms to that of first 21 terms.

Sol. a + 10 d 3 1
Given =
a + 16 d 4
 4a + 40d = 3a + 48d
1
 a = 8d (i)
a a + 4d 3
therefore 5 = =7 using( i) 1
a 21 a + 20 d

a5 : a21 = 3 : 7
5
s5 (2a + 4 d ) 5  20 d 25
= 2 = = 2
s 21 21
(2a + 20 d ) 21  36 d 189
2
Therefore, S5:S21=25:189

OR
33(b). 250 logs are stacked in the following manner:
22 logs in the bottom row, 21 in the next row, 20 in the row next to it
and so on (as shown by an example). In how many rows, are the 250
logs placed and how many logs are there in the top row?

Sol. Let the number of rows be n.


A.P. formed is 22, 21, 20, 19, .........
Here a = 22, d = – 1 Sn = 250
1
15
MS_X_Mathematics_041_30/1/3_2022-23
n 1
 250 = [44 + (n – 1) (– 1)]
2
1
 n2 – 45n + 500 = 0
 (n – 25) (n – 20) = 0
n  25  n = 20 1

logs in top row = a20 = 22 + 19 (– 1) = 3 1


34(a). Sides AB and AC and median AM of a ABC are proportional to sides DE
and DF and median DN of another DEF . Show that Δ𝐴𝐵𝐶 ∼ Δ𝐷𝐸𝐹
Sol.

1 mark
for
figure

Extend AM to A' so that AM = A'M and DN to D' so that DN = D'N. 1


Join A'C and D'F. 2

1
 AMB ~ A'MC

 AB = A'C.
1
Similarly, DE = D'F 2
𝐴𝐵 𝐴𝐶 𝐴𝑀
Given 𝐷𝐸 = 𝐷𝐹 = 𝐷𝑁

AC A' C AA' / 2
 DF = D' F = D D' / 2

  AA'C  DD'F 1

1=2

Similarly,  3 =  4

1+3=A=2+4=D

16
MS_X_Mathematics_041_30/1/3_2022-23
Hence  ABC   DEF (SAS) 1
OR
34(b). ABCD is a parallelogram, P is a point on side BC and DP when produced
meets AB produced at L. Prove that
𝐷𝑃 𝐷𝐶
(i) 𝑃𝐿 = 𝐵𝐿
𝐷𝐿 𝐴𝐿
(ii) 𝐷𝑃 = 𝐷𝐶
(iii) If LP : PD = 2 : 3 then find BP : BC

Sol.

(i)  DPC   LPB

𝐷𝑃 𝑃𝐶 𝐷𝐶
 = 𝑃𝐵 = 𝐵𝐿 __ (i) 1
𝑃𝐿

(ii) As BC ∥ AD

  LPB   LDA 1

In ∆DLA, AD ll BP
𝐿𝑃 𝐿𝐵
 𝐷𝑃 = 𝐴𝐵

𝐿𝑃 𝐿𝐵
 +1= +1
𝐷𝑃 𝐴𝐵

𝐷𝐿 𝐴𝐿 1
 𝐷𝑃 = 𝐴𝐵

𝐷𝐿 𝐴𝐿
 𝐷𝑃 = 𝐶𝐷 (AB = CD) 1

𝐿𝑃 𝑃𝐵
(iii) 𝐿𝐷 = 𝐴𝐷 ( LPB   LDA)

2 𝑃𝐵
 5 = 𝐵𝐶 (AD = BC) 1

17
MS_X_Mathematics_041_30/1/3_2022-23
35. An aeroplane when flying at a height of 3000 m from the ground passes
vertically above another aeroplane at an instant when the angles of
elevation of the two planes from the same point on the ground are 60 0
and 450 respectively. Find the vertical distance between the aeroplanes
at that instant. Also, find the distance of first plane from the point of
observation. (Take 3 = 1.73)

Sol.

1 mark
for
figure

Let planes be located at points C and D

3000 – h
tan 45 = 1 =  x = 3000 – h ------(i) 1
x

3000 1
tan 60 = 3 =  x = 1000 3 ------(ii)
x

Using (i) and (ii) h = 3000 – 1730

= 1270 1

∴ vertical distance between the aeroplanes = 1270 m

3 3000 1
Also sin 60 = =  z = 2000 3 = 3460
2 z

Distance of the first plane from the point of observation = 3460 m


SECTION E
This section comprises of 3 case-study based questions of 4 marks each.
18
MS_X_Mathematics_041_30/1/3_2022-23
36. India meteorological department observes seasonal and annual rainfall
every year in different sub-divisions of our country.

It helps them to compare and analyse the results. The table given below shows
sub-division wise seasonal (monsoon) rainfall (mm) in 2018:
Rainfall (mm) Number of Sub-divisions
200-400 2
400-600 4
600-800 7
800-1000 4
1000-1200 2
1200-1400 3
1400 -1600 1
1600-1800 1
Based on the above information, answer the following questions :
(I) Write the modal class.
(II) Find the median of the given data.
OR
(II) Find the mean rainfall in this season.
(III) If sub-division having at least 1000 mm rainfall during monsoon season,
is considered good rainfall sub-division, then how many sub-divisions
had good rainfall?
Sol. (i) Modal Class is 600-800 1

19
MS_X_Mathematics_041_30/1/3_2022-23
𝑁
(ii) = 12, median class is 600 – 800 ½
2

Rainfall xi fi cf.
200 – 400 300 2 2
400 – 600 500 4 6
600 – 800 700 7 13 ½ for
correct
800 – 1000 900 4 17 table
1000 – 1200 1100 2 19
1200 – 1400 1300 3 22
1400 – 1600 1500 1 23
1600 – 1800 1700 1 24
24

200
Median = 600 + (12 – 6)
7

5400
1
= or 771·4
7

OR

(ii)
Rainfall xi fi f ixi
200 – 400 300 2 600
400 – 600 500 4 2000

20
MS_X_Mathematics_041_30/1/3_2022-23
600 – 800 700 7 4900
1 for
800 – 1000 900 4 3600 correct
1000 – 1200 1100 2 2200 table

1200 – 1400 1300 3 3900


1400 – 1600 1500 1 1500
1600 – 1800 1700 1 1700
24 20400

20400
Mean = = 850 1
24

(iii) Sub-divisions having good rainfall = 2 + 3 + 1 + 1 = 7. 1


37. The discus throw is an event in which an athlete attempts to
throw a discus. The athlete spins anti-clockwise around one and
a half times through a circle, then releases the throw. When released,
the discus travels along tangent to the circular spin orbit.

In the given figure, AB is one such tangent to a circle of radius 75 cm. Point
O is centre of the circle and ABO = 300 . PQ is parallel to OA.

21
MS_X_Mathematics_041_30/1/3_2022-23
Based on above information:
(a) find the length of AB.
(b) find the length of OB.
(c) find the length of AP.
OR
Find the length of PQ

Sol. 1
1 75 2
(i)tan 30 = =
3 AB

1
 AB = 75 3 cm 2

1 75 1
(ii)sin 30 = 2 = 𝑂𝐵 2

 OB = 150 cm 1
2

(iii) QB = 150 – 75 = 75 cm

 Q is mid- point. of OB 1

Since PQ ll AO therefore P is mid-point of AB


1
75 3
Hence AP = cm.
2

OR

(iii) QB = 150 – 75 = 75 cm 1
2
Now,  BQP   BOA
𝑄𝐵 𝑃𝑄
 = 𝑂𝐴
𝑂𝐵

22
MS_X_Mathematics_041_30/1/3_2022-23
1 𝑃𝑄 1
2= 75

75
1
 PQ = cm 2
2

38. While designing the school year book, a teacher asked the student that the
length and width of a particular photo is increased by x units each to double
the area of the photo. The original photo is 18 cm long and 12 cm wide.
Based on the above information, answer the following questions:
(I) Write an algebraic equation depicting the above information.
(II) Write the corresponding quadratic equation in standard form.
(III) What should be the new dimensions of the enlarged photo?

OR
Can any rational value of x make the new area equal to 220cm 2

23
MS_X_Mathematics_041_30/1/3_2022-23
Sol.
(i) (18 + x) (12 + x) = 2(18 ×12) 1

(ii) x2 + 30x – 216 = 0


1

(iii) Solving: x2 + 30x – 216 = 0

 (x + 36) (x – 6) = 0

x  -36   x = 6. 1

new dimensions are 24 cm  18 cm 1

OR

(iii) If (18 + x) (12 + x) = 220

then x2 + 30x – 4 = 0 1

Here D = 900 + 16 = 916 which is not a perfect square. 1

Thus, we can’t have any such rational value of x.

24
MS_X_Mathematics_041_30/1/3_2022-23
Marking Scheme
Strictly Confidential
(For Internal and Restricted use only)
Secondary School Examination, 2023
MATHEMATICS PAPER CODE 30/2/1

General Instructions: -

1 You are aware that evaluation is the most important process in the actual and correct
assessment of the candidates. A small mistake in evaluation may lead to serious problems
which may affect the future of the candidates, education system and teaching profession. To
avoid mistakes, it is requested that before starting evaluation, you must read and understand
the spot evaluation guidelines carefully.
2 “Evaluation policy is a confidential policy as it is related to the confidentiality of the
examinations conducted, Evaluation done and several other aspects. Its’ leakage to
public in any manner could lead to derailment of the examination system and affect the
life and future of millions of candidates. Sharing this policy/document to anyone,
publishing in any magazine and printing in News Paper/Website etc may invite action
under various rules of the Board and IPC.”
3 Evaluation is to be done as per instructions provided in the Marking Scheme. It should not
be done according to one’s own interpretation or any other consideration. Marking Scheme
should be strictly adhered to and religiously followed. However, while evaluating, answers
which are based on latest information or knowledge and/or are innovative, they may be
assessed for their correctness otherwise and due marks be awarded to them.
4 The Marking scheme carries only suggested value points for the answers.
These are in the nature of Guidelines only and do not constitute the complete answer. The
students can have their own expression and if the expression is correct, the due marks should
be awarded accordingly.
5 The Head-Examiner must go through the first five answer books evaluated by each evaluator
on the first day, to ensure that evaluation has been carried out as per the instructions given
in the Marking Scheme. If there is any variation, the same should be zero after deliberation
and discussion. The remaining answer books meant for evaluation shall be given only after
ensuring that there is no significant variation in the marking of individual evaluators.
6 Evaluators will mark (✓) wherever answer is correct. For wrong answer CROSS ‘X” be
marked. Evaluators will not put right (✓) while evaluating which gives an impression that
answer is correct and no marks are awarded. This is most common mistake which
evaluators are committing.
7 If a question has parts, please award marks on the right-hand side for each part. Marks
awarded for different parts of the question should then be totaled up and written in the left-
hand margin and encircled. This may be followed strictly.
8 If a question does not have any parts, marks must be awarded in the left-hand margin and
encircled. This may also be followed strictly.

1
MS_X_Mathematics_041_30/2/1_2022-23
9 In Q1-Q20, if a candidate attempts the question more than once (without canceling the previous
attempt), marks shall be awarded for the first attempt only and the other answer scored out
with a note “Extra Question”.
10 In Q21-Q38, if a student has attempted an extra question, answer of the question deserving
more marks should be retained and the other answer scored out with a note “Extra Question”.
11 No marks to be deducted for the cumulative effect of an error. It should be penalized only once.
12 A full scale of marks __________(example 0 to 80/70/60/50/40/30 marks as given in
Question Paper) has to be used. Please do not hesitate to award full marks if the answer
deserves it.
13 Every examiner has to necessarily do evaluation work for full working hours i.e., 8 hours
every day and evaluate 20 answer books per day in main subjects and 25 answer books per
day in other subjects (Details are given in Spot Guidelines). This is in view of the reduced
syllabus and number of questions in question paper.
14 Ensure that you do not make the following common types of errors committed by the
Examiner in the past:-
● Leaving answer or part thereof unassessed in an answer book.
● Giving more marks for an answer than assigned to it.
● Wrong totaling of marks awarded on an answer.
● Wrong transfer of marks from the inside pages of the answer book to the title page.
● Wrong question wise totaling on the title page.
● Wrong totaling of marks of the two columns on the title page.
● Wrong grand total.
● Marks in words and figures not tallying/not same.
● Wrong transfer of marks from the answer book to online award list.
● Answers marked as correct, but marks not awarded. (Ensure that the right tick mark is
correctly and clearly indicated. It should merely be a line. Same is with the X for
incorrect answer.)
● Half or a part of answer marked correct and the rest as wrong, but no marks awarded.
15 While evaluating the answer books if the answer is found to be totally incorrect, it should be
marked as cross (X) and awarded zero (0) Marks.
16 Any unassessed portion, non-carrying over of marks to the title page, or totaling error
detected by the candidate shall damage the prestige of all the personnel engaged in the
evaluation work as also of the Board. Hence, in order to uphold the prestige of all concerned,
it is again reiterated that the instructions be followed meticulously and judiciously.
17 The Examiners should acquaint themselves with the guidelines given in the “Guidelines for
spot Evaluation” before starting the actual evaluation.
18 Every Examiner shall also ensure that all the answers are evaluated, marks carried over to
the title page, correctly totaled and written in figures and words.
19 The candidates are entitled to obtain photocopy of the Answer Book on request on payment
of the prescribed processing fee. All Examiners/Additional Head Examiners/Head
Examiners are once again reminded that they must ensure that evaluation is carried out
strictly as per value points for each answer as given in the Marking Scheme.

2
MS_X_Mathematics_041_30/2/1_2022-23
MARKING SCHEME
MATHEMATICS (Subject Code–041)
(PAPER CODE: 30/2/1)

Q. No. EXPECTED OUTCOMES/VALUE POINTS Marks


SECTION A
Questions no. 1 to 18 are multiple choice questions (MCQs) and questions
number 19 and 20 are Assertion-Reason based questions of 1 mark each

1.

Sol. (b) – x2 + 4x + 4 = 0 1
2.

Sol. (a) 22 cm 1
3.

Sol. (c) 83 1


4.

Sol. (c) q 1
5.

Sol. (c) (3, 4) 1

3
MS_X_Mathematics_041_30/2/1_2022-23
6.

Sol. (d) 2 1

7.

Sol. 1 1
(c) 25
8.

Sol. (d) intersecting at (a, b) 1


9.

Sol. (a) – 4 1
10.

Sol. 3 1
(d) 4 𝜋𝑑2
4
MS_X_Mathematics_041_30/2/1_2022-23
11.

Sol. 4 1
(b) 8
12.

Sol. 9 1
(d) 2
13.

Sol. 35 ° 1
(b) ( 2 )
14.

Sol. (d) – 1, – 3 1

5
MS_X_Mathematics_041_30/2/1_2022-23
15.

Sol. (c) PS 1
16.

Sol. (b) a2 + 2b 1
17.

Sol. 1 1
(b) 2 ab

6
MS_X_Mathematics_041_30/2/1_2022-23
18.

Sol. (d) 1 cm 1

19.

Sol. (b) Both Assertion (A) and Reason (R) are true, but Reason (R) is not the 1
correct explanation of the Assertion (A).
20.

Sol. (d) Assertion (A) is false, but Reason (R) is true. 1

7
MS_X_Mathematics_041_30/2/1_2022-23
SECTION B
This section comprises of Very Short Answer (VSA) type questions of 2
marks each.
21.

Sol. Let us assume that 2 + 3 is rational


p
Let 2 + 3 = ; q  0 and p, q are integers ½
q
p – 2q ½
 3=
q
p and q are integers,  p – 2q is an integer
p – 2q
 is a rational number ½
q
 3 is a rational number which contradicts our assumption that 3 is an
irrational number.
½
 2 + 3 is an irrational number
22(a).

Sol. 3
4 cot2 45 – sec2 60 + sin2 60 + p = 4
2
√3 3
 4(1)2 – (2)2 + ( 2 ) + p = 4 1

3 3 ½
4–4+ +p=
4 4
p=0 ½
OR
22(b).

Sol. cos A + cos2 A = 1  cos A = 1 – cos2 A = sin2 A 1


 sin2 A + sin4 A = cos A + cos2 A (∵ sin2 A = cos A)
=1 1
23.

Sol. Let the given points be A (– 2, 3), B (8, 3) and C (6, 7)


Then, AB = 10, BC = 4 + 16 = 20 , 1
AC = 64 + 16 = 80 ½
8
MS_X_Mathematics_041_30/2/1_2022-23
 AB2 = BC2 + AC2 ½
∴ the given points are the vertices of a right angled triangle.
24(a).

Sol.

Let AB be the tower of height ‘h’.


 AC = 3 h

AB h 1
In  ABC, tan  = =
AC 3 h
1
⟹ tan  = ½
3
  = 30 ½
OR
24(b).

Sol.

Height of tower = AB
AB
In  ABC, tan 30 =
30 1
30
 AB = = 10 3
3 1
∴ Height of Tower is 10 3 m
9
MS_X_Mathematics_041_30/2/1_2022-23
25.

Sol.  BAC +  BOC = 180 1


⟹ BOC = 180– 65
⟹ BOC = 115 1
SECTION C
This section comprises of Short Answer (SA) type questions of 3 marks
each.
26(a).

Sol. 18180 = 22 × 32 × 5 × 101 ½


7575 = 3 × 52 × 101 ½
LCM = 22 × 32 × 52 × 101 = 90900 1
HCF = 3 × 5 × 101 = 1515 1
OR
26(b).

Sol. LCM of 6, 12, 18 = 36 2


So, all the three bells ring together after 36 minutes at 6 : 36 AM
1
27.

Sol. 1 1
LHS = (𝑐𝑜𝑠 – 𝑐𝑜𝑠 𝜃) (𝑠𝑖𝑛 – 𝑠𝑖𝑛 𝜃)
𝜃 𝜃

10
MS_X_Mathematics_041_30/2/1_2022-23
1 – 𝑐𝑜𝑠2 𝜃 1 – 𝑠𝑖𝑛2 𝜃
=( )( ) ½
𝑐𝑜𝑠 𝜃 𝑠𝑖𝑛 𝜃
𝑠𝑖𝑛2 𝜃 𝑐𝑜𝑠2 𝜃
= × 1
𝑐𝑜𝑠 𝜃 𝑠𝑖𝑛 𝜃
= sin  cos 
1 1
RHS = = 𝑠𝑖𝑛 𝜃 𝑐𝑜𝑠 𝜃
𝑡𝑎𝑛 𝜃 + 𝑐𝑜𝑡 𝜃 + ½
𝑐𝑜𝑠 𝜃 𝑠𝑖𝑛 𝜃
𝑐𝑜𝑠 𝜃 𝑠𝑖𝑛 𝜃
= 1
𝑠𝑖𝑛2 𝜃 + 𝑐𝑜𝑠 2 𝜃
= sin  cos 
∴ LHS = RHS
28.
Sol. PQ = QR  PQ2 = QR2
(5 – 0)2 + (– 3 – 1)2 = (x – 0)2 + (6 – 1)2 1
 25 + 16 = x2 + 25 1
 x2 = 16
 x = 4, x = – 4 ½+½
29.

Sol. 22 120 
Area cleaned by 1 blade = × 21 × 21 × 1½
7 360 
= 462 1
 Total area cleaned = 2 × 462 = 924 ½
∴ Total area cleaned is 924 cm2
30 (a).

Sol. system has infinite number of solutions


2 3 7
 2𝑎 = 𝑎 + 𝑏 = 28 1
1 1
𝑎=4a=4 1
and a + b = 12  b = 8
1
OR
11
MS_X_Mathematics_041_30/2/1_2022-23
30(b).

Sol. 217 x + 131 y = 913 


131 x + 217 y = 827 
Adding 348 (x + y) = 1740

x+y=5 1
Subtracting, 86 (x – y) = 86
x–y=1 1
 x = 3, y = 2 ½+½
31.

Sol. OA = OP
 In  OAP,  OPA =  OAP ... (i) 1
  OPA +  APR = 90 1
  OAP +  APR = 90 Using (i) ½
  QAP +  APR = 90 ½
SECTION D
This section comprises of Long Answer (LA) type questions of 5 marks
each.
32.

Sol. 45, 39, 33, .......


a = 45, d = – 6 ½
Sn = 180
n
180 = [2 × 45 + (n – 1) (– 6)]
2
n
 180 = [90 – 6n + 6] 1
2
12
MS_X_Mathematics_041_30/2/1_2022-23
 360 = 96n – 6n2
 6n2 – 96n + 360 = 0 1
 n2 – 16n + 60 = 0  (n – 10) (n – 6) = 0 1
n – 10 = 0, n – 6 = 0  n = 10, 6 1
We get two values of ‘n’ as sum of 7th term to 10th term is zero as some terms
are negative and some are positive. ½
33(a).

Sol. 1 for
correct
figure

PQ = Height of Light house = 75 m


 XQS =  QSP = 30
 XQR =  QRP = 60
R and S are position of ships.
In  PQR,
75 75
= tan 60 = 3  PR = = 25 3 1½
PR 3
75
In  PQS, = tan 30
PS
 PS = 75 3 1

13
MS_X_Mathematics_041_30/2/1_2022-23
 Distance between the ships, RS = PS – PR
= 75 3 – 25 3 = 50 3 1
= 50 × 1·73 = 86·5 ½
∴ Distance between the ships is 86.5 m
OR
33(b).

Sol. 1 for
correct
figure

Height of building AB = 30 m
BP = transmission tower = h(say)
 ACB = 30,  ACP = 60
AB 
In  ABC , tan 30  =
AC 

 AC = 30 3 
1 30 1½
 =
3 AC 
AP
In  APC, tan 60 =
AC

14
MS_X_Mathematics_041_30/2/1_2022-23
30 + h
3=  30 3 × 3 = 30 + h 1½
30 3
 h = 30 (3 – 1)
 h = 60 1
 Height of transmission tower = 60 m
34.

Sol.
Number of xi fi xifi c.f.
cars
0 – 10 5 7 35 7
10 – 20 15 14 210 21
20 – 30 25 13 325 34
30 – 40 35 12 420 46
40 – 50 45 20 900 66
50 – 60 55 11 605 77
60 – 70 65 15 975 92
70 – 80 75 8 600 100
Total 100 4070
Correct table 2
 x ifi 4070
Mean = = = 40·7 1
 fi 100
Median class : 40 – 50 ½
50 – 46
Median = 40 + × 10 = 42 1½
20
35(a).

15
MS_X_Mathematics_041_30/2/1_2022-23
Sol. 1 for
correct
figure

In  ABC and  PQR


𝐴𝐵 𝐵𝐶 𝐴𝐷
= 𝑄𝑅 = 𝑃𝑀
𝑃𝑄
AB 2 BD AD
= = 1
PQ 2 QM PM
(∵ D is midpoint of BC and M is midpoint of QR)
AB BD AD
= =   ABD   PQM 1
PQ QM PM
  B =  Q ⎯ (i) ½
Now, In  ABC and  PQR
𝐴𝐵 𝐵𝐶
= (given)
𝑃𝑄 𝑄𝑅
B=Q from (i) ½
  ABC   PQR 1
OR
35(b).

16
MS_X_Mathematics_041_30/2/1_2022-23
Sol. 1 for
correct
figure

In  BMC and  EMD


MC = MD
 CMB =  EMD
 MBC =  MED
  BMC ≌  EMD 1
 BC = DE
But AD = BC
 AD = DE
 AE = 2 BC 1
 AEL   CBL ½
𝐸𝐿 𝐴𝐸
 𝐵𝐿 = 𝐵𝐶
𝐸𝐿 2𝐵𝐶
⟹ 𝐵𝐿 = ½
𝐵𝐶
𝐸𝐿
⟹ 𝐵𝐿 = 2
 EL = 2 BL 1

17
MS_X_Mathematics_041_30/2/1_2022-23
SECTION E
This section comprises of 3 case-study based questions of 4 marks each.
36.

Sol. 22 90 77
(i)Area of sector ODCO = ×7×7× = or 38·5 ½+½
7 360 2
77
∴ Area of sector ODCO is or 38·5 cm2
2
1
(ii) ar ( AOB) = × 10 × 10 = 50 1
2
∴ ar ( AOB) is 50 cm2
(iii) (a) Required cost = (50 – 38·5) × 20 1
= 230 1
∴ required cost is ₹ 230.
OR
90 22
(iii) (b) Length of arc CD = ×2× ×7 1
360 7
= 11 1
∴ Length of arc CD is 11 cm.
18
MS_X_Mathematics_041_30/2/1_2022-23
37.

Sol. 22 7 7 77
(i) Area of base of the cylindrical cup = × × = or 38.5 1
7 2 2 2
77
∴ Area of base of the cylindrical cup is or 38.5 cm2
2
2 22 21 21 21
(ii) (a) Capacity of hemispherical cup = × × × × 1
3 7 2 2 2
4851
= or 2425.5 1
2
4851
∴ Capacity of hemispherical cup is cm3 or 2425.5 cm3
2
OR
22
(ii) (b) Capacity of cylindrical cup = 7 × (7)2 × 14 1
= 539 1
∴ Capacity of cylindrical cup is 539 cm3
22 7
(iii) External Curved surface area of cylindrical cup = 2 × 7 × 2 ×14 = 308 1
∴ External Curved surface area of cylindrical cup is 308 cm2

19
MS_X_Mathematics_041_30/2/1_2022-23
38.

Sol. 80
(i) P (more than 100 computers) = or 0·08 1
1000
(ii)(a) 50 or fewer computers = 250 + 200 + 290 = 740 1
740 1
Required probability = or 0·74
1000
OR
(ii)(b) No more than 20 computers = 250 + 200 = 450 1
450 1
Required probability = or 0·45
1000
250 1
(iii) P (10 or less than 10 computer) = or 0·25
1000
20
MS_X_Mathematics_041_30/2/1_2022-23
Marking Scheme
Strictly Confidential
(For Internal and Restricted use only)
Secondary School Examination, 2023
MATHEMATICS PAPER CODE 30/2/2

General Instructions: -

1 You are aware that evaluation is the most important process in the actual and correct
assessment of the candidates. A small mistake in evaluation may lead to serious problems
which may affect the future of the candidates, education system and teaching profession. To
avoid mistakes, it is requested that before starting evaluation, you must read and understand
the spot evaluation guidelines carefully.
2 “Evaluation policy is a confidential policy as it is related to the confidentiality of the
examinations conducted, Evaluation done and several other aspects. Its’ leakage to
public in any manner could lead to derailment of the examination system and affect the
life and future of millions of candidates. Sharing this policy/document to anyone,
publishing in any magazine and printing in News Paper/Website etc may invite action
under various rules of the Board and IPC.”
3 Evaluation is to be done as per instructions provided in the Marking Scheme. It should not
be done according to one’s own interpretation or any other consideration. Marking Scheme
should be strictly adhered to and religiously followed. However, while evaluating, answers
which are based on latest information or knowledge and/or are innovative, they may be
assessed for their correctness otherwise and due marks be awarded to them.
4 The Marking scheme carries only suggested value points for the answers.
These are in the nature of Guidelines only and do not constitute the complete answer. The
students can have their own expression and if the expression is correct, the due marks should
be awarded accordingly.
5 The Head-Examiner must go through the first five answer books evaluated by each evaluator
on the first day, to ensure that evaluation has been carried out as per the instructions given
in the Marking Scheme. If there is any variation, the same should be zero after deliberation
and discussion. The remaining answer books meant for evaluation shall be given only after
ensuring that there is no significant variation in the marking of individual evaluators.
6 Evaluators will mark (✓) wherever answer is correct. For wrong answer CROSS ‘X” be
marked. Evaluators will not put right (✓) while evaluating which gives an impression that
answer is correct and no marks are awarded. This is most common mistake which
evaluators are committing.
7 If a question has parts, please award marks on the right-hand side for each part. Marks
awarded for different parts of the question should then be totaled up and written in the left-
hand margin and encircled. This may be followed strictly.
8 If a question does not have any parts, marks must be awarded in the left-hand margin and
encircled. This may also be followed strictly.

1
MS_X_Mathematics_041_30/2/2_2022-23
9 In Q1-Q20, if a candidate attempts the question more than once (without canceling the previous
attempt), marks shall be awarded for the first attempt only and the other answer scored out
with a note “Extra Question”.
10 In Q21-Q38, if a student has attempted an extra question, answer of the question deserving
more marks should be retained and the other answer scored out with a note “Extra Question”.
11 No marks to be deducted for the cumulative effect of an error. It should be penalized only once.
12 A full scale of marks __________(example 0 to 80/70/60/50/40/30 marks as given in
Question Paper) has to be used. Please do not hesitate to award full marks if the answer
deserves it.
13 Every examiner has to necessarily do evaluation work for full working hours i.e., 8 hours
every day and evaluate 20 answer books per day in main subjects and 25 answer books per
day in other subjects (Details are given in Spot Guidelines). This is in view of the reduced
syllabus and number of questions in question paper.
14 Ensure that you do not make the following common types of errors committed by the
Examiner in the past:-
● Leaving answer or part thereof unassessed in an answer book.
● Giving more marks for an answer than assigned to it.
● Wrong totaling of marks awarded on an answer.
● Wrong transfer of marks from the inside pages of the answer book to the title page.
● Wrong question wise totaling on the title page.
● Wrong totaling of marks of the two columns on the title page.
● Wrong grand total.
● Marks in words and figures not tallying/not same.
● Wrong transfer of marks from the answer book to online award list.
● Answers marked as correct, but marks not awarded. (Ensure that the right tick mark is
correctly and clearly indicated. It should merely be a line. Same is with the X for
incorrect answer.)
● Half or a part of answer marked correct and the rest as wrong, but no marks awarded.
15 While evaluating the answer books if the answer is found to be totally incorrect, it should be
marked as cross (X) and awarded zero (0) Marks.
16 Any unassessed portion, non-carrying over of marks to the title page, or totaling error
detected by the candidate shall damage the prestige of all the personnel engaged in the
evaluation work as also of the Board. Hence, in order to uphold the prestige of all concerned,
it is again reiterated that the instructions be followed meticulously and judiciously.
17 The Examiners should acquaint themselves with the guidelines given in the “Guidelines for
spot Evaluation” before starting the actual evaluation.
18 Every Examiner shall also ensure that all the answers are evaluated, marks carried over to
the title page, correctly totaled and written in figures and words.
19 The candidates are entitled to obtain photocopy of the Answer Book on request on payment
of the prescribed processing fee. All Examiners/Additional Head Examiners/Head
Examiners are once again reminded that they must ensure that evaluation is carried out
strictly as per value points for each answer as given in the Marking Scheme.

2
MS_X_Mathematics_041_30/2/2_2022-23
MARKING SCHEME
MATHEMATICS (Subject Code–041)
(PAPER CODE: 30/2/2)

Q. No. EXPECTED OUTCOMES/VALUE POINTS Marks


SECTION A
Questions no. 1 to 18 are multiple choice questions (MCQs) and questions
number 19 and 20 are Assertion-Reason based questions of 1 mark each.
1.

Sol. (b) 2 1
2.

Sol. (b) 3 : 7 1
3.

Sol. 𝟑 1
(d) 𝝅𝒅𝟐
𝟒
4.

Sol. (a) 22 cm 1

3
MS_X_Mathematics_041_30/2/2_2022-23
5.

Sol. (c) 0.1 1


6.

Sol. (d) 1 cm 1
7.

Sol. (c) q 1
8.

Sol. (c) 83 1

4
MS_X_Mathematics_041_30/2/2_2022-23
9.

Sol. (d) intersecting at (a, b) 1


10.

Sol. 𝟏 1
(b) ab
𝟐
11.

Sol. 𝟏 1
(a)
𝟗
12.

Sol. 𝟏 1
(d) a = ,c=5
𝟐

5
MS_X_Mathematics_041_30/2/2_2022-23
13.

Sol. 𝟏 1
(c) 𝟐𝟓
14.

Sol. 𝟗 1
(d) 𝟐
15.

Sol. (d) 2 1
16.

Sol. 𝟑𝟓 ° 1
(b) ( 𝟐 )

6
MS_X_Mathematics_041_30/2/2_2022-23
17.

Sol. (c) PS 1
18.

Sol. (b) a2 + 2b 1

19.

Sol. (d) Assertion (A) is false, but Reason (R) is true. 1


7
MS_X_Mathematics_041_30/2/2_2022-23
20.

Sol. (a) Both Assertion (A) and Reason (R) are true, and Reason (R) is the correct 1
explanation of the Assertion (A).
SECTION B
This section comprises of Very Short Answer (VSA) type questions of 2
marks each.
21(a).

Sol.

Let AB be the tower of height ‘h’.


 AC = 3 h
AB h
In  ABC, tan  = = 1
AC 3 h
1
⟹ tan  = ½
3
  = 30 ½
OR
21(b).

8
MS_X_Mathematics_041_30/2/2_2022-23
Sol.

Height of tower = AB

AB
In  ABC, tan 30 = 1
30
30
 AB = = 10 3 1
3
∴ Height of Tower is 10 3 m
22.

Sol.  BAC +  BOC = 180 1


⟹ BOC = 180– 65
⟹ BOC = 115 1
23(a).

Sol. 3
4 cot2 45 – sec2 60 + sin2 60 + p = 4
2
√3 3 1
 4(1)2 – (2)2 + ( 2 ) + p = 4
3 3 ½
4–4+ +p=
4 4
p=0 ½
OR
9
MS_X_Mathematics_041_30/2/2_2022-23
23(b).

Sol. cos A + cos2 A = 1  cos A = 1 – cos2 A = sin2 A 1


 sin2 A + sin4 A = cos A + cos2 A (∵ sin2 A = cos A)
=1 1
24.

Sol. Let the given points be A (– 2, 3), B (8, 3) and C (6, 7)


Then, AB = 10, BC = 4 + 16 = 20 , 1

AC = 64 + 16 = 80 ½
 AB2 = BC2 + AC2 ½
∴ the given points are the vertices of a right angled triangle.
25.

Sol. If the number 4n, for any n, were to end with digit zero, it would be divisible
by 5. So, the prime factorization of 4n should contain the prime factor 5. 1
But in 4n = (2 × 2)n = 22n, the only prime factor is 2.
 By fundamental theorem of arithmetic, there is no natural number n for
1
which 4n ends with digit zero.
SECTION C
This section comprises of Short Answer (SA) type questions of 3 marks
each.
26.

Sol. sin 2  + (1 + cos )2


LHS = 1
(1 + cos ) sin 
1 + 1 + 2 cos 
= 1
(1 + cos ) sin 
2 (1 + cos )
= ½
(1 + cos ) sin 
2
= = 2 cosec  = RHS ½
sin 

10
MS_X_Mathematics_041_30/2/2_2022-23
27(a).

Sol. system has infinite number of solutions


2 3 7 1
 = =
2𝑎 𝑎+𝑏 28
1 1
 = a=4 1
𝑎 4
and a + b = 12  b = 8 1
OR
27(b).

Sol. 217 x + 131 y = 913 


131 x + 217 y = 827 
Adding 348 (x + y) = 1740

x+y=5 1
Subtracting, 86 (x – y) = 86
x–y=1 1
 x = 3, y = 2 ½+½
28(a).

Sol. 18180 = 22 × 32 × 5 × 101 ½


7575 = 3 × 52 × 101 ½
LCM = 22 × 32 × 52 × 101 = 90900 1
HCF = 3 × 5 × 101 = 1515 1
OR
28(b).

11
MS_X_Mathematics_041_30/2/2_2022-23
Sol. LCM of 6, 12, 18 = 36 2
So, all the three bells ring together after 36 minutes at 6 : 36 AM 1
29.

Sol. OA = OP
 In  OAP,  OPA =  OAP ... (i) 1
  OPA +  APR = 90 1
  OAP +  APR = 90 Using (i) ½
  QAP +  APR = 90 ½
30.

Sol. PQ = QR  PQ2 = QR2


(5 – 0)2 + (– 3 – 1)2 = (x – 0)2 + (6 – 1)2 1
 25 + 16 = x2 + 25 1
 x2 = 16
 x = 4, x = – 4 ½+½
31.

Sol.
Diagonal of rectangle = 6 2 + 8 2 = 10 1
10
 Radius of circle r = =5 ½
2
12
MS_X_Mathematics_041_30/2/2_2022-23
Area of circle = 3·14 × 5 × 5
= 78·5 ½
Area of rectangle = 6 × 8 = 48 ½
Area of shaded region = 78·5 – 48
= 30·5 cm2 ½
∴Area of shaded region is 30.5 cm2
SECTION D
This section comprises of Long Answer (LA) type questions of 5 marks
each.
32.

Sol.
Number of xi fi xifi c.f.
cars
0 – 10 5 7 35 7
10 – 20 15 14 210 21
20 – 30 25 13 325 34
30 – 40 35 12 420 46
40 – 50 45 20 900 66
50 – 60 55 11 605 77
60 – 70 65 15 975 92
70 – 80 75 8 600 100
Total 100 4070
Correct table 2
 x ifi 4070
Mean = = = 40·7 1
 fi 100
Median class : 40 – 50 ½
50 – 46
Median = 40 + × 10 = 42 1½
20

13
MS_X_Mathematics_041_30/2/2_2022-23
33(a).

Sol. 1 for
correct
figure

In  ABC and  PQR


𝐴𝐵 𝐵𝐶 𝐴𝐷
= =
𝑃𝑄 𝑄𝑅 𝑃𝑀
AB 2 BD AD 1
= =
PQ 2 QM PM
(∵ D is midpoint of BC and M is midpoint of QR)
AB BD AD
= =   ABD   PQM 1
PQ QM PM
  B =  Q ⎯ (i) ½
Now, In  ABC and  PQR
𝐴𝐵 𝐵𝐶
= (given)
𝑃𝑄 𝑄𝑅
B=Q from (i) ½
  ABC   PQR 1
OR
33(b).

14
MS_X_Mathematics_041_30/2/2_2022-23
Sol. 1 for
correct
figure

In  BMC and  EMD


MC = MD
 CMB =  EMD
 MBC =  MED
  BMC ≌  EMD 1
 BC = DE
But AD = BC
 AD = DE
 AE = 2 BC 1
 AEL   CBL ½
𝐸𝐿 𝐴𝐸
 =
𝐵𝐿 𝐵𝐶
𝐸𝐿 2𝐵𝐶 ½
⟹ =
𝐵𝐿 𝐵𝐶
𝐸𝐿
⟹ =2
𝐵𝐿
 EL = 2 BL 1
34.

Sol. 1 + 4 + 7 + 10 + ...... + x = 287


a = 1, d = 3 Last term = x ½
 1 + (n – 1) 3 = x
x+2
3n–2=xn= ⎯ (i) 1
3
Sn = 287
n (x + 2 ) (x + 1 )
[1 + x] = 287  = 287 ⎯ using (i) 1
2 6
x2 + 3x + 2 = 1722 ½
x2 + 3x – 1720 = 0
 (x + 43) (x –40)=0 1
 x = – 43, 40
x  – 43
 x = 40 1
15
MS_X_Mathematics_041_30/2/2_2022-23
35(a).

Sol. (a)PQ = height of 1st tower = 30 m 1 for


correct
AB = height of 2nd tower = h (say) figure
 PAQ = 60,  APB = 30
Let AP = x

30 30
In  APQ, tan 60 = x= 1½
x 3
= 10 3 ½
 Distance between two towers = 10 3 m
h 1 h
In  APB, tan 30 =  = 1½
x 3 10 3
 h = 10 ½
 Height of 2nd tower = 10 m
OR

16
MS_X_Mathematics_041_30/2/2_2022-23
35(b).

Sol. 1 for
correct
figure

AB = Height of tower = 100 m


P and Q are position of cars
 XBP =  APB = 30
 YBQ =  AQB = 45
In  ABQ,
AB 100
tan 45 = 1= 1½
AQ x
 x = 100
In  ABP,
AB
tan 30 =
AP
1 100 1½
=  y = 100 3
3 y
= 100(1.73) = 173
Distance between cars = x + y
= 100 + 173 = 273 1
 Distance between cars is 273 m.

17
MS_X_Mathematics_041_30/2/2_2022-23
SECTION E
This section comprises of 3 case-study based questions of 4 marks each.
36.

Sol. 80
(i) P (more than 100 computers) = or 0·08 1
1000
(ii)(a) 50 or fewer computers = 250 + 200 + 290 = 740 1
740
Required probability = or 0·74
1000 1
OR
(ii)(b) No more than 20 computers = 250 + 200 = 450 1
450
Required probability = or 0·45 1
1000
250
(iii) P (10 or less than 10 computer) = or 0·25 1
1000
18
MS_X_Mathematics_041_30/2/2_2022-23
37.

Sol. 22 90 77
(i)Area of sector ODCO = ×7×7× = or 38·5 ½+½
7 360 2
77
∴ Area of sector ODCO is or 38·5 cm2
2
1
(ii) ar ( AOB) = × 10 × 10 = 50 1
2
∴ ar ( AOB) is 50 cm2
(iii) (a) Required cost = (50 – 38·5) × 20 1
= 230 1
∴ required cost is ₹ 230.
OR
90 22
(iii) (b) Length of arc CD = ×2× ×7 1
360 7 1
= 11
∴ Length of arc CD is 11 cm.
19
MS_X_Mathematics_041_30/2/2_2022-23
38.

Sol. 22 7 7 77
(i) Area of base of the cylindrical cup = × × = or 38.5 1
7 2 2 2
77
∴ Area of base of the cylindrical cup is or 38.5 cm2
2
2 22 21 21 21
(ii) (a) Capacity of hemispherical cup = × × × × 1
3 7 2 2 2
4851
= or 2425.5 1
2
4851
∴ Capacity of hemispherical cup is cm3 or 2425.5 cm3
2
OR
22 1
(ii) (b) Capacity of cylindrical cup = 7 × (7)2 × 14
= 539 1
∴ Capacity of cylindrical cup is 539 cm3
22 7
(iii) External Curved surface area of cylindrical cup = 2 × 7 × 2 ×14 = 308
∴ External Curved surface area of cylindrical cup is 308 cm2 1

20
MS_X_Mathematics_041_30/2/2_2022-23
Marking Scheme
Strictly Confidential
(For Internal and Restricted use only)
Secondary School Examination, 2023
MATHEMATICS PAPER CODE 30/2/3

General Instructions: -

1 You are aware that evaluation is the most important process in the actual and correct
assessment of the candidates. A small mistake in evaluation may lead to serious problems
which may affect the future of the candidates, education system and teaching profession. To
avoid mistakes, it is requested that before starting evaluation, you must read and understand
the spot evaluation guidelines carefully.
2 “Evaluation policy is a confidential policy as it is related to the confidentiality of the
examinations conducted, Evaluation done and several other aspects. Its’ leakage to
public in any manner could lead to derailment of the examination system and affect the
life and future of millions of candidates. Sharing this policy/document to anyone,
publishing in any magazine and printing in News Paper/Website etc may invite action
under various rules of the Board and IPC.”
3 Evaluation is to be done as per instructions provided in the Marking Scheme. It should not
be done according to one’s own interpretation or any other consideration. Marking Scheme
should be strictly adhered to and religiously followed. However, while evaluating, answers
which are based on latest information or knowledge and/or are innovative, they may be
assessed for their correctness otherwise and due marks be awarded to them.
4 The Marking scheme carries only suggested value points for the answers.
These are in the nature of Guidelines only and do not constitute the complete answer. The
students can have their own expression and if the expression is correct, the due marks should
be awarded accordingly.
5 The Head-Examiner must go through the first five answer books evaluated by each evaluator
on the first day, to ensure that evaluation has been carried out as per the instructions given
in the Marking Scheme. If there is any variation, the same should be zero after deliberation
and discussion. The remaining answer books meant for evaluation shall be given only after
ensuring that there is no significant variation in the marking of individual evaluators.
6 Evaluators will mark( √ ) wherever answer is correct. For wrong answer CROSS ‘X” be
marked. Evaluators will not put right (✓)while evaluating which gives an impression that
answer is correct and no marks are awarded. This is most common mistake which
evaluators are committing.
7 If a question has parts, please award marks on the right-hand side for each part. Marks
awarded for different parts of the question should then be totaled up and written in the left-
hand margin and encircled. This may be followed strictly.
8 If a question does not have any parts, marks must be awarded in the left-hand margin and
encircled. This may also be followed strictly.

1
MS_X_Mathematics_041_30/2/3_2022-23
9 In Q1-Q20, if a candidate attempts the question more than once (without canceling the previous
attempt), marks shall be awarded for the first attempt only and the other answer scored out
with a note “Extra Question”.
10 In Q21-Q38, if a student has attempted an extra question, answer of the question deserving
more marks should be retained and the other answer scored out with a note “Extra Question”.
11 No marks to be deducted for the cumulative effect of an error. It should be penalized only once.
12 A full scale of marks __________(example 0 to 80/70/60/50/40/30 marks as given in
Question Paper) has to be used. Please do not hesitate to award full marks if the answer
deserves it.
13 Every examiner has to necessarily do evaluation work for full working hours i.e., 8 hours
every day and evaluate 20 answer books per day in main subjects and 25 answer books per
day in other subjects (Details are given in Spot Guidelines). This is in view of the reduced
syllabus and number of questions in question paper.
14 Ensure that you do not make the following common types of errors committed by the
Examiner in the past:-
● Leaving answer or part thereof unassessed in an answer book.
● Giving more marks for an answer than assigned to it.
● Wrong totaling of marks awarded on an answer.
● Wrong transfer of marks from the inside pages of the answer book to the title page.
● Wrong question wise totaling on the title page.
● Wrong totaling of marks of the two columns on the title page.
● Wrong grand total.
● Marks in words and figures not tallying/not same.
● Wrong transfer of marks from the answer book to online award list.
● Answers marked as correct, but marks not awarded. (Ensure that the right tick mark is
correctly and clearly indicated. It should merely be a line. Same is with the X for
incorrect answer.)
● Half or a part of answer marked correct and the rest as wrong, but no marks awarded.
15 While evaluating the answer books if the answer is found to be totally incorrect, it should be
marked as cross (X) and awarded zero (0) Marks.
16 Any unassessed portion, non-carrying over of marks to the title page, or totaling error
detected by the candidate shall damage the prestige of all the personnel engaged in the
evaluation work as also of the Board. Hence, in order to uphold the prestige of all concerned,
it is again reiterated that the instructions be followed meticulously and judiciously.
17 The Examiners should acquaint themselves with the guidelines given in the “Guidelines for
spot Evaluation” before starting the actual evaluation.
18 Every Examiner shall also ensure that all the answers are evaluated, marks carried over to
the title page, correctly totaled and written in figures and words.
19 The candidates are entitled to obtain photocopy of the Answer Book on request on payment
of the prescribed processing fee. All Examiners/Additional Head Examiners/Head
Examiners are once again reminded that they must ensure that evaluation is carried out
strictly as per value points for each answer as given in the Marking Scheme.

2
MS_X_Mathematics_041_30/2/3_2022-23
MARKING SCHEME
MATHEMATICS (Subject Code–041)
(PAPER CODE: 30/2/3)

Q. No. EXPECTED OUTCOMES/VALUE POINTS Marks


SECTION A
Questions no. 1 to 18 are multiple choice questions (MCQs) and questions
number 19 and 20 are Assertion-Reason based questions of 1 mark each.
1.

Sol. (b) (𝟐 × 𝟓)𝐧 1


2.

Sol. 𝟏 1
(d) 𝟓
3.

Sol. (c) 10 1
4.

Sol. (a) 2 cm 1
3
MS_X_Mathematics_041_30/2/3_2022-23
5.

Sol. (c) 0 1
6.

Sol. (d) 5 cm 1
7.

Sol. (a) 22 cm 1
8.

Sol. (c) (3, 4) 1


9.

Sol. 𝟏 1
(b) 𝟐 ab
10.

Sol. 𝟑𝟓 ° 1
(b) ( 𝟐 )
4
MS_X_Mathematics_041_30/2/3_2022-23
11.

Sol. (d) – 1, – 3 1
12.

Sol. (d) 1 cm 1
13.

Sol. (c) PS 1

5
MS_X_Mathematics_041_30/2/3_2022-23
14.

Sol. (a) – 4 1
15.

Sol. 𝟒 1
(b) 𝟖
16.

Sol. (d) 2 1

17.

Sol. (c) 83 1


18.

Sol. (b) – x2 + 4x + 4 = 0 1

6
MS_X_Mathematics_041_30/2/3_2022-23
19.

Sol. (b) Both Assertion (A) and Reason (R) are true, but Reason (R) is not the 1
correct explanation of the Assertion (A).
20.

Sol. (d) Assertion (A) is false, but Reason (R) is true. 1


SECTION B
This section comprises of Very Short Answer (VSA) type questions of 2
marks each.
21(a).

Sol.

Let AB be the tower of height ‘h’.


 AC = 3 h
AB h
In  ABC, tan  = = 1
AC 3 h

7
MS_X_Mathematics_041_30/2/3_2022-23
1
⟹ tan  = ½
3
  = 30 ½
OR
21(b).

Sol.

Height of tower = AB
AB
In  ABC, tan 30 = 1
30
30
 AB = = 10 3 1
3
∴ Height of Tower is 10 3 m
22.

Sol. Let the given points be A (– 2, 3), B (8, 3) and C (6, 7)


Then, AB = 10, BC = 4 + 16 = 20 , 1
AC = 64 + 16 = 80 ½
 AB2 = BC2 + AC2 ½
∴ the given points are the vertices of a right angled triangle.
23

8
MS_X_Mathematics_041_30/2/3_2022-23
Sol.  BAC +  BOC = 180 1
⟹ BOC = 180– 65
⟹ BOC = 115 1
24(a).

Sol. 3
4 cot2 45 – sec2 60 + sin2 60 + p = 4
2
√3 3
 4(1)2 – (2)2 + ( 2 ) + p = 4 1
3 3
4–4+ +p= ½
4 4
p=0 ½
OR
24(b).

Sol. cos A + cos2 A = 1  cos A = 1 – cos2 A = sin2 A 1


 sin2 A + sin4 A = cos A + cos2 A (∵ sin2 A = cos A)
=1 1
25.

Sol. Let us assume that 6 – 7 is rational


p
 6 – 7 = ; q  0 and p, q are integers ½
q
6q – p
 7 = ½
q
p, q are integers,  6q – p is an integer
6q – p
 is a rational number ½
q
 7 is rational number which contradicts our assumption that 7 is an
irrational number
6 – 7 is an irrational number ½
SECTION C
This section comprises of Short Answer (SA) type questions of 3 marks each.
26.

9
MS_X_Mathematics_041_30/2/3_2022-23
Sol. 𝑐𝑜𝑠2 𝜃 𝑠𝑖𝑛3 𝜃
LHS = +
1 – 𝑡𝑎𝑛 𝜃 𝑠𝑖𝑛 𝜃 – 𝑐𝑜𝑠 𝜃
𝑐𝑜𝑠2 𝜃 𝑠𝑖𝑛3 𝜃
= ½
𝑠𝑖𝑛 𝜃 + 𝑠𝑖𝑛 𝜃 – 𝑐𝑜𝑠 𝜃
1–
𝑐𝑜𝑠 𝜃
𝑐𝑜𝑠3 𝜃 𝑠𝑖𝑛3 𝜃
= – 1
𝑐𝑜𝑠 𝜃 – 𝑠𝑖𝑛 𝜃 𝑐𝑜𝑠 𝜃 – 𝑠𝑖𝑛 𝜃
𝑐𝑜𝑠3 𝜃 – 𝑠𝑖𝑛3 𝜃
=
𝑐𝑜𝑠 𝜃 – 𝑠𝑖𝑛 𝜃
(𝑐𝑜𝑠 𝜃 – 𝑠𝑖𝑛 𝜃) (𝑐𝑜𝑠2 𝜃 + 𝑠𝑖𝑛2 𝜃 + 𝑐𝑜𝑠 𝜃 𝑠𝑖𝑛 𝜃) 1
=
(𝑐𝑜𝑠 𝜃 – 𝑠𝑖𝑛 𝜃)
= 1 + cos  sin  = RHS ½
27.

Sol.

3·14 × (14)2
Area of minor sector = = 102.57 1
6
1·73
Area of  AOB = (14)2 = 84·77 1
4
Area of minor segment = Area of minor sector – Area of  AOB
= 102·57 – 84·77 = 17·8 1
∴ Area of minor segment is17·8 cm2
28(a).

Sol. 18180 = 22 × 32 × 5 × 101 ½


7575 = 3 × 52 × 101 ½
LCM = 22 × 32 × 52 × 101 = 90900 1
HCF = 3 × 5 × 101 = 1515 1
OR
28(b).

10
MS_X_Mathematics_041_30/2/3_2022-23
Sol. LCM of 6, 12, 18 = 36 2
So, all the three bells ring together after 36 minutes at 6 : 36 AM 1
29.

Sol. OA = OP
 In  OAP,  OPA =  OAP ... (i) 1
  OPA +  APR = 90 1
  OAP +  APR = 90 Using (i) ½
  QAP +  APR = 90 ½
30.

Sol. PQ = QR  PQ2 = QR2


(5 – 0)2 + (– 3 – 1)2 = (x – 0)2 + (6 – 1)2 1
 25 + 16 = x2 + 25 1
 x2 = 16
 x = 4, x = – 4 ½+½
31(a).

Sol. system has infinite number of solutions


2 3 7
 = = 1
2𝑎 𝑎+𝑏 28
1 1
 = a=4 1
𝑎 4
and a + b = 12  b = 8 1
OR

11
MS_X_Mathematics_041_30/2/3_2022-23
31(b).

Sol. 217 x + 131 y = 913 


131 x + 217 y = 827 
Adding 348 (x + y) = 1740

x+y=5 1
Subtracting, 86 (x – y) = 86
x–y=1 1
 x = 3, y = 2 ½+½
SECTION D
This section comprises of Long Answer (LA) type questions of 5 marks each.
32.

Sol. Modal Class: 45 – 60 ½


Mode = 55
15 – a
55 = 45 + × 15 2
30 – (a + 10 )
a=5 1
6 + 7 + a + 15 + 10 + b = 51 1
 a + b = 13
 b = 13 – 5 = 8 ½
33.

12
MS_X_Mathematics_041_30/2/3_2022-23
Sol. ATQ,
32 + 36 + 40 + ..... = 2,000
a = 32, d =4 ½+½
Let ‘n’ be the number of months.
Sn = 2000
n
[2(32) + (n – 1) 4] = 2000 1
2
n2 + 15n – 1000 = 0 1
n2 + 40n – 25n – 1000 = 0
(n + 40) (n – 25) = 0 1
 n = – 40, n = 25
(rejecting)
 n = 25 1
So, Prerna will save ₹ 2,000, in 25 months.
34(a).

Sol. 1 for
correct
figure

In  ABC and  PQR


𝐴𝐵 𝐵𝐶 𝐴𝐷
= =
𝑃𝑄 𝑄𝑅 𝑃𝑀
AB 2 BD AD 1
= =
PQ 2 QM PM
(∵ D is midpoint of BC and M is midpoint of QR)
AB BD AD
= =   ABD   PQM 1
PQ QM PM
  B =  Q ⎯ (i) ½
Now, In  ABC and  PQR
𝐴𝐵 𝐵𝐶
= (given)
𝑃𝑄 𝑄𝑅
13
MS_X_Mathematics_041_30/2/3_2022-23
B=Q from (i) ½
  ABC   PQR 1
OR
34(b).

Sol. 1 for
correct
figure

In  BMC and  EMD


MC = MD
 CMB =  EMD
 MBC =  MED
  BMC ≌  EMD 1
 BC = DE
But AD = BC
 AD = DE
 AE = 2 BC 1
 AEL   CBL ½
𝐸𝐿 𝐴𝐸
 =
𝐵𝐿 𝐵𝐶
𝐸𝐿 2𝐵𝐶
⟹ = ½
𝐵𝐿 𝐵𝐶
𝐸𝐿
⟹ =2
𝐵𝐿
 EL = 2 BL 1
35(a).

14
MS_X_Mathematics_041_30/2/3_2022-23
Sol. 1 for
correct
figure

PQ = Height of Light house = 75 m


 XQS =  QSP = 30
 XQR =  QRP = 60
R and S are position of ships.
In  PQR,
75 75
= tan 60 = 3  PR = = 25 3 1½
PR 3
75
In  PQS, = tan 30
PS
 PS = 75 3 1
 Distance between the ships, RS = PS – PR
= 75 3 – 25 3 = 50 3 1
= 50 × 1·73 = 86·5 ½
∴ Distance between the ships is 86.5 m
OR
35(b).

15
MS_X_Mathematics_041_30/2/3_2022-23
Sol. 1 for
correct
figure

Height of building AB = 30 m
BP = transmission tower = h(say)
 ACB = 30,  ACP = 60
AB 
In  ABC , tan 30  =
AC 
 1½
 AC = 30 3 
1 30
 =
3 AC 
AP
In  APC, tan 60 =
AC
30 + h
3=  30 3 . 3 = 30 + h 1½
30 3
h = 30 (3 – 1)
h = 60 1
 Height of transmission tower = 60 m

16
MS_X_Mathematics_041_30/2/3_2022-23
SECTION E
This section comprises of 3 case-study based questions of 4 marks each.
36.

Sol. 22 90 77
(i)Area of sector ODCO = ×7×7× = or 38·5 ½+½
7 360 2
77
∴ Area of sector ODCO is cm2 or 38·5 cm2
2
1
(ii) ar ( AOB) = × 10 × 10 = 50
2 1
∴ ar ( AOB) is 50 cm2
(iii) (a) Required cost = (50 – 38·5) × 20
= 230 1
∴ required cost is ₹ 230. 1
OR
90 22 1
(iii) (b) Length of arc CD = ×2× ×7
360 7
= 11 1
∴ Length of arc CD is 11 cm.

17
MS_X_Mathematics_041_30/2/3_2022-23
37.

Sol. 22 7 7 77
(i) Area of base of the cylindrical cup = × × = or 38.5 1
7 2 2 2
77
∴ Area of base of the cylindrical cup is or 38.5 cm2
2
2 22 21 21 21
(ii) (a) Capacity of hemispherical cup = × × × × 1
3 7 2 2 2
4851
= or 2425.5 1
2
4851
∴ Capacity of hemispherical cup is cm3 or 2425.5 cm3
2
OR
22
(ii) (b) Capacity of cylindrical cup = 7 × (7)2 × 14 1
= 539 1
∴ Capacity of cylindrical cup is 539 cm3
22 7
(iii) External Curved surface area of cylindrical cup = 2 × 7 × 2 ×14 = 308 1
∴ External Curved surface area of cylindrical cup is 308 cm2
18
MS_X_Mathematics_041_30/2/3_2022-23
38.

Sol. 80
(i) P (more than 100 computers) = or 0·08 1
1000
(ii)(a) 50 or fewer computers = 250 + 200 + 290 = 740 1
740
Required probability = or 0·74 1
1000
OR
(ii)(b) No more than 20 computers = 250 + 200 = 450 1
450
Required probability = or 0·45 1
1000
250 1
(iii) P (10 or less than 10 computer) = or 0·25
1000

19
MS_X_Mathematics_041_30/2/3_2022-23
Marking Scheme
Strictly Confidential
(For Internal and Restricted use only)
Secondary School Examination, 2023
MATHEMATICS PAPER CODE 30/4/1

General Instructions: -

1 You are aware that evaluation is the most important process in the actual and correct assessment of
the candidates. A small mistake in evaluation may lead to serious problems which may affect the
future of the candidates, education system and teaching profession. To avoid mistakes, it is requested
that before starting evaluation, you must read and understand the spot evaluation guidelines carefully.
2 “Evaluation policy is a confidential policy as it is related to the confidentiality of the
examinations conducted, Evaluation done and several other aspects. Its’ leakage to public in
any manner could lead to derailment of the examination system and affect the life and future
of millions of candidates. Sharing this policy/document to anyone, publishing in any magazine
and printing in News Paper/Website etc may invite action under various rules of the Board and
IPC.”
3 Evaluation is to be done as per instructions provided in the Marking Scheme. It should not be done
according to one’s own interpretation or any other consideration. Marking Scheme should be strictly
adhered to and religiously followed. However, while evaluating, answers which are based on
latest information or knowledge and/or are innovative, they may be assessed for their
correctness otherwise and due marks be awarded to them.
4 The Marking scheme carries only suggested value points for the answers.
These are in the nature of Guidelines only and do not constitute the complete answer. The students
can have their own expression and if the expression is correct, the due marks should be awarded
accordingly.
5 The Head-Examiner must go through the first five answer books evaluated by each evaluator on the
first day, to ensure that evaluation has been carried out as per the instructions given in the Marking
Scheme. If there is any variation, the same should be zero after deliberation and discussion. The
remaining answer books meant for evaluation shall be given only after ensuring that there is no
significant variation in the marking of individual evaluators.
6 Evaluators will mark ( √ ) wherever answer is correct. For wrong answer CROSS ‘X” be marked.
Evaluators will not put right (✓) while evaluating which gives an impression that answer is correct
and no marks are awarded. This is most common mistake which evaluators are committing.
7 If a question has parts, please award marks on the right-hand side for each part. Marks awarded for
different parts of the question should then be totaled up and written in the left-hand margin and
encircled. This may be followed strictly.
8 If a question does not have any parts, marks must be awarded in the left-hand margin and encircled.
This may also be followed strictly.
9 In Q1-Q20, if a candidate attempts the question more than once (without canceling the previous
attempt), marks shall be awarded for the first attempt only and the other answer scored out
with a note “Extra Question”.
10 In Q21-Q38, if a student has attempted an extra question, answer of the question deserving
more marks should be retained and the other answer scored out with a note “Extra Question”.
11 No marks to be deducted for the cumulative effect of an error. It should be penalized only once.
12 A full scale of marks __________ (example 0 to 80/70/60/50/40/30 marks as given in Question
Paper) has to be used. Please do not hesitate to award full marks if the answer deserves it.

1
MS_X_Mathematics_041_30/4/1_2022-23
13 Every examiner has to necessarily do evaluation work for full working hours i.e., 8 hours every day
and evaluate 20 answer books per day in main subjects and 25 answer books per day in other subjects
(Details are given in Spot Guidelines). This is in view of the reduced syllabus and number of
questions in question paper.
14 Ensure that you do not make the following common types of errors committed by the Examiner in
the past:-
● Leaving answer or part thereof unassessed in an answer book.
● Giving more marks for an answer than assigned to it.
● Wrong totaling of marks awarded on an answer.
● Wrong transfer of marks from the inside pages of the answer book to the title page.
● Wrong question wise totaling on the title page.
● Wrong totaling of marks of the two columns on the title page.
● Wrong grand total.
● Marks in words and figures not tallying/not same.
● Wrong transfer of marks from the answer book to online award list.
● Answers marked as correct, but marks not awarded. (Ensure that the right tick mark is correctly
and clearly indicated. It should merely be a line. Same is with the X for incorrect answer.)
● Half or a part of answer marked correct and the rest as wrong, but no marks awarded.
15 While evaluating the answer books if the answer is found to be totally incorrect, it should be marked
as cross (X) and awarded zero (0) Marks.
16 Any un assessed portion, non-carrying over of marks to the title page, or totaling error detected by
the candidate shall damage the prestige of all the personnel engaged in the evaluation work as also
of the Board. Hence, in order to uphold the prestige of all concerned, it is again reiterated that the
instructions be followed meticulously and judiciously.
17 The Examiners should acquaint themselves with the guidelines given in the “Guidelines for spot
Evaluation” before starting the actual evaluation.
18 Every Examiner shall also ensure that all the answers are evaluated, marks carried over to the title
page, correctly totaled and written in figures and words.
19 The candidates are entitled to obtain photocopy of the Answer Book on request on payment of the
prescribed processing fee. All Examiners/Additional Head Examiners/Head Examiners are once
again reminded that they must ensure that evaluation is carried out strictly as per value points for
each answer as given in the Marking Scheme.

2
MS_X_Mathematics_041_30/4/1_2022-23
MARKING SCHEME
MATHEMATICS (Subject Code–041)
(PAPER CODE: 30/4/1)

Q. No. EXPECTED OUTCOMES/VALUE POINTS Marks


SECTION A
Questions no. 1 to 18 are multiple choice questions (MCQs) and
questions number 19 and 20 are Assertion-Reason based questions of 1
mark each

1.

Sol. (a) 1 : 2 1
2.

Sol. (a) 2, – 5 1
3.

Sol. (b) 96 1
4.

Sol. (b) 7 1
5.

Sol. 1 2 1
(c) d
8
6.

3
MS_X_Mathematics_041_30/4/1_2022-23
Sol. (a) Mode = 3 Median – 2 Mean 1

7.

Sol. (c) Coincident 1

8.

Sol. (d) 0 1

9.

Sol. (a) 60° 1


10.

Sol. 1
1 + cot 2 
(c)
cot 
11.

Sol. 1 1
(c) 6

4
MS_X_Mathematics_041_30/4/1_2022-23
12.

Sol. (b) 2.5 cm 1


13.

Sol. (d) 10 1

14.

Sol. (b) 90° 1


15.

Sol. (a) 2√3 cm 1

5
MS_X_Mathematics_041_30/4/1_2022-23
16.

Sol. (b) 20 cm 1

17.

Sol. 𝟑 1
(d) − 𝟕

18.

Sol. 𝟏𝟐 1
(d) 𝟏𝟑

6
MS_X_Mathematics_041_30/4/1_2022-23
19.

Sol. (c) Assertion (A) is true but Reason (R) is false 1

20.

Sol. (b) Both Assertion (A) and Reason (R) are true and Reason (R) is 1
not the correct explanation of Assertion (A).
SECTION B
This section comprises very short answer (VSA) type questions of 2
marks each.
21.

Sol.
Let the numbers be 2x, 3x

1
LCM = 6x = 180  x = 30
 Numbers are 60, 90
HCF (60, 90) = 30 1
22.

7
MS_X_Mathematics_041_30/4/1_2022-23
Sol.
p(x) = 6x2 + 37x – (k – 2)
𝟏
1
Let the zeroes be , 𝟐

1 (k – 2) 1
Product of zeroes =  .
=–
 6
𝟏
6=–k+2k=–4
𝟐
23(A).

Sol.
2x2 – 9x + 4 = 0

a = 2, b = – 9, c = 4
Let ,  be roots of 2x2 – 9x + 4 = 0
b 9
Sum =  +  = – = 1
a 2
c 4
Product of roots =  = = =2
a 2 1

OR
23(B).

Sol.
4x2 – 5 = 0

a = 4, b = 0, c = – 5
𝟏
Discriminant = b2 – 4ac = 0 – 4 (4) (– 5) = 80 > 0 1𝟐
 roots are real and distinct. 𝟏
𝟐
24.

8
MS_X_Mathematics_041_30/4/1_2022-23
Sol.
Total outcomes are HH, HT, TH, TT 𝟏
𝟐
Favourable outcomes are HT, TH, TT 𝟏
𝟐
3 1
P (at most one head) =
4

25(A).

Sol.
5 cos 2 60  + 4 s ec 2 30  – tan 2 45 
sin 2 30  + cos 2 30 

5(1/2)2 + 4(2/√3)2 – (1)2 𝟏


= 1𝟐
1
5/4 + 16/3 – 1 67 𝟏
= =
1 12 𝟐

OR
25(B).

Sol.
sin (A – B) = 0  A – B = 0° 𝟏
𝟐
1
cos (A + B) =  A + B = 60 𝟏
2
𝟐
 A = 30, B = 30 1

SECTION C
This section comprises of Short Answer (SA) type questions of 3 marks
each.
26(A).

9
MS_X_Mathematics_041_30/4/1_2022-23
Sol.
a = – 14, a5 = 2  a + 4d = 2 1

1
– 14 + 4d = 2  d = 4
an = 62  a + (n – 1)d = 62
– 14 + (n – 1)4 = 62  n = 20 1

OR
26(B).

Sol.
65, 61, 57, 53, ...

a = 65, d = – 4 𝟏
𝟐
Let an be the first negative term
an < 0  a + (n – 1)d < 0
1
65 + (n – 1) (– 4) < 0  69 – 4n < 0
69
n>
4 1
69
 Least positive integral value of n which satisfies n > 4 is 18
𝟏
 1st negative term of the AP = 18 𝟐

27.

Sol. Let √𝟓 be a rational number.


𝐩 ½
∴ √𝟓 = 𝐪 , where q≠0 and let p & q be co-primes.
5q2 = p2 ⟹ p2 is divisible by 5 ⟹ p is divisible by 5 1
⟹ p = 5a, where ‘a’ is some integer ----- (i)
25a2 = 5q2 ⟹ q2 = 5a2 ⟹q2 is divisible by 5 ⟹ q is divisible by 5 ½
⟹ q = 5b, where ‘b’ is some integer ----- (ii)
(i) and (ii) leads to contradiction as ‘p’ and ‘q’ are co-primes. 1
∴ √𝟓 is an irrational number.
28.

10
MS_X_Mathematics_041_30/4/1_2022-23
Sol. PA and PB are tangents drawn from the external point P to the circle with
centre O.

1 mark
for
correct
figure

In quad. OAPB,
 OAP +  APB +  OBP +  AOB = 360 𝟏

𝟏
90 +  APB + 90 +  AOB = 360 (Tangent ⊥ radius) 𝟐
𝟏
 APB +  AOB = 360 – 180 = 180 𝟐

29(A).

Sol.
sin A – 2 sin 3 A sin A (1 – 2 sin 2 A ) 1
LHS = =
3 2
2 cos A – cos A cos A (2 cos A – 1)

sin A[1 – 2(1 – cos 2 A )] sin A[1 – 2 + 2 cos 2 A ] 1


= =
cos A[2 cos 2 A – 1 ] cos A [2 cos 2 A – 1 ]
sin A[2 cos 2 A – 1 ] 1
= = tan A = RHS
2
cos A [2 cos A – 1 ]

11
MS_X_Mathematics_041_30/4/1_2022-23
OR

29(B).

Sol.
LHS = sec A (1 – sin A) (sec A + tan A)

1  1 sin A 
= (1 – sin A)  +  1
cos A  cos A cos A 
1 (1 + sin A )
= (1 – sin A) 1
cos A cos A
1 – sin 2 A cos 2 A
= = = 1=RHS 1
cos 2 A cos 2 A
30.

Sol. AB is the chord of larger circle touching the smaller circle at P.

𝟏
𝟐

OA = 5 cm, OP = 3 cm
To find AB
OP ⊥ AB (radius ⊥ tangent)
AB is the chord of larger circle and OP ⊥ AB
 AP = PB 1

In right-angled  AOP, AP2 = 52 – 32 = 16


AP = 4 cm 1

12
MS_X_Mathematics_041_30/4/1_2022-23
𝟏
 AB = 2AP = 8 cm
𝟐

31.

Sol. 𝟏
px(x – 2) + 6 = 0  px2 – 2px + 6 = 0
𝟐

a = p, b = – 2p, c = 6
𝟏
Quadratic equation has equal roots,  D = 0
𝟐

b2 – 4ac = 0  4p2 – 24p = 0 1


𝟏
4p (p – 6) = 0
𝟐

𝑝 = 0, 𝑝 = 6 𝟏
𝑝 = 0 rejected  𝑝 = 6 𝟐
SECTION D
This section comprises of Long Answer (LA) type questions of 5 marks
each.
32(A).

Sol.

1 mark
for
correct
figure

13
MS_X_Mathematics_041_30/4/1_2022-23
AB = Height of tower = 75 m

P, Q are positions of cars

 XBQ =  BQA = 30

 XBP =  BPA = 60

75 75
In  APB, tan 60 =  AP = = 25 3 1𝟐
𝟏
AP 3
75
In  AQB, tan 30 =  AQ = 75 3 𝟏
AQ 𝟏
𝟐
Distance between the cars = PQ = AQ – AP
𝟏
= 75 3 – 25 3 = 50 3 𝟐

= 50  1·73 = 86·5 m 𝟏
𝟐

OR
32(B).

14
MS_X_Mathematics_041_30/4/1_2022-23
Sol.

1 mark
for
correct
figure

Let AC be h m, BC = DE = 7 m, AB = (h−7) m
∠𝐴𝐸𝐵 = 60° and ∠𝐵𝐸𝐶 = 30°
∴ ∠𝐸𝐶𝐷 = 30°
Let CD be 𝑥 m 𝟏
1𝟐
𝐷𝐸 7
= 𝑥 = tan 30∘ ⟹ 𝑥 = 7√3
𝐶𝐷
⟹ 𝐵𝐸 = 7√3
𝐴𝐵
Again 𝐵𝐸 = tan 60° 1

ℎ−7 1
⟹ 7√3
= √3

⟹ ℎ = 28 𝟏
𝟐
∴ Height of tower = 28 𝑚

33(A).

15
MS_X_Mathematics_041_30/4/1_2022-23
Sol.

1 mark
for
correct
figure

In  ABC, D is a point on side BC such that  ADC =  BAC


In  CBA and  CDA
1
 C =  C (common)
 BAC =  ADC (given) 1

  CBA   CAD (By AA similarity) 1


 their corresponding sides are proportional
CB CA
 =  CA2 = CB . CD 1
CA CD

OR
33(B).

Sol.

1 mark
for
correct
figure

 ABC   PQR

16
MS_X_Mathematics_041_30/4/1_2022-23
AD and AM are medians of  ABC and  PQR respectively.
 ABC   PQR
AB BC
 =
PQ QR
AB 2 BD 𝟏
= 1𝟐
PQ 2QM

AB BD
=
PQ QM
Also  B =  Q ( ABC   PQR)
𝟏
  ABD   PQM (SAS similarly) 1𝟐

𝑨𝑩 𝑨𝑫
 = 1
𝑷𝑸 𝑷𝑴
34.

Sol. 3
Radius of each cone = Radius of cylinder = cm
2
Height of each cone ‘H’ = 2 cm

Height of cylinder ‘h’ = 12 – 4 = 8 cm 1

Volume of air = Volume of cylinder + Volume of 2 cones


1
= r2h + 2 r2H
3

 2  22 3 3  2  𝟏 𝟏
= r2  h + H  =   8 +  2  1𝟐+1𝟐
 3  7 2 2  3 
22 9 28
=   = 66 cm3 1
7 4 3

17
MS_X_Mathematics_041_30/4/1_2022-23
35.

Sol.
Monthly Exp. (in ₹) x fi cf d xifi

1000 – 1500 1250 24 24 –3 – 72

1500 – 2000 1750 40 64 –2 – 80

2000 – 2500 2250 33 97 –1 – 33

2500 – 3000 2750 x=28 125 0 0 2 for


correct
3000 – 3500 3250 30 155 1 30 table

3500 – 4000 3750 22 177 2 44

4000 – 4500 4250 16 193 3 48

4500 – 5000 4750 7 200 4 28

Total – 35

172 + x = 200  x = 28 1

l = lower limit of median class = 2500


N 200
= = 100
2 2
C = 97, f = 28, h = 500
18
MS_X_Mathematics_041_30/4/1_2022-23
N
–C
Median = l + 2 h
f
100 – 97
= 2500 +  500
28
3
= 2500 +  500 = 2553·6 1
28
Median Expenditure = ₹ 2553·6

35  500 1
Mean = 2750 – = 2750 – 87·5 = 2662·5
200

Mean Expenditure = ₹ 2662·5

SECTION E
This section comprises of 3 case-study based questions of 4 marks each.
36.

Sol.
𝟏
(i) 5x + 4y = 9500 ____________ (1)
𝟐
4x + 3y = 7370 ____________ (2) 𝟏
𝟐

19
MS_X_Mathematics_041_30/4/1_2022-23
(ii) (a) Solving (1) and (2), x = 980 2
 Prize Amount for Hockey = ₹ 980
OR
1
(ii) (b) On solving x = 980, y = 1,150
 Prize Amount for Cricket is more by ₹ (1,150 – 980) = ₹ 170 1

(iii) 2(x + y) = 2(980 + 1150) = 2(2130) = ₹ 4,260 1

37.

20
MS_X_Mathematics_041_30/4/1_2022-23
Sol.
𝟏 𝟏
(i) R(200, 400), S(– 200, 400) +
𝟐 𝟐

(ii) (a) side PQ = (200+200) m = 400 m 1

Area of square PQRS = 400  400

= 160000 sq. units 1


OR

(ii) (b) Diagonal PR = (400 )2 + (400 )2 1

= √3200 or 400 2 1

(iii) 𝐶 (−600,0); 𝐴(200,800); 𝑆(−200,400)

S divides CA in the ratio 𝑘: 1


𝑘(200)+1(−600)
−200 = 𝑘+1

⟹𝑘=1 1

21
MS_X_Mathematics_041_30/4/1_2022-23
38.

Sol.
(i) Total perimeter = r + 2r

22 7
=  + 7 = 18 units 1
7 2

1 1 22 7 7 77
(ii) (a) Area of parking = r2 =    = 1
2 2 7 2 2 4
22 1 44 𝟏
Area of quadrants = 2 . 22 =
7 4 7 𝟐

77 44 715 𝟏
Total Area = + = 28 or 25.54 sq. units
4 7 𝟐
22
MS_X_Mathematics_041_30/4/1_2022-23
OR

Area of playground 98 56
(ii) (b) = 77/4 = 11 = 56 : 11 1+1
Area of parking

2𝜋𝑟
(iii) Required Perimeter = 2(𝑙 + 𝑏) + 2

22 7 𝟏
= 2(14 + 7) + × 2 = 53 units
7 𝟐

Cost of fencing = 53 × 2 = ₹ 106 𝟏


𝟐

23
MS_X_Mathematics_041_30/4/1_2022-23
Marking Scheme
Strictly Confidential
(For Internal and Restricted use only)
Secondary School Examination, 2023
MATHEMATICS PAPER CODE 30/4/2

General Instructions: -

1 You are aware that evaluation is the most important process in the actual and correct assessment of
the candidates. A small mistake in evaluation may lead to serious problems which may affect the
future of the candidates, education system and teaching profession. To avoid mistakes, it is requested
that before starting evaluation, you must read and understand the spot evaluation guidelines carefully.
2 “Evaluation policy is a confidential policy as it is related to the confidentiality of the
examinations conducted, Evaluation done and several other aspects. Its’ leakage to public in
any manner could lead to derailment of the examination system and affect the life and future
of millions of candidates. Sharing this policy/document to anyone, publishing in any magazine
and printing in News Paper/Website etc may invite action under various rules of the Board and
IPC.”
3 Evaluation is to be done as per instructions provided in the Marking Scheme. It should not be done
according to one’s own interpretation or any other consideration. Marking Scheme should be strictly
adhered to and religiously followed. However, while evaluating, answers which are based on
latest information or knowledge and/or are innovative, they may be assessed for their
correctness otherwise and due marks be awarded to them.
4 The Marking scheme carries only suggested value points for the answers.
These are in the nature of Guidelines only and do not constitute the complete answer. The students
can have their own expression and if the expression is correct, the due marks should be awarded
accordingly.
5 The Head-Examiner must go through the first five answer books evaluated by each evaluator on the
first day, to ensure that evaluation has been carried out as per the instructions given in the Marking
Scheme. If there is any variation, the same should be zero after deliberation and discussion. The
remaining answer books meant for evaluation shall be given only after ensuring that there is no
significant variation in the marking of individual evaluators.
6 Evaluators will mark ( √ ) wherever answer is correct. For wrong answer CROSS ‘X” be marked.
Evaluators will not put right (✓) while evaluating which gives an impression that answer is correct
and no marks are awarded. This is most common mistake which evaluators are committing.
7 If a question has parts, please award marks on the right-hand side for each part. Marks awarded for
different parts of the question should then be totaled up and written in the left-hand margin and
encircled. This may be followed strictly.
8 If a question does not have any parts, marks must be awarded in the left-hand margin and encircled.
This may also be followed strictly.
9 In Q1-Q20, if a candidate attempts the question more than once (without canceling the previous
attempt), marks shall be awarded for the first attempt only and the other answer scored out
with a note “Extra Question”.
10 In Q21-Q38, if a student has attempted an extra question, answer of the question deserving
more marks should be retained and the other answer scored out with a note “Extra Question”.
11 No marks to be deducted for the cumulative effect of an error. It should be penalized only once.
12 A full scale of marks __________ (example 0 to 80/70/60/50/40/30 marks as given in Question
Paper) has to be used. Please do not hesitate to award full marks if the answer deserves it.

1
MS_X_Mathematics_041_30/4/2_2022-23
13 Every examiner has to necessarily do evaluation work for full working hours i.e., 8 hours every day
and evaluate 20 answer books per day in main subjects and 25 answer books per day in other subjects
(Details are given in Spot Guidelines). This is in view of the reduced syllabus and number of
questions in question paper.
14 Ensure that you do not make the following common types of errors committed by the Examiner in
the past:-
● Leaving answer or part thereof unassessed in an answer book.
● Giving more marks for an answer than assigned to it.
● Wrong totaling of marks awarded on an answer.
● Wrong transfer of marks from the inside pages of the answer book to the title page.
● Wrong question wise totaling on the title page.
● Wrong totaling of marks of the two columns on the title page.
● Wrong grand total.
● Marks in words and figures not tallying/not same.
● Wrong transfer of marks from the answer book to online award list.
● Answers marked as correct, but marks not awarded. (Ensure that the right tick mark is correctly
and clearly indicated. It should merely be a line. Same is with the X for incorrect answer.)
● Half or a part of answer marked correct and the rest as wrong, but no marks awarded.
15 While evaluating the answer books if the answer is found to be totally incorrect, it should be marked
as cross (X) and awarded zero (0)Marks.
16 Any un assessed portion, non-carrying over of marks to the title page, or totaling error detected by
the candidate shall damage the prestige of all the personnel engaged in the evaluation work as also
of the Board. Hence, in order to uphold the prestige of all concerned, it is again reiterated that the
instructions be followed meticulously and judiciously.
17 The Examiners should acquaint themselves with the guidelines given in the “Guidelines for spot
Evaluation” before starting the actual evaluation.
18 Every Examiner shall also ensure that all the answers are evaluated, marks carried over to the title
page, correctly totaled and written in figures and words.
19 The candidates are entitled to obtain photocopy of the Answer Book on request on payment of the
prescribed processing fee. All Examiners/Additional Head Examiners/Head Examiners are once
again reminded that they must ensure that evaluation is carried out strictly as per value points for
each answer as given in the Marking Scheme.

2
MS_X_Mathematics_041_30/4/2_2022-23
MARKING SCHEME
MATHEMATICS (Subject Code–041)
(PAPER CODE: 30/4/2)

Q. No. EXPECTED OUTCOMES/VALUE POINTS Marks


SECTION A
Questions no. 1 to 18 are multiple choice questions (MCQs) and questions
number 19 and 20 are Assertion-Reason based questions of 1 mark each

1.

Sol. (c) 𝑠𝑒𝑐 2 𝜃 − 𝑡𝑎𝑛2 𝜃 = 1 1


2.

Sol. (a) 3 1
3.

Sol. (b) 20 cm 1
4.

Sol. (a) 1 : 2 1
5.

Sol. 𝟏𝟐 1
(d) 𝟏𝟑

3
MS_X_Mathematics_041_30/4/2_2022-23
6.

Sol. (b) 2.5 cm 1

7.

Sol. (a) 2, – 5 1

8.

Sol. (a) 60° 1


9.

Sol. (d) 10 1
10.

Sol. 𝟏 1
(c) 𝟖d2
11.

Sol. 1
(b) 25

4
MS_X_Mathematics_041_30/4/2_2022-23
12.

Sol. (a) 40 cm 1
13.

Sol. (c) 100° 1

14.

Sol. (b) 15 1
15.

Sol. 1
(d) 0
16.

Sol. 𝟑 1
(d) − 𝟕
17.

Sol. 1
(c) Coincident
5
MS_X_Mathematics_041_30/4/2_2022-23
18.

Sol. (b) 7 1

19.

Sol. (b) Both Assertion (A) and Reason (R) are true and Reason (R) is 1
not the correct explanation of Assertion (A).
20.

Sol. (c) Assertion (A) is true but Reason (R) is false 1

SECTION B
This section comprises very short answer (VSA) type questions of 2
marks each.
21(A).

6
MS_X_Mathematics_041_30/4/2_2022-23
Sol.
5 1
+ – cot2 45 + 2 sin2 90
cot 30 2
sin 2
60 

5 1 5 4 1
= + – (1)2 + 2(1)2 = + –1+2
2 2 3 3
( 3) ( 3 / 2)
𝟗
=𝟑+1=4 1
OR
21(B).

Sol. sin  1
sin  = cos   = 1  tan  = 1  cot  = 1
cos 

tan2  + cot2  – 2 = (1)2 + (1)2 – 2 = 0 1

22.

Sol.
Total outcomes are HH, HT, TH, TT 𝟏
𝟐
Favourable outcomes are HT, TH, TT 𝟏
𝟐
3 1
P (at most one head) =
4
23.

Sol.
Let the numbers be 2x, 3x

1
LCM = 6x = 180  x = 30
 Numbers are 60, 90
1
HCF (60, 90) = 30
24(A).

7
MS_X_Mathematics_041_30/4/2_2022-23
Sol.
2x2 – 9x + 4 = 0

a = 2, b = – 9, c = 4
Let ,  be roots of 2x2 – 9x + 4 = 0
b 9
Sum =  +  = – = 1
a 2
c 4
Product of roots =  = = =2 1
a 2

OR
24(B).

Sol.
4x2 – 5 = 0

a = 4, b = 0, c = – 5
𝟏
Discriminant = b2 – 4ac = 0 – 4 (4) (– 5) = 80 > 0 1𝟐
 roots are real and distinct. 𝟏
𝟐

25.

Sol.
p(x) = 6x2 + 37x – (k – 2)
𝟏
1
Let the zeroes be , 𝟐

1 (k – 2) 1
Product of zeroes =  . =–
 6
𝟏
6=–k+2k=–4 𝟐

SECTION C
This section comprises of Short Answer (SA) type questions of 3 marks
each.

8
MS_X_Mathematics_041_30/4/2_2022-23
26.

Sol. AB is the chord of larger circle touching the smaller circle at P.

𝟏
𝟐

OA = 5 cm, OP = 3 cm
To find AB
OP ⊥ AB (radius ⊥ tangent)
AB is the chord of larger circle and OP ⊥ AB
 AP = PB 1

In right-angled  AOP, AP2 = 52 – 32 = 16


AP = 4 cm 1

𝟏
 AB = 2AP = 8 cm 𝟐

27.

Sol. PA and PB are tangents drawn from the external point P to the circle with
centre O.

9
MS_X_Mathematics_041_30/4/2_2022-23
1 mark
for
correct
figure

In quad. OAPB,
 OAP +  APB +  OBP +  AOB = 360 𝟏

𝟏
90 +  APB + 90 +  AOB = 360 (Tangent ⊥ radius) 𝟐
𝟏
 APB +  AOB = 360 – 180 = 180 𝟐

28.

Sol. 𝟏
px(x – 2) + 6 = 0  px2 – 2px + 6 = 0
𝟐

a = p, b = – 2p, c = 6
𝟏
Quadratic equation has equal roots,  D = 0
𝟐

b2 – 4ac = 0  4p2 – 24p = 0 1


𝟏
4p (p – 6) = 0
𝟐
𝑝 = 0, 𝑝 = 6
𝟏
𝑝 = 0 rejected  𝑝 = 6
𝟐

10
MS_X_Mathematics_041_30/4/2_2022-23
29(A).

Sol.
a = 15, S15 = 750

15 1
[2a + 14d] = 750
2
2(15) + 14d = 100
d=5 1

a20 = a + 19d = 15 + 19(5) = 110 1

OR

29(B).

Sol. 𝟏
A.P formed is 1000, 1100, 1200, …
𝟐
a = 1000, d = 100

a30 = a + 29d = 3900 1

Amount paid in 30th instalment = ₹ 3,900

30 𝟏
S30 = [2  1000 + 29  100] = 15  4900 = 73,500 1+𝟐
2

 Total amount paid after 30th instalment = ₹ 73,500

30.

Sol. Let √𝟑 be a rational number.


𝐩 ½
∴ √𝟑 = , where q≠0 and let p & q be co-primes.
𝐪
3q2 = p2 ⟹ p2 is divisible by 3 ⟹ p is divisible by 3
11
MS_X_Mathematics_041_30/4/2_2022-23
⟹ p = 3a, where ‘a’ is some integer ----- (i) 1
9a2 = 3q2 ⟹ q2 = 3a2 ⟹q2 is divisible by 3 ⟹ q is divisible by 3
⟹ q = 3b, where ‘b’ is some integer ----- (ii) ½
(i) and (ii) leads to contradiction as ‘p’ and ‘q’ are co-primes.
∴ √𝟑 is an irrational number. 1
31(A).

Sol.
sin A – 2 sin 3 A sin A (1 – 2 sin 2 A ) 1
LHS = =
3 2
2 cos A – cos A cos A (2 cos A – 1)

sin A[1 – 2(1 – cos 2 A )] sin A[1 – 2 + 2 cos 2 A ] 1


= =
cos A[2 cos 2 A – 1 ] cos A [2 cos 2 A – 1 ]
sin A[2 cos 2 A – 1 ] 1
= = tan A = RHS
cos A [2 cos 2 A – 1 ]
OR

31(B).

Sol.
LHS = sec A (1 – sin A) (sec A + tan A)

1  1 sin A 
= (1 – sin A)  +  1
cos A  cos A cos A 
1 (1 + sin A )
= (1 – sin A) 1
cos A cos A
1 – sin 2 A cos 2 A
= = = 1=RHS 1
cos 2 A cos 2 A

SECTION D
This section comprises of Long Answer (LA) type questions of 5 marks
each.

12
MS_X_Mathematics_041_30/4/2_2022-23
32.

Sol.
Height of cylinder h = 20 cm
𝟏
radius of cylinder = 6 cm = Radius of cone
𝟐
Height of cone = 8 cm

Slant height l = 8 2 + 6 2
= 64 + 36 = 10 cm 1

Surface area of remaining solid

= CSA of cylinder + CSA of cone + Area of base of cylinder


= 2rh + rl + r2 = r[2h + l + r]
22 22 1+1+1
=  6[2  20 + 10 + 6] =  6  56
7 7
𝟏
= 1056 cm2 𝟐

33.

Sol.
Monthly Exp. (in ₹) x fi cf d xifi

1000 – 1500 1250 24 24 –3 – 72

1500 – 2000 1750 40 64 –2 – 80

13
MS_X_Mathematics_041_30/4/2_2022-23
2000 – 2500 2250 33 97 –1 – 33

2500 – 3000 2750 x=28 125 0 0 2 for


correct
table
3000 – 3500 3250 30 155 1 30

3500 – 4000 3750 22 177 2 44

4000 – 4500 4250 16 193 3 48

4500 – 5000 4750 7 200 4 28

Total – 35

1
172 + x = 200  x = 28

l = lower limit of median class = 2500


N 200
= = 100
2 2
C = 97, f = 28, h = 500
N
–C
Median = l + 2 h
f
100 – 97
= 2500 +  500
28
3
= 2500 +  500 = 2553·6 1
28
Median Expenditure = ₹ 2553·6

35  500 1
Mean = 2750 – = 2750 – 87·5 = 2662·5
200

Mean Expenditure = ₹ 2662·5

14
MS_X_Mathematics_041_30/4/2_2022-23
34(A).

Sol.

1 mark
for
correct
figure

AB = Height of tower = 75 m

P, Q are positions of cars

 XBQ =  BQA = 30

 XBP =  BPA = 60

75 75 𝟏
In  APB, tan 60 =  AP = = 25 3 1𝟐
AP 3
75
In  AQB, tan 30 =  AQ = 75 3 𝟏
AQ 𝟏
𝟐
Distance between the cars = PQ = AQ – AP

= 75 3 – 25 3 = 50 3 𝟏
𝟐
𝟏
= 50  1·73 = 86·5 m
𝟐

OR

15
MS_X_Mathematics_041_30/4/2_2022-23
34(B).

Sol.

1 mark
for
correct
figure

Let AC be h m, BC = DE = 7 m, AB = (h−7) m
∠𝐴𝐸𝐵 = 60° and ∠𝐵𝐸𝐶 = 30°
∴ ∠𝐸𝐶𝐷 = 30°
Let CD be 𝑥 m
𝐷𝐸 7 𝟏
= 𝑥 = tan 30∘ ⟹ 𝑥 = 7√3 1𝟐
𝐶𝐷
⟹ 𝐵𝐸 = 7√3
𝐴𝐵
Again 𝐵𝐸 = tan 60°
1
ℎ−7
⟹ = √3 1
7√3

⟹ ℎ = 28 𝟏
𝟐
∴ Height of tower = 28 𝑚

16
MS_X_Mathematics_041_30/4/2_2022-23
35(A).

Sol.
In  ACB and  ADC,

 ACB =  ADC 1

A=A 1

  ACB   ADC 𝟏
𝟐
AC AB 8 AB
 =  = 1
AD AC 3 8
𝟏
64
 AB = 𝟐
3
64 55
BD = AB – AD = –3= cm 1
3 3

OR
35(B).

Sol. 𝟏
Correct given, to prove, figure and construction 𝟐
×𝟒=𝟐

Correct Proof 3

SECTION E
This section comprises of 3 case-study based questions of 4 marks each.

17
MS_X_Mathematics_041_30/4/2_2022-23
36.

Sol.
𝟏 𝟏
(i) R(200, 400), S(– 200, 400) +
𝟐 𝟐

(ii) (a) side PQ = (200+200) m = 400 m 1

Area of square PQRS = 400  400

18
MS_X_Mathematics_041_30/4/2_2022-23
= 160000 sq. units 1
OR
(ii) (b) Diagonal PR = (400 )2 + (400 )2 1

= √3200 or 400 2 1

(iii) 𝐶 (−600,0); 𝐴(200,800); 𝑆(−200,400)

S divides CA in the ratio 𝑘: 1


𝑘(200)+1(−600)
−200 = 𝑘+1

⟹𝑘=1 1

19
MS_X_Mathematics_041_30/4/2_2022-23
37.

Sol.
(i) Total perimeter = r + 2r

22 7
=  + 7 = 18 units 1
7 2

1 1 22 7 7 77
(ii) (a) Area of parking = r2 =    = 1
2 2 7 2 2 4
22 1 44 𝟏
Area of quadrants = 2 . 22 =
7 4 7 𝟐

77 44 715 𝟏
Total Area = + = 28 or 25.54 sq. units
4 7 𝟐
OR
20
MS_X_Mathematics_041_30/4/2_2022-23
Area of playground 98 56 1+1
(ii) (b) = 77/4 = 11 = 56 : 11
Area of parking

2𝜋𝑟
(iii) Required Perimeter = 2(𝑙 + 𝑏) + 2
𝟏
22 7
= 2(14 + 7) + × 2 = 53 units 𝟐
7

𝟏
Cost of fencing = 53 × 2 = ₹ 106 𝟐
38.

Sol.
(i) 5x + 4y = 9500 ____________ (1) 𝟏
𝟐
4x + 3y = 7370 ____________ (2) 𝟏
𝟐
(ii) (a) Solving (1) and (2), x = 980 1
 Prize Amount for Hockey = ₹ 980 1
OR
(ii) (b) On solving x = 980, y = 1,150 1

 Prize Amount for Cricket is more by ₹ (1,150 – 980) = ₹ 170 1


(iii) 2(x + y) = 2(980 + 1150) = 2(2130) = ₹ 4,260 1

21
MS_X_Mathematics_041_30/4/2_2022-23
22
MS_X_Mathematics_041_30/4/2_2022-23
Marking Scheme
Strictly Confidential
(For Internal and Restricted use only)
Secondary School Examination, 2023
MATHEMATICS PAPER CODE 30/4/3

General Instructions: -

1 You are aware that evaluation is the most important process in the actual and correct assessment of
the candidates. A small mistake in evaluation may lead to serious problems which may affect the
future of the candidates, education system and teaching profession. To avoid mistakes, it is requested
that before starting evaluation, you must read and understand the spot evaluation guidelines carefully.
2 “Evaluation policy is a confidential policy as it is related to the confidentiality of the
examinations conducted, Evaluation done and several other aspects. Its’ leakage to public in
any manner could lead to derailment of the examination system and affect the life and future
of millions of candidates. Sharing this policy/document to anyone, publishing in any magazine
and printing in News Paper/Website etc may invite action under various rules of the Board and
IPC.”
3 Evaluation is to be done as per instructions provided in the Marking Scheme. It should not be done
according to one’s own interpretation or any other consideration. Marking Scheme should be strictly
adhered to and religiously followed. However, while evaluating, answers which are based on
latest information or knowledge and/or are innovative, they may be assessed for their
correctness otherwise and due marks be awarded to them.
4 The Marking scheme carries only suggested value points for the answers.
These are in the nature of Guidelines only and do not constitute the complete answer. The students
can have their own expression and if the expression is correct, the due marks should be awarded
accordingly.
5 The Head-Examiner must go through the first five answer books evaluated by each evaluator on the
first day, to ensure that evaluation has been carried out as per the instructions given in the Marking
Scheme. If there is any variation, the same should be zero after deliberation and discussion. The
remaining answer books meant for evaluation shall be given only after ensuring that there is no
significant variation in the marking of individual evaluators.
6 Evaluators will mark ( √ ) wherever answer is correct. For wrong answer CROSS ‘X” be marked.
Evaluators will not put right (✓) while evaluating which gives an impression that answer is correct
and no marks are awarded. This is most common mistake which evaluators are committing.
7 If a question has parts, please award marks on the right-hand side for each part. Marks awarded for
different parts of the question should then be totaled up and written in the left-hand margin and
encircled. This may be followed strictly.
8 If a question does not have any parts, marks must be awarded in the left-hand margin and encircled.
This may also be followed strictly.
9 In Q1-Q20, if a candidate attempts the question more than once (without canceling the previous
attempt), marks shall be awarded for the first attempt only and the other answer scored out
with a note “Extra Question”.
10 In Q21-Q38, if a student has attempted an extra question, answer of the question deserving
more marks should be retained and the other answer scored out with a note “Extra Question”.
11 No marks to be deducted for the cumulative effect of an error. It should be penalized only once.
12 A full scale of marks __________ (example 0 to 80/70/60/50/40/30 marks as given in Question
Paper) has to be used. Please do not hesitate to award full marks if the answer deserves it.

1
MS_X_Mathematics_041_30/4/3_2022-23
13 Every examiner has to necessarily do evaluation work for full working hours i.e., 8 hours every day
and evaluate 20 answer books per day in main subjects and 25 answer books per day in other subjects
(Details are given in Spot Guidelines). This is in view of the reduced syllabus and number of
questions in question paper.
14 Ensure that you do not make the following common types of errors committed by the Examiner in
the past:-
● Leaving answer or part thereof unassessed in an answer book.
● Giving more marks for an answer than assigned to it.
● Wrong totaling of marks awarded on an answer.
● Wrong transfer of marks from the inside pages of the answer book to the title page.
● Wrong question wise totaling on the title page.
● Wrong totaling of marks of the two columns on the title page.
● Wrong grand total.
● Marks in words and figures not tallying/not same.
● Wrong transfer of marks from the answer book to online award list.
● Answers marked as correct, but marks not awarded. (Ensure that the right tick mark is correctly
and clearly indicated. It should merely be a line. Same is with the X for incorrect answer.)
● Half or a part of answer marked correct and the rest as wrong, but no marks awarded.
15 While evaluating the answer books if the answer is found to be totally incorrect, it should be marked
as cross (X) and awarded zero (0) Marks.
16 Any un assessed portion, non-carrying over of marks to the title page, or totaling error detected by
the candidate shall damage the prestige of all the personnel engaged in the evaluation work as also
of the Board. Hence, in order to uphold the prestige of all concerned, it is again reiterated that the
instructions be followed meticulously and judiciously.
17 The Examiners should acquaint themselves with the guidelines given in the “Guidelines for spot
Evaluation” before starting the actual evaluation.
18 Every Examiner shall also ensure that all the answers are evaluated, marks carried over to the title
page, correctly totaled and written in figures and words.
19 The candidates are entitled to obtain photocopy of the Answer Book on request on payment of the
prescribed processing fee. All Examiners/Additional Head Examiners/Head Examiners are once
again reminded that they must ensure that evaluation is carried out strictly as per value points for
each answer as given in the Marking Scheme.

2
MS_X_Mathematics_041_30/4/3_2022-23
MARKING SCHEME
MATHEMATICS (Subject Code–041)
(PAPER CODE: 30/4/3)

Q. No. EXPECTED OUTCOMES/VALUE POINTS Marks


SECTION A
Questions no. 1 to 18 are multiple choice questions (MCQs) and
questions number 19 and 20 are Assertion-Reason based questions of 1
mark each

1.

Sol. 1
(c) Coincident
2.

Sol. 1
(a) 2√3 cm
3.

Sol. (a) 1 : 2 1
4.

Sol. (a) 60° 1

3
MS_X_Mathematics_041_30/4/3_2022-23
5.

Sol. (b) 2.5 cm 1

6.

Sol. (d) 10 1

7.

Sol. 1
(d) √112

8.

Sol. 1
(b) 1
9.

Sol. 1
1
(c) 6

4
MS_X_Mathematics_041_30/4/3_2022-23
10.

Sol. 𝟏𝟐 1
(d) 𝟏𝟑

11.

Sol. (a) 2, – 5 1

12.

Sol. 1
(d) 0

13.

Sol. 𝟑 1
(d) − 𝟕
14.

Sol. 1 2 1
(c) d
8
15.

Sol. (c) 30-40 1

5
MS_X_Mathematics_041_30/4/3_2022-23
16.

Sol. (d) 115° 1

17.

Sol. (a) 8 cm 1

18.

Sol. (b) isosceles triangle 1

19.

6
MS_X_Mathematics_041_30/4/3_2022-23
Sol. (c) Assertion (A) is true but Reason (R) is false 1

20.

Sol. (b) Both Assertion (A) and Reason (R) are true and Reason (R) is 1
not the correct explanation of Assertion (A).
SECTION B
This section comprises very short answer (VSA) type questions of 2
marks each.
21(A).

Sol.
2x2 – 9x + 4 = 0

a = 2, b = – 9, c = 4
Let ,  be roots of 2x2 – 9x + 4 = 0
b 9
Sum =  +  = – = 1
a 2
c 4
Product of roots =  = = =2 1
a 2
21(B).

Sol.
4x2 – 5 = 0

a = 4, b = 0, c = – 5
𝟏
Discriminant = b2 – 4ac = 0 – 4 (4) (– 5) = 80 > 0 1𝟐
𝟏
 roots are real and distinct. 𝟐
7
MS_X_Mathematics_041_30/4/3_2022-23
22(A).
Sol.
2 sec2 45 + 3 cosec2 45 – 2 sin 45 cos 45

=2 ( 2 )2 + 3 ( 2 )2 – 2 1  1 
  1𝟐
𝟏
 2 2
=4+6–1=9 𝟏
𝟐
22(B).

Sol.
sin  – cos  = 0  sin  = cos   tan  = 1

𝟏
  = 45
𝟐
4 4
 1   1 
sin4 45 + cos4 45 =   +  1
 2  2
1 1 1 𝟏
= + =
4 4 2 𝟐

23.

Sol.
Total outcomes are HH, HT, TH, TT 𝟏
𝟐
Favourable outcomes are HT, TH, TT 𝟏
𝟐
3 1
P (at most one head) =
4

24.

8
MS_X_Mathematics_041_30/4/3_2022-23
Sol.
Let the numbers be 2x, 3x

1
LCM = 6x = 180  x = 30
 Numbers are 60, 90
1
HCF (60, 90) = 30
25.

Sol.
p(x) = 6x2 + 37x – (k – 2)
𝟏
1
Let the zeroes be , 𝟐

1 (k – 2) 1
Product of zeroes =  . =–
 6
𝟏
6=–k+2k=–4 𝟐

SECTION C
This section comprises of Short Answer (SA) type questions of 3 marks
each.
26.

Sol.
Let roots of the quadratic equation be , 6

px2 – 14x + 8 = 0

14 14 2 1
∴  + 6 =  7 = =
p p p

8 8 4 8
and  . 6 =  62 = 6. = 1+𝟐
𝟏
p p p2 p
𝟏
p=3
𝟐

9
MS_X_Mathematics_041_30/4/3_2022-23
27.

Sol.
Given : PA and PB are tangents drawn from an external point P to the
circle with centre O.

1 mark
for
correct
figure

To prove:  OPA =  OPB


𝟏
Construction: Join OA, OB 𝟐

Proof: In  OPA and  OPB

OP = OP (common)

OA = OA (radii)

 OAP =  OBP (each 90, radius ⊥ tangents)

∴  OPA   OPB (RHS) 1

𝟏
  OPA =  OPB (CPCT)
𝟐

10
MS_X_Mathematics_041_30/4/3_2022-23
28.

Sol. AB is the chord of larger circle touching the smaller circle at P.

𝟏
𝟐

OA = 5 cm, OP = 3 cm
To find AB
OP ⊥ AB (radius ⊥ tangent)
AB is the chord of larger circle and OP ⊥ AB
 AP = PB 1

In right-angled  AOP, AP2 = 52 – 32 = 16


AP = 4 cm 1

𝟏
 AB = 2AP = 8 cm 𝟐

29(A).

Sol.
a = – 14, a5 = 2  a + 4d = 2 1

1
– 14 + 4d = 2  d = 4
an = 62  a + (n – 1)d = 62
– 14 + (n – 1)4 = 62  n = 20 1

11
MS_X_Mathematics_041_30/4/3_2022-23
29(B).

Sol.
65, 61, 57, 53, ...

a = 65, d = – 4 𝟏
𝟐
Let an be the first negative term
an < 0  a + (n – 1)d < 0
1
65 + (n – 1) (– 4) < 0  69 – 4n < 0
69 1
n>
4
69
 Least positive integral value of n which satisfies n > 4 is 18
𝟏
 1st negative term of AP = 18 𝟐
30(A).

Sol.
sin A – 2 sin 3 A sin A (1 – 2 sin 2 A ) 1
LHS = =
2 cos 3 A – cos A cos A (2 cos 2 A – 1)

sin A[1 – 2(1 – cos 2 A )] sin A[1 – 2 + 2 cos 2 A ] 1


= =
cos A[2 cos 2 A – 1 ] cos A [2 cos 2 A – 1 ]
sin A[2 cos 2 A – 1 ] 1
= = tan A = RHS
2
cos A [2 cos A – 1 ]
30(B).
Sol.
LHS = sec A (1 – sin A) (sec A + tan A)

1  1 sin A 
= (1 – sin A)  +  1
cos A  cos A cos A 

12
MS_X_Mathematics_041_30/4/3_2022-23
1
1 (1 + sin A )
= (1 – sin A)
cos A cos A
1 – sin 2 A cos 2 A
= = = 1=RHS 1
cos 2 A cos 2 A
31.

Sol. Let √𝟓 be a rational number.


𝐩 ½
∴ √𝟓 = 𝐪 , where q≠0 and let p & q be co-primes.
5q2 = p2 ⟹ p2 is divisible by 5 ⟹ p is divisible by 5 1
⟹ p = 5a, where ‘a’ is some integer ----- (i)
2 2 2 2 2
25a = 5q ⟹ q = 5a ⟹q is divisible by 5 ⟹ q is divisible by 5 ½
⟹ q = 5b, where ‘b’ is some integer ----- (ii)
(i) and (ii) leads to contradiction as ‘p’ and ‘q’ are co-primes. 1
∴ √𝟓 is an irrational number.
SECTION D
This section comprises of Long Answer (LA) type questions of 5 marks
each.
32(A).

Sol.

1 mark
for
correct
figure

PN × NR = QN2

PN QN
=
QN NR

13
MS_X_Mathematics_041_30/4/3_2022-23
 PNQ =  QNR
𝟏
 PNQ   QNR 1𝟐

1
  2 =  P and  1 =  R

1+2=P+R

  PQR =  P +  R 𝟏

In  PQR,  P +  PQR +  R = 180


𝟏
 2  PQR = 180   PQR = 90 𝟐

32(B).

14
MS_X_Mathematics_041_30/4/3_2022-23
Sol.

Draw AL ⊥ BC and DM ⊥ BC 1

In  𝐴𝑂𝐿 and  𝐷𝑂𝑀,

 𝐴𝑂𝐿 =  𝐷𝑂𝑀

 𝐴𝐿𝑂 =  𝐷𝑀𝑂

 𝐴𝑂𝐿   𝐷𝑂𝑀 2

𝐴𝐿 𝐴𝑂 𝟏
 = ……(i)
𝐷𝑀 𝐷𝑂 𝟐
1
𝑎𝑟(𝛥 ABC) 2
× BC × AL
= 1 1
𝑎𝑟(𝛥 DBC) × BC × DM
2

𝐴𝐿 𝐴𝑂 𝟏
= = [using (i)]
𝐷𝑀 𝐷𝑂 𝟐

33.

Sol.
Height of cylinder = 10 cm

15
MS_X_Mathematics_041_30/4/3_2022-23
7 1
Radius of cylinder = radius of hemisphere = 3·5 = cm
2

Total surface area of the article

= CSA of cylinder + CSA of 2 hemispheres

= 2rh + 2 × 2r2

= 2r(h + 2r)

22 7 7 𝟏 𝟏
=2× × (10 + 2 × ) 1𝟐 +1𝟐
7 2 2

= 22 × 17 = 374 cm2 1

34.

Sol.
Monthly Exp. (in ₹) x fi cf d xifi

1000 – 1500 1250 24 24 –3 – 72

1500 – 2000 1750 40 64 –2 – 80

2000 – 2500 2250 33 97 –1 – 33

2500 – 3000 2750 x=28 125 0 0 2 for


correct
3000 – 3500 3250 30 155 1 30 table

16
MS_X_Mathematics_041_30/4/3_2022-23
3500 – 4000 3750 22 177 2 44

4000 – 4500 4250 16 193 3 48

4500 – 5000 4750 7 200 4 28

Total – 35

172 + x = 200  x = 28 1

l = lower limit of median class = 2500


N 200
= = 100
2 2
C = 97, f = 28, h = 500
N
–C
Median = l + 2 h
f
100 – 97
= 2500 +  500
28
3 1
= 2500 +  500 = 2553·6
28
Median Expenditure = ₹ 2553·6

35  500
Mean = 2750 – = 2750 – 87·5 = 2662·5 1
200

Mean Expenditure = ₹ 2662·5


35(A).

Sol.

17
MS_X_Mathematics_041_30/4/3_2022-23
1 mark
for
correct
figure

AB = Height of tower = 75 m

P, Q are positions of cars

 XBQ =  BQA = 30

 XBP =  BPA = 60

75 75 𝟏
In  APB, tan 60 =  AP = = 25 3 1𝟐
AP 3
75
In  AQB, tan 30 =  AQ = 75 3 𝟏
AQ 𝟏
𝟐
Distance between the cars = PQ = AQ – AP

𝟏
= 75 3 – 25 3 = 50 3
𝟐

= 50  1·73 = 86·5 m 𝟏
𝟐

35(B).

Sol.
18
MS_X_Mathematics_041_30/4/3_2022-23
1 mark
for
correct
figure

Let AC be h m, BC = DE = 7 m, AB = (h−7) m
∠𝐴𝐸𝐵 = 60° and ∠𝐵𝐸𝐶 = 30°
∴ ∠𝐸𝐶𝐷 = 30°
Let CD be 𝑥 m
𝐷𝐸 7
= 𝑥 = tan 30∘ ⟹ 𝑥 = 7√3 𝟏
1𝟐
𝐶𝐷
⟹ 𝐵𝐸 = 7√3
𝐴𝐵
Again 𝐵𝐸 = tan 60°
1
ℎ−7
⟹ = √3 1
7√3

⟹ ℎ = 28 𝟏
𝟐
∴ Height of tower = 28 𝑚

SECTION E
This section comprises of 3 case-study based questions of 4 marks each.

19
MS_X_Mathematics_041_30/4/3_2022-23
36.

Sol.
(i) Total perimeter = r + 2r

22 7
=  + 7 = 18 units 1
7 2

1 1 22 7 7 77
(ii) (a) Area of parking = r2 =    = 1
2 2 7 2 2 4
22 1 44 𝟏
Area of quadrants = 2 . 22 =
7 4 7 𝟐

77 44 715 𝟏
Total Area = + = 28 or 25.54 sq. units
4 7 𝟐
20
MS_X_Mathematics_041_30/4/3_2022-23
OR

Area of playground 98 56
(ii) (b) = 77/4 = 11 = 56 : 11 1+1
Area of parking

2𝜋𝑟
(iii) Required Perimeter = 2(𝑙 + 𝑏) + 2

22 7 𝟏
= 2(14 + 7) + × 2 = 53 units
7 𝟐

Cost of fencing = 53 × 2 = ₹ 106 𝟏


𝟐

37.

Sol.
𝟏
(i) 5x + 4y = 9500 ____________ (1)
𝟐
4x + 3y = 7370 ____________ (2) 𝟏
𝟐
(ii) (a) Solving (1) and (2), x = 980 1
1
 Prize Amount for Hockey = ₹ 980
OR
(ii) (b) On solving x = 980, y = 1,150 1
21
MS_X_Mathematics_041_30/4/3_2022-23
1
 Prize Amount for Cricket is more by ₹ (1,150 – 980) = ₹ 170
(iii) 2(x + y) = 2(980 + 1150) = 2(2130) = ₹ 4,260 1

38.

Sol.
𝟏 𝟏
(i) R(200, 400), S(– 200, 400) +
𝟐 𝟐

22
MS_X_Mathematics_041_30/4/3_2022-23
(ii) (a) side PQ = (200+200) m = 400 m 1

Area of square PQRS = 400  400


1
= 160000 sq. units
OR
1
(ii) (b) Diagonal PR = (400 )2 + (400 )2
1
= √3200 or 400 2

(iii) 𝐶 (−600,0); 𝐴(200,800); 𝑆(−200,400)

S divides CA in the ratio 𝑘: 1


𝑘(200)+1(−600)
−200 = 𝑘+1
⟹𝑘 =1
1

23
MS_X_Mathematics_041_30/4/3_2022-23
Marking Scheme
Strictly Confidential
(For Internal and Restricted use only)
Secondary School Examination, 2023
MATHEMATICS PAPER CODE 30/5/1

General Instructions: -

1 You are aware that evaluation is the most important process in the actual and correct
assessment of the candidates. A small mistake in evaluation may lead to serious problems
which may affect the future of the candidates, education system and teaching profession. To
avoid mistakes, it is requested that before starting evaluation, you must read and understand
the spot evaluation guidelines carefully.
2 “Evaluation policy is a confidential policy as it is related to the confidentiality of the
examinations conducted, Evaluation done and several other aspects. Its’ leakage to
public in any manner could lead to derailment of the examination system and affect the
life and future of millions of candidates. Sharing this policy/document to anyone,
publishing in any magazine and printing in News Paper/Website etc may invite action
under various rules of the Board and IPC.”
3 Evaluation is to be done as per instructions provided in the Marking Scheme. It should not
be done according to one’s own interpretation or any other consideration. Marking Scheme
should be strictly adhered to and religiously followed. However, while evaluating, answers
which are based on latest information or knowledge and/or are innovative, they may be
assessed for their correctness otherwise and due marks be awarded to them.
4 The Marking scheme carries only suggested value points for the answers.
These are in the nature of Guidelines only and do not constitute the complete answer. The
students can have their own expression and if the expression is correct, the due marks should
be awarded accordingly.
5 The Head-Examiner must go through the first five answer books evaluated by each evaluator
on the first day, to ensure that evaluation has been carried out as per the instructions given
in the Marking Scheme. If there is any variation, the same should be zero after deliberation
and discussion. The remaining answer books meant for evaluation shall be given only after
ensuring that there is no significant variation in the marking of individual evaluators.
6 Evaluators will mark ( √ ) wherever answer is correct. For wrong answer CROSS ‘X” be
marked. Evaluators will not put right (✓) while evaluating which gives an impression that
answer is correct and no marks are awarded. This is most common mistake which
evaluators are committing.
7 If a question has parts, please award marks on the right-hand side for each part. Marks
awarded for different parts of the question should then be totaled up and written in the left-
hand margin and encircled. This may be followed strictly.
8 If a question does not have any parts, marks must be awarded in the left-hand margin and
encircled. This may also be followed strictly.

1
MS_X_Mathematics_041_30/5/1_2022-23
9 In Q1-Q20, if a candidate attempts the question more than once (without canceling the previous
attempt), marks shall be awarded for the first attempt only and the other answer scored out
with a note “Extra Question”.
10 In Q21-Q38, if a student has attempted an extra question, answer of the question deserving
more marks should be retained and the other answer scored out with a note “Extra Question”.
11 No marks to be deducted for the cumulative effect of an error. It should be penalized only once.
12 A full scale of marks __________ (example 0 to 80/70/60/50/40/30 marks as given in
Question Paper) has to be used. Please do not hesitate to award full marks if the answer
deserves it.
13 Every examiner has to necessarily do evaluation work for full working hours i.e., 8 hours
every day and evaluate 20 answer books per day in main subjects and 25 answer books per
day in other subjects (Details are given in Spot Guidelines). This is in view of the reduced
syllabus and number of questions in question paper.
14 Ensure that you do not make the following common types of errors committed by the
Examiner in the past: -
● Leaving answer or part thereof unassessed in an answer book.
● Giving more marks for an answer than assigned to it.
● Wrong totaling of marks awarded on an answer.
● Wrong transfer of marks from the inside pages of the answer book to the title page.
● Wrong question wise totaling on the title page.
● Wrong totaling of marks of the two columns on the title page.
● Wrong grand total.
● Marks in words and figures not tallying/not same.
● Wrong transfer of marks from the answer book to online award list.
● Answers marked as correct, but marks not awarded. (Ensure that the right tick mark is
correctly and clearly indicated. It should merely be a line. Same is with the X for
incorrect answer.)
● Half or a part of answer marked correct and the rest as wrong, but no marks awarded.
15 While evaluating the answer books if the answer is found to be totally incorrect, it should be
marked as cross (X) and awarded zero (0) Marks.
16 Any un assessed portion, non-carrying over of marks to the title page, or totaling error
detected by the candidate shall damage the prestige of all the personnel engaged in the
evaluation work as also of the Board. Hence, in order to uphold the prestige of all concerned,
it is again reiterated that the instructions be followed meticulously and judiciously.
17 The Examiners should acquaint themselves with the guidelines given in the “Guidelines for
spot Evaluation” before starting the actual evaluation.
18 Every Examiner shall also ensure that all the answers are evaluated, marks carried over to
the title page, correctly totaled and written in figures and words.
19 The candidates are entitled to obtain photocopy of the Answer Book on request on payment
of the prescribed processing fee. All Examiners/Additional Head Examiners/Head
Examiners are once again reminded that they must ensure that evaluation is carried out
strictly as per value points for each answer as given in the Marking Scheme.

2
MS_X_Mathematics_041_30/5/1_2022-23
MARKING SCHEME
MATHEMATICS (Subject Code–041)
(PAPER CODE: 30/5/1)

Q. No. EXPECTED OUTCOMES/VALUE POINTS Marks

SECTION A

Questions no. 1 to 18 are multiple choice questions (MCQs) and questions


number 19 and 20 are Assertion-Reason based questions of 1 mark each

1.

Sol. (b) Infinite 1

2.

Sol. (d) ab = 6 1

3.

Sol. (b) 3 1

3
MS_X_Mathematics_041_30/5/1_2022-23
4.

Sol. (d) 10 1

5.

Sol. (a) x2 – 4x + 1 = 0 1

6.

Sol. 𝟏𝟕 1
(a) − 𝟕

7.

4
MS_X_Mathematics_041_30/5/1_2022-23
Sol. (d) 3 units 1

8.

Sol. (c) 6·5 cm 1

9.

Sol. (c) 8000 m3 1

10.

Sol. (b) 21 1

5
MS_X_Mathematics_041_30/5/1_2022-23
11.

Sol. (a) 3 cm 1

12.

Sol. (a) sin 60 1

13.

Sol. (b)  B =  D 1

14.

6
MS_X_Mathematics_041_30/5/1_2022-23
Sol. (c) – 32 1

15.

Sol. 𝟑 1
(c) 𝟒

16.

Sol. (a) 30 1

17.

Sol. 𝟕 1
(b) 𝟎.𝟎𝟏

7
MS_X_Mathematics_041_30/5/1_2022-23
18.

Sol. (a) decreases by 2 1

19.

Sol. (c) Assertion (A) is true, but Reason (R) is false 1

20.

8
MS_X_Mathematics_041_30/5/1_2022-23
Sol. (c) Assertion (A) is true, but Reason (R) is false 1

SECTION B

This section comprises of very short answer (VSA) type questions of 2 marks
each.

21(a).

Sol. AP : AB = 2 : 5  AP : PB = 2 : 3 𝟏
𝟐

𝟖 + 12 𝟏𝟎 – 15 𝟏
x= 𝟓
= 4, y = 𝟓
=–1
𝟏
Point P is (4, – 1) 𝟐

OR

21(b).

Sol. 2 2 2 2
PA = PB  (x – 5) + (y – 1) = (x – 1) + (y – 5)
2 2 𝟏

x=y 𝟏

9
MS_X_Mathematics_041_30/5/1_2022-23
22.

Sol. PA . PB = (PC – AC).(PC + BC) 𝟏

= (PC – AC)(PC + AC) [AC = BC] 𝟏


𝟐
𝟏
2 2 𝟐
= PC – AC

23.

ol. 96 = 2 x 2 x 2 x 2 x 2 x 3 𝟏
𝟐
5
=2 x3

120 = 2 x 2 x 2 x 3 x 5
𝟏
3 𝟐
=2 x3x5
𝟏
HCF = 24
𝟐
LCM = 480 𝟏
𝟐

24.

10
MS_X_Mathematics_041_30/5/1_2022-23
K 1
Sol. • k • •
A(5, –6) P(0, y) B(–1, –4)

𝟏
Let the point of division be P(0, y) which divides AB in the ratio K : 𝟐
1

–K+5
0= K=5 𝟏
K+1

𝟏
Ratio is 5:1
𝟐

25(a).

m2 + n2 = (a cos  + b sin )2 + (a sin  – b cos )2


Sol. 𝟏
𝟐

= a2(cos2 + sin2) + b2(sin2  + cos2 )


𝟏

𝟏
2 𝟐
= a2 + b

OR

25(b).

Sol. √𝒔𝒆𝒄 A – 1 √𝒔𝒆𝒄 A + 1


LHS = +
√𝒔𝒆𝒄 A + 1 √𝒔𝒆𝒄 A – 1

𝒔𝒆𝒄 A – 1 + sec A + 1
=
√𝒔𝒆𝒄𝟐 𝑨 – 1 𝟏

11
MS_X_Mathematics_041_30/5/1_2022-23
𝟐 sec A 𝟏
= tan A
𝟐

= 2 cosec A = RHS 𝟏
𝟐

SECTION C

This section comprises of short answer (SA) type questions of 3 marks each.

26(a).

Sol. Let √𝟑 be a rational number.


𝐩 ½
∴ √𝟑 = 𝐪 , let p & q be co-primes and q≠0

3q2 = p2 ⟹ p2 is divisible by 3 ⟹ p is divisible by 3


1
⟹ p = 3a, where ‘a’ is some integer ----- (i)

9a2 = 3q2 ⟹ q2 = 3a2 ⟹q2 is divisible by 3 ⟹ q is divisible by 3


½
⟹ q = 3b, where ‘b’ is some integer ----- (ii)

(i) and (ii) leads to contradiction as ‘p’ and ‘q’ are co-primes.
1
∴ √𝟑 is an irrational number.

OR

26(b)

12
MS_X_Mathematics_041_30/5/1_2022-23
LCM = 432
Sol. 𝟐

𝟒𝟑𝟐
i.e. = 7 min 12 sec.
𝟔𝟎

 traffic lights will change simultaneously again at 7 : 7 : 12 a.m. 𝟏

27.

Sol. ap = a + (p – 1)d = q ________ (i) 𝟏


𝟐
aq = a + (q – 1)d = p ________ (ii) 𝟏
𝟐
Solving (i) and (ii)

d = – 1, a = q + p – 1 𝟏 𝟏
+
𝟐 𝟐
an = (q + p – 1) + (n – 1)(– 1) = q + p - n
𝟏

28(a).

13
MS_X_Mathematics_041_30/5/1_2022-23
Sol.  EFG   ADG

𝑬𝑭 𝑭𝑮 𝟏
 𝑨𝑫 = 𝑫𝑮 _________ (i)

 EFC   BDC

𝑬𝑭
 𝑩𝑫 = 𝑪𝑫
𝑪𝑭 𝟏

𝑬𝑭 𝑪𝑭
 = {BD = AD} _________ (ii) 𝟏
𝑨𝑫 𝑪𝑫
𝟐
Using (i) and (ii)

𝑭𝑮
= 𝑪𝑫
𝑪𝑭 𝟏
𝑫𝑮
𝟐

OR

28(b).

14
MS_X_Mathematics_041_30/5/1_2022-23
Sol.  ALE   CLB

𝑨𝑳 𝑬𝑳 𝟏
 𝑪𝑳 = 𝑩𝑳 _________ (i)

Also  CLM   ALB

𝑨𝑳
 𝑪𝑳 = 𝑪𝑴
𝑨𝑩 𝟏

𝑨𝑳 𝑪𝑫
 𝑪𝑳 = 𝑪𝑴 {AB = CD} _________ (ii) 𝟏
𝟐
Using (i) and (ii)

𝑬𝑳 𝟐𝑪𝑴
=
𝑩𝑳 𝑪𝑴

𝟏
 EL = 2BL 𝟐

29.

Sol. Let walking speeds be x km/hr. and y km/hr. (x > y)

𝟏
ATQ, 2x + 2y = 16
𝟏
and 8x – 8y = 16
𝟏 𝟏
+
Solving to get x = 5, y = 3 𝟐 𝟐

Speeds are 5 km/hr. 3 km/hr.

15
MS_X_Mathematics_041_30/5/1_2022-23
30.

𝒔𝒊𝒏 𝜽 𝒄𝒐𝒔 𝜽
Sol. cos 𝜽 sin 𝜽
𝟏
LHS = cos 𝜽 + sin 𝜽
1– 1–
sin 𝜽 cos 𝜽

𝒔𝒊𝒏𝟐 𝜽 𝒄𝒐𝒔𝟐 𝜽
= cos 𝜽(sin 𝜽 – cos 𝜽) + sin 𝜽(cos 𝜽 – sin 𝜽)

𝟏
𝒔𝒊𝒏𝟑 𝜽 – cos𝟑 𝜽
= sin 𝜽 cos 𝜽(sin 𝜽 – cos 𝜽)

𝒔𝒊𝒏𝟐 𝜽 + cos𝟐 𝜽 + sin 𝜽 cos 𝜽 𝟏


= sin 𝜽 cos 𝜽 𝟐

𝟏
= 𝒔𝒊𝒏 𝜽 cos 𝜽 + 1
𝟏
= 1 + cosec  sec  = RHS 𝟐

31.

𝒙 – 42·5
C.I. x f u= fu
Sol. 𝟓
25 – 30 27·5 14 –3 – 42
30 – 35 32·5 22 –2 – 44
35 – 40 37·5 16 –1 – 16 For
40 – 45 42·5 6 0 0 correct
table
45 – 50 47·5 5 1 5
50 – 55 52·5 3 2 6 2 Marks
55 – 60 57·5 4 3 12
70 – 79

16
MS_X_Mathematics_041_30/5/1_2022-23
𝟏
𝟕𝟗
Mean = 42·5 – x 5 = 36·86
𝟕𝟎

SECTION D

This section comprises of long answer (LA) type questions of 5 marks each.

32.

Sol.

Correct
Figure

1 Mark

Let B is the basket of hot air balloon

𝒉
tan 60 = √𝟑 = 𝒙  h = x√𝟑 _________ (i) 𝟏

𝟏 𝒉
𝟏
tan 30 = =  x = h√𝟑 – 100 _________ (ii)
√𝟑 𝒙 + 100

17
MS_X_Mathematics_041_30/5/1_2022-23
using (i) and (ii)
𝟏
(a) h= (h√𝟑 –100)√𝟑 = 3h – 100√𝟑  h = 50√𝟑 m 𝟐

√𝟑 𝒉 𝟓𝟎√𝟑 𝟏
(b) sin 60 = =𝒚=  y = 100 m
𝟐 𝒚

𝒉
(c) x = = 50 m  x+100=150 m 𝟏
√𝟑
𝟐
_________________________________________________________________

# ANOTHER SOLUTION AS PER BELOW FIGURE IS ALSO POSSIBLE

Correct
Figure

1 Mark

Let B is the basket of hot air balloon. D and C be the positions of the
first and second observer’s respectively.

𝒉
tan 60 = √𝟑 = 𝒙  h = x√𝟑 _________ (i) 𝟏

𝟏 𝒉 𝟏
tan 30 = = 𝟏𝟎𝟎−𝒙  √𝟑𝒉 = 𝟏𝟎𝟎 − 𝒙 _________ (ii)
√𝟑

(a) using (i) and (ii)

18
MS_X_Mathematics_041_30/5/1_2022-23
𝒉 = √𝟑 (100 - √𝟑𝒉)  𝒉 = 25√𝟑 m 𝟏
𝟐
√𝟑 𝒉
(b) sin 60 = = 𝑩𝑫  BD = 50 m 𝟏
𝟐

𝒉
(c) x = = 25 m
√𝟑

 AC = 100 – x = 75 m 𝟏
𝟐

33(a).

Sol.

19
MS_X_Mathematics_041_30/5/1_2022-23
Join OA, OB, OC and draw OE ⊥ AC and OF ⊥ AB. 1
𝟏
𝟐

BF = 10 cm, CE = 8 cm, Let AF = AE = x


𝟏
1
ar  ABC = ar  BOC + ar  COA + ar  AOB 𝟐

𝟏
90 = 𝟐. 4 (BC + CA + AB)

90 = 2(18 + 8 + x + 10 + x) 1

90 = 4(18 + x)

x = 4·5

AB = 14·5 cm and AC = 12·5 cm


1

OR

33(b).

20
MS_X_Mathematics_041_30/5/1_2022-23
Sol. OO = √𝟔𝟐 + 8𝟐 = 10 cm {OP ⊥ OP} 𝟏
𝟐
Let OA = x, OA = 10 – x

𝟏
AP2 = 36 – x2
𝟐
𝟏
Also AP2 = 64 – (10 – x)2
𝟐

Therefore 36 – x2 = 64 – (10 – x)2

 36 – x2 = 64 – 100 – x2 + 20 x

 x = 3·6 𝟐

In  PAO, AP2 = 36 – (3·6)2 = 23·04


𝟏
 AP = 4·8
𝟏
Length PQ = 2 x AP = 9·6 cm 𝟐

34(a).

Sol. Let first average speed of the train be x km/hr.

𝟓𝟒 𝟔𝟑 2
+ 𝒙+6 = 3
𝒙

 54x + 324 + 63x = 3x2 + 18x

21
MS_X_Mathematics_041_30/5/1_2022-23
 3x2 – 99x – 324 = 0 or x2 – 33x – 108 = 0 2

 (x – 36) (x + 3) = 0

 x = 36, -3 (rejected)
1

Therefore, first average speed of the train was 36 km/hr.

OR

34(b).

Sol. Let the time taken by smaller diameter tap be x hrs.

Time taken by larger diameter tap is (x – 2) hrs.

𝟏 𝟏 𝟖
Therefore 𝒙 – 2 + 𝒙 = 𝟏𝟓
2

 15(2x – 2) = 8x(x – 2)

 8x2 – 46x + 30 = 0

 4x2 – 23x + 15 = 0 1

 (4x – 3)(x – 5) = 0

𝟑
 x = 𝟒, x = 5
1

22
MS_X_Mathematics_041_30/5/1_2022-23
𝟑
x  𝟒 as x – 2 < 0

Smaller diameter tap fills in 5 hrs.

Larger diameter tap fills in 3 hrs. 1

35.

Sol. Area of that part of the field in which the horse can graze by means 1
𝟏
of a 5 m long rope = 𝟒 × 𝟑. 𝟏𝟒 × (𝟓)𝟐

= 𝟏𝟗. 𝟔𝟐𝟓 𝒎𝟐 1

Area of that part of the field in which the horse can graze by means
𝟏
of a 10 m long rope = 𝟒 × 𝟑. 𝟏𝟒 × (𝟏𝟎)𝟐 1

1
= 𝟕𝟖. 𝟓 𝒎𝟐

Increase in grazing area = 𝟕𝟖. 𝟓 𝒎𝟐 − 𝟏𝟗. 𝟔𝟐𝟓 𝒎𝟐 = 𝟓𝟖. 𝟖𝟕𝟓 𝒎𝟐


1

SECTION E

This section comprises of 3 case-study based questions of 4 marks each.

23
MS_X_Mathematics_041_30/5/1_2022-23
36.

𝟐𝟐 𝟏𝟕𝟔
Sol. (i) SA = 2r2 = 2 x x4= 𝐦𝐦𝟐 or 25·1 mm2 1
𝟕 𝟕

𝟐 𝟐𝟐
(ii) Volume of material dug out to make one dimple = 𝟑 x x8
𝟕

𝟑𝟓𝟐 1
= 𝐦𝐦𝟑 or 16·76 mm3
𝟐𝟏

24
MS_X_Mathematics_041_30/5/1_2022-23
(iii)(a) radius of ball = 21 mm

Total surface area exposed to surroundings

= 4(21)2 – 315 x (2)2 + 315 x 2(2)2


1
𝟐𝟐 𝟐𝟐
=4x x 21 x 21 + x 315 x 4
𝟕 𝟕

1
= 9504 mm2

OR

𝟒 𝟐 1
(iii) (b) Volume of the golf ball = 𝟑(21)3 – 315 x 𝟑(2)3

= 33528 mm3
1

25
MS_X_Mathematics_041_30/5/1_2022-23
37.

Sol. (i) All possible outcomes: RR, RG, RB, GR, GB, GG, YR, YB, YG 1

(ii) Number of favourable outcome (RB) = 1

26
MS_X_Mathematics_041_30/5/1_2022-23
𝟏 1
P (Making purple) = 𝟗

𝟏
(iii)(a) As P(winning) = 𝟗

𝟏 𝟏
Therefore, number of people must win = 𝟗 x 99 = 11
𝟐

 Game lost by 88 persons. 𝟏


𝟐

Funds collected = 5 x 88 – 10 x 11 = ₹ 330 1

OR

(iii)(b) Number of participants = 99

𝟏
P(winning the game) = 𝟗
𝟏
Number of persons won = 11 𝟐
𝟏
Number of persons lost = 88
𝟐

Funds collected = 88 x 5 – 11 x 5 = ₹ 385 1

27
MS_X_Mathematics_041_30/5/1_2022-23
38.

28
MS_X_Mathematics_041_30/5/1_2022-23
Sol. (i) – 16t2 + 20t = 0  4t(– 4t + 5) = 0
1
𝟓
t = 0, t =
𝟒

(ii) (a) 1

𝟑 𝟗 𝟑 1
(iii)(a) At t = , h = – 16 x + 20 x = – 36 + 30 = – 6
𝟐 𝟒 𝟐

𝟑
It means after 𝟐 seconds, dolphin has reached 6 cm below water 1
level.

OR

(iii)(b) Speed of dolphin = 20 cm per second.

In one second, distance covered = 20 cm


2
𝟓 𝟓
In seconds, distance covered = 20 x = 25 cm
𝟒 𝟒

29
MS_X_Mathematics_041_30/5/1_2022-23
Marking Scheme
Strictly Confidential
(For Internal and Restricted use only)
Secondary School Examination, 2023
MATHEMATICS PAPER CODE 30/5/2

General Instructions: -

1 You are aware that evaluation is the most important process in the actual and correct
assessment of the candidates. A small mistake in evaluation may lead to serious problems
which may affect the future of the candidates, education system and teaching profession. To
avoid mistakes, it is requested that before starting evaluation, you must read and understand
the spot evaluation guidelines carefully.
2 “Evaluation policy is a confidential policy as it is related to the confidentiality of the
examinations conducted, Evaluation done and several other aspects. Its’ leakage to
public in any manner could lead to derailment of the examination system and affect the
life and future of millions of candidates. Sharing this policy/document to anyone,
publishing in any magazine and printing in News Paper/Website etc may invite action
under various rules of the Board and IPC.”
3 Evaluation is to be done as per instructions provided in the Marking Scheme. It should not
be done according to one’s own interpretation or any other consideration. Marking Scheme
should be strictly adhered to and religiously followed. However, while evaluating, answers
which are based on latest information or knowledge and/or are innovative, they may be
assessed for their correctness otherwise and due marks be awarded to them.
4 The Marking scheme carries only suggested value points for the answers.
These are in the nature of Guidelines only and do not constitute the complete answer. The
students can have their own expression and if the expression is correct, the due marks should
be awarded accordingly.
5 The Head-Examiner must go through the first five answer books evaluated by each evaluator
on the first day, to ensure that evaluation has been carried out as per the instructions given
in the Marking Scheme. If there is any variation, the same should be zero after deliberation
and discussion. The remaining answer books meant for evaluation shall be given only after
ensuring that there is no significant variation in the marking of individual evaluators.
6 Evaluators will mark ( √ ) wherever answer is correct. For wrong answer CROSS ‘X” be
marked. Evaluators will not put right (✓) while evaluating which gives an impression that
answer is correct and no marks are awarded. This is most common mistake which
evaluators are committing.
7 If a question has parts, please award marks on the right-hand side for each part. Marks
awarded for different parts of the question should then be totaled up and written in the left-
hand margin and encircled. This may be followed strictly.
8 If a question does not have any parts, marks must be awarded in the left-hand margin and
encircled. This may also be followed strictly.

1
MS_X_Mathematics_041_30/5/2_2022-23
9 In Q1-Q20, if a candidate attempts the question more than once (without canceling the previous
attempt), marks shall be awarded for the first attempt only and the other answer scored out
with a note “Extra Question”.
10 In Q21-Q38, if a student has attempted an extra question, answer of the question deserving
more marks should be retained and the other answer scored out with a note “Extra Question”.
11 No marks to be deducted for the cumulative effect of an error. It should be penalized only once.
12 A full scale of marks __________ (example 0 to 80/70/60/50/40/30 marks as given in
Question Paper) has to be used. Please do not hesitate to award full marks if the answer
deserves it.
13 Every examiner has to necessarily do evaluation work for full working hours i.e., 8 hours
every day and evaluate 20 answer books per day in main subjects and 25 answer books per
day in other subjects (Details are given in Spot Guidelines). This is in view of the reduced
syllabus and number of questions in question paper.
14 Ensure that you do not make the following common types of errors committed by the
Examiner in the past:-
● Leaving answer or part thereof unassessed in an answer book.
● Giving more marks for an answer than assigned to it.
● Wrong totaling of marks awarded on an answer.
● Wrong transfer of marks from the inside pages of the answer book to the title page.
● Wrong question wise totaling on the title page.
● Wrong totaling of marks of the two columns on the title page.
● Wrong grand total.
● Marks in words and figures not tallying/not same.
● Wrong transfer of marks from the answer book to online award list.
● Answers marked as correct, but marks not awarded. (Ensure that the right tick mark is
correctly and clearly indicated. It should merely be a line. Same is with the X for
incorrect answer.)
● Half or a part of answer marked correct and the rest as wrong, but no marks awarded.
15 While evaluating the answer books if the answer is found to be totally incorrect, it should be
marked as cross (X) and awarded zero (0) Marks.
16 Any unassessed portion, non-carrying over of marks to the title page, or totaling error
detected by the candidate shall damage the prestige of all the personnel engaged in the
evaluation work as also of the Board. Hence, in order to uphold the prestige of all concerned,
it is again reiterated that the instructions be followed meticulously and judiciously.
17 The Examiners should acquaint themselves with the guidelines given in the “Guidelines for
spot Evaluation” before starting the actual evaluation.
18 Every Examiner shall also ensure that all the answers are evaluated, marks carried over to
the title page, correctly totaled and written in figures and words.
19 The candidates are entitled to obtain photocopy of the Answer Book on request on payment
of the prescribed processing fee. All Examiners/Additional Head Examiners/Head
Examiners are once again reminded that they must ensure that evaluation is carried out
strictly as per value points for each answer as given in the Marking Scheme.

2
MS_X_Mathematics_041_30/5/2_2022-23
MARKING SCHEME
MATHEMATICS (Subject Code–041)
(PAPER CODE: 30/5/2)

Q. No. EXPECTED OUTCOMES/VALUE POINTS Marks

SECTION A

Questions no. 1 to 18 are multiple choice questions (MCQs) and questions


number 19 and 20 are Assertion-Reason based questions of 1 mark each

1.

Sol. (b) cos 60 1

2.

Sol. (b)  B =  D 1

3
MS_X_Mathematics_041_30/5/2_2022-23
3.

(d) – 64
Sol. 1

4.

Sol. 𝟏
(c) 1
𝟏𝟐

5.

(c) 6·5 cm
Sol. 1

4
MS_X_Mathematics_041_30/5/2_2022-23
6.

Sol. (c) 8000 m3 1

7.

(b) 21
Sol. 1

8.

(a) 3 cm
Sol. 1

5
MS_X_Mathematics_041_30/5/2_2022-23
9.

(a) 30
Sol. 1

10.

Sol. 𝟕 1
(b) 𝟎.𝟎𝟏

11.

(a) decreases by 2
Sol. 1

6
MS_X_Mathematics_041_30/5/2_2022-23
12.

Sol. (d) 10 1

13.

Sol. (a) x2 – 4x + 1 = 0 1

14.

Sol. 𝟏𝟕 1
(a) −
𝟕

7
MS_X_Mathematics_041_30/5/2_2022-23
15.

Sol. (b) √𝟏𝟑 units 1

16.

Sol. (d) infinite 1

17.

Sol. (d) ab = 6 1

18.

Sol. (c) 5 1

8
MS_X_Mathematics_041_30/5/2_2022-23
19.

Sol. (c) Assertion (A) is true, but Reason (R) is false 1

20.

Sol. (c) Assertion (A) is true, but Reason (R) is false 1

SECTION B

This section comprises of very short answer (VSA) type questions of 2 marks
each.

21

9
MS_X_Mathematics_041_30/5/2_2022-23
Sol. 96 = 2 x 2 x 2 x 2 x 2 x 3 𝟏
𝟐
5
=2 x3

120 = 2 x 2 x 2 x 3 x 5
𝟏
3 𝟐
=2 x3x5

𝟏
HCF = 24
𝟐
LCM = 480 𝟏
𝟐

22.

Let the ratio be 𝒌: 𝟏


Sol.

𝒌+𝟔
x =k+1 1
2

𝟔𝒌 – 3 1
y= k+1 2

10
MS_X_Mathematics_041_30/5/2_2022-23
P(x, y) lies on y = x
1
 k + 6 = 6k –3 2

𝟗 1
k=𝟓 2

Ratio is 9 : 5

23(a)

m2 + n2 = (a cos  + b sin )2 + (a sin  – b cos )2


Sol. 𝟏
𝟐

= a2(cos2 + sin2) + b2(sin2  + cos2 )


𝟏

𝟏
2 𝟐
= a2 + b

OR

23(b).

√𝒔𝒆𝒄 A – 1 √𝒔𝒆𝒄 A + 1
LHS = +
Sol. √𝒔𝒆𝒄 A + 1 √𝒔𝒆𝒄 A – 1

11
MS_X_Mathematics_041_30/5/2_2022-23
𝒔𝒆𝒄 A – 1 + sec A + 1 𝟏
=
√𝒔𝒆𝒄𝟐 𝑨 – 1
𝟏
𝟐 sec A 𝟐
= tan A
𝟏
= 2 cosec A = RHS 𝟐

24(a).

Sol. AP : AB = 2 : 5  AP : PB = 2 : 3 𝟏
𝟐

𝟖 + 12 𝟏𝟎 – 15
x= = 4, y = =–1
𝟓 𝟓
𝟏
Point P is (4, – 1)
𝟏
𝟐

OR

24(b).

12
MS_X_Mathematics_041_30/5/2_2022-23
Sol. 2 2 2 2 2
PA = PB  (x – 5) + (y – 1) = (x – 1) + (y – 5)
2 𝟏

x=y 𝟏

25

 QPT = 55
Sol.
1
  OPQ = 90 – 55 = 35 2

  OQP = 35

 POQ = 180 – 70 = 110 1

And reflex  POQ = 250

Hence  PRQ = 125 𝟏


𝟐

SECTION C

13
MS_X_Mathematics_041_30/5/2_2022-23
This section comprises of short answer (SA) type questions of 3 marks each.

26

Sol.

𝑥 – 52·5
Classes x f u= fu
15
0 – 15 7·5 17 –3 – 51
15 – 30 22·5 20 –2 – 40 Correct
Table
30 – 45 37·5 18 –1 – 18
2
45 – 60 52·5 21 0 0 Marks
60 – 75 67·5 15 1 15
75 – 90 82·5 9 2 18
100 – 76
–𝟕𝟔
Mean = 52·5 + 15 × (𝟏𝟎𝟎)

1
= 41·1

27.

Let unit’s digit be x and ten’s digit be y.


Sol.
∴ Number = 10y + x

10y + x = 7(x + y)  3y – 6x = 0
1
y = 2x ……………………………………………………….. (i)

14
MS_X_Mathematics_041_30/5/2_2022-23
10y + x – (10x + y) = 18

9y – 9x = 18 or y – x = 2 …………………………………. (ii) 1

On solving (i) and (ii), x = 2, y = 4 1


2

∴ required number is 42 1
2

28.

𝒔𝒊𝒏 𝜽 𝒄𝒐𝒔 𝜽
Sol. cos 𝜽 sin 𝜽
𝟏
LHS = cos 𝜽 + sin 𝜽
1– 1–
sin 𝜽 cos 𝜽

𝒔𝒊𝒏𝟐 𝜽 𝒄𝒐𝒔𝟐 𝜽
= + 𝟏
cos 𝜽(sin 𝜽 – cos 𝜽) sin 𝜽(cos 𝜽 – sin 𝜽)

𝒔𝒊𝒏𝟑 𝜽 – cos𝟑 𝜽
= sin 𝜽 cos 𝜽(sin 𝜽 – cos 𝜽)

𝒔𝒊𝒏𝟐 𝜽 + cos𝟐 𝜽 + sin 𝜽 cos 𝜽 𝟏


= sin 𝜽 cos 𝜽 𝟐
𝟏
= 𝒔𝒊𝒏 𝜽 cos 𝜽 + 1
𝟏
= 1 + cosec  sec  = RHS 𝟐

29(a).

Sol. Let √𝟑 be a rational number.

15
MS_X_Mathematics_041_30/5/2_2022-23
𝐩
∴ √𝟑 = 𝐪 , let p & q be co-primes and q≠0 ½

3q2 = p2 ⟹ p2 is divisible by 3 ⟹ p is divisible by 3


1
⟹ p = 3a, where ‘a’ is some integer ----- (i)

9a2 = 3q2 ⟹ q2 = 3a2 ⟹q2 is divisible by 3 ⟹ q is divisible by 3


½
⟹ q = 3b, where ‘b’ is some integer ----- (ii)

(i) and (ii) leads to contradiction as ‘p’ and ‘q’ are co-primes.
1
∴ √𝟑 is an irrational number.

OR

29(b).

LCM = 432
Sol. 𝟐
𝟒𝟑𝟐
i.e. = 7 min 12 sec.
𝟔𝟎
𝟏
 traffic lights will change simultaneously again at 7 : 7 : 12 a.m.

30.

Here Sn = 6n – n2
Sol.
1
n = 1, S1 = a = 5 2

n = 2, a + (a + d) = 12 – 4 or 2a + d = 8 1

16
MS_X_Mathematics_041_30/5/2_2022-23
Putting a = 5, d = – 2 1
2

Hence a30 = 5 + 29(– 2) = – 53


1

31(a).

 EFG   ADG
Sol.
𝑬𝑭 𝑭𝑮
 𝑨𝑫 = 𝑫𝑮 _________ (i) 𝟏

 EFC   BDC

𝑬𝑭 𝑪𝑭
 𝑩𝑫 = 𝑪𝑫 𝟏

𝑬𝑭 𝑪𝑭 𝟏
 = {BD = AD} _________ (ii) 𝟐
𝑨𝑫 𝑪𝑫

Using (i) and (ii)


𝟏
𝑭𝑮 𝑪𝑭 𝟐
= 𝑪𝑫
𝑫𝑮

OR

17
MS_X_Mathematics_041_30/5/2_2022-23
31(b).

 ALE   CLB
Sol.
𝑨𝑳 𝑬𝑳
 𝑪𝑳 = 𝑩𝑳 _________ (i) 𝟏

Also  CLM   ALB


𝟏
𝑨𝑳 𝑨𝑩
 𝑪𝑳 = 𝑪𝑴

𝑨𝑳 𝑪𝑫 𝟏
 𝑪𝑳 = 𝑪𝑴 {AB = CD} _________ (ii)
𝟐

Using (i) and (ii)

𝑬𝑳 𝟐𝑪𝑴
=
𝑩𝑳 𝑪𝑴 𝟏
𝟐
 EL = 2BL

SECTION D

This section comprises of long answer (LA) type questions of 5


marks each.

18
MS_X_Mathematics_041_30/5/2_2022-23
32(a).

Sol.

𝟏
Join OA, OB, OC and draw OE ⊥ AC and OF ⊥ AB. 1𝟐

BF = 10 cm, CE = 8 cm, Let AF = AE = x


𝟏
1𝟐
ar  ABC = ar  BOC + ar  COA + ar  AOB

19
MS_X_Mathematics_041_30/5/2_2022-23
𝟏
90 = 𝟐. 4 (BC + CA + AB)
1
90 = 2(18 + 8 + x + 10 + x)

90 = 4(18 + x)

x = 4·5
1
AB = 14·5 cm and AC = 12·5 cm

OR

32(b).

OO = √𝟔𝟐 + 8𝟐 = 10 cm {OP ⊥ OP}


Sol. 𝟏
𝟐
Let OA = x, OA = 10 – x
𝟏
𝟐
AP2 = 36 – x2
𝟏
Also, AP2 = 64 – (10 – x)2 𝟐

20
MS_X_Mathematics_041_30/5/2_2022-23
Therefore 36 – x2 = 64 – (10 – x)2

 36 – x2 = 64 – 100 – x2 + 20 x

 x = 3·6 𝟐

In  PAO, AP2 = 36 – (3·6)2 = 23·04


𝟏
 AP = 4·8
𝟏
Length PQ = 2 x AP = 9·6 cm 𝟐

33.

Area of that part of the field in which the horse can graze by means
Sol. 𝟏
of a 5 m long rope = 𝟒 × 𝟑. 𝟏𝟒 × (𝟓)𝟐 1

1
𝟐
= 𝟏𝟗. 𝟔𝟐𝟓 𝒎

Area of that part of the field in which the horse can graze by means 1
𝟏
of a 10 m long rope = 𝟒 × 𝟑. 𝟏𝟒 × (𝟏𝟎)𝟐

= 𝟕𝟖. 𝟓 𝒎𝟐 1

1
Increase in grazing area = 𝟕𝟖. 𝟓 𝒎𝟐 − 𝟏𝟗. 𝟔𝟐𝟓 𝒎𝟐 = 𝟓𝟖. 𝟖𝟕𝟓 𝒎𝟐

21
MS_X_Mathematics_041_30/5/2_2022-23
34

Sol.

Correct
Figure

1
Mark
Let AD be the tower

𝒉 1
tan 60 = √𝟑 = 𝒙  h = 𝒙√𝟑 2

𝟏 𝒉 – 10
tan 30 = = 1
√𝟑 𝒙
2

𝒙
 h = 10 +
√𝟑

22
MS_X_Mathematics_041_30/5/2_2022-23
𝒙
𝒙√𝟑 = 10 +  x = 𝟓√𝟑
√𝟑

1
(a) h = 15 m

(b) x = 𝟓√𝟑 m 1

𝑥
(c) = cos 60  PD = 10√3 m 1
𝑃𝐷

35(a).

Let first average speed of the train be x km/hr.


Sol.
𝟓𝟒 𝟔𝟑
+ 𝒙+6 = 3 2
𝒙

 54x + 324 + 63x = 3x2 + 18x

 3x2 – 99x – 324 = 0 or x2 – 33x – 108 = 0 2

 (x – 36) (x + 3) = 0
1
 x = 36, -3 (rejected)

Therefore, first average speed of the train was 36 km/hr.

OR

23
MS_X_Mathematics_041_30/5/2_2022-23
35(b).

Let the time taken by smaller diameter tap be x hrs.


Sol.
Time taken by larger diameter tap is (x – 2) hrs.

𝟏 𝟏 𝟖 2
Therefore 𝒙 – 2 + 𝒙 = 𝟏𝟓

 15(2x – 2) = 8x(x – 2)

 8x2 – 46x + 30 = 0

1
 4x2 – 23x + 15 = 0

 (4x – 3)(x – 5) = 0

𝟑
 x = 𝟒, x = 5 1

𝟑
x  𝟒 as x – 2 < 0

Smaller diameter tap fills in 5 hrs.

Larger diameter tap fills in 3 hrs. 1

SECTION E

This section comprises of 3 case-study based questions of 4 marks each.

24
MS_X_Mathematics_041_30/5/2_2022-23
36.

(i) All possible outcomes: RR, RG, RB, GR, GB, GG, YR, YB, YG
Sol. 1
(ii) Number of favourable outcome (RB) = 1

25
MS_X_Mathematics_041_30/5/2_2022-23
𝟏 1
P(Making purple) = 𝟗

𝟏
(iii)(a) As P(winning) = 𝟗

𝟏 𝟏
therefore number of people must win = 𝟗 x 99 = 11
𝟐

 Game lost by 88 persons. 𝟏


𝟐

Funds collected = 5 x 88 – 10 x 11 = ₹ 330 1

OR

(iii)(b) Number of participants = 99

𝟏
P(winning the game) = 𝟗
𝟏
Number of persons won = 11 𝟐
𝟏
Number of persons lost = 88
𝟐

Funds collected = 88 x 5 – 11 x 5 = ₹ 385 1

26
MS_X_Mathematics_041_30/5/2_2022-23
37.

𝟐𝟐 𝟏𝟕𝟔
(i) SA = 2r2 = 2 x x4= 𝐦𝐦𝟐 or 25·1 mm2
Sol. 𝟕 𝟕 1

𝟐 𝟐𝟐
(ii) Volume of material dug out to make one dimple = 𝟑 x x8
𝟕

𝟑𝟓𝟐
= 𝐦𝐦𝟑 or 16·76 mm3
𝟐𝟏 1

27
MS_X_Mathematics_041_30/5/2_2022-23
(iii)(a) radius of ball = 21 mm

Total surface area exposed to surroundings

= 4(21)2 – 315 x (2)2 + 315 x 2(2)2


1
𝟐𝟐 𝟐𝟐
=4x x 21 x 21 + x 315 x 4
𝟕 𝟕

1
= 9504 mm2

OR

𝟒 𝟐 1
(iii) (b) Volume of the golf ball = 𝟑(21)3 – 315 x 𝟑(2)3

= 33528 mm3
1

28
MS_X_Mathematics_041_30/5/2_2022-23
38.

29
MS_X_Mathematics_041_30/5/2_2022-23
(i) – 16t2 + 20t = 0  4t(– 4t + 5) = 0
Sol.
𝟓 1
t = 0, t = 𝟒

(ii) (a)
1
𝟑 𝟗 𝟑
(iii)(a) At t = 𝟐, h = – 16 x 𝟒 + 20 x 𝟐 = – 36 + 30 = – 6 1

𝟑
It means after 𝟐 seconds, dolphin has reached 6 cm below water
level. 1

OR

(iii)(b) Speed of dolphin = 20 cm per second.

In one second, distance covered = 20 cm

𝟓 𝟓 2
In 𝟒 seconds, distance covered = 20 x 𝟒 = 25 cm

30
MS_X_Mathematics_041_30/5/2_2022-23
Marking Scheme
Strictly Confidential
(For Internal and Restricted use only)
Secondary School Examination, 2023
MATHEMATICS PAPER CODE 30/5/3

General Instructions: -

1 You are aware that evaluation is the most important process in the actual and correct
assessment of the candidates. A small mistake in evaluation may lead to serious problems
which may affect the future of the candidates, education system and teaching profession. To
avoid mistakes, it is requested that before starting evaluation, you must read and understand
the spot evaluation guidelines carefully.
2 “Evaluation policy is a confidential policy as it is related to the confidentiality of the
examinations conducted, Evaluation done and several other aspects. Its’ leakage to
public in any manner could lead to derailment of the examination system and affect the
life and future of millions of candidates. Sharing this policy/document to anyone,
publishing in any magazine and printing in News Paper/Website etc may invite action
under various rules of the Board and IPC.”
3 Evaluation is to be done as per instructions provided in the Marking Scheme. It should not
be done according to one’s own interpretation or any other consideration. Marking Scheme
should be strictly adhered to and religiously followed. However, while evaluating, answers
which are based on latest information or knowledge and/or are innovative, they may be
assessed for their correctness otherwise and due marks be awarded to them.
4 The Marking scheme carries only suggested value points for the answers.
These are in the nature of Guidelines only and do not constitute the complete answer. The
students can have their own expression and if the expression is correct, the due marks should
be awarded accordingly.
5 The Head-Examiner must go through the first five answer books evaluated by each evaluator
on the first day, to ensure that evaluation has been carried out as per the instructions given
in the Marking Scheme. If there is any variation, the same should be zero after deliberation
and discussion. The remaining answer books meant for evaluation shall be given only after
ensuring that there is no significant variation in the marking of individual evaluators.
6 Evaluators will mark ( √ ) wherever answer is correct. For wrong answer CROSS ‘X” be
marked. Evaluators will not put right (✓) while evaluating which gives an impression that
answer is correct and no marks are awarded. This is most common mistake which
evaluators are committing.
7 If a question has parts, please award marks on the right-hand side for each part. Marks
awarded for different parts of the question should then be totaled up and written in the left-
hand margin and encircled. This may be followed strictly.
8 If a question does not have any parts, marks must be awarded in the left-hand margin and
encircled. This may also be followed strictly.

1
MS_X_Mathematics_041_30/5/3_2022-23
9 In Q1-Q20, if a candidate attempts the question more than once (without canceling the previous
attempt), marks shall be awarded for the first attempt only and the other answer scored out
with a note “Extra Question”.
10 In Q21-Q38, if a student has attempted an extra question, answer of the question deserving
more marks should be retained and the other answer scored out with a note “Extra Question”.
11 No marks to be deducted for the cumulative effect of an error. It should be penalized only once.
12 A full scale of marks __________ (example 0 to 80/70/60/50/40/30 marks as given in
Question Paper) has to be used. Please do not hesitate to award full marks if the answer
deserves it.
13 Every examiner has to necessarily do evaluation work for full working hours i.e., 8 hours
every day and evaluate 20 answer books per day in main subjects and 25 answer books per
day in other subjects (Details are given in Spot Guidelines).This is in view of the reduced
syllabus and number of questions in question paper.
14 Ensure that you do not make the following common types of errors committed by the
Examiner in the past:-
● Leaving answer or part thereof unassessed in an answer book.
● Giving more marks for an answer than assigned to it.
● Wrong totaling of marks awarded on an answer.
● Wrong transfer of marks from the inside pages of the answer book to the title page.
● Wrong question wise totaling on the title page.
● Wrong totaling of marks of the two columns on the title page.
● Wrong grand total.
● Marks in words and figures not tallying/not same.
● Wrong transfer of marks from the answer book to online award list.
● Answers marked as correct, but marks not awarded. (Ensure that the right tick mark is
correctly and clearly indicated. It should merely be a line. Same is with the X for
incorrect answer.)
● Half or a part of answer marked correct and the rest as wrong, but no marks awarded.
15 While evaluating the answer books if the answer is found to be totally incorrect, it should be
marked as cross (X) and awarded zero (0) Marks.
16 Any un assessed portion, non-carrying over of marks to the title page, or totaling error
detected by the candidate shall damage the prestige of all the personnel engaged in the
evaluation work as also of the Board. Hence, in order to uphold the prestige of all concerned,
it is again reiterated that the instructions be followed meticulously and judiciously.
17 The Examiners should acquaint themselves with the guidelines given in the “Guidelines for
spot Evaluation” before starting the actual evaluation.
18 Every Examiner shall also ensure that all the answers are evaluated, marks carried over to
the title page, correctly totaled and written in figures and words.
19 The candidates are entitled to obtain photocopy of the Answer Book on request on payment
of the prescribed processing fee. All Examiners/Additional Head Examiners/Head
Examiners are once again reminded that they must ensure that evaluation is carried out
strictly as per value points for each answer as given in the Marking Scheme.

2
MS_X_Mathematics_041_30/5/3_2022-23
MARKING SCHEME
MATHEMATICS (Subject Code–041)
(PAPER CODE: 30/5/3)

Q. No. EXPECTED OUTCOMES/VALUE POINTS Marks

SECTION A

Questions no. 1 to 18 are multiple choice questions (MCQs) and questions


number 19 and 20 are Assertion-Reason based questions of 1 mark each

1.

Sol. (d) 3 units 1

2.

Sol. (c) 6·5 cm 1

3
MS_X_Mathematics_041_30/5/3_2022-23
3.

Sol. (c) 8000 m3 1

4.

Sol. (c) 16 1

5.

Sol. (c) – 32 1

6.

Sol. 𝟒 1
(b) 𝟓𝟐

4
MS_X_Mathematics_041_30/5/3_2022-23
7.

Sol. (a) 𝟓√𝟑 cm 1

8.

𝟕
Sol. (b) 1
𝟎.𝟎𝟏

9.

Sol. (d) 3 1

5
MS_X_Mathematics_041_30/5/3_2022-23
10.

Sol. (a) x2 – 6x + 7 = 0 1

11.

Sol. (a) decreases by 2 1

12.

Sol. (b) Infinite 1

13.

Sol. (c) x = a, y = b 1

6
MS_X_Mathematics_041_30/5/3_2022-23
14.

Sol. (b) 3 1

15.

Sol. (d) 10 1

16.

Sol. (b)  B =  D 1

7
MS_X_Mathematics_041_30/5/3_2022-23
17.

Sol. (a) sin 60 1

18.

Sol. (a) 3 cm 1

8
MS_X_Mathematics_041_30/5/3_2022-23
19.

Sol. (c) Assertion (A) is true, but Reason (R) is false 1

20.

Sol. (c) Assertion (A) is true, but Reason (R) is false 1

SECTION B

This section comprises of very short answer (VSA) type questions of 2 marks
each.

21.

Sol. LCM of 18, 24, 36 is 72 1

Required greatest 3-digit number = 936. 1

22(a).

2 2 𝟏
m2 + n2 = (a cos  + b sin ) + (a sin  – b cos )
Sol.
𝟐

2 𝟏
= a2(cos2 + sin2) + b (sin2  + cos2 )

9
MS_X_Mathematics_041_30/5/3_2022-23
𝟏
2
= a2 + b 𝟐

OR

22(a).

Sol. √𝒔𝒆𝒄 A – 1 √𝒔𝒆𝒄 A + 1


LHS = +
√𝒔𝒆𝒄 A + 1 √𝒔𝒆𝒄 A – 1
𝟏
𝒔𝒆𝒄 A – 1 + sec A + 1
= 𝟏
√𝒔𝒆𝒄𝟐 𝑨 – 1
𝟐
𝟐 sec A
= 𝟏
tan A
𝟐
= 2 cosec A = RHS

23.

K 1
Sol. • k • •
A(5, –6) P(0, y) B(–1, –4)

𝟏
Let the point of division be P(0, y) which divides AB in the ratio K : 1 𝟐

–K+5 𝟏
0= K=5
K+1
𝟏
Ratio is 5:1 𝟐

10
MS_X_Mathematics_041_30/5/3_2022-23
24.

Sol.

Correct
figure
1 Mark

Let PQ and RS be tangents at the end of diameter AB.

 PAO =  RBO = 90 ½

½
 PQ ‖ RS

25(a).

11
MS_X_Mathematics_041_30/5/3_2022-23
Sol. AP : AB = 2 : 5  AP : PB = 2 : 3 𝟏
𝟐

𝟏
𝟖 + 12 𝟏𝟎 – 15
x= 𝟓
= 4, y = 𝟓
=–1
𝟏
Point P is (4, – 1) 𝟐

OR

25(b).

Sol. 2 2 2 2 2
PA = PB  (x – 5) + (y – 1) = (x – 1) + (y – 5)
2 𝟏

x=y 𝟏

SECTION C

This section comprises of short answer (SA) type questions of 3 marks each.

12
MS_X_Mathematics_041_30/5/3_2022-23
26(a).

Sol.  EFG   ADG

𝑬𝑭 𝑭𝑮 𝟏
 𝑨𝑫 = 𝑫𝑮 _________ (i)

 EFC   BDC

𝑬𝑭
 𝑩𝑫 = 𝑪𝑫
𝑪𝑭 𝟏

𝑬𝑭 𝑪𝑭
 = {BD = AD} _________ (ii) 𝟏
𝑨𝑫 𝑪𝑫
𝟐
Using (i) and (ii)

𝑭𝑮
= 𝑪𝑫
𝑪𝑭 𝟏
𝑫𝑮
𝟐

OR

13
MS_X_Mathematics_041_30/5/3_2022-23
26(b)

Sol.  ALE   CLB

𝑨𝑳 𝑬𝑳 𝟏
 𝑪𝑳 = 𝑩𝑳 _________ (i)

Also  CLM   ALB

𝑨𝑳
 𝑪𝑳 = 𝑪𝑴
𝑨𝑩 𝟏

𝑨𝑳 𝑪𝑫
 𝑪𝑳 = 𝑪𝑴 {AB = CD} _________ (ii) 𝟏
𝟐
Using (i) and (ii)

𝑬𝑳 𝟐𝑪𝑴
=
𝑩𝑳 𝑪𝑴

𝟏
 EL = 2BL 𝟐

27.

14
MS_X_Mathematics_041_30/5/3_2022-23
𝒙
Sol. Let the fraction be 𝒚

𝒙–1 𝟏
= 𝟑  3x – y = 3 1
𝒚

𝒙 𝟏
= 𝟒  4x – y = 8 1
𝒚+8

Solving to get x = 5, y = 12

𝟓 1
Fraction is
𝟏𝟐

28.

Sol. 𝒕𝒂𝒏 𝑨(1 – sec A) – tan A(1 + sec A)


LHS = 1
𝟏 – sec𝟐𝑨

– 2 tan A sec A
= 1
– tan𝟐𝑨

𝒄𝒐𝒔 𝑨 𝟏
= 2× ×
sin A cos A

= 2 cosec A = RHS
1

15
MS_X_Mathematics_041_30/5/3_2022-23
29.

𝒙 – 42·5
C.I. x f u= fu
Sol. 𝟓
25 – 30 27·5 14 –3 – 42
30 – 35 32·5 22 –2 – 44
35 – 40 37·5 16 –1 – 16 For
40 – 45 42·5 6 0 0 correct
table
45 – 50 47·5 5 1 5
50 – 55 52·5 3 2 6 2 Marks
55 – 60 57·5 4 3 12
70 – 79

𝟕𝟗
Mean = 42·5 – x 5 = 36·86
𝟕𝟎 𝟏

30(a).

Sol. Let √𝟑 be a rational number.


𝐩 ½
∴ √𝟑 = 𝐪 , let p & q be co-primes and q≠0

3q2 = p2 ⟹ p2 is divisible by 3 ⟹ p is divisible by 3


1
⟹ p = 3a, where ‘a’ is some integer ----- (i)

9a2 = 3q2 ⟹ q2 = 3a2 ⟹q2 is divisible by 3 ⟹ q is divisible by 3


½
⟹ q = 3b, where ‘b’ is some integer ----- (ii)

(i) and (ii) leads to contradiction as ‘p’ and ‘q’ are co-primes.
1
∴ √𝟑 is an irrational number.

16
MS_X_Mathematics_041_30/5/3_2022-23
OR

30(b).

LCM = 432
Sol. 𝟐
𝟒𝟑𝟐
i.e. = 7 min 12 sec.
𝟔𝟎

 traffic lights will change simultaneously again at 7 : 7 : 12 a.m. 𝟏

31.

Sol. a = 8, l = 65

𝒏
730 = 𝟐 [8 + 65]
½
𝟕𝟑𝟎 × 2
𝒏= = 20 1
73

∴ l = a + 19 d ⟹ 65 = 18 + 19 d 1

⟹d=3 ½

SECTION D

This section comprises of long answer (LA) type questions of 5 marks each.

17
MS_X_Mathematics_041_30/5/3_2022-23
32(a).

Sol. Let first average speed of the train be x km/hr.

𝟓𝟒 𝟔𝟑 2
+ 𝒙+6 = 3
𝒙

 54x + 324 + 63x = 3x2 + 18x

 3x2 – 99x – 324 = 0 or x2 – 33x – 108 = 0 2

 (x – 36) (x + 3) = 0

 x = 36, -3 (rejected) 1

Therefore, first average speed of the train was 36 km/hr.

OR

32(b).

Sol. Let the time taken by smaller diameter tap be x hrs.

Time taken by larger diameter tap is (x – 2) hrs.

18
MS_X_Mathematics_041_30/5/3_2022-23
𝟏 𝟏 𝟖 2
Therefore 𝒙 – 2 + 𝒙 = 𝟏𝟓

 15(2x – 2) = 8x(x – 2)

 8x2 – 46x + 30 = 0

1
 4x2 – 23x + 15 = 0

 (4x – 3)(x – 5) = 0

𝟑
 x = 𝟒, x = 5 1

𝟑
x  𝟒 as x – 2 < 0

Smaller diameter tap fills in 5 hrs.

Larger diameter tap fills in 3 hrs. 1

33.

Sol. Area of that part of the field in which the horse can graze by means of a 5 1
𝟏
m long rope = 𝟒 × 𝟑. 𝟏𝟒 × (𝟓)𝟐

= 𝟏𝟗. 𝟔𝟐𝟓 𝒎𝟐 1

Area of that part of the field in which the horse can graze by means of a
𝟏
10 m long rope = 𝟒 × 𝟑. 𝟏𝟒 × (𝟏𝟎)𝟐 1

1
= 𝟕𝟖. 𝟓 𝒎𝟐

19
MS_X_Mathematics_041_30/5/3_2022-23
Increase in grazing area = 𝟕𝟖. 𝟓 𝒎𝟐 − 𝟏𝟗. 𝟔𝟐𝟓 𝒎𝟐 = 𝟓𝟖. 𝟖𝟕𝟓 𝒎𝟐 1

34(a).

Sol.

𝟏
Join OA, OB, OC and draw OE ⊥ AC and OF ⊥ AB. 1𝟐

BF = 10 cm, CE = 8 cm, Let AF = AE = x


𝟏
1𝟐
ar  ABC = ar  BOC + ar  COA + ar  AOB

20
MS_X_Mathematics_041_30/5/3_2022-23
𝟏
90 = 𝟐. 4 (BC + CA + AB)

90 = 2(18 + 8 + x + 10 + x)
1
90 = 4(18 + x)

x = 4·5
1
AB = 14·5 cm and AC = 12·5 cm

OR

34(b).

Sol. OO = √𝟔𝟐 + 8𝟐 = 10 cm {OP ⊥ OP} 𝟏


𝟐
Let OA = x, OA = 10 – x

𝟏
AP2 = 36 – x2
𝟐

21
MS_X_Mathematics_041_30/5/3_2022-23
Also AP2 = 64 – (10 – x)2 𝟏
𝟐

Therefore 36 – x2 = 64 – (10 – x)2

 36 – x2 = 64 – 100 – x2 + 20 x

 x = 3·6
𝟐

In  PAO, AP2 = 36 – (3·6)2 = 23·04


𝟏
 AP = 4·8
𝟏
Length PQ = 2 x AP = 9·6 cm 𝟐

35.

Sol.

Correct
figure

1 Mark

22
MS_X_Mathematics_041_30/5/3_2022-23
Let PQ and RS be the pillars.

𝒉 + 20 1
tan 60 = √𝟑 =  h + 20 = 𝒙√𝟑 _________ (i)
𝒙

𝟏 𝒉
tan 30 = = 80 – x  𝒉√𝟑 = 80 – x __________ (ii) 1
√𝟑

Using (i) and (ii) x = 28·65, h = 29·56

AP = 28·65 m, AR = 51·35 m ½+½

½+½
PQ = h + 20 = 49.56 m and RS = 29.56 m

SECTION E

This section comprises of 3 case-study based questions of 4 marks each.

23
MS_X_Mathematics_041_30/5/3_2022-23
36.

24
MS_X_Mathematics_041_30/5/3_2022-23
Sol. (i) – 16t2 + 20t = 0  4t(– 4t + 5) = 0
1
𝟓
t = 0, t = 𝟒

(ii) (a) 1

𝟑 𝟗 𝟑 1
(iii)(a) At t = 𝟐, h = – 16 x 𝟒 + 20 x 𝟐 = – 36 + 30 = – 6

𝟑
It means after 𝟐 seconds, dolphin has reached 6 cm below water level. 1

OR

(iii)(b) Speed of dolphin = 20 cm per second.

In one second, distance covered = 20 cm

25
MS_X_Mathematics_041_30/5/3_2022-23
𝟓 𝟓 2
In 𝟒 seconds, distance covered = 20 x 𝟒 = 25 cm

37.

𝟐𝟐 𝟏𝟕𝟔
Sol. (i) SA = 2r2 = 2 x x4= 𝐦𝐦𝟐 or 25·1 mm2 1
𝟕 𝟕

𝟐 𝟐𝟐
(ii) Volume of material dug out to make one dimple = 𝟑 x x8
𝟕

26
MS_X_Mathematics_041_30/5/3_2022-23
𝟑𝟓𝟐 1
= 𝐦𝐦𝟑 or 16·76 mm3
𝟐𝟏

(iii)(a) radius of ball = 21 mm

Total surface area exposed to surroundings

1
= 4(21)2 – 315 x (2)2 + 315 x 2(2)2

𝟐𝟐 𝟐𝟐
=4x x 21 x 21 + x 315 x 4
𝟕 𝟕

= 9504 mm2 1

OR

𝟒 𝟐
(iii) (b) Volume of the golf ball = 𝟑(21)3 – 315 x 𝟑(2)3 1

1
= 33528 mm3

27
MS_X_Mathematics_041_30/5/3_2022-23
38.

Sol. (i) All possible outcomes : RR, RG, RB, GR, GB, GG, YR, YB, YG 1

(ii) Number of favourable outcome (RB) = 1

28
MS_X_Mathematics_041_30/5/3_2022-23
𝟏 1
P(Making purple) = 𝟗

𝟏
(iii)(a) As P(winning) = 𝟗

𝟏 𝟏
therefore number of people must win = 𝟗 x 99 = 11
𝟐

 Game lost by 88 persons. 𝟏


𝟐

Funds collected = 5 x 88 – 10 x 11 = ₹ 330 1

OR

(iii)(b) Number of participants = 99

𝟏
P(winning the game) = 𝟗
𝟏
Number of persons won = 11 𝟐
𝟏
Number of persons lost = 88
𝟐

Funds collected = 88 x 5 – 11 x 5 = ₹ 385 1

29
MS_X_Mathematics_041_30/5/3_2022-23
Marking Scheme
Strictly Confidential
(For Internal and Restricted use only)
Secondary School Examination, 2023
MATHEMATICS PAPER CODE 30/6/1

General Instructions: -

1 You are aware that evaluation is the most important process in the actual and correct
assessment of the candidates. A small mistake in evaluation may lead to serious problems
which may affect the future of the candidates, education system and teaching profession. To
avoid mistakes, it is requested that before starting evaluation, you must read and understand
the spot evaluation guidelines carefully.
2 “Evaluation policy is a confidential policy as it is related to the confidentiality of the
examinations conducted, Evaluation done and several other aspects. Its’ leakage to
public in any manner could lead to derailment of the examination system and affect the
life and future of millions of candidates. Sharing this policy/document to anyone,
publishing in any magazine and printing in News Paper/Website etc may invite action
under various rules of the Board and IPC.”
3 Evaluation is to be done as per instructions provided in the Marking Scheme. It should not
be done according to one’s own interpretation or any other consideration. Marking Scheme
should be strictly adhered to and religiously followed. However, while evaluating, answers
which are based on latest information or knowledge and/or are innovative, they may be
assessed for their correctness otherwise and due marks be awarded to them.
4 The Marking scheme carries only suggested value points for the answers.
These are in the nature of Guidelines only and do not constitute the complete answer. The
students can have their own expression and if the expression is correct, the due marks should
be awarded accordingly.
5 The Head-Examiner must go through the first five answer books evaluated by each evaluator
on the first day, to ensure that evaluation has been carried out as per the instructions given
in the Marking Scheme. If there is any variation, the same should be zero after deliberation
and discussion. The remaining answer books meant for evaluation shall be given only after
ensuring that there is no significant variation in the marking of individual evaluators.
6 Evaluators will mark ( √ ) wherever answer is correct. For wrong answer CROSS ‘X” be
marked. Evaluators will not put right (✓) while evaluating which gives an impression that
answer is correct and no marks are awarded. This is most common mistake which
evaluators are committing.
7 If a question has parts, please award marks on the right-hand side for each part. Marks
awarded for different parts of the question should then be totaled up and written in the left-
hand margin and encircled. This may be followed strictly.
8 If a question does not have any parts, marks must be awarded in the left-hand margin and
encircled. This may also be followed strictly.

1
MS_X_Mathematics_041_30/6/1_2022-23
9 In Q1-Q20, if a candidate attempts the question more than once (without canceling the previous
attempt), marks shall be awarded for the first attempt only and the other answer scored out
with a note “Extra Question”.
10 In Q21-Q38, if a student has attempted an extra question, answer of the question deserving
more marks should be retained and the other answer scored out with a note “Extra Question”.
11 No marks to be deducted for the cumulative effect of an error. It should be penalized only once.
12 A full scale of marks __________ (example 0 to 80/70/60/50/40/30 marks as given in
Question Paper) has to be used. Please do not hesitate to award full marks if the answer
deserves it.
13 Every examiner has to necessarily do evaluation work for full working hours i.e., 8 hours
every day and evaluate 20 answer books per day in main subjects and 25 answer books per
day in other subjects (Details are given in Spot Guidelines). This is in view of the reduced
syllabus and number of questions in question paper.
14 Ensure that you do not make the following common types of errors committed by the
Examiner in the past:-
● Leaving answer or part thereof unassessed in an answer book.
● Giving more marks for an answer than assigned to it.
● Wrong totaling of marks awarded on an answer.
● Wrong transfer of marks from the inside pages of the answer book to the title page.
● Wrong question wise totaling on the title page.
● Wrong totaling of marks of the two columns on the title page.
● Wrong grand total.
● Marks in words and figures not tallying/not same.
● Wrong transfer of marks from the answer book to online award list.
● Answers marked as correct, but marks not awarded. (Ensure that the right tick mark is
correctly and clearly indicated. It should merely be a line. Same is with the X for
incorrect answer.)
● Half or a part of answer marked correct and the rest as wrong, but no marks awarded.
15 While evaluating the answer books if the answer is found to be totally incorrect, it should be
marked as cross (X) and awarded zero (0) Marks.
16 Any un assessed portion, non-carrying over of marks to the title page, or totaling error
detected by the candidate shall damage the prestige of all the personnel engaged in the
evaluation work as also of the Board. Hence, in order to uphold the prestige of all concerned,
it is again reiterated that the instructions be followed meticulously and judiciously.
17 The Examiners should acquaint themselves with the guidelines given in the “Guidelines for
spot Evaluation” before starting the actual evaluation.
18 Every Examiner shall also ensure that all the answers are evaluated, marks carried over to
the title page, correctly totaled and written in figures and words.
19 The candidates are entitled to obtain photocopy of the Answer Book on request on payment
of the prescribed processing fee. All Examiners/Additional Head Examiners/Head
Examiners are once again reminded that they must ensure that evaluation is carried out
strictly as per value points for each answer as given in the Marking Scheme.

2
MS_X_Mathematics_041_30/6/1_2022-23
MARKING SCHEME
MATHEMATICS (Subject Code–041)
(PAPER CODE: 30/6/1)

Q. No. EXPECTED OUTCOMES/VALUE POINTS Marks


SECTION A
Questions no. 1 to 18 are multiple choice questions (MCQs) and questions
number 19 and 20 are Assertion-Reason based questions of 1 mark each
1.

Sol. (C) rational 1


2.

Sol. (C)8 units 1


3.

Sol. (D)more than 3 1

4.

Sol. (A) (0, – 3) 1

3
MS_X_Mathematics_041_30/6/1_2022-23
5.

Sol. (D) 4 : 5 1

6.

Sol. (D) 0 1

7.

Sol. (D) – 1 1

8.

Sol. (B) 27 cm 1
9.

Sol. 𝑏2 1
(D)
4𝑎

4
MS_X_Mathematics_041_30/6/1_2022-23
10.

Sol. (B) 2·25 cm 1


11.

Sol. 7 1
(C) 20

12.

Sol. (D) 416 cm3 1


13.

Sol. (C) 3x + y = 900, 5x + 3y = 2100 1

5
MS_X_Mathematics_041_30/6/1_2022-23
14.

Sol. 1 1
(A) 62
2
15.

Sol. 3 1
(B) 13
16.

Sol. (D) tan2– sec2  = 1 1

17.

Sol. (D) a = 0, b = – 6 1
18.

Sol. (D) 4 1

6
MS_X_Mathematics_041_30/6/1_2022-23
19.

Sol. (A) 1

20.

Sol. (A) 1

21(A).
Sol. n
If 6 ends with digit 0, it would be divisible by 5. So, prime factorization of
n n n n
6 would contain 5. But 6 = (2 × 3) , the only prime factorization of 6
are 2 and 3 as per fundamental theorem of Arithmetic . There is no other
n
prime in the factorization of 6 . So, there is no natural number n for which 2
n
6 ends with digit zero.

OR

21(B)

7
MS_X_Mathematics_041_30/6/1_2022-23
Sol.
72=23 X 32
120=23 X 3 X 5
HCF = 24 1
LCM=360 1

22.

Sol. 1
Let the coordinates of P and Q be (0, y) and
2
(x, 0) respectively.

∵ R(2, 5) is the midpoint of PQ


0+x 𝑦+0 1 1
= 2 and =5 2
+2
2 2

∴ x = 4, y = 10

1
P(0, 10) and Q(4, 0)
2

23.

Sol.

Pole of height AB = 18 m
AP = length of shadow 1
18
In  APB, tan  = 𝐴𝑃
1
6 18
= 2
7 𝐴𝑃 1
 AP = 21 m 2

8
MS_X_Mathematics_041_30/6/1_2022-23
24.

Sol. 1
 AOB = 180 – 130 = 50
2

 OAP = 90 1
2
∴  APB = 180 – (50 + 90) = 40
1

25(A).

Sol.
In  ABC, DE ∥ BC
𝐴𝐷 𝐴𝐸 𝑥 𝑥+2 1
∴ =  =
𝐷𝐵 𝐸𝐶 𝑥–2 𝑥–1
x(x – 1) = (x + 2)(x – 2)
x2 – x = x2 – 4  x = 4 1

OR

9
MS_X_Mathematics_041_30/6/1_2022-23
25(B).

Sol.
In  AOB and  COD,

 OAB =  OCD

 OBA =  ODC
1
Therefore,  AOB   COD 1
2

𝑂𝐴 𝑂𝐵 1
∴ =
𝑂𝐶 𝑂𝐷 2

26.

Sol.
Let P(x, 0) be the point on x axis which divides AB in the ratio k : 1 1
k:1 2
A(6, 3) • B(– 2, – 5)
– 5k + 3 3 P
=0k= 2
𝑘+1 5
Ratio is 3 : 5 1
2

27(A).

Sol. 26= 13 x 2
65= 13 x 5 1
117= 13 x 3 x 3

10
MS_X_Mathematics_041_30/6/1_2022-23
∴ HCF = 13 1
LCM = 13 x 2 x 3 x 5 x 3 = 1170 1

OR

27(B)

Sol. Let √𝟐 be a rational number.


𝐩 ½
∴ √2 = , where q≠0 and let p & q be co-primes.
𝐪
2q2 = p2 ⟹ p2 is divisible by 2 ⟹ p is divisible by 2
1
⟹ p = 2a, where ‘a’ is some integer ----- (i)
4a2 = 2q2 ⟹ q2 = 2a2 ⟹q2 is divisible by 2 ⟹ q is divisible by 2
½
⟹ q = 2b, where ‘b’ is some integer ----- (ii)
(i) and (ii) leads to contradiction as ‘p’ and ‘q’ are co-primes.
1
∴ √𝟐 is an irrational number.
28.

Sol. ABC is an isosceles triangle

∴ 𝐴𝐵 = 𝐴𝐶 ⇒ ∠𝐵 = ∠𝐶 1

In  ABD and  ECF,

 ADB =  EFC
1
 ABD =  ECF
1
∴  ABD   ECF

11
MS_X_Mathematics_041_30/6/1_2022-23
29(A).

Sol. 1
Let one number be x  another number = 15 – x
2

1 1 3
Therefore, + = 1
𝑥 15 – x 10

15 – x + x 3
=  150 = 3x(15 – x)
𝑥(15 – x) 10

3x2 – 45x + 150 = 0 1


2
x2 – 15x + 50 = 0  (x – 10)(x – 5) = 0
1
⇒x = 10, 5
2
1
Numbers are 10, 5 or 5, 10 2

OR

29(B).

Sol.
x2 – 7x + 10 = 0
1
 +  = 7,  = 10 2

1
2 + 2 = ( + )2 – 2 = 49 – 20 = 29

22 = (10)2 = 100 1

Quadratic Equation with roots 2, 2 is

12
MS_X_Mathematics_041_30/6/1_2022-23
∴x2 – (𝛼 2 + 𝛽 2 )x + 𝛼 2 𝛽 2 = 0
1
2
i.e. x2 – 29x + 100 = 0

30.

Sol. 1
1 + sec A 1+
cos A
LHS = = 1
𝑠𝑒𝑐 𝐴
𝑐𝑜𝑠 𝐴

= 1 + cos A 1

(1 – cos A)(1 + cos A)


= 1
(1 – cos A)

1− 𝑐𝑜𝑠2 𝐴
=
1−cos 𝐴

𝑠𝑖𝑛2 𝐴
= = RHS 1
1−cos 𝐴

31.

Sol.
A=
60
×
22
× 21 × 21 = 231 𝑐𝑚 2 1
360 7 1
2
60 22
Length of arc = × 2 × × 21
360 7
= 22 cm 1
1
2

13
MS_X_Mathematics_041_30/6/1_2022-23
32(A).

Sol.
TP = TQ
1
⇒  TPQ =  TQP

Let  PTQ be 

180° – 𝜃 𝜃
⇒  TPQ =  TQP = = 90 – 12
1
2 2

Now  OPT = 90

𝜃 𝜃 1
⇒ OPQ = 90 – (90 – ) = 12
2 2

 PTQ = 2  OPQ
1

OR

14
MS_X_Mathematics_041_30/6/1_2022-23
32(B).

Sol. 1
AQ = AR
1
2AQ = AQ + AR
1
= AB + BQ + AC + CR 2
1
= AB + AC + (BP + CP) 2

= AB + AC + BC 1

1 1 1
AQ = 2 (AB + AC + BC) = 2 (Perimeter of △ ABC)

33.

Sol.
Radius of cone = radius of hemisphere = 7 cm
1
∴Height of cone = 14 cm

Volume of solid = Volume of hemisphere + volume of cone

15
MS_X_Mathematics_041_30/6/1_2022-23
2 1 1 1
= 3(7)3 + (7)214 12 + 1 2
3

1 22
= × × 7 × 7(14 + 14)
3 7

154 4312
= × 28 = 𝑐𝑚2 or 1437.33 𝑐𝑚2 1
3 3

34(A).

Sol. 𝑎 + 10d 2
= 1
a + 17d 3

3a + 30d = 2a + 34d  a = 4d 1

𝑎 + 4d 4𝑑 + 4d 8𝑑 1
Therefore, = = = 1
a + 20d 4d + 20d 24𝑑 3

5
𝑆5 [2𝑎 + 4d] 5[8𝑑 + 4d]
2
= 21 =
𝑆21 [2𝑎 + 20d] 21[8𝑑 + 20d] 1
2

5 × 12d 5
= = or 𝑆5 : 𝑆21 = 5 ∶ 49
21 × 28d 49 1

OR

34(B).

Sol. 6
S6 = 36  [2a + 5d] = 36 1
2

16
MS_X_Mathematics_041_30/6/1_2022-23
 2a + 5d = 12 ----------- (1) 1

16
S16 = 256  [2a + 15d] = 256
2

 2a + 15d = 32 ------------ (2) 1

Solving (1) and (2)

1
d=2

a=1

10
S10 = [2(1) + 9(2)]
2
1
= 100

35.

Sol.
(i) 20 + 60 + 70 + x + 60 = 250
1
x = 250 – 210 = 40
Mass 80 – 100 100 – 120 120 – 140 140 – 160 160 – 180 Total
No. of 250
apples fi 20 60 70 x = 40 60
1 for
xi 90 110 130 150 170 correct
xifi 1800 6600 9100 6000 10200 33700 table

33700
Mean mass = = 134·8 1
250

1
Mean mass = 134.8 g
2

17
MS_X_Mathematics_041_30/6/1_2022-23
(ii) Modal class = 120-140

(70 – 60)
Mode = 120 + (140 – 60 – 40) × 20 1

= 125
1
Hence modal mass = 125 gm
2

36.

18
MS_X_Mathematics_041_30/6/1_2022-23
Sol.
(i) 20x + 5y = 9000
1
5x + 25y = 26000

(ii) Solving the equations x = 200

Monthly fee paid by poor child = ₹200 2

OR

1
(ii) getting x=200 and y= 1000 1+
2

1
Difference in the fee = 1000 – 200 = ₹ 800 2

(iii)10x + 20y = 10(200) + 20(1000)

= ₹ 22000
1

19
MS_X_Mathematics_041_30/6/1_2022-23
37.

Sol. 𝑂𝑃
1
(i) In  OBP, cos 30 = 𝑂𝐵 2

√3 36 72
= 𝑂𝐵  OB = 1
2 √3
= 24√3 cm 2

20
MS_X_Mathematics_041_30/6/1_2022-23
𝑃𝐵 36
(ii)In  OBP, tan 30 = 36  PB =
√3

1
PB = 12√3

𝐴𝑃 1
In  OAP, tan 45 = 36  AP = 36 cm
2
1
AB = AP – PB = 36 – 12√3 = 12(3 – √3) cm 2

OR

1
(ii)Area of  OPB = × OP × PB
2

1
= × 36 × 12√3 = 216√3 cm2
2 1+1

(iii) AP = 36 cm 1

21
MS_X_Mathematics_041_30/6/1_2022-23
38.

Sol. 1
(i)P (drawing ball bearing number 8) = 1
15

1
(ii)Even numbers = 2, 4, 6, 8, 10, 12, 14 2

No. of favourable outcomes = 7

7 1
P (even number ball) = 12
15

OR

(ii)Multiples of 3 are 3, 6, 9, 12, 15 1


2

22
MS_X_Mathematics_041_30/6/1_2022-23
No. of favourable outcomes = 5

5 1 1
12
∴ P(multiple of 3) = =
15 3

(iii) Solid colour and even number 2, 4, 6, 8

4
P(solid colour and bear an even no.) = 1
15

23
MS_X_Mathematics_041_30/6/1_2022-23
Marking Scheme
Strictly Confidential
(For Internal and Restricted use only)
Secondary School Examination, 2023
MATHEMATICS PAPER CODE 30/6/2

General Instructions: -

1 You are aware that evaluation is the most important process in the actual and correct
assessment of the candidates. A small mistake in evaluation may lead to serious problems
which may affect the future of the candidates, education system and teaching profession. To
avoid mistakes, it is requested that before starting evaluation, you must read and understand
the spot evaluation guidelines carefully.
2 “Evaluation policy is a confidential policy as it is related to the confidentiality of the
examinations conducted, Evaluation done and several other aspects. Its’ leakage to
public in any manner could lead to derailment of the examination system and affect the
life and future of millions of candidates. Sharing this policy/document to anyone,
publishing in any magazine and printing in News Paper/Website etc may invite action
under various rules of the Board and IPC.”
3 Evaluation is to be done as per instructions provided in the Marking Scheme. It should not
be done according to one’s own interpretation or any other consideration. Marking Scheme
should be strictly adhered to and religiously followed. However, while evaluating, answers
which are based on latest information or knowledge and/or are innovative, they may be
assessed for their correctness otherwise and due marks be awarded to them.
4 The Marking scheme carries only suggested value points for the answers.
These are in the nature of Guidelines only and do not constitute the complete answer. The
students can have their own expression and if the expression is correct, the due marks should
be awarded accordingly.
5 The Head-Examiner must go through the first five answer books evaluated by each evaluator
on the first day, to ensure that evaluation has been carried out as per the instructions given
in the Marking Scheme. If there is any variation, the same should be zero after deliberation
and discussion. The remaining answer books meant for evaluation shall be given only after
ensuring that there is no significant variation in the marking of individual evaluators.
6 Evaluators will mark ( √ ) wherever answer is correct. For wrong answer CROSS ‘X” be
marked. Evaluators will not put right (✓) while evaluating which gives an impression that
answer is correct and no marks are awarded. This is most common mistake which
evaluators are committing.
7 If a question has parts, please award marks on the right-hand side for each part. Marks
awarded for different parts of the question should then be totaled up and written in the left-
hand margin and encircled. This may be followed strictly.
8 If a question does not have any parts, marks must be awarded in the left-hand margin and
encircled. This may also be followed strictly.

1
MS_X_Mathematics_041_30/6/2_2022-23
9 In Q1-Q20, if a candidate attempts the question more than once (without canceling the previous
attempt), marks shall be awarded for the first attempt only and the other answer scored out
with a note “Extra Question”.
10 In Q21-Q38, if a student has attempted an extra question, answer of the question deserving
more marks should be retained and the other answer scored out with a note “Extra Question”.
11 No marks to be deducted for the cumulative effect of an error. It should be penalized only once.
12 A full scale of marks __________ (example 0 to 80/70/60/50/40/30 marks as given in
Question Paper) has to be used. Please do not hesitate to award full marks if the answer
deserves it.
13 Every examiner has to necessarily do evaluation work for full working hours i.e., 8 hours
every day and evaluate 20 answer books per day in main subjects and 25 answer books per
day in other subjects (Details are given in Spot Guidelines). This is in view of the reduced
syllabus and number of questions in question paper.
14 Ensure that you do not make the following common types of errors committed by the
Examiner in the past:-
● Leaving answer or part thereof unassessed in an answer book.
● Giving more marks for an answer than assigned to it.
● Wrong totaling of marks awarded on an answer.
● Wrong transfer of marks from the inside pages of the answer book to the title page.
● Wrong question wise totaling on the title page.
● Wrong totaling of marks of the two columns on the title page.
● Wrong grand total.
● Marks in words and figures not tallying/not same.
● Wrong transfer of marks from the answer book to online award list.
● Answers marked as correct, but marks not awarded. (Ensure that the right tick mark is
correctly and clearly indicated. It should merely be a line. Same is with the X for
incorrect answer.)
● Half or a part of answer marked correct and the rest as wrong, but no marks awarded.
15 While evaluating the answer books if the answer is found to be totally incorrect, it should be
marked as cross (X) and awarded zero (0)Marks.
16 Any un assessed portion, non-carrying over of marks to the title page, or totaling error
detected by the candidate shall damage the prestige of all the personnel engaged in the
evaluation work as also of the Board. Hence, in order to uphold the prestige of all concerned,
it is again reiterated that the instructions be followed meticulously and judiciously.
17 The Examiners should acquaint themselves with the guidelines given in the “Guidelines for
spot Evaluation” before starting the actual evaluation.
18 Every Examiner shall also ensure that all the answers are evaluated, marks carried over to
the title page, correctly totaled and written in figures and words.
19 The candidates are entitled to obtain photocopy of the Answer Book on request on payment
of the prescribed processing fee. All Examiners/Additional Head Examiners/Head
Examiners are once again reminded that they must ensure that evaluation is carried out
strictly as per value points for each answer as given in the Marking Scheme.

2
MS_X_Mathematics_041_30/6/2_2022-23
MARKING SCHEME
MATHEMATICS (Subject Code–041)
(PAPER CODE: 30/6/2)

Q. No. EXPECTED OUTCOMES/VALUE POINTS Marks


SECTION A
Questions no. 1 to 18 are multiple choice questions (MCQs) and questions
number 19 and 20 are Assertion-Reason based questions of 1 mark each

1.

Sol. (D) a = 0, b = – 6 1
2.

Sol. (D)more than 3 1


3.

Sol. (D) 4 1

4.

Sol. (D) – 1 1

3
MS_X_Mathematics_041_30/6/2_2022-23
5.

Sol. (B) 2·25 cm 1

6.

Sol. (C) 20 1

7.

Sol. (B) 4 1

8.

Sol. (A) – 10 1
9.

Sol. (A) (0, – 3) 1

4
MS_X_Mathematics_041_30/6/2_2022-23
10.

Sol. 𝑏2 1
(D)
4𝑎
11.

3
Sol. (B) 1
13

12.

Sol. (B) 27 cm 1
13.

Sol. (D) 416 cm3 1

14.

Sol. (D) 4 : 5 1

5
MS_X_Mathematics_041_30/6/2_2022-23
15.

Sol. 1 1
(A) 622
16.

Sol. 𝟒 1
(C) 𝟒𝟓

17.

Sol. 𝟓 1
(C) (– , 0)
𝟒

18.

Sol. (D) 2 cos2 A – 1 1

6
MS_X_Mathematics_041_30/6/2_2022-23
19.

Sol. (A) 1

20.

Sol. (A) 1

21.

Sol.

Pole of height AB = 18 m
AP = length of shadow
18 1
In  APB, tan  = 𝐴𝑃
6 18 1
= 2
7 𝐴𝑃
1
2
 AP = 21 m
7
MS_X_Mathematics_041_30/6/2_2022-23
22(A).

Sol.
In  ABC, DE ∥ BC
𝐴𝐷 𝐴𝐸 𝑥 𝑥+2 1
= 𝐸𝐶  𝑥 – 2 =
𝐷𝐵 𝑥–1

x(x – 1) = (x + 2)(x – 2)
x2 – x = x2 – 4  x = 4 1

OR
22(B).

Sol.
In  AOB and  COD,

 OAB =  OCD

 OBA =  ODC
1
1
Therefore,  AOB   COD 2
𝑂𝐴 𝑂𝐵
∴ 𝑂𝐶 = 𝑂𝐷 1
2
23(A).

8
MS_X_Mathematics_041_30/6/2_2022-23
Sol. n
If 6 ends with digit 0, it would be divisible by 5. So, prime factorization of
n n n n
6 would contain 5. But 6 = (2 × 3) , the only prime factorization of 6
are 2 and 3 as per fundamental theorem of Arithmetic. There is no other
n
prime in the factorization of 6 . So, there is no natural number n for which
n 2
6 ends with digit zero.

OR
23(B).

Sol.
72=23 X 32
120=23 X 3 X 5
HCF = 24 1
LCM=360 1

24.

Sol. 𝟏
Let the point on x-axis be P(x, 0) 𝟐

PA = 10  PA2 = 100

1
(x – 11)2 + (0 + 8)2 = 100

(x – 11)2 = 100 – 64 = 36

x – 11 = 6, – 6

𝟏
x = 17, 5
𝟐

9
MS_X_Mathematics_041_30/6/2_2022-23
25.

Sol. 1
 AOB = 180 – 30 = 50
2

 OAP = 90 1
2
∴ APB = 180 – (50 + 90) = 40 1

SECTION C
This section comprises of Short Answer (SA) type questions of 3 marks
each.
26.

10
MS_X_Mathematics_041_30/6/2_2022-23
Sol. 1 22
Area of quadrant BOC = × ×7×7
4 7

77 1
= cm2
2

1 1
Area of  BOC = × OB × OC = × 7 × 7
2 2

49
= cm2 1
2

77 49
Area of shaded region = 2 [ − ] = 28 cm2
2 2 1

27.

Sol.
sin  + cos  = p, sec  + cosec  = q

LHS = q(p2 – 1)

= (sec  + cosec)[(𝒔𝒊𝒏  + cos )2 − 1]

𝟏 𝟏 1
=[ + ] [𝒔𝒊𝒏𝟐  + 𝑐𝑜𝑠 2  + 2 𝑠𝑖𝑛cos − 1]
𝒄𝒐𝒔 𝒔𝒊𝒏 

𝑠𝑖𝑛 𝜃 + cos 𝜃
=( ) [1 + 2 sin  cos  – 1] 1
𝑐𝑜𝑠 𝜃 sin 𝜃

(𝑠𝑖𝑛 𝜃 + cos 𝜃)
= (2 sin  cos )
𝑐𝑜𝑠 𝜃 sin 𝜃

= 2(sin𝜃 + cos𝜃)

= 2p = RHS
1

11
MS_X_Mathematics_041_30/6/2_2022-23
28(A).

Sol. 1
Let one number be x  another number = 15 – x
2
1 1 3
Therefore, 𝑥 + 15 – x = 10 1

15 – x + x 3
= 10  150 = 3x(15 – x)
𝑥(15 – x)

3x2 – 45x + 150 = 0


1
x2 – 15x + 50 = 0  (x – 10)(x – 5) = 0 2

1
⇒x = 10, 5
2
1
Numbers are 10, 5 or 5, 10 2

OR

28(B).

12
MS_X_Mathematics_041_30/6/2_2022-23
Sol.
x2 – 7x + 10 = 0
1
 +  = 7,  = 10 2

2 + 2 = ( + )2 – 2 = 49 – 20 = 29 1

1
22 = (10)2 = 100 2

Q.E. with roots 2, 2 is

∴ x2 – (𝛼 2 + 𝛽 2 )x + 𝛼 2 𝛽 2 = 0

i.e. x2 – 29x + 100 = 0 1

29.

Sol.
Let P(x, 0) be the point on x axis which divides AB in the ratio k : 1 1
k:1
• 2
A(6, 3) B(– 2, – 5)
– 5k + 3 3 P
=0k= 2
𝑘+1 5

Ratio is 3 : 5 1
2

30.

13
MS_X_Mathematics_041_30/6/2_2022-23
Sol.
Given ABC is an isosceles triangle, ∴ 𝐴𝐵 = 𝐴𝐶 ⇒ ∠𝐵 = ∠𝐶
1

In  ABD and  ECF,

 ADB =  EFC (90 each, given)

 ABD =  ECF 1

∴  ABD   ECF 1
31(A).

Sol. 26= 13 x 2
65= 13 x 5 1
117= 13 x 3 x 3
∴ HCF = 13 1
LCM = 13 x 2 x 3 x 5 x 3 = 1170 1

OR
31(B).

Sol. Let √𝟐 be a rational number.


𝐩 ½
∴ √2 = 𝐪 , where q≠0 and let p & q be co-primes.
2q2 = p2 ⟹ p2 is divisible by 2 ⟹ p is divisible by 2
1
⟹ p = 2a, where ‘a’ is some integer ----- (i)
4a = 2q ⟹ q = 2a ⟹q is divisible by 2 ⟹ q is divisible by 2
2 2 2 2 2
½
⟹ q = 2b, where ‘b’ is some integer ----- (ii)
(i) and (ii) leads to contradiction as ‘p’ and ‘q’ are co-primes.
1
∴ √𝟐 is an irrational number.

SECTION D
This section comprises of Long Answer (LA) type questions of 5 marks
each.

14
MS_X_Mathematics_041_30/6/2_2022-23
32.

Sol.
(i)20 + 60 + 70 + x + 60 = 250

1
x = 250 – 210 = 40
Mass 80 – 100 100 – 120 120 – 140 140 – 160 160 – 180 Total
No. of 250
apples fi 20 60 70 x = 40 60
xi 90 110 130 150 170
1
xifi 1800 6600 9100 6000 10200 33700

33700
Mean mass = 250
= 134·8
Mean mass = 134.8 g 1

1
(ii) Modal class = 120-140
2
(70 – 60)
Mode = 120 + (140 – 60 – 40) × 20
1
= 125

Hence modal mass = 125 g 1


2

15
MS_X_Mathematics_041_30/6/2_2022-23
33(A).

Sol.
TP = TQ

⇒  TPQ =  TQP
1

Let  PTQ be 

180° – 𝜃 𝜃
⇒  TPQ =  TQP = = 90 – 2 1
1
2 2

Now  OPT = 90

𝜃 𝜃 1
⇒ OPQ = 90 – (90 – 2 ) = 2 12

 PTQ = 2  OPQ 1

OR

16
MS_X_Mathematics_041_30/6/2_2022-23
33(B).

Sol. 1
AQ = AR

1
2AQ = AQ + AR
1
= AB + BQ + AC + CR
2
1
= AB + AC + (BP + CP) 2

= AB + AC + BC 1

1 1
AQ = 2 (AB + AC + BC) =2(Perimeter of ∆ 𝐴𝐵𝐶) 1

34.

17
MS_X_Mathematics_041_30/6/2_2022-23
Sol. 7
Radius of hemisphere = Radius of cone = 3·5 cm = cm
2

Height of cone = 9·5 – 3·5 = 6 cm 1

Volume of solid = volume of hemisphere + volume of cone

2 22 7 3 1 22 7 2
𝟏 𝟏
= × ×( ) + × ×( ) ×6 𝟏 +𝟏
3 7 2 3 7 2 𝟐 𝟐

77 1001
= × 13 = = 166·8 cm3 1
6 6

9.5 cm
3.5 cm

35(A)

Sol. 𝟏
(i) Integers divisible by 9 are 108, 117, 126, …., 198 𝟐

a = 108, d = 9

a + (n – 1)d = 198

⇒108 + (n – 1)9 = 198  n = 11 1

𝑛 11 1
S11 = 2 (a + l) = (108 + 198)
2
𝟏
= 1683 𝟐

𝟏
(ii) Integers are 101, 102, 103,……,199 𝟐

18
MS_X_Mathematics_041_30/6/2_2022-23
99
Sum of all integers = (101 + 199)
2

99
= × 300 = 14850 1
2

Sum of integers not divisible by 9 = 14850 – 1683

= 13167 𝟏
𝟐

OR

35(B)

Sol.
– 4 + (– 1) + 2 + 5 + ….. + x = 437
𝟏
Here a = – 4, d = 3 𝟐

𝑥+7
-4 + (n – 1)3 = x  n =
3 1

Sn = 437

𝑥+7 1
⇒ ( 3 ) . (– 4 + x) = 437
2 1

x2 + 3x – 28 = 437 × 6 = 2622

x2 + 3x – 2650 = 0 1

(x + 53)(x – 50) = 0

1
x ≠– 53, x = 50 12

19
MS_X_Mathematics_041_30/6/2_2022-23
SECTION E
This section comprises of 3 case-study based questions of 4 marks each.
36.

Sol. 1
(i)P (drawing ball bearing number 8) = 15
1

(ii)Even numbers = 2, 4, 6, 8, 10, 12, 14 1


2

No. of favourable outcomes = 7

7
P (even number ball) = 15 1
1
2

OR

(ii)Multiples of 3 are 3, 6, 9, 12, 15


1
2

20
MS_X_Mathematics_041_30/6/2_2022-23
No. of favourable outcomes = 5

5 1 1
∴ P(multiple of 3) = 15 = 3 12

(iii) Solid colour and even number 2, 4, 6, 8

4 1
P(solid colour and bear an even no.) = 15

21
MS_X_Mathematics_041_30/6/2_2022-23
37.

Sol. 𝑂𝑃 1
(i) In  OBP, cos 30 = 𝑂𝐵 2

1
2

22
MS_X_Mathematics_041_30/6/2_2022-23
√3 36 72
= 𝑂𝐵  OB =
2 √3
= 24√3 cm

𝑃𝐵 36
(ii)In  OBP, tan 30 = 36  PB = 1
√3

1
PB = 12√3
2
1
𝐴𝑃
In  OAP, tan 45 = 36  AP = 36 cm 2

AB = AP – PB = 36 – 12√3 = 12(3 – √3) cm

OR

1 1+1
(ii)Area of  OPB = 2 × OP × PB

1
= 2 × 36 × 12√3 = 216√3 cm2 1

(ii) AP = 36 cm

23
MS_X_Mathematics_041_30/6/2_2022-23
38.

Sol.
(i)20x + 5y = 9000

1
5x + 25y = 26000

(ii)Solving the equations x = 200, y = 1000


2
Monthly fee paid by poor child = ₹200

OR

24
MS_X_Mathematics_041_30/6/2_2022-23
1
(ii) getting x=200 and y= 1000 1+2

Difference in the fee = 1000 – 200 = ₹ 800 1


2

(iii)10x + 20y = 10(200) + 20(1000)

= ₹ 22000 1

25
MS_X_Mathematics_041_30/6/2_2022-23
Marking Scheme
Strictly Confidential
(For Internal and Restricted use only)
Secondary School Examination, 2023
MATHEMATICS PAPER CODE 30/6/3

General Instructions: -

1 You are aware that evaluation is the most important process in the actual and correct
assessment of the candidates. A small mistake in evaluation may lead to serious problems
which may affect the future of the candidates, education system and teaching profession. To
avoid mistakes, it is requested that before starting evaluation, you must read and understand
the spot evaluation guidelines carefully.
2 “Evaluation policy is a confidential policy as it is related to the confidentiality of the
examinations conducted, Evaluation done and several other aspects. Its’ leakage to
public in any manner could lead to derailment of the examination system and affect the
life and future of millions of candidates. Sharing this policy/document to anyone,
publishing in any magazine and printing in News Paper/Website etc may invite action
under various rules of the Board and IPC.”
3 Evaluation is to be done as per instructions provided in the Marking Scheme. It should not
be done according to one’s own interpretation or any other consideration. Marking Scheme
should be strictly adhered to and religiously followed. However, while evaluating, answers
which are based on latest information or knowledge and/or are innovative, they may be
assessed for their correctness otherwise and due marks be awarded to them.
4 The Marking scheme carries only suggested value points for the answers.
These are in the nature of Guidelines only and do not constitute the complete answer. The
students can have their own expression and if the expression is correct, the due marks should
be awarded accordingly.
5 The Head-Examiner must go through the first five answer books evaluated by each evaluator
on the first day, to ensure that evaluation has been carried out as per the instructions given
in the Marking Scheme. If there is any variation, the same should be zero after deliberation
and discussion. The remaining answer books meant for evaluation shall be given only after
ensuring that there is no significant variation in the marking of individual evaluators.
6 Evaluators will mark ( √ ) wherever answer is correct. For wrong answer CROSS ‘X” be
marked. Evaluators will not put right (✓) while evaluating which gives an impression that
answer is correct and no marks are awarded. This is most common mistake which
evaluators are committing.
7 If a question has parts, please award marks on the right-hand side for each part. Marks
awarded for different parts of the question should then be totaled up and written in the left-
hand margin and encircled. This may be followed strictly.
8 If a question does not have any parts, marks must be awarded in the left-hand margin and
encircled. This may also be followed strictly.

1
MS_X_Mathematics_041_30/6/3_2022-23
9 In Q1-Q20, if a candidate attempts the question more than once (without canceling the previous
attempt), marks shall be awarded for the first attempt only and the other answer scored out
with a note “Extra Question”.
10 In Q21-Q38, if a student has attempted an extra question, answer of the question deserving
more marks should be retained and the other answer scored out with a note “Extra Question”.
11 No marks to be deducted for the cumulative effect of an error. It should be penalized only once.
12 A full scale of marks __________(example 0 to 80/70/60/50/40/30 marks as given in
Question Paper) has to be used. Please do not hesitate to award full marks if the answer
deserves it.
13 Every examiner has to necessarily do evaluation work for full working hours i.e., 8 hours
every day and evaluate 20 answer books per day in main subjects and 25 answer books per
day in other subjects (Details are given in Spot Guidelines).This is in view of the reduced
syllabus and number of questions in question paper.
14 Ensure that you do not make the following common types of errors committed by the
Examiner in the past:-
● Leaving answer or part thereof unassessed in an answer book.
● Giving more marks for an answer than assigned to it.
● Wrong totaling of marks awarded on an answer.
● Wrong transfer of marks from the inside pages of the answer book to the title page.
● Wrong question wise totaling on the title page.
● Wrong totaling of marks of the two columns on the title page.
● Wrong grand total.
● Marks in words and figures not tallying/not same.
● Wrong transfer of marks from the answer book to online award list.
● Answers marked as correct, but marks not awarded. (Ensure that the right tick mark is
correctly and clearly indicated. It should merely be a line. Same is with the X for
incorrect answer.)
● Half or a part of answer marked correct and the rest as wrong, but no marks awarded.
15 While evaluating the answer books if the answer is found to be totally incorrect, it should be
marked as cross (X) and awarded zero (0)Marks.
16 Any un assessed portion, non-carrying over of marks to the title page, or totaling error
detected by the candidate shall damage the prestige of all the personnel engaged in the
evaluation work as also of the Board. Hence, in order to uphold the prestige of all concerned,
it is again reiterated that the instructions be followed meticulously and judiciously.
17 The Examiners should acquaint themselves with the guidelines given in the “Guidelines for
spot Evaluation” before starting the actual evaluation.
18 Every Examiner shall also ensure that all the answers are evaluated, marks carried over to
the title page, correctly totaled and written in figures and words.
19 The candidates are entitled to obtain photocopy of the Answer Book on request on payment
of the prescribed processing fee. All Examiners/Additional Head Examiners/Head
Examiners are once again reminded that they must ensure that evaluation is carried out
strictly as per value points for each answer as given in the Marking Scheme.

2
MS_X_Mathematics_041_30/6/3_2022-23
MARKING SCHEME
MATHEMATICS (Subject Code–041)
(PAPER CODE: 30/6/3)

Q. No. EXPECTED OUTCOMES/VALUE POINTS Marks


SECTION A
Questions no. 1 to 18 are multiple choice questions (MCQs) and questions
number 19 and 20 are Assertion-Reason based questions of 1 mark each

1.

Sol. (B) 5 units 1


2.

𝒚
Sol. (B) 1
√𝒙𝟐 + y 𝟐
3.

Sol. 𝟏 1
(A) ,–4
𝟑
4.

Sol. (D) 4 : 5 1

3
MS_X_Mathematics_041_30/6/3_2022-23
5.

Sol. (D) 4 1

6.

Sol. (D) a = 0, b = – 6 1

7.

Sol. (C) rational 1

8.

Sol. (D) tan2 – sec2  = 1 1


9.

Sol. (A) (0, – 3) 1

4
MS_X_Mathematics_041_30/6/3_2022-23
10.

Sol. 1 1
(A) 622
11.

Sol. (D) 0 1

12.

Sol. (B) 27 cm 1
13.

Sol. (B) 2·25 cm 1

5
MS_X_Mathematics_041_30/6/3_2022-23
14.

Sol. (D) 416 cm3 1


15.

Sol. 3 1
(B) 13
16.

Sol. (D) pq 1

17.

Sol. 𝟏 1
(B) 𝟐

18.

Sol. (A) am  bl 1

6
MS_X_Mathematics_041_30/6/3_2022-23
19.

Sol. (A) 1

20.

Sol. (A) 1

21.

Sol. k:1
A(5,–6) B(– 1,– 4)
P
Let P(0, y) be the point on y axis which divides AB in the ratio k : 1 𝟏
𝟐

–k+5
=0 k=5 1
k+ 1

7
MS_X_Mathematics_041_30/6/3_2022-23
𝟏
Ratio is 5 : 1
𝟐

22(A).

Sol. In  ABC, DE ∥ BC
𝐴𝐷 𝐴𝐸 𝑥 𝑥+2
=  =
𝐷𝐵 𝐸𝐶 𝑥–2 𝑥–1
x(x – 1) = (x + 2)(x – 2) 1
x2 – x = x2 – 4  x = 4

1
OR
22(B).

Sol.
In  AOB and  COD,

 OAB =  OCD

 OBA =  ODC
1
Therefore,  AOB   COD 1
2
𝑂𝐴 𝑂𝐵
∴ 𝑂𝐶 = 𝑂𝐷 1
2

8
MS_X_Mathematics_041_30/6/3_2022-23
23(A).
Sol. n
If 6 ends with digit 0, it would be divisible by 5. So, prime factorization of
n n n n
6 would contain 5. But 6 = (2 × 3) , the only prime factorization of 6
are 2 and 3 as per fundamental theorem of Arithmetic . There is no other
n
prime in the factorization of 6 . So, there is no natural number n for which
n 2
6 ends with digit zero.

OR

23(B).
Sol.
72=23 X 32
120=23 X 3 X 5
HCF = 24 1
LCM=360 1

24.

Sol. Pole of height AB = 18 m


AP = length of shadow
18 1
In  APB, tan  = 𝐴𝑃
6 18 1
=
7 𝐴𝑃 2
 AP = 21 m 1
2

25.

9
MS_X_Mathematics_041_30/6/3_2022-23
Sol. 1
 AOB = 180 – 30 = 50
2

 OAP = 90 1
2
∴ APB = 180 – (50 + 90) = 40 1

SECTION C
This section comprises of Short Answer (SA) type questions of 3 marks
each.
26.

Sol. 𝑠𝑖𝑛 𝜃 cos 𝜃


LHS = (sin  + cos )( cos 𝜃 + ) 1
sin 𝜃

𝑠𝑖𝑛 2 𝜃 + cos2 𝜃
= (sin  + cos )( ) 1
cos 𝜃 sin 𝜃

(𝑠𝑖𝑛 𝜃 + cos 𝜃) .(1)


= 𝟏
𝑐𝑜𝑠 𝜃 sin 𝜃
𝟐

= sec  + cosec  = RHS 𝟏


𝟐

27(A).

Sol.
Let the natural number be x

160 1
ATQ, x + 12 =
𝑥

x2 + 12x = 160

x2 + 12x – 160 = 0 1

(x + 20)(x – 8) = 0

x ≠ – 20, x= 8
1

10
MS_X_Mathematics_041_30/6/3_2022-23
⇒ Required natural number is 8

OR

27(B).

Sol.
x2 + 12x – k = 0

Let the roots be , 3


𝟏
𝟐
 + 3 = – 12   = – 3
1
 . 3 = – k  32 = – k 1

 k = – 27 𝟏
𝟐

28.

Sol. Let P(x, 0) be the point on x axis which divides AB in the ratio k : 1
1
A(6, 3) k:1 B(– 2, – 5) 2
– 5k + 3 3 •
=0k= P
2
𝑘+1 5

Ratio is 3 : 5 1
2

29.

Sol. 60
A = 360 ×
22
× 21 × 21 = 231 cm2 1
7 1
2
60 22
Length of arc = 360 × 2 × 7
× 21
= 22 cm 1
1
2

11
MS_X_Mathematics_041_30/6/3_2022-23
30.

Sol. 1
Given ABC is an isosceles triangle, ∴ 𝐴𝐵 = 𝐴𝐶 ⇒ ∠𝐵 = ∠𝐶

In  ABD and  ECF,

 ADB =  EFC (90 each, given)

 ABD =  ECF 1

∴  ABD   ECF 1
31(A).

Sol. 26= 13 x 2
65= 13 x 5 1
117= 13 x 3 x 3
∴ HCF = 13 1
LCM = 13 x 2 x 3 x 5 x 3 = 1170 1

OR
31(B).

Sol. Let √𝟐 be a rational number.


𝐩 ½
∴ √2 = 𝐪 , where q≠0 and let p & q be co-primes.
2q2 = p2 ⟹ p2 is divisible by 2 ⟹ p is divisible by 2 1
⟹ p = 2a, where ‘a’ is some integer ----- (i)
4a2 = 2q2 ⟹ q2 = 2a2 ⟹q2 is divisible by 2 ⟹ q is divisible by 2 ½
⟹ q = 2b, where ‘b’ is some integer ----- (ii)
(i) and (ii) leads to contradiction as ‘p’ and ‘q’ are co-primes. 1
∴ √𝟐 is an irrational number.
SECTION D

12
MS_X_Mathematics_041_30/6/3_2022-23
This section comprises of Long Answer (LA) type questions of 5 marks
each.
32(A).

Sol.
Let a be the first term and d be the common difference.

7
S7 = 182  (2a + 6d) = 182 1
2

182 × 2
 2a + 6d = = 52
7

a + 3d = 26 -------- (i) 1

𝑎4 1 𝑎 + 3d 1
=  =  5a + 15d = a + 16d 1
𝑎17 5 a + 16d 5

4a = d --------(ii) 𝟏
𝟐

Solving (i) and (ii)

a = 2 and d=8 𝟏 1
𝟐
+2

∴ AP is 2, 10, 18, 26, …. 𝟏


𝟐

OR

32(B).

Sol.
Sq = 63q – 3q2

13
MS_X_Mathematics_041_30/6/3_2022-23
∴ S1 = 60  a1 = 60 (1st term) 1

S2 = 63(2) – 3(2)2 = 126 – 12 = 114

a1 + a2 = 114  a2 = 114 – 60 = 54 1

d = a2 – a1 = 54 – 60 = – 6 1

ap = – 60

60 + (p – 1)d = – 60

(p – 1)(– 6) = – 120  p = 21 1

1
a11 = a + 10d = 60 + 10(– 6) = 0

33(A).
Sol.
ABCD is a parallelogram touching the circle at P, Q, R, S by sides

AB, BC, CD, DA respectively.

1 for
correct
figure

We know that tangents drawn from the external point to a circle are
equal.

∴ AP = AS --------(i)

14
MS_X_Mathematics_041_30/6/3_2022-23
PB = BQ --------(ii)

CR = CQ --------(iii)
2

DR = DS ---------(iv)

Adding (i), (ii), (iii), (iv)

1
(AP + PB) + (CR + DR) = (AS + DS) + (BQ + CQ)

AB + CD = AD + BC

ABCD is a parallelogram
𝟏
𝟐
⇒AB = CD, AD = BC

⇒2AB = 2AD  AB = AD
𝟏
 ABCD is a rhombus. 𝟐

OR

33(B)

15
MS_X_Mathematics_041_30/6/3_2022-23
Sol.
PQ = PR (tangents drawn from an external point to the circle )

∴  PQR =  PRQ
1
1
In  PQR,  PQR =  PRQ = 2 (180 – 30) = 75

Draw a perpendicular QL from Q to QP 1

Now,  PQL = 90

∴ RQL = 90 – 75 = 15 1

RQL   SQL (SAS )


1

∴  RQL =  SQL = 15

∴  RQS = 15 + 15 = 30


1
34.

Sol.
(i)20 + 60 + 70 + x + 60 = 250
1
x = 250 – 210 = 40
Mass 80 – 100 100 – 120 120 – 140 140 – 160 160 – 180 Total
No. of 250
apples fi 20 60 70 x = 40 60
xi 90 110 130 150 170 1 for
correct
xifi 1800 6600 9100 6000 10200 33700 table
33700
Mean mass = = 134·8 1
250

16
MS_X_Mathematics_041_30/6/3_2022-23
Mean mass = 134.8 g

1
(ii) Modal class = 120-140
2
(70 – 60)
Mode = 120 + (140 – 60 – 40) × 20 1

1
= 125 2

Hence modal mass = 125 g

35.

Sol.
Radius of cone = radius of hemisphere = 7 cm
1
∴Height of cone = 14 cm

Volume of solid = Volume of hemisphere + volume of cone


1 1
2 1 1 +1
= 3(7)3 + (7)214 2 2
3

1 22
= × × 7 × 7(14 + 14)
3 7

154 4312
= × 28 = 𝑐𝑚2 or 1437.33 𝑐𝑚2 1
3 3

SECTION E
This section comprises of 3 case-study based questions of 4 marks each.

17
MS_X_Mathematics_041_30/6/3_2022-23
36.

Sol. 𝑂𝑃 1
(i) In  OBP, cos 30 = 𝑂𝐵 2

√3 36 72
= 𝑂𝐵  OB =
2 √3
= 24√3 cm 1
2

18
MS_X_Mathematics_041_30/6/3_2022-23
𝑃𝐵 36
(ii)In  OBP, tan 30 = 36  PB =
√3

PB = 12√3
1
𝐴𝑃
In  OAP, tan 45 = 36  AP = 36 cm
1
AB = AP – PB = 36 – 12√3 = 12(3 – √3) cm 2
1
2
OR

1
(ii)Area of  OPB = 2 × OP × PB

1
= 2 × 36 × 12√3 = 216√3 cm2
1+1
(ii) AP = 36 cm

19
MS_X_Mathematics_041_30/6/3_2022-23
37.

Sol. 1
(i)P (drawing ball bearing number 8) = 1
15

1
(ii)Even numbers = 2, 4, 6, 8, 10, 12, 14 2

No. of favourable outcomes = 7

7
P (even number) = 15 12
1

OR
1
(ii)Multiples of 3 are 3, 6, 9, 12, 15 2

No. of favourable outcomes = 5

20
MS_X_Mathematics_041_30/6/3_2022-23
5 1 1
∴ P(multiple of 3) = 15 = 3 12

(iii) Solid colour and even number 2, 4, 6, 8

4
P(solid colour and bear an even no.) = 15 1

38.

Sol.
(i)20x + 5y = 9000
1
5x + 25y = 26000

21
MS_X_Mathematics_041_30/6/3_2022-23
(ii)Solving the equations x = 200, y = 1000

Monthly fee paid by poor child = ₹200 2

OR

1
(ii) getting x=200 and y= 1000 1+
2

Difference in the fee = 1000 – 200 = ₹ 800 1


2

(iii)10x + 20y = 10(200) + 20(1000)

= ₹ 22000 1

22
MS_X_Mathematics_041_30/6/3_2022-23

You might also like